Автор: Nowak M.T.   Kaczor W.J.  

Теги: mathematics   mathematical analysis  

ISBN: 0-8218-2050-8

Год: 2001

Текст
                    STUDENT MATHEMATICAL LIBRARY
Volume 11


Problems in
Mathematical
Analysis II
Continuity and
Differentiation


w. J. Kaczor
M. T. Nowak


11


( )


)


)


(


)


lL


( J


- -


]n


Ji, I


(
,





Selected Titles in This Series 12 W. J. Kaczor and M. T. Nowak, Problems in mathematical analysis II: Continuity and differentiation, 2001 11 Michael Mesterton-Gibbons, An introduction to game-theoretic modelling, 2000 10 John Oprea, The mathematics of soap films: Explorations with Maple@ , 2000 9 David E. Blair, Inversion theory and conformal mapping, 2000 8 Edward B. Burger, Exploring the number jungle: A journey into diophantine analysis, 2000 7 Judy L. Walker, Codes and curves, 2000 6 Gerald Tenenbaum. and Michel Mendes France, The prime numbers and their distribution, 2000 5 Alexander Mehlm.ann, The game's afoot! Game theory in myth and paradox, 2000 4 W. J. Kaczor and M. T. Nowak, Problems in mathematical analysis I: Real numbers, sequences and series, 2000 3 Roger Knobel, An introduction to the mathematical theory of waves, 2000 2 Gregory F. Lawler and Lester N. Coyle, Lectures on contemporary probability, 1999 1 Charles Radin, Miles of tiles, 1999
Problems in Mathematical Analysis II Continuity and Differentiation
STUDENT MATHEMATICAL LIBRARY Volume 12 Problems in Mathematical Analysis II Continuity and Differentiation w. J. Kaczor M. T. Nowak . S "I3 ltNiiED '\ AM.BRICAN .MAT1iBM.ATICAL SOCIETY
Editorial Board David Bressoud Robert Devaney, Chair Carl Pomerance Hung-Hsi Wu Originally published in Polish, as Zadania z Analizy Matematycznej. CZfJSC Druga Funkcje Jednej Zmiennej-Rachunek R6zniczowy @ 1998, Wydawnictwo Uniwersytetu Marii Curie-Sklodowskiej, Lublin. Translated, revised and augmented by the authors. 2000 Mathematics Subject Classification. Primary 00A07; Secondary 26A06, 26A15, 26A24. Library of Congress Cataloging-in-Publication Data Kaczor, W. J. (Wieslawa J.), 1949- [Zadania z analizy matematycznej. English] Problems in mathematical analysis. I. Real numbers, sequences and series / W. J. Kaczor, M. T. Nowak. p. em. - (Student mathematical library, ISSN 1520-9121 j v. 4) Includes bibliographical references. ISBN 0-8218-2050-8 (softcover; alk. paper) 1. Mathematical analysis I. Nowak, M. T. (Maria T.), 1951- II. Title. III. Series. QA300K32513 2000 515'.076-dc21 99-087039 Copying and reprinting. Individual readers of this publication, and nonprofit libraries acting for them, are permitted to make fair use of the materiaI, such as to copy a chapter for use in teaching or research. Permission is granted to quote brief passages from this publication in reviews, provided the customary acknowledgment of the source is given. Republication, systematic copying, or multiple reproduction of any material in this publication is permitted only under license from the American Matp.ematical Society. Requests for such permission should be addressed to the Assistant to the Publisher t American Mathematical Society, P. O. Box 6248, Providence, Rhode Island 02940-6248. Requests can also be made bye-mail to reprint-permissionQams. org. @ 2001 by the American Mathematical Society. All rights reserved. The American Mathematical Society retains all rights except those granted to the United States Government. Printed in the United States of America. @ The paper used in this book is acid-free and falls within the guidelines established to ensure permanence and durability. Visit the AMS home page at URL: http://vww . ams. org/ 10 9 8 7 6 5 4 3 2 1 06 05 04 03 02 01
Contents Preface . Xl Notation and Terminology ... Xlll Problems Chapter 1. Limits and Continuity 1.1. The Limit of a Function 1.2. Properties of Continuous Functions 1.3. Intermediate Value Property 1.4. Semicontinuous Functions 1.5. Uniform Continuity 1.6. Functional Equations 1.7. Continuous Functions in l\1Ietric Spaces 3 9 14 18 24 21 32 Chapter 2. Differentiation 2.1. The Derivative of a Real Function 2.2. Mean Value Theorems 2.3. Taylor's Forn1ula and L'Hospital's Rule 2.4. Convex Functions 31 45 52 61 - vii
... vw Contents 2.5. Applications of Derivatives 68 2.6. Strong Differentiability and Schwarz Differentiability 77 Chapter 3. Sequences and Series of Functions 3.1. Sequences of Functions, Uniform Convergence 81 3.2. Series of FUnctions, Uniform Convergence 87 3.3. Power Series 96 3.4. Taylor Series 102 Solutions Chapter 1. Limits and Continuity 1.1. The Limit of a Function 111 1.2. Properties of Continuous Functions 129 1.3. Intermediate Value Property 146 1.4. Semicontinuous FUnctions 160 1.5. Uniform Continuity 171 1.6. FUnctional Equations 181 1.7. Continuous Functions in Metric Spaces 198 Chapter 2. Differentiation 2.1. The Derivative of a Real FUnction 211 2.2. Mean Value Theorems 233 2.3. Taylor's Formula and L'Hospital's Rule 245 2.4. Convex Functions 267 2.5. Applications of Derivatives 285 2.6. Strong Differentiability and Schwarz Differentiability 310 Chapter 3. Sequences and Series of Functions 3.1. Sequences of Functions, Uniform Convergence 317 3.2. Series of Functions, Uniform Convergence 336 3.3. Power Series 355 
Contents be 3.4. Taylor Series Bibliography - Books Index 372 393 397 
Chapter 1 Limits and Continuity 1.1. The Limit of a Function We adopt tbe following definitions. Definition 1. A real function I is said to be increasing (resp. strictly increasing, decreasing, strictly decreasing) on a nonempty set A C Ii if XI < X2, X., X2 E A, implies f(xIJ f(x2) (resp. f(x1) < f(x2), f(x!) > f(x2), f(x!) > f(x2». A function which is either in- creasing or decreasing (resp. strictly increasing or strictly 4 ecre.a. ct jn g) is called monotone (resp. strictly monotone). Definition 2. By a deleted neighborhood of a point a e III we mean the set (a - E,a + E) \ {a}, where E > O. l.l.t. Find the limits or state that they do not exist. (c) lim [ ! ] , a,b> 0, z-.O a x (d) lim [ 1 ] %-.0 x ; , limI=l , z-.o x (a) lim xcos!, z-.o X (b) (e) Jim X ( v'X 2 + 1- {/x 3 + 1 ) , %-"+00 (f) lim cos ( coox) z-+o sin{sinx) · - 3
4 Problems. 1: Limits and Continuity 1.1.2. Assume that f: (-a, a) \ {OJ -+ IR. Show that (a) lim J{x) = I if and only if lim J{sinx) = I, %o zo (b) if lim f(x) = I, then lim f(lxl) = I. Does the other impli- %O zo cation hold? 1.1.3. Suppose a function j: (-a, a) \ {OJ  (0,+00) satisfies lim ( f(z) +  ) = 2. Show that lim fez) = 1. zo I\J zo 1.1.4. Assume f is defined on a deleted neighborhood of a and lim ( f(z) +  ) = o. Determine lirn f(x). ZQ I/\/I ZQ 1.1.5. Prove that if f is a bounded function on [0, I} satisfying J(ax) = 6f(x) for 0  x   and a,6 > 1, then z+ f(x) = 1(0). 1.1.6. Calculate (a) (:c2(1+2+3+".+[])), (b) z (x ([  ] + [  ] +...+ [ : ])). k e K 1.1.7. Compute lim ,where P(x) is a polynomial with positive :r:-+oo J>{[ZJ) coefficients. 1.1.8. Show by an example that the condition (.) lim(/(x) + /(2x» = 0 zo does not imply that f has a limit at O. Prove that if there exists a function cp such tbat in a deleted neighborhood of zero the inequal- ity f(x) > (z) is satisfied and lim tp(x) = 0, then (*) implies z-+o lim fez) = o. %-+0 1.1.9. (a) Give an example of a function f satisfying the condition lim (f(x)j(2x» = 0 .z ...0 
1.1. The Limit of a Function 5 .and such that lim f(x) does not exist. -.o (b) Show that if in a deleted neighborhood of zero the inequalities f(x)  Ixl O t i < a < 1, and f(x)f(2x)  Ixl hold, then lim f(x) = o. z-.o 1.1.10. Given a real Q, assume that lim I%) = g(a) for each pos- Z-'oo itive a. Show that there is c such that g( a) = ca O . 1.1.11. Suppose that f: Ii -+ IR is a monotonic function such that lim ' , ([z») = 1. Show that also 1im ' , \C: / = 1 for each c > o. z-.oo z-.oo 1.1.12. Prove that if a > 1 and Q E Ii, then (a) a lim - = +00, -too Z (b) a Urn - = +00. z-+oo zQ 1.1.13. Show that if Q > 0, then lim I;: = o. z-.oo 1.1.14. For a > 0, show that 1im a Z = 1. Use this equality to prove z-+o the continuity of the exponential function. 1.1.15. Show that (a) lim ( 1 + ! ) Z = e, -+oo X (c) lim(! + x)  = e. z-t-O (b) lim ( 1 + .!. ) Z = e, z-.-oo x 1.1.16. Show that lim m(t +x) = O. Using this equality, deduce that z-tO the logarithmic function is continuous on (0,00). (c) 1.1.17. Determine the following limits: lim !n(l + z) a Z - 1 (a) ,(b) 1im , a > 0, z-tO X z-+O Z 1im (1 + x)O - 1 ICI , aEa. z-tO Z 
6 Problems. 1: Limits and Continuity 1.1.18. Find (a) (c) (e) lim (Inx)'!-, %-+00 Jim (cosx) .ia. , %-+0 lim (sin X) rf.. z-+o+ (b) lim z8in Z t %-.0+ (d) lim (e Z -l), z-+oo 1.1.19. Find the following limits: () lim sin 2x + 2 arctan3x + 3x 2 a %-+0 m( 1 + 3x + 5in 2 x) + xe Z ' . v i - e- Z - v i - cosx (c) lim , %-+0+ vs inx ( b ) lim In cos x z-+o tan x 2 ' (d) lim (1 + x 2 )cot%. z-+o 1.1.20. Calculate ( 1rx ) !- (a) lim tan 2 1 ' -+oo X + (b) zx(In(I+ ; )-ln ; ). 1.1.21. Suppose tbat lim g(x) = 0 and that there are Q E 1R and %-+0+ positive 1n and M such that m  I:)  JIll for positive x from a neighborhood of zero. Show that if 0: lim g(x) In x = 'Y, then z-+o+ lim f(x)9(z) = e"". In the case where'Y = 00 or "'f = -00 we assume Z'-+o+ that e oo = 00 and e- OO = O. 1.1.22. Assume that lim f(x) = 1 and lim g(x) = 00. Show that if %-+0 z-+o lim g(x)(f(x) -1) = '1, then lim l(x)9(%) = e. z-+O %-+0 1.1.23. Calculate (a) lim ( 2 sin Vi + Vi sin .!. ) % t z-+o+ X ( ) e-:' lim 1 + xe-;'- sin 14 ' z-+o X e Iim ( l+e-*arctan  +xe-*Sin ) . %-+0 X X (b) (c) 
1.1. The Limit of a Function 7 1.1.24. Let f: [0, +00)  R be a function such that each sequence {f{a+n)}, a > 0, converges to zero. Does the limit lim f(x) exist? :&-+00 1.1.25. Let f : [0, +(0) -+ IR be a function such that, for any positive a, the sequence {f (an)} converges to zero. Does the limit fun f(x) exist? %400 1.1.26. Let f : [0, +00) -+ IR be a function such that, for each a > 0 and each b > 0, the sequence {f(a+bn)} converges to zero. Does the limit Urn f(x) exist? %-i'00 1.1.27. Prove that if lim f(x) = 0 and lirn 1(2%)-/(z) = 0, then zo %40 Z lirn fez) = o. Z-i'O Z 1.1.28. Suppose that f defined on (a, +00) is bounded on each finite interval (a, b), 0 < b. Prove that if Urn (f(x + 1) - f{x» = I, then z-++oo also Iirn Iz) = I. z+oo 1.1.29. Let f defined on (a, +00) be bounded below on each finite interval (a, b), a < b. Show that if lim {/(x + 1) - f(x» = +00, %-++00 then also Jim fez) = +00. %-++00 % 1.1.30. Let f defined on (a, +00) be bounded on each finite interval (a,b), a < b. H for a nonnegative integer k, lim J(:r;+-J(z) exists, z-++oo then Ii fex) 1 r f(x + 1) - fez) :r;-+oo zk+l = k + 1 :t'-!oo zk · 1.1.31. Let / defincd on (0, +00) be bounded on each finite interval (a, b), a < b, and assume that f(x)  c > 0 for z E (a, +00). Show that if Jim Jj()l) exists, then lim (f(z» also exists and z-++oo z-++oo lim (f(x»  = lim f{x + 1) . Z+CO %4+00 f(x) 
8 Problems. 1: Limits and Continuity 1.1.32. Assume that lim / ( rt]-l ) = O. Does this imply that the -..o limit lim / (x) exists? z-..o 1.1.33. Let / : R  III be such that, for any a e Ii, the sequence {J ( : )} converges to zero. Does f have a limit at zero? 1.1.34. Prove that if lim / (x (  - r])) = 0, then lim lex) = o. z-..o zo (a) 1.1.35. Show that if J is monotonically increasing (decreasing) on (a, b), then for any :Co e (a, b), /(x:) = lim /(x) = inf /(:c) ( f(X:) = sup f(X» ) , z-t>zt z>zo Z>ZO f( xo ) = lim f(x) = sup f(x) ( /(x o ) = inf f(X» ) , Z-+-Zo %<zo %<%0 /( xo ) :s /(%0) :s /(xt) (/(xo)  /(xo)  text» . (b) (c) 1.1.36. Show that if / is monotonically increasing on (a, b), then for any Xo E (a, b), (a) (b) 1im I(x-) = I(xt), zz+ o lim f(z+) = I(x;). zzo 1.1.37. Prove the following Cauchy theorem. In order that / have a finite limit when x tends to a, a necessary and sufficient condition is that for every E > 0 there exists 0 > 0 such that I/(x) -/(z')1 < E whenever 0 < Ix - al < 0 and 0 < Ix' -al < o. Formulate and prove aD analogous necessary and sufficient condition in order that fun J(x) z-.oo exist. 1.1.38. Show that if lim f(x) = A and lim g(y) = B, then Z-'4 u-..A lim 9(/(X» = B, provided (9 0 f)(x) = g(f(x» is well defined and ;p;-.o f does not attain A in a deleted neighborhood of a. 
1.2. Properties of Continuous Functions 9 1.1.39. Find functions I and 9 such that lirn I(x) = A and lim g(y) z-+a JI-+A = B, but lim g(/{x» :F B. z-+o 1.1.40. Suppose I: Ii. -+ IR is an increasing function and x J-+ I(x) - x has the period 1. Denote by In the nth iterate 011; that is, II = f and In = 10 I n - 1 for n > 2. Prove that if lim In(O) exists, n-too n then for every x E IR, lirn Jnez) = lirn JraeO} . n-+oo n n-+(X) n 1.1.41. Suppose I: lIt R is an increasing function and x H- I(x} - x has the period 1. Moreover, suppose that 1(0) > 0 and p is a fixed positive integer. Let In denote the nth iterate of I. Prove that if m" is the least positive integer such that jmp(O) > p, then L < lim 1"(0) < lim In(o) < L + 1 + 1(0) . ffl" - n n - n-too n - m" m" 1.1.42. Suppose I: IR -+ IR is an increasing function and x t-+ /(x) - x has the period 1. Show tbat lirn In(z! exists and its value n-+oo n is the same for each z E IR, where In denotes the nth iterate of I. 1.2. Properties of Continuous Functions 1.2.1. Find all points of continuity of f defined by { 0 if x irrational, I{x) = sin Ixl if x is rational. 1.2.2. Determine the set of points of continuity of J given by f(x) = { z2 -1 z ir tional. o if x IS ratIonal. 1.2.3. Study the continuity of the following functions: o if x is irrational or x = 0, (a) I{x) = l/q if x = plq, p E Z, q E N, and p,q are co-prime,
10 Problems. 1: Limits and Continuity Ixl if x is irrational or x = 0, (b) /(x) = qxj(q + 1) if % = p/q, p E Z, q e N, and p, q are co-prime. (The function defined in (a) is called the Riemann function.) 1.2.4. Prove that jf I E C([a, b]), then 1/1 e C([a, b». Show by an example that the converse is Dot true. 1.2.5. Determine all an and b n for which the function defined by { an + sin 7rX if z E [2n, 2n + 1], n e Z, J(z) = b n + COS7fZ if x e (2n -I, 2n), neZ, is continuous on III 1.2.6. Let lex) = [x2] sin 'lrX for z E I!L Study the continuity of /. 1.2.7. Let /(x) = [xl + (x - [x»lz] for x   . Show that / is continuous and that it is strictly increasing on [1,(0). 1.2.8. Study the continuity of the following functions and sketch their graphs: (a) n Z - n- z I(x) = Urn 1 % E R, n-too n Z + n- z lex) = lirn x 2 e n% + x t x e  n-+oo e nz + 1 f(x) = lim m(e n + 3;n) 1 X  0, n-foOO n (b) (c) (d) (e) f(x) = lim ft 4 n + X 2n +..!...., X:F 0, n--.oo 3;2n f(z) = lim 2 V'C OS 2n Z + sin 2n x, z e III n--.oo 1.2.9. Show that if J : IR -+ 111 is continuous and periodic, then it at tains its supremum and infimum. 
1.2. Properties of Continuous Functions 11 1.2.10. For P(x) = x 2n + a2n_lx 2n - 1 + .. . + alX + ao, show that there is x. E R such that P(x.) = inf{P(x): % E Ii}. Show also that the absolute value of any polynomial P attains its infimum; that is, there is x. E IR such that IP(x*)1 = inf{lP(x)l: x E IIi}. 1.2.11. (a) Give an example of a bounded function on [0, 1] which achieves neither an infimum nor a supremum. (b) Give an example of a bounded function on [0, 1] which does Dot achieve its infimum on any [a, b] C [0,1], a < b. 1.2.12. For J : lR -t Ii, Xo E R and 6 > 0, set WJ(Xo, 6) = suptlf(x) -f(xo)l: x E IR, Ix - xol < 6} and wJ(xo) = lim w/(xo,6). Show that / is continuous at Xo if and &-..0+ only if w/(xo) = O. 1.2.13. (a) Let /,g E C([a,b)) and for x E [a,b] let hex) = mint/ex), g(x)} and H(x) = max{f(x), g(x)}. Show that h,H E C([a,b]). (b) Let 1),/2,J3 E C([a,b» and for x e [a,b] let J(x) denote that one of the three values fl(X),J2(X) and /3(X) that lies between the other two. Show that f E C([a,b)). 1.2.14. Prove that if f E C([a, b]), then the functions defined by setting m(x) = inf{/C,): ,e [Cl,XJ} and Af(x) = sup{/«(): 'E [a, x]} are also continuous on [a, b]. 1.2.15. Let f be a bounded function on [a, b]. Show that the functions defined by m(x) = inf{!«(): (E [a, x)} and kf(x) = sup{f«(): {E [a,x)} are continuous from the left on (at b). 
12 Problems. 1: Limits and Continuity 1.2.16. Verify whether under the assumptions of the foregoing prob- lem the functions m*(x)=inf{j(): (E [a, x]} and .t\-l*(x)=sup{j«): (E [a,x]} are continuous from the left on ( a, b). 1.2.17. Suppose f is continuous on [a,oo) and lim f(x) is finite. :z:-+oo Show that I is bounded on [a,oo). 1.2.18. Let I be continuous on Ii and let {xn} be a bounded se- quence. Do the equalities !!!!! J(x n ) = f( !!!!! x n ) and iiiii f(x n ) = f( liiii x n ) n-tooo n-too n-+oo n-+oo hold? 1.2.19. Let / : IR -+ R be increasing and continuous and let {x n } be a bounded sequence. Show that (a) (b) !!!!! f(xn) = I( !!!!! x n ), n-tooo n-too iiiii f(zn) = f( iiiii x n ). n-+oo n-+oo 1.2.20. Let f : R -+ Ii be decreasing and continuous and let {xn} be a bounded sequence. Show that (a) (b) !!m f(xn) = f( nm x n ), n-too n-+oo liiD f(xn} = I( lim x n ). n-tooo n-+oo 1.2.21. Suppose that f is continuous on Ii, Urn f(x) = -00 and %-+-00 1im f(x) = +00. Define 9 by setting z-+oo g(x) = sup{t: jet) < x} for x E R (a) Prove that 9 is continuous from the left. (b) Is 9 continuous? 
1.2. Properties of Continuous Functions 13 1.2.22. Let I: 1R --. III be a continuous periodic function with two incommensurate periods T 1 and T2; that is, R- is irrational. Prove that I is a constant function. Give an example of a nonconstant periodic function with two incommensurate periods. 1.2.23. (a) Show that if /: Ii --.  is nonconstant, periodic and continuous, then it has a smallest positive period, the so-called fundamental period. (b) Give an example of a noncoDstant periodic function without a fundamental period. (c) Prove that if /: R -+ Ii is a periodic function without a funda- mental period, then the set of all periods of f is dense in III 1.2.24. (a) Prove that the theorem in part (a) of the preceding problem re- m8-1ns true when the continuity of J on Ii is replaced by the continuity at one point. (b) Show that if J : III -+ IR is a periodic function without a funda- mental period and if it is continuous at least at one point, then it is constant. 1.2.25. Show that if /, 9 : Ii -+ It are continuous and periodic and lim (f(x) - g(x» = 0, then f = g. z-tco 1.2.26. Give an example of two periodic functions f and 9 such that any period of J is not commensurate with any period of 9 and such that /+g (a) is not periodic, (b) is periodic. 1.2.27. Let /,g: R -+ R be continuous and periodic with positive fundamental periods Tl and T 2 , respectively. Prove that if R-  Q, then h = f + 9 is not a periodic function. 
14 Problems. 1: Limits and Continuity 1.2.28. Let I, 9 : IR  R be periodic and suppose that I is continu- ous and no period of 9 is commensurate with the fundamental period of I. Prove that f + 9 is not a periodic function. 1.2.29. Prove that the set of points of discontinuity of a monotonic function I: Ii --. Ii is at most countable. 1.2.30. Suppose I is continuous on [0,1]. Prove that 1 n ( k ) lim - (-l)kl - = 0. n-+oo n L..J n k=J 1.2.31. Let I be continuous on [0, 1 J. Prove that ; F  (_l)k()1 () = o. 1.2.32. Suppose I : (0, 00) --. Ii is a continuous function such that f(x)  f(nx) for all positive z and natural n. Show that lim I(z) z-+oo exists (finite or ionni te). 1.2.33. A function I defined on an interval I C Ii is said to be convex on I if I(Ja. + (1 - A)Z2) < A/(xl) + (1 - A)/(X2) whenever Z],Z2 E I and  E (0,1). Prove that if I is convex on an open interval, then it is continuous. Iust a convex function on an arbitrary interval be continuous? 1.2.34. Prove that if a sequence {In} of continuous functions on A converges uniformly to J on A, then J is continuous on A. 1.3. Intermediate Value Property Recall the following: Definition. A real function I has the intermediate value property on an interval I containing [a, b) if f(a) < v < f(b) or J(b) < v < J(a); 
1.3. Intermediate Value Property 15 that is, if v is between f(a) and f(b), there is between a and b a c such that fee) = v. 1.3.1. Give examples of functions which have the intermediate value property on an interval I but are not continuous on this interval. 1.3.2. Prove that a strictly increasing function f: [a, b) Ii which has the intermediate vcdue property is continuous on [a, b]. 1.3.3. Let f: (0,1] -. [OJ 1] be continuous. Show that f has a fixed point in [O,IJ; that is, there exists Xo E [0, 1] such that f(xo) = xo. 1.3.4. Assume that f, 9 : [a, b] Ii are continuous and such that f(a) < g(a) and f(b) > g(b). Prove that there exists Xo e (a, b) for which f(xo} = g(xo). 1.3.5. Let I: IR -+ IR be continuous and periodic with period T > 0. Prove tbat there is Xo such that f (xo + ) = f(xo). 1.3.6. A function I : (a, b) -+ IR is continuous. Prove that, given x., X2, ... ,X n in (a, b), there exists Xo E (a, b) such that 1 f(xo) = - (f(x.) + f(X2) +... + f(x n ». n 1.3.7. (a) Prove that the equation (I - x) cosx = sinx has at least one solution in (0,1). (b) For a nonzero polynomial P, show that tbe equation IP(x)1 = ez has at least one solution. 1.3.8. For no < be < al < b I < ... < an < b n , show that all roots of the polynomial n n P(x) = II (x + all:) + 2 II (x + ble), x e II(, k =O k=O are real.
16 Problems. 1: Limits and Continuity 1.3.9. Suppose that f and 9 have the intermediate value property on [a, b]. lVIust I + 9 possess the intermediate value property on that . terva} " m . 1.3.10. Assume that I E C([0, 2]) and f(O) = f(2). Prove that there exist Xl and X2 in [0,2] such that X2 - Xl = 1 and l(x2) = I(xl). Give a geometric interpretation of this fact. 1.3.11. Let I E C([O,2]). Show that there are Xl and X2 in [0,2] such that X2 - Xl = 1 and 1 l(x2) - f(x1) = 2 (f{2) -/(0». 1.3.12. For n e N, let I E C([O, n]) be such that 1(0) = fen). Prove that there are %1 and %2 in [0, n] satisfying X2 - Xl = 1 and l(x2) = I(xl). 1.3.13. A continuous function f on [0, n), n E N, satisfies f(O) = fen). Show that for every k E {1,2,...,n -l} there are Xi and!tt such that I(x,,) = I(x), where X" - xk = k or X" - xl = n - k. Is it true that for every k e {l, 2, . . . , n - I} there are Xle and x k such that f(x,,) = I(xt), where x" - xl = k? 1.3.14. For n e N, let I E C([O, n]) be such that 1(0) = fen). Prove that the equation f (x) = f(y) has at least n solutions with x - YEN. 1.3.15. Suppose that real continuous functions / and 9 defined on R commute; that is, I(g(%» = g(/(x» for X E III Prove that if the equation /2(x) = g2(X) has a solution, then the equation /(x) = g(x) also has (here f 2 (z) = f{f(x» and g2(x) = g(g(x»). Show by example that the assumption of continuity of f and 9 in the foregoing problem  nn ot be omitted. 1.3.16. Prove that a continuous injection I: R -+ Ii is either strictly decreasing or strictly increasing. 
1.3. Intermediate Value Property 17 1.3.17. Assume that J : JR -+ R is a continuous injection. Prove that if there exists n such that the nth iteration of / is an identity, that is, /n(x) = x for all x E IR, then (a) I(x) = z, z E Ii, if f is strictly increasing, (b) /2(x) = x, z E If, if I is strictly decreasing. 1.3.18. Assume J: R -+ IR satisfies the condition I(/{x» = /2(X) = -x, :r; E 1R. Show that f nn ot be continuous. 1.3.19. Find all functions I : 1R. -+ R which have the intermediate value property and such that there is n e N for which In(x) = -x, Z E IR, where In denotes the nth iteration of /. 1.3.20. Prove that if J : R -+ Ii has the intermediate value property and J-l({q}) is closed for every rational q, then I is continuous. 1.3.21. Assume that f : (a, (0) -+ IR is continuous and bounded. Prove that, given T, there exists a sequence {x n } such that lim X n = +00 and Urn (f(xn + T) - I(zn» = o. n-too n...oo 1.3.22. Give an example of a continuous function /: Ii -+ R which attains eacb of its values exactly three times. Does there exist a continuous function J : Ii -+ IR which attains each of its values exactly two times? 1.3.23. Let f : (0,1) -+ III be continuous and piecewise strictly mono- tone. (A function J is said to be piecewise strictly monotone on [0,1], if there exists a partition of [0,1] into finitely many subinter- vals [ti-lt td, where i = 1,2,... ,n and 0 = to < tl < · .. < t n = 1, such that f is strictly monotone on each of these subintervals.) Prove that J attains at least one of its values an odd number of times. 1.3.24. A continuous function J : [0,1] -t Ii at tin each of its values finitely many times and 1(0) #: 1(1). Sho\v that / attains at least one of its values an odd number of times. 
18 Problems. 1: Limits and Continuity 1.3.25. Assume that / : K 4- K is continuous on a CODlpact set K C III Moreover t assume that an Xo E K is such that each limit point of the sequence of iterates {/"(xo)} is a fixed point of I. Prove tbat {/n(xo)} is convergent. 1.3.26. A function I : III -+ Ii is increasing, continuous, and such that F defined by F(x) = I(x) - x is periodic with period 1. Prove that if a(/) = lim Jft O} , then there is Xo E [0,1] such that F(xo) = n-too a(/). Prove also that I has a fixed point in [0,1] if and only if a(f} = O. (See Problems 1.1.40 - 1.1.42.) 1.3.27. A function / : [0,1] -+ III satisfies 1(0) < 0 and /(1) > 0, and there exists a function 9 continuous on [0, 1] and such that I + 9 is decreasing. Prove that the equation J(x) = 0 has a solution in the open interval (Ot 1). 1.3.28. Show that every bijection I : Ii -to (0,00) has in f1n i tely many points of discontinuity. 1.3.29. Recall that each x E (0, I) can be represented by a binary fraction .a)a2a3..., where ai E {O,I}, i = 1,2,... . In the case where x has two distinct binary expansions we choose the one with infinitely many digits equal to 1. Next let a function I : (0, 1) -+ [0, 1] be defined by _ 1 n I(x) = Urn - " ai. n-too n L.-i i=l Prove tbat f is discontinuous at eacb x E: (0,1) but nevertheless it has the intermediate value property. 1.4. Semicontinuous Functions Definition 1. The extended real number system iR consists of the real number system to which two symbols, +00 and -00, have been adjoined, with the following properties: (i) H x is real, then -00 < x < +00, and x + 00 = +00, x - 00 = -00 and -:L = 2- = o. +00 -00 (ii) H x> 0, then x. (+00) = +00, x. (-00) = -00.
1.4. Semicontinuous Functions 19 (ill) IT x < 0, then x. (+00) = -, x. (-00) = +00. Definition 2. HAC R is a nonempty set, then sup A (resp. inf A) is the smallest (resp. greatest) extended real number which is greater (resp. slnalJer) than or equal to each element. of A. Let 1 be a real-valued function defined on a nonempty set A c '" Definition 3. IT Xo is a limit point of A, then the li1nit inferior (resp. the limit superior) of f(x) as x  Xo is defined as the infimunl (resp. the supremum) of the set of ally E IR such that there is a sequence {x n } of points in A which is convergent to xo, whose terms are aU different from Xo and y = lim f{x n ). The liInit inferior and the limit n-too superior of f(x) as x  Xo are denoted by Jim f(x) and lim f(x), z-tozo Z-+ZO respecti vely. Definition 4. A real-v-c1llled function is said to be lower (resp. upper) semicontinuo'US at all Xo E A which is a limit point of A if lim f(x) > Z-+ZO f(xo) (resp. lilll I(x) < f{xo». IT Xo is an isolated point of A, then '%-+Zo we assume that f is lower and upper semicontinuous at that point. 1.4.1. Show that if Xo is a Iimit point of A and / : A --) IR, then (a) (b) lim f(x) = sup inC{f(x): x E A, 0 < Ix - xol < oj, z-+Zo 6>0 TIm f(x} = inf sup{f(x): x e A, 0 < Ix - xol < 6}. ;e-tozo 6>0 1.4.2. Show that if Xo is a limit point of A and J : A  IR, then (a) (b) !!!!! f(x) = lirn inr{f(x): x E A, 0 < Ix - xol < oJ, z-toZO 6...0+ lim f(x) = lim snp{f(x): x E A, 0 < Ix - xol < oj. Z-"Zo 6-+0+ 
20 Problems. 1: Limits and Continuity 1.4.3. Prove that Yo e IR is the limit inferior of f : A -+ IIi at a limit point Xo of A if and only if for every E > 0 the following two conditions are satisfied: (i) there is 6 > 0 such that f(x) > Yo - E for all x E A with o < Ix - xol < 6, (ii) for every 6 > 0 there is z' E A such that 0 < Ix' - xol < 6 and f(x') < Yo + E. Establish an analogous statement for the limit superior of f at Xo. 1.4.4. Let f : A -+ .Ii and let Xo be a limit point of A. Prove that (a) lim f(x) = -00 if and only ilfor any real y and for any 6> 0 %Zo there exists x' e A such that 0 < Ix' - zol < 6 and f(x') < y. (b) lim f(x) = +00 if and only if for any real y and for any 6 > 0 z-tozo there exists x' e A such that 0 < Ix' - xol < 6 and fex') > y. 1.4.5. Suppose f : A -+ B and Zo is a limit point of A. Show that if I = Jim f(x) (resp. L = Iim f(x», then there is a sequence Z-+ZO z-tozo {xn}, X n e A, X n :F Xo, converging to Xo such that I = lim f(xn) n-tooo (resp. L = lim f(x n ». n-.oo 1.4.6. Let /: A -+ R and let Xo be a limit point of A. Prove that !!!!! (-/(x» = - nm lex) and iIiii (- f(x» = - lim f(x). %-+zo z-tozo z"'zo Z-'Zo 1.4.7. Let f : A -+ (0,00) and let Xo be a limit point of A. Show that lim 1 = _1 and lim 1 = 1 %-:+;o f(x) lim lex) zzo f(x) lim f(z). z-tozo Z-lo%O (We assume that + = 0 and r/F = +00.) 1.4.8. Assume that f, 9 : A -+ Ji and that Xo is a limit point of A. Prove that (excluding the indeterminate forms of the type +00 - 00 
1.4. Semicontinuous Functions 21 and -00 + 00) the following inequalities hold: !ill! I{x)+ !ill! g(x)  fun (f(x) + g{x» < !!!!! I(x)+ lirn g(x) %-+%0 z-tzo z-u:o z-+%O z-tzo =:; lirn (f(x) + g(x»:5 iiiii f(z) + iiiii g(x). %-+%0 %-+ZO %-+%0 Give examples of functions for which ":5 " in the above inequalities is replaced by "< "_ 1.4.9. Assume that I, 9 : A -. [0, 00) and that Xo is a limit point of A_ Prove that (excluding the indeterminate forms of the type 0-(+00) and (+00). 0) the following inequalities hold: lim I(x). lim g(x) < Urn (f(x) - g(x»  !ill! I(x). lim g(x) %-+%0 %-+ZO %-+ZO z-+%o z-+zo =:; lim (/(x) . g(x»:5 iiiii f{x). iWi g(x). %-+%0 %-tzo z-tzo Give examples of functions for which "=:; " in the above inequalities is replaced by "<". 1.4.10. Prove that if 1im I(x) exists, then (excluding tbe indeter- %-+%0 minate forms of the type +00 - 00 and -co + (0) fu!! (/(x) + g(x» = lim f(x) + !ill! g(x), %-+:1:0 z-u:o z-.%o Jim (f(z) + g(z» = lim I(x) + iiiii g(x). %-foZO z-tzo z-.zo Moreover, if f and 9 are nonnegative, then (excluding the indetermi- nate forms of the type o. (+00) and (+00) .0) !!!!! (f(x) . g(x» = lim f(x) - lim g{x), z-.%O z-.%o z-+zo liiii (/(x) · g(x» = lim I(x). lim g(z). z-tzo %-+%0 z-+%O 1.4.11. Prove that if I is continuous on (a, b), , = Urn f(x) and %-+a L = Jim f(x), then for every  E [I, L] tbere is a sequence {x n } of %-.a points in (a, b) converging to a and such that Jim I (zn) = . n-too 
22 Problems. 1: Limits and Continuity 1.4.12. Find the points at which f: IR --. Ii defined by I(x) = { o. smx if x is irrational, if x is rational is semicontinuous. 1.4.13. Determine points at which the function f defined by { 2 1 x - f(x) = 0 if x is irrational, if x is rational is semicontinuous. 1.4.14. Show that the function given by setting o if x is irrational or x = 0, f(x) = if x = :' p e Z, q e N, and p, q are co-prune is upper scmicontinuous. 1.4.15. Find the points at which tbe function defined by Ixl (a) f(x) = .!1=-. q+l if x is irrational or x = 0, if x = , p e Z, q e N, and p, q are co-prime, {-I tip 9+ 1 if x E Qn (0, 1] and x = t p,q e N, and p, q are co-prime, if x E (0, 1) is irrational (b) f(x) = o is neither upper nor lower semicontinuous. 1.4.16. Let /,g : A --+ JR he lower (resp. upper) semicontinuous at Xo e A. Show that (a) if a > 0, then af is lower (resp. upper) semicontinuous at Xo- H a < 0, then of is upper (resp_ lower) semi continuous at :Co- (h) f + 9 is lower (resp. upper) semi continuous at Zo-
1.4. Semicontinuous Functions 23 1.4.17. Assume that fn : A -+ R, n E N, are lower (resp. upper) semi continuous at Xo E A. Show that sup In (resp. inf In) is lower nEN nEN (resp. upper) semicontinuous at Xo. 1.4.18. Prove that a pointwise limit of an increasing (resp. decreas- ing) sequence of lower (resp. upper) semicontinuous functions is lower (resp. upper) semicontinuous. 1.4.19. For f: A -+ JR and x a limit point of A define the oscillation of f at x by o/(x) = lim sup{l/(z) - f(u)l: z, u E A, Iz - xl < 6, In - xl < 6} 6-.0+ Show that o/(x) = II (x) - f2(X), where fl(x) = max{f(x), iiiii fez)} and f2(X) = min{f(x), lim fez)}. zz  1.4.20. Let fl, f2t and 01 be as in the foregoing problem. Show that II and 01 are upper semicontinuous, and f2 is lower semicontinuous. 1.4.21. Prove that in order that I : A -+ Ii be lower (resp. upper) semicontinuous at Xo E A, a necessary and sufficient condition is that for every a < I(xo) (resp. a > f(xo» there is 6 > 0 such that f(x) > a (resp. I(x) < a) whenever Ix - xol < 6, x E A. 1.4.22. Prove that in order that f : A -+ R be lower (resp. upper) semicontinuous on At a necessary and sufficient condition is that for every a E IR the set {x E A: f{x) > a} (resp. {x e A: f(x) < a}) be open in A. 1.4.23. Prove that f : Ii -+ Ii is lower semicontinuous if and only if the set {(x,y) E R 2 : y > f(x)} is closed in Ii:!. Formulate and prove an analogous necessary and sufficient con- dition for upper semicontinuity of f on III 1.4.24. Prove the following theorem of Baire. Every lower (resp. up- per) semicontinuous f: A -+ JR is the pointwise limit of an increasing (resp. decreasing) sequence of continuous functions on A. 
24 Problems. 1: Limits and Continuity 1.4.25. Prove that if I : A -to III is upper semicontinuous, 9 : A -to It is lower semicontinuous and I(z) < g(z) everywhere on A, then there is a continuous function h on A such that I(x)  h(x) < g(z), x e A. 1.5. Uniform Continuity Definition. A real function 1 defined on A c Ii is said to be uni- formly continuous on A if, given E > 0, there exists 6 > 0 such that for all x and y in A with Iz - yl < 6 we have I/(x) -/(y)1 < E. 1.5.1. Verify whether the following functions are uniformly continu- ous on (0,1) : (a) I(x) = e Z , (b) fez) = sin!, x (c) f(x) = x sin !, (d) 1 I(z) = e. , x (e) I(z) = e-!- , (f) 1 fez) = e% cos-, x (g) I(:c) = In, (h) 7r I{x) = C05X. COS- , z (i) f(x) = ootz. 1.5.2. Which of the following functions are uniformly continuous on [0,00)1 (a) J(z) = -IX, (c) J(x) = 5in 2 z, (e) f(x) = e 1 (g) J(z) = sin (sin x), (i) fez) = sin.vz. (b) (d) (f) (h) J(z) = z sin x, f(x) = sin(x 2 ), I(x) = e Sin (z2), I(z) = sin(xsinz), 1.5.3. Show that if J is uniformly continuous on (a, b), a, b E 1R, then Um I(z) and fun j(z) exist as finite limits. z-ta+ z-+6- 
1.5. Uniform Continuity 25 1.5.4. Suppose I and 9 are uniformly continuous on (a, b) ([a,oo}). Does this imply the uniform continuity on (a, b) ([a, 00» of the func- tions (a) f + 9, (b) /g, (c) x t-+ fez) sin x? 1.5.5. (a) Show that if f is uniformly continuous on (a,b] and on [b,e), then it is also uniformly continuous on (a, c). (b) Suppose A and B are closed sets in Ii and let I : A U B -+- IR be uniformly continuous on A and on B. Must I be uniformly continuous on AU B? 1.5.6. Prove that any function continuous and periodic on 1R. must be uniformly continuous on III 1.5.7. (a) Show that if / : 1R -+ 1R is continuous and such that lim f(x) -t-oo and lirn f(x) are finite, then I is uniformly continuous on III -toOO (b) Show that if / : [a,oo) R is continuous and Iim fez) is finite, z-too then J is uniformly continuous on [a, (0). 1.5.8. E x min e the uniform continuity of (a) I(x) = arctanx on (-00,00), (b) fez) = zsin on (0,00), (c) lex) = e-: on (0,00). 1.5.9. Assume that f is uniformly continuous on (0, (0). Must the limits lim lex) and lim I(x) exist? z-+o+ -IoOO 1.5.10. Prove that any function which is bounded, monotonic and continuous on an interval I C Ii is uniformly continuous on I. 1.5.11. Assume f is uniformly continuous and unbounded on [0,00). Is it true that either lirn I(x) = +00 or li.m f(x) = -oo? z-too Z-loOO
26 Problems. 1: Limits and Continuity 1.5.12. A function f : [0,00) -). JR is uniformly continuous and for any x 2= 0 the se<)uence {f(x + n)} converges to zero. Prove that lim f(x) = O. -too 1.5.13. Suppose that / : [1,00) -). JR is uniformly continuous. Prove that there is a positive M such that J/%>l < kf for x 2: 1. 1.5.14. Let f : (0,00) -). IR be uniformly continuous. Prove that there is a positive !vI with the following property: sup{l/(z + u) - f(u)l} < /vf(x + 1) for every x > o. u>o 1.5.15. Let I : A  IR, A c IR, be uniformly continuous. Prove that if {xn} is any Cauchy sequence of elements in A, then {f(xn}} is also a Cauchy sequence. 1.5.16. Suppose A C IR is bOWlded_ Prove that if J : A -). R transforms Cauchy sequences of elelnents of A into Cauchy sequences, then f is uniformly continuous on A. Is the boundedness of A an essential assumption? 1.5.17. Prove that f is uniformly continuous on A C IR if and only if for any sequences {xn} and {Yn} of elements of A, lim (x n - Yn) = 0 implies lim (/(x n ) - I(Yn» = o. n-+oo n-+oo 1.5.18. Suppose that I : (0, (0)  (0,00) is uniformly continuous. Does this imply that I - / (x +  ) - I ? 1m - . ;z:-+co I(x) 1.5.19. A function f: IR -7 Ii is continuous at zero and satisfies the following conditions f(O) = 0 and f(xl + X2) =:; I(xl) + l(x2) for any XI, X2 e III Prove that I is uniformly continuous on IR. 
1.6. Functional Equations 27 1.5.20. For / : A  IR, A C IR, we define w/(6) = suP{I/(Xl) - l{x2)1: Xt,X2 e A, Ix 1 - x21 < o} and call wIthe modulus of co71tintlity of f. Show that I is uniformly continuous on A if and only if lim W J (6) = o. .5 -+U'" 1.5.21. Let f : R  R be wliformly continuous. Prove that the following statements are equivalent. (a) For an). uniforntly continuous function 9 : ri  Hi, f. 9 is uni- Formly continuous on IR. (b) The function x t-+ Ixl/(x) is uniforully continuous on I!l 1.5.22. Prove that the following condition is necessary and sufficient for / to be uniformly continuous on an interval I. Given E > 0, there is N > 0 such that for every XI,X2 E I, Xl  X2, f(XI) - l(x2) I > N iml)lies I/(xl) - !(x2)1 < E. Xl - X2 1.6. Functional Equations 1.6.1. Prove that the only functions continuous on Ii and satisfying the Cauchy functional equation f{x + y) = I(x) + fey) are the linear functions of the foml f(x) = ax. 1.6.2. Prove that if f: IR  R satisfies the Cauchy functional equa- tion f(x + V) = f(x) + f(y) and one of the conditions (a) / is continuous at an Xo E Ii, (b) f is bounded above on some interval (a,b), (c) I is monotonic on Ii, then lex) = ax. 
28 Problems. 1: Limits and Continuity 1.6.3. Determine all continuous functions I : R ]i uch that /(1) > 0 and J(x + y) = f(x)f(y). 1.6.4. Show tbat the only solutioDS of the functional equation J(xy) = I(z) + I(y) which are not identically zero and are continuous on (0,00) are the logarithmic functions. 1.6.5. Show that the only solutions of the functional equation I(zy) = f(x)/(Y) which are not identically zero and are continuous on (0, 00) are the power functions of the form I(x) = xo. 1.6.6. Find all continuous functions I: R -4 Ii such that I(z) -/(Y) is rational for rational z - y. 1.6.7. For Iql < 1, find all functions I : R -4 ni. continuous at zero and satisfying the functional equation J(x) + f(qx) = O. 1.6.8. Find all functions I : R R continuous at zero and satisfying the equation I(x) + I (: x) = x. 1.6.9. Determine all solutions f: R R of the functional equation 2/(2%) = f(x) + z which are continuous at zero. 1.6.10. Find all continuous functions I : IR -+ Ii satisfying the Jensen equation I e ;, ) = f(x); I(y) .
1.6. Functional Equations 29 1.6.11. Find all functions continuous on (at b)t at bEll, satisfying the Jensen equation f e;y ) = f{x);f{Y) , 1.6.12. Determine all solutions I: Ii  Ii of the functional equation /(2x + 1) = /(x) which are continuous at -1. 1.6.13. For a real a, show that if I: Ii --. Ii is a continuous solution of the equation /(x + y) = I(x) + fey) + axy, then f(x) = i x2 + bx, where b = /(1) - i. 1.6.14. Determine all continuous at zero solutions of the functional equation f{x) = f ( 1  x ), x =F 1. 1.6.15. Let I : [0, 1] --. [0, 1] be continuous, monotonically decreas- ing and such that I(/(x» = z for x E [0,1]. Is lex) = 1- z the only such function? 1.6.16. Suppose that I and 9 satisfy the equation f(x + y) + I(x - y) = 2/(x)g(y), z,y E III Show that if I is not identically zero and I/(x) 1 < 1 for x E Ill, then also 10(x)1 S 1 for x E III 1.6.17. Find all continuous functions / : Ii --. JR satisfying the func- tional equation I(x + y) = l(x)e Y + I(y)e z . 1.6.18. Determine all continuous at zero solutions I : Ii -. .Ii of I(x + y) -/(x - y) = J(x)f(y). 
30 Problems. 1: Limits and Continuity 1.6.19. Solve the functional equation ( X -1 ) lex) + I :c = 1 + x for x # 0, 1. 1.6.20. A sequence {xn} converyes in the Cesaro sense if C Ii I . Xl + X2 + %3 +... + X n - m X n = lID n-..oo n-+oo n exists and is finite. Find all functions which are Cesaro continuous, that is, I(C- lim xn} = C- lim f(xn) n-..oo noo for every Cesaro convergent sequence {x,}. 1.6.21. Let I : [0, 1]  [0, 1] be an injection such that f(2x - I(x» = x for X E [0,1]. Prove that f(x) = x, X E [0,1]. 1.6.22. For m different from zero, prove that if a continuous function I : IR  IR satisfies the equation f ( 2x - f» ) = mx, then f(x) = m(x - c). 1.6.23. Show that the only solutions of the functional equation f(x + y) + J(y - x) = 2J(x)J(y) continuous on II( and not identically zero are J (x) = cos( ax) and f(x) = cosh(ax) with a real. 1.6.24. Determine all continuous on (-1,1) solutions of f ( t ::V ) = f(x) + f(y). 1.6.25. Find all polynomials P such that P(2x - x 2 ) = (p(x»2. 
1.6. Functional Equations 31 1.6.26. Let m, n 2 be integers. Find all functions I : [0,00) -+ R continuous at at least one point in [0, 00) and such that f ( ! tXi ) =! t(J(Xi»,n for Xi > 0, i = 1,2,...,n. 11 . 1 11 . 1 1= .= 1.6.27. Find all not identically zero functions I : IR IR satisfying the equations I(xy) = I(x)f(y) and f(x + z) = f(x) + fez) with some z ;:f; o. 1.6.28. Find all functions I : Ii \ {OJ -. Ii such that J(x) = -I G ), X O. 1.6.29. Find all solutions I : Ii \ {OJ -+ IR of the functional equation I(x) + l(x2) = 1 G) + 1 ( ). x O. 1.6.30. Prove that the functions I, g, t/J : IR -. R satisfy the equation I(x) - g(y) = l/J ( x + y ) , Y =F x, x-y 2 if and only if there exist a, b and c such that f(x) = O(x) = ax 2 + bx + c, ,p(x) = 2ax + b. 1.6.31. Prove that there is a function I : Ii -+ Q satisfying the following three conditions: (a) I(x + y) = I(x) + I(y) for %,y E lit, (b) I(x) = x for x E Q, (c) I is not continuous on III
32 Problems. 1: Limits and Continuity 1.7. Continuous Functions in Metric Spaces In this section X and Y will stand for metric spaces (X, d 1 ) and (Y, ), respectively. To shorten notation we say that X is a metric space instead of saying that (X,d t ) is a metric space. H not stated otherwise, JR and an are always assumed to be equipped with the Euclidean metric. 1.7.1. Let (X,d.) and (Y,) be metric spaces and let I : X 4 Y. Prove that the following conditions are equivalent. (a) The function J is continuous. (b) For each closed set FeY the set 1-1 (F) is closed in X. (c) For each open set G c Y the set I-I( G) is open in X. (d) For each subset A of X, I(A.) C I(A). (e) For each subset B ofY, 1- 1 (B) c l-l( B ). 1.7.2. Let (X,d}) and (Y,d2) be metric spaces and let I : X -. Y be continuous. Prove that the inverse image 1-1 (B) of a Borel set B in (Y, th) is a Borel set in (X, d 1 ). 1.7.3. Give an example of a continuous function I : X -. Y such that the image I(F) (resp. I(G» is not closed (resp. open) in Y for a closed F (resp. open G) in X. 1.7.4. Let (X,d.) and (Y,) be metric spaces and let I : X -+ Y be continuous. Prove that the image of each compact set F in X is compact in Y. 1.7.5. Let I be defined on the union of closed sets Fit F 2, .. . , F m- Prove that if the restriction of I to each Fit i = 1, 2, . . . ,m, is con- tinuous, then I is continuous on Fl U F2 U... U F m- Show by example that the statement does not hold in the case of infinitely many sets F i. 1.7.6. Let I be defined on the union of open sets G h t E T. Prove that if for each t e T the restriction I,G. is continuous, then I is continuous on U G t - tET 
1.7. Continuous Functions in Metric Spaces 33 1.7.7. Let (X,d}) and (Y,) be metric spaces. Prove that I: X -i> Y is continuous if and only if for each compact A in X the function /IA is continuous. 1.7.8. Assume that / is a continuous bijection of a compact metric space X onto a metric space Y. Prove that the inverse function /-1 is continuous on Y. Prove also that compactness cann ot be omitted from the hypotheses. 1.7.9. Let / be a continuous mapping of a compact metric space X into a metric space Y. Show that I is uniformly continuous on X. 1.7.10. Let (X,d) be a metric space and let A be anonempty subset of X. Prove that the function f : X  [0,00) defined by f(x) = dist(x,A) = inf{d(x,y): tI E A} is uniformly continuous on X. 1.7.11. Assume that / is a continuous mapping of a connected metric space X into a metric space Y. Show that I(X) is connected in Y. 1.7.12. Let /: A  Y, 0  A C X. For x E A define 0,(x,6) = diam(/(A n B(x,6»). The oscillation off at x is defined as o/(x) = Jim o/(x, 6). 6-.0+ Prove that f is continuous at Xo E A if and only if o/(xo) = 0 (compare with 1.4.19 and 1.4.20). 1.7.13. Let f : A -i> Y, 0  A c X and for:z: E A let o/(x) be the oscillation of f at x defined in the foregoing problem. Prove that for each E > 0 the set {x E A: o/(x) > E} is closed in X. 1.7.14. Show that the set of points of continuity of I : X -+ Y is a countable intersection of open sets, that is, a fi& in (X, d 1 ). Show also that the set of points of discontinuity of I is a countable union of closed sets, that is, an :F fT in (X, d 1 ). 
34 Problems. 1: Limits and Continuity 1.7.15. Give an example of a fun<.-tion f : 1R -+ JR. whose set of points of discontinuity is Q. 1.7.16. Prove that every :Fu subset of IR is the set of points of dis- continuity for some f : JR. -+ nt 1.7.17. Let A be an:FtT subset of a DlctriC space X. Iust there exist a function f : X -+ R whose set of points of discontinuity is A? 1.7.18. Let XA be the characteristic function of A C X. Show that {x EX: 0XA (x) > O} = oA, where o,(x) is the osciUation of I at x defined in 1.7.12. Conclude that XA is continuous on X if and only if A is both open and closed in X. 1.7.19. Assume tbat 91 and 92 are continuous mappings of a metric space (X,d l ) into a metric space (Y,d 2 ), and that a set A with a void interior is dense in X. Prove that if { 91(X) for x E A, lex) = 92 (x) for x EX \A, then o,(x) = d 2 (Ul(X),92(X», x E X, where o/(x) is the oscillation of / at x defined in 1.7.12. 1.7.20. We say that a real function I defined on a metric space X is in the first Baire class if I is a pointwise limit of a sequence of continuous functions on X. Prove that if f is in the first Baire class, then the set of points of discontinuity of I is a set of the first category; that is, it is the union of countably many nowhere dense sets. 1.7.21. Prove that if X is a complete met.ric space and f is in the first Baire class on X, then the set of points of continuity of f is dense in X. 1.7.22. Let f : (0,00) -+ IR be continuous and such that, for each positive x, the sequence {I ( : )} converges to zero. Does this imply tbat fun f(x) = O? (Compare with 1.1.33.) zo+ 
1.7. Continuous Functions in Metric Spaces 5 1.7.23. Let F denote a family of real functions continuous on a com- plete metric space X such that for every x E X tbere is M z such that I/(x)1 < 1\1;z for all I E F. Prove that there exist a positive constant flrJ and a Donempty open set G C X such that I/(x)1 ::;!vl for every I E:F and every x E G. 1.7.24. Let FI ::> F 2 ::> Fa ::> ... be a nested collection of nonempty closed subsets of a complete metric space X such that lim diam F n = n-foOO O. Prove that if f is continuous on X, then f ( f\ Fn) = D f(Fn). 1.7.25. Let (X, d 1 ) be a metric space and p a fixed point in X.. For u e X define tbe function fu by lu{x) = d1(u, x) - d1(P,x), x e x.. Prove that 'U t-+ I u is a distance preserving mapping, that is, an isometry of (X, d 1 ) into the space C(X, IR) of real functions continuous on X endowed with the metric d(j,g) = sup{lj(x) - g(x)1 : x EX}.. 1.7.26. Prove that a nletric space X  compact if and only if every continuous function f : X -+ IR is bounded. 1.7.27. Let (X,d.) be a metric space and for x E X define p(x) = dist(x, X \ {x}).. Prove that the following two conditions are equiva- lent. (a) Each continuous function f : X -+ R is uniformly continuous. (b) Every sequence {xn} of elements in X such that Urn p(xn) = 0 n-.oo contains a convergent subsequence. 1.728. Show that a nletric space X is compact if and only if every real function continuous on X is uniformly continuous and for every e > 0 the set {x EX: p(x) > E}, where p is defined in 1.7.27, is finite. 
36 Problems. 1: Limits and Continuity 1.7.29. Give an example of a noncompact metric space X such that every continuous f : X -+ IR is uniformly continuous on X.
Chapter 2 Differentiation 2.1. The Derivative of a Real Function 2.1.1. Find the derivatives (if they exist) of the following functions: (a) (b) (c) (d) (e) (f) fez) = zlxl, x E IR, f(x) = , x E Ii, fez) = [x] sin 2 (1rz), Z E Ii, fez) = (z - [x]) sin 2 (1rz), X E Ii, fez) = In lxi, x E R \ to}, 1 fez) = arccos, Ixl> 1. 2.1.2. Find the derivatives of the following functions: (a) I(x) = log 2, z > 0, X:F 1, (b) f(x) = lo cosx, x E (O,) \ {I}. 2.1.3. Study differentiability of the following functions: { arctan Z if Izl S I, (a) fez) = : sgnz + %2 1 if Ixl > 1, - 37 
38 Problems. 2: Differentation (b) { x2e-z2 if Ixl  I, J(x) =  if Ixl > I, (c) { arctan b if x:F 0, f(x) = lt 2 IZI 1 . f X =0. 2.1.4. Show that the function given by { x21cos I if I (x) = 0 if x ;e 0, x=o is not differentiable at Zn = 2nI ' nEZ, but is differentiable at zero, which is a limit point of {x nt n E Z}. 2.1.5. Determine the constants a, b, c and d so that I is differentiable on Ii: 4x if z  0, (a) fez) = ax 2 +bx+c if o < z < 1, 3-2x if x  I, ax+b if x S 0, (b) I(x) = c:x 2 + dx if o < x < 1, 1- 1 if x> 1, Z ax+b if x < 1, (c) f(x) = ") if 1 < x S 2, ax-+c dz2 + 1 if x> 2. z 2.1.6. Find the following sums: (a) n Lke kz , x E JR, Ic=O I:(-1)J: ( 2n ) kn, n  1, Ic=O k (b) n (c) L kcos(kx), x E r-t k=l 
2.1. The Derivative of a Real Function 39 2.1.7. Prove that if lal sin x + a2 sin 2x + . . . + an sin nxl < I sin xl for x E IR, then lal + 2a2 + ... + nanl < 1. 2.1.8. Assume that / and 9 are differentiable at Q. Find (a) lim xlCa) - al(x) , z-Ioa X - a (b) I " I(x)g(a) - /(a)g(x) 1m . z a x-a 2.1.9. Suppose that I(a) > 0 and that I is differentiable at a. De- termine the limits I (a) Urn ( f (a + ) ) ;; , n-.oo I(a) b) . ( /(X» ) J a..!. l aa hm /() , a > O. z-i>a a 2.1.10. Let I be differentiable at a. Find the following limits: (a) lim an I(x) - x n I(a) , n E N, z-to X - a " l(x)e Z - I(a) , (b) 11m Ie ) I( ) ' a = 0, I (0) 0, z-ta X COSX - a (c) n co n(i(a+ ;u+/(a+ +...+J(a+ -k/(a)), kEN, (d) n (/(a+ ) +/(a+ ) +".+/(a+ ) -n/(a»). 2.1.11. For a > 0 and Tn, keN, calculate lim ( en + l)m + (n + 2)m +... + en + k)m _ 1m ) J n-too n m - 1 (a) (b) ( J ) III 2 ) n ( k ) " lirn a + n \ a + ii . ." a + n n-too ank J (c) lim (( 1 + ) ( 1 + 2a ) ... ( 1 + )) . n-looo n 2 n 2 n 2
40 Problems. 2: DifFerentation 2.1.12. Assume that 1(0) = 0 and that I is differentiable at zero. For a positive integer k find ; (/(x) + 1 (j) + 1 ( ; ) + . .. + 1 ( : )) · 2.1.13. Let I be differentiable at a and let {xn} and {zn} be two sequences converging to a and such that X n f; a, Zn #: a, X n f; Zn for n e N. Give an example of I for which lim f{x n ) - I(zn) n....oo X n - Zn (a) is equal to I'(a), (b) does not exist or exists but is different from /' (a). 2.1.14. Let I be differentiable at a and let {xn} and {zn} be two sequences converging to a and such that X n < a < Zn for n E N. Prove that lim fex n } -/{zn) = /'(a). n....oo X n - Zn 2.1.15. (a) Show that / defined on (0,2) by setting { z2 for rational x e (0,2), I(z) = 2x -1 for irrational x e (0,2) is differentiable only at x = 1 and tbat I' (1) f; O. Is the inverse function differentiable at 1 = y = /(1)? (b) Let A = {y e (0,3) : y e Q, Jji  Q}, 1 B={x:x=2(y+4), yeA}. Define I by setting I(z) = z2 2z-1 2z-4 for rational z E (0,2), for irrational z E (0,2), for x E B. 
2.1. The Derivative of a Real Function 41 Show that the interval (0, 3) is contained in the range of I, and that the inverse function is not differentiable at 1. 2.1.16. Consider the function I defined on IR as follows: { 0 if x is irrational or x = 0, lex) = . . Oq If % = , p e Z, q E N, and p,q are co-pnme, where the sequence {Oq} is such that lirn n"an = 0 for some integer n-too k > 2. Prove that I is differentiable at each irrational which is alga. braic of degree at most k (that is, at each algebraic surd of degree at most k). 2.1.17. Let P be a polynomial of degree n with n different real roots Xl, %2, . . . ,X n and let Q be a polynomial of degree at most n -1. Show that Q(x) Q(Xk) P(x ) - L..J P'(x,,){x - XIc ) k=l n for % E R \ {XhX2,.. .,x n } and find the sum E P'( ) ' n > 2. k=l · 2.1.18. Using the result of the foregoing problem, establish the fol- lowing equalities: (a) n ( n ) (-l)tc n! f; k x+k = x(x+l)(x+2)...(x+n) for x e R \ {-n,-(n-l),...,-I,O}, (b) n ( n ) (_l)k n!2 R t; k x+2k = x(x+2)(x+4)... (x + 2n) for x e IR \ {-2n, -2(n - 1),..., -2, OJ. 2.1.19. Let I be differentiable on III Describe the points of differen- tiability of If I.
42 Problems. 2: Differentation 2.1.20. Assume that II) 12,. .. , In are defined in some neighborhood of :c, are different from zero at x, and are differentiable at x. Prove that (1]1 f.)' (x) = t 1Hz) . ii II: k=l Ik(z) k=l 2.1.21. Assume that functions 11,/2,..., In; 9t,!h,.. - ,On are de- fined in some neighborhood of z, are different from zero at x, and are differentiable .at z. Prove that (IT l! ) ' (x) = IT II< (x) t ( lk(Z) _ !4(X» ) . V=l 01: l 9k k= 1 Ik(z) Ok(Z) 2.1.22. Study the differentiability of J and 1/1 when { X if Z e Q, (a) I(z) = sin x if 3: E R \ Q. (b) (z) = { Z - if :c e Qn [ 21r 1 ' 21r 2 ) k 2, I sin (x - M if x E (Ii \ Q) n L . .. :/-2 }' k > 2. 2.1.23. Show that if the one.sided derivatives I (xo) and I+.(:co) exist, then / is continuous at Xo- 2.1.24. Prove that if 1 : (a, b) -to IIi assumes its largest value at c e (at b), that is, fCe) = max{J(x): x e (a, b)}, and there exist one-sided derivatives I (c) and f (e), then I!..(c) > 0 and I (c) < o. Establish the analogous necessary condition that 1 assumes its smallest value. 2.1.25. Prove that if J e C([a,b», I(a) = I(b), and I exists 00 (a, b), then inf{/ (x): :c E (a, b)} < 0 < sup{/ (x): x e (a, b)}.
2.1. The Derivative of a Real Function 43 2.1.26. Prove that if j E C«a, b]) and 1'- exists on (a, b), then ine {f (x): x E (a, b)} f(b = (a) sup{J (x): x E (a, b)}. 2.1.21. Prove that if I exists and is continuous on (a, b), then f is differentiable on (a, b) and I'(x) = 1'- (x) Cor x E (a, b). 2.1.28. Does there exist a function f : (1, 2) lIt such that f'- (x) = x and I (z) = 2x for z E (1,2)? 2.1.29. Let f be differentiable on [a, b) and such that (i) I(a) = f(b) = 0, (ii) I'(a) = f (a) > 0, I'(b) = I (b) > o. Prove that there is c e (a, b) such that f(c) = 0 and f'(e) o. 2.1.30. Show that f(x) = arctan x satisfies the equation (1 + x 2 )j C n) (x) + 2(n -l)xfCn-l)(x) + (n - 2)(n - 1)j(n-2)(x) = 0 for x E IR and n > 2. Show also that for m 0, 1(2m)(0) = 0, j(2m+l)(o) = (-1)m(2m)!. 2.1.31. Show that (a) (ezsinx)(n) = 2 i e z sin (x+n ), x e IR, n > 1, (b) (x n Inx)'nl = n!(lnx+ 1 + +... + ), x> 0, n > 1, (c) C:X ) (n l _ (_I}nn!x- n - 1 (InX -1- -... -;), x > 0, n > 1, (d) (xn-1e! )'n l = (_I)n x:: 1 , X#: 0, n > 1.
44 Problems. 2: Differentation 2.1.32. Prove the following identities: (a) i () sin (Z+k ; ) = 2t sin (z+n  ), z E B, n  1, (b)  (-1 )I:+l! ( n ) = 1 + 1 + . .. +.!. n > 1  k k 2 n' -. "..-1 2.1.33. Let I(z) = vz 2 - 1 for x > 1. Show that f(n)(x) > 0 if n is odd, and f(n)(x) < 0 if n is positive and even. 2.1.34. For 12n(x) = m(! + z2 n ), n E N, show that fJn)( -1) = O. 2.1.35. For a polynomial P of degree R, prove that t P(k)(O), zHI = t(-I)k P(k)(z z"+1. 1:=0 (k + 1). 1: =0 (k + 1). 2.1.36. Let,\1, '\2,...,'\n be such that ,\t + ,\ + ... +,\ > 0 for any kEN. Then J given by 1 I(x) = (1- '\lx)(I- '\2X)... (1 - '\nX) is well defined in some neighborhood of zero. Show that for keN, f(le) (0) > 0 2.1.37. Let f be n times differentiable on (0, (0). Prove that for positive x, znJ (n) G) = (_I)n (zn-I f G )) (n) . 2.1.38. Let I, J be open intervals and let I : J  IR, 9 : I  J be infinitely differentiable on J and I, respectively. Prove the Faa di Bruno formula for the nth derivative of h = fog: (n) _  n! (JI) { g(1)(t» kl ( 9(2)(t) ) k2 ... ( gCR)(t» k a h (t)-L..J kl!k2!...k n ! / (g(t)} 1! 2! n! ' where k = kl + k2 + · · . + k n and summation is over all kl, k 2 , . · . t k n BUch that kl + 2 + . · · + nk n = n. 
2.2. Mean Value Theorems 45 2.1.39. Show that the functions (a) { I e-;2' f(x) = 0 if x 0, if x = 0, (b) 9(X) = { e o - n if X> 0, x 0, (c) { e- + if x e (a,b), hex) = 0 if d ( b ,\ X 7= a, h are in Coo(IR). 2.1.40. Let I be differentiable on (a,b) and such that for x e (a,b) we have f'(x) = g(f(x», where 9 E COO(Ii). Prove that f e COO (a, b). 2.1.41. Assume that f is twice differentiable on (a,b) and that for some real a,{J,'Y such that a 2 + {32 > 0, ol"(x) + {3f'(x) + "II (x) = 0, x e (a, b). Prove that J e COO «a, b». 2.2. Mean Value Theorems 2.2.1. Prove that if J is continuous on a closed interval [a, b], differ- entiable on the open interval (a,6), and if /(a) = /(6) = 0, then for a real 0 there is an x e ( a, b) such that a/(x) + f'(x) = 0. 2.2.2. Let f and 9 be functions continuous on [a, b), differentiable on the open interval ( at b), and let I( a) = I( b) = O. Show that there is a point x E (a, b) such that g'(x)/(x) + I'(x} = o.
46 Problems. 2: Differentation 2.2.3. Assume that f is continuous on (a, b], a > 0, and differentiable on the open interval (a, b). Show that if I(a) f(b) a =6' then there is Xo E (a, b) such that xo/'(xo) = f(xo). 2.2.4. Suppose I is continuous on [a, b] and differentiable on the open interval (a, b). Prove that if /2 (b) - /2 (a) = 1J2 - a 2 , then the equation 1'(x)/(x) = x has at least one root in (a,b). 2.2.5. Assume that f and 9 are continuous and never vanishing on [a, b], and differentiable on (a, b). Prove that if I(a)g(b) = I(b)g(a), then there is Xo E ( a, b) such that I'(xo) o'(xo) f(xo) - g(xo). 2.2.6. Assume that ao,a.,... ,an are real numbers such that ao al an-l +-+...+ +an=O. n+l n 2 Prove that the polynomial P(x} = aoxn + 01X n - 1 +... + an has at least one root in (0, 1). 2.2.7. For real constants ao, aI, . . . ,an such that ao 201 2 2 a2 2n- 1 an_l 2 n a n - + - + + .. . + + = 0, 1 2 3 n n+I show that the function f(x) = an Inn X + . .. + 02 In 2 X + alln X + ao has at least one root in (1, e 2 ). 2.2.8. Prove that if all roots of a polynomial P of degree n > 2 are real, then all roots of pi are also real. 2.2.9. Let I be continuously differentiable on [a, b) and twice differ- entiable on (a, b), and suppose that f(el,) = f'(a) = f(b) = O. Prove that there is XI E (a, b) such that /"(XI) = O.
2.2. Mean Value Theorems 47 2.2.10. Let 1 be continuously differentiable on [a, b] and twice differ- entiable on (a, b), and suppose that I(a) = I(b) and f'(a) = f'(6) = 0. Show that there are Xl ,%2 E ( a, b), Xl :F X2, such that I"(xl) = 1"(x2). 2.2.11. Show that each of the equations (a) X 13 + 7x 3 - 5 = 0, (b) 3 Z + 4% = 5 z has exactly one real root. 2.2.12. For nonzero a., a2,. . . ,an and for aa, a2t. . . , an such that Qi -:f: Qj for i :F j, prove that the equation alx Q1 + U2XQ2 + ... + unx Qn = 0, x E (0,00), has at most n - 1 roots in (0,00). 2.2.13. Prove that under the assumptions of the foregoing problem the equation ale0 1Z + a2e02Z +... + ane QnZ = 0 has at most n - 1 real roots. 2.2.14. For functions I, 9 and h continuous on [a,b] and differen- tiable on ( a, b), define f(x) g(x) hex) F(x) = det I(a} g(a) h(a) , x E [a, b]. I(b) 9(b) h(b) Show that there is Xo E (a, b) such that F'(xo) = 0. Use this to derive the mean value theorem and the generalized mean value theorem. 2.2.15. Let 1 be continuous on [0,2] and twice differentiable on (0,2). Show that if 1(0) = 0,/(1) = 1 and 1(2) = 2, then there is Xo E (0,2) such that I"(xo) = O. 
48 Problems. 2: Differentation 2.2.16. Suppose that / is continuous on [a, h] and differentiable on (a,h). Prove that if / is not a linear function, then there are Xl and X2 in ( a, b) such tbat 1'(:1:1) < I(b = (a) < /'(:1:2). 2.2.17. Let I be continuous on [0,1] and differentiable on (0,1). Sup- pose that /(0) = /(1) = 0 and that there is Xo E (0,1) such that f{xo} = 1. Prove that II'(C)I > 2 for some c e (0,1). 2.2.18. Let / be continuous on [at b], a > 0, and differentiable on (a, b). Show that there is Xl E (a,b) such that b/(a) - a/(b) /( ) I ' ( ) b = Xl - Xl Xl. -a 2.2.19. Show that the functions X  1n{1 + x}, X  In{! + r) and z H arctan z are uniformly continuous on [0,00). 2.2.20. Assume that / is twice differentiable on (a, b), and that there is M > 0 such that I 1"(x) I  M for all x E (a,b). Prove that / is uniformly continuous on (a, b). 2.2.21. Suppose that / : [a, b] -+ R, b - a  4, is differentiable on the open interval (a,b). Prove that there is Xo e (a, b) such that /'(xo) < 1 + /2(xO). 2.2.22. Prove that if / is differentiable on (a, b), and if (i) (ll) lim /(x) = +00, lim I(z) = -00, Z-foc+ z-+b- /'(x) + 1 2 (x) + 1  0 for X E (a, b), then b - a  1r. 2.2.23. Let / be continuous on [a, b] and differentiable on (a, b). Show that if lim /'(x) = A, then /(b) = A. z-tb- 
2.2. Mean Value Theorems 49 2.2.24. Suppose / is differentiable on (0, (0) and /'(x) = O(x) as x -+ 00. Prove that I(x) = 0(x 2 ) as x -+ 00. 2.2.25. Let /1, /2, . . . , / R and 91,92,..., 9n be continuous on [a, b] and differentiable on (a, b). Suppose, further, that gl.;(a) :F gk(b) for k = 1,2,..., n. Prove that there is C E (a, b) for which tf (c) == tgHc /,,(b) - f,,(a) . i=l k=1 91c(b) - gt(a) 2.2.26. Assume that J is differentiable on an open interval I and that (a, b) C I. We say that f is uniformly differentiable on (a, b], if for any e"> 0 there is 6 > 0 such that f(x + h - f(x) _ /,(x) I < e for all x E [a, b] and Ihl < 6, x + h e I. Prove that / is uniformly differentiable on [at b] if and only if I' is continuous on [a, b). 2.2.21. Let f be continuous on [a, b], 9 differentiable on [a, b], and g(a) = o. Prove that if there is a :F 0 such that Ig(x)f(x) + "\g'(x)1 < Ig(x)1 for x e [a,b]t then g(x) = 0 on [a,bJ. 2.2.28. Let I be differentiable on (0,00). Show that if 1im fez) = z...+oo Z 0, then lim 1/'(z)1 = o. %-++00 2.2.29. Show that the only functions /: R -+ Ii satisfying the equa- tion f(x + h h ) -/(x) _ _I ' (x + 2 1 h) for x, hell, h:; 0, are polynomials of second degree.
50 Problems. 2: Differentation 2.2.30. For positive p and q such that p + q = 1, find all functions I : B  R satisfying the equation I(x) - I(y) = I'(PX + qy) for x, y e JR, X:F y. x-v 2.2.31. Prove that if I is differentiable on an interval I, then I' enjoys tbe intermediate value property on I. 2.2.32. Let I be differentiable on (0,00). Show that (a) if Urn {I (x) + I'(x» = 0, then lim I(x) = 0, Z+ Z+ (b) if lim (I (x) + 2vxl'(x» = 0, then lim f(x) = O. Z+ Z+ 2.2.33. Prove that if I e (/l([a, b) has at least three distinct zeros in [a, b), then the equation I(x) + f"(x) = 2/'(x) has at least one root in [a, b]. 2.2.34. Prove that if a polynomial P has n distinct zeros greater than 1, then the polynomial Q(x) = (x 2 + l)P(x)P'(x) + x (p(X»2 + (p'(x»2) bas at least 2n - 1 distinct real zeros. 2.2.35. Let a polynomial P(x) = amx m +am_IX m - 1 +.. .+alx+ao with am > 0 have m distinct real zeros. Show that the polynomial Q(x) = (p(x»2 - P'(x) has (1) exactly m + 1 distinct real zeros if Tn is odd, (2) exactly m distinct real zeros if m is even. 2.2.36. Assume that all zeros of a polynomial P of degree n > 3 are real and write P(x) = (x - al)(x - 62)... (x - an), where aj S ai+1 t i = 1,..., n - 1, and P'(x) = n(x - Cl)(X - C2)... (x - Cn-I), 
2.2. Mean Value Theorems 51 where ai < Ci < 0i+lt i = 1,...,71 -1. Show that if Q(x) = (x - al)(z - a2)... (x - an-I), Q'(x) = (n -1)(x - dd(x -)... (x - dn-2), then d  c; for i = I, . . . , n - 2. Moreover t show that if R(x) = (x - a2){x - a3) . · · (x - 0'1)' R'(x) = (n -1)(x - el)(x - e2)... (z - en-2), then e. :S (:;+1 for i = 1,2,..., n - 2. 2.2.37. Under the assumptions of the foregoing problem, show that (1) if S(x) = (x-at -e)(z -a2)... (x -an), where e  0 is such that at + E =5 an-It and if 8'(x) = n(x - 11)(x - /2)... (x - In-I), then I n-1 > Cn-It (2) if T(x) = (x - at)(x - a2) · .. (x - an + c:), where E > 0 is such that an - e > a2, and if T'(z) = n(x - 9.)(X - 92) ... (x - 9n-I), then 91 S Cl. 2.2.38. Show that under the assumptions of 2.2.36, ai+l - a; a;+1 - ai . ai + . 1 < Ci < ai+l - . 1 ' 1. = 1,2, · · · , n - 1. n-,+ - - &+ 2.2.39. Prove that if f is differentiable on [0,1], and if (i) /(0) = 0, (il) there is K > 0 such that If'(x)1  Klf(x)1 for x e (0,1], then I(x) = o. 2.2.40. Let f be in Coo on tbe interval (-1,1), and let J C (-1,1) be an interval whose length is A. Suppose that J is decomposed into three consecutive intervals J 1 ,J2 and J 3 whose lengths are At'.\2 and -"3, respectively. (So we have J 1 UJ 2 U J 3 = J and Al + A2 + A3 = A.) Prove that if mj:(J) = inf{lf(k)(x)l: x e J}, k e N J then 1 mk(J)  "\ (mk-l(J 1 ) + mk-l(J 3 ». .1\2 
52 Problems. 2: DifFerentation 2.2.41. Prove that under the assumptions of the foregoing problem, if I/(x)1  1 for x e (-1,1), then 2 .(.:1) kk mk(J)  ),.k ' keN. 2.2.42. Assume that a polynomial P(x) = tlnX n + lIn_lX n - 1 +... + alZ + ao has n distinct real zeros. Prove that if there is p, 1 :S p S n -1, such that a" = 0 and ai f; 0 for all i :F p, then ap-lap+l < o. 2.3. Taylor's Formula and L'Hospital's Rule 2.3.1. Suppose that I : [a, b]  R is n - 1 times differentiable on [a, b]. If I(n) (xo) exists, then for every x e [a, b), lex) = /(xo) + I'o) (x - xo) + I"o) (x - xO)2 f(n) (xo) + . .. + I (x - :ro)n + o«x - xo)n). n. (This formula is called Taylor's formula with the Peano lorm lor the remainder. ) 2.3.2. Suppose that I : [a, b) -.. 11 is n times continuously differen- tiable on [a,b], and I(n+l) exists in the open interval (a, b). Prove that for any X,ZO e [a,b] and any p > 0 there exists S E (0,1) such that f'(x ) I"(x ) I(x) = I(xo) + 11 0 (x - xo) + 21 0 (x - xO)2 f(n) (xo) + .. · + n! (x - xo)n + rn(x), where rn(x) = I(n+l) (xo + S(x - zo» (1 _ 8)n+l-p(x _ xo)n+l nIp is the Schlomilch-Roche form for the remainder. 
2.3. Taylor's Formula and L'Hospital's Rule 53 2.3.3. Using the above result, derive the following forms for the re- mainder: (a) ( ) I(n+l)(xo + 8(x - xo» ( ) n+l Tn X = (n + I)! X - Xo (the Lagrange form), (b) T ( x) = f(n+l)(xo + 8(x - xo» (1- 8 ) n(x _ xO)R+1 n n! (the Cauchy form). 2.3.4. Let 1 : [a, b] R be n + 1 times differentiable on [a, b]. For x, Xo e [a, b] prove the following Taylor Ionn.ula with integral remain- d I(x) = I(xo) + I' ) (x - xo) + J" :o) (x - XO)2 I (n) ( % ) 1 1 % + ... + I 0 (x - xo)n + .. j(n+l)(t)(x - t)ndt. n. n. %0 2.3.5. Let 1 : [a, b) Ii be n + 1 times differentiable on [a, b]. For x, Xo e [a, b) prove the following Taylor formula: lex) = I(xo) + J' ) (x - xo) + J" :o) (x - XO)2 f(n) (xo) +... + r (x - xo)n + Rn+l(X), n. where 1 z 1 ta+l 1 tft 1 t2 Rn+l(X) = ... f(n+l)(ti)dtl .. .dt n dt n + 1 . zg zo zo zo 2.3.6. Show that the approximation formula 1 1 2 v l+x 1+ -x--x 2 8 gives "; 1 + x with the error not greater than Ix1 3 , if Ixl < .
54 Problems. 2: Differentation 2.3.7. For x > -1, x i: 0, show that (a) (1 + x)O > 1 + ax if a > 1 or a < 0, (b) (1 + X)CI < 1 + ax if 0 < a < 1. 2.3.8. Suppose that j,g E C2([0, 1]) and o'(x)  ° for x E (0,1), and J'(O)g"(O)  1"(O)g'(O). For x e (0,1), let 8(x) be one of the numbers for which the assertion of the generalized mean value theorem holds, that is, Compute lex) - 1(0) I' (8(x» g(x) - g(O) - g'(8(x». Urn 8(x) . 01:-.0+ X 2.3.9. Let J : TIt -+- IIi be n + 1 times differentiable on III Prove that for every x E Ii there is 8 E (0,1) such that (a) x 2 x R I(x) = /(0) + x/'(x) - - J"(x) +... + (_I)"+I_/(n)(x) 2 n! n+l + (_1)"+2 Z /(R+l) ({Jx) (n + I)! ' (b) ( X ) z2 x2n /(n)(x) = x - I X +... + -1 n I 1 + x J() 1 + x / ( ) () (1 + x)" n! 2n+2 /(n+l) ( %+9%2 ) ) "+1 X 1+z-J, + (-1 (1 +x)n+l (n + I)! ,x.,.. -1. 2.3.10. Let I : Ii  R be 2n + 1 times differentiable on R. Prove that for every x e Ii there is {J E (0,1) such that lex) = 1(0) + TII' G ) () + :, /(3) (; ) G f . . . 2 (2n-l) (  ) (  ) 2n-l + + (2n - I)! I 2 2 2 (2n+l) ( X ) 2n+l + (2n + 1)! 1 (8x) 2' · 
2.3. Taylor's Formula and L'Hospital's Rule 55 2.3.11. Using the result in the foregoing problem, prove that n 1 ( ) 21:+1 In(l + x) > 2 E 2k 1 2 x k=O + + x for n = 0, 1, . .. and x > 0 2.3.12. Show that if J" (x) exists, then r J(x + h) - 2f(x) + f(x - h) - f " ( ) h h 2 - x , r J(x + 2h) - 2/(x + h) + I(x) - f " ( ) h h 2 - X . (a) (b) 2.3.13. Show that if J"'(x) exists, then . - J(x + 3h) - 3J(x + 2h) + 3J(x + h) - f(x) - f lll ( )  h 3 - x. h-tO 2.3.14. For x > 0, establish the following inequalities: (a) n Ie z x e > L, k! ' k=O X 2 X 3 3: 4 X 2 X 3 x- - + - - - < In(l+x) <X- -+- 2 3 4 2 3' 1 1 2 V 1 1? 13 1 + -x - -x < 1 + x < 1 + -z - -x- + -x 2 8 2 8 16. (b) (c) 2.3.15. Prove that if f(n+l)(x) exists and is different from zero, and 8(h) is a number defined by the Taylor formula h n - J h n f(x+h) = J(x)+hJ'(xj+...+ ( _l), J(n-l)(x)+, f(n) (x+8(h)h), n. n. then Jim 9( h) = 1 l ' h-+O n + 
56 Problems. 2: Differentation 2.3.16. Suppose that I is differentiable on [0,1] and that 1(0) = 1(1) = O. Suppose, further, that I" exists on (0,1) and is bounded (say 1/"(x)1 5 A for z E (0,1». Prove that A 1/'(x)1  '2 for z e [0,1]. 2.3.17. Suppose I : [-c,c] .... It is twice differentiable on [-c, c), and set Mk = sup{l/{k){z)l: Z E [-c,c]} for k = 0,1,2. Prove that (a) 1/'(x)1 < Mo + (x 2 + cfl) 2 2 for x E [-c,c], - c c (b) Ml2 ';MoM2 for c JZ: . 2.3.18. Let I be twice differentiable on (a, (0), a e IR, and let Mle = sup{l/(i) (z)1 : Z E (a,oo)} < 00, k = 0,1,2. Prove that M 1  2 vMoM2- Give an example of a function for which the equality M 1 = 2 v kfoM2 holds. 2.3.19. Let I be twice differentiable on IR, and let Mle = sup{l/(k) (x) 1 : x E R} < 00, k = 0,1,2. Prove that Ml  .../ 2 MoM2 . 2.3.20. Let I be p times differentiable on Ii, and let Mk = sup{l/(k){x)l: x e Il} < 00, k = 0,1,... ,p, p > 2. Prove that .(p-.) 1-.1 S M" 5 2 :I JlJ o P M; for k = 1,2, . . . ,p - 1. 2.3.21. Assume that I" exists and is bounded on (0,00). Prove that if lim I(x) = 0, then Jim I'(x) = O. z-tooo -+oo 
2.3. Taylor's Formula and L'Hospital's Rule 57 2.3.22. Assume that I is twice continuously differentiable on (0,00), Urn xf(x) = 0 and lim xfl/(x) = o. %....+00 Z +OO Prove that lim x I' (x) = o. %-t+oo 2.3.23. Assume I is twice continuously differentiable on (0, 1) and such that (i) Jim f(x) = 0, z-H- (ll) there is ]vI > 0 such that (1- x)2If"(X)1 < M for x E (0,1). Prove that lim (1- x)f'(x) = O. z....1 - 2.3.24. Let I be differentiable on [atb] and let f'(a) = f'(b) = O. Prove that if I" exists in (a, b), then there is c e (a, b) such that II"(c)1 (b a)2 lf(b) - f(a)l. 2.3.25. Let f : [-1,1] -+ Ii be three times differentiable and let I( -1) = 1(0) = 0, f(l) = 1 and f'(O) = O. Show that there exists c E (-1,1) such that I"'(c) > 3. 2.3.26. Let I be n times continuously differentiable on [a, b) and let I(x) -/(t) Q(t) = , x, t E [a, b], t x. x-t Prove the following version of Taylor's formula: I' (xo) f(n) (xo) f(x) = I(xo) + I! (x - xo) + . . . + n! (x - xo)n + rn(x), where rn(x) = q(" zo) (x - xo)n+l. 2.3.27. Assume that I : (-1, 1) Ii is differentiable at zero. For -1 < X n < Yn < 1, n E N, such that lirn Xn = lim Yn = 0, form n-+oo n....oo the quotient Dn = f(Yn) - f(xn) . Yn-Xn Prove that (a) if X n < 0 < Yn, then Urn Dn = /'(0), n....oo
58 Problems. 2: Differentation (b) if 0 < X n < Yn and the sequence { 1/.. Zn } is bounded, then lim Dn = f'(O), n-t>oo (c) if I' exists on (-1, 1) and is continuous at 0, then lim Dn = n-+oo f'(O). (Compare with 2.1.13 and 2.1.14.) 2.3.28. For meN, define the polynomial P by setting P(x) = 'f ( m; 1) (-1)"(x - k)m. x e IR. k=0 Show that P(x) == O. 2.3.29. Suppose that f(n+2) is continuous on [0, x]. Prove that there is 9 e (0, 1) such that /' ( 0 ) /(n-l) ( 0) J(n) ( 2- ) I(x) = 1(0) + z+ ... + xn-l + rJ+1 x n I! (n - I)! n! n+2 n f (n+2) ( 8 ) x + 2(n+l) x (n+2)r 2.3.30. Suppose that f(n+l') exists in [a, b] and is continuous at an Xo E [a, b]. Prove that if j(n+;) (xo) = 0. for j = 1,2,... ,p - 1, and j(n+p)(xo) :F 0, and /'(xo) /(n-l)(xo) n-l I(x) = /(xo) + I! (z - zo) +... + (n -I)! (x - xo) f(n)(xo + 9(x) (x - :to» ( ) n + I X-Xo , n. then 1 lim 9(x) = ( n + P ) -it . Z-+ZO n 2.3.31. Suppose that I is twice continuously differentiable on (-1,1) and that 1(0) = o. Find [*J lim  I(kx). :1:....0+ L..J k=l 
2.3. Taylor's Formula and L'Hospital's Rule 59 2.3.32. Let f be infinitely differentiable on (a, b). Prove that if / vanishes at infinitely many points in tbe closed interv-c1l [e, dJ C (a, b), and sup{lf(n)(x)1 : x E (a,b)} = O(n!) as -n -+ 00, then / vanishes on an open subinterval of (a, b). 2.3.33. Suppose (i) f is infinitely differentiable on IR, (ii) there is L > 0 such that I/(n) (x) I  L for all x E Ii and all -n E 1'1, (ill) / (  ) = 0 for 1t e N. Prove that f(x) == 0 on III 2.3.34. Use I'Hospital's rule to evaluate the following limits: arctan  (a) lim z-+1 , zl x-I (b) lim x ( ( 1 + ! ) Z - e ) , %+oo x I Urn ( Sin x ) z , %o+ x (c) lim(6 - x), z-+5 (e) lim ( SinX ) "*. z-+o+ X (d) 2.3.35. Prove tbat if / is twice continuously differentiable on R such that 1(0) = 1, /'(0) = 0, and 1"(0) = -1, then for a E IR, %]!l!oo(J(  ) r = e- U; . 2.3.36. For a > 0, at:: 1, evaluate ( aZ-1 ) ! Urn . +oo x(a -1) 
60 Problems. 2: DifFerentation 2.3.37. Can }'Hospital's rule be applied to evaluate the following limits? (a) (b) (c) (d) I _ x - sin x 1Dl 2 . t z-+oo X + S1D z lim 2x + sin2x + 1 %-+00 (2x + sin2x)(sinx + 3)2' lim ( 2Sin VZ + vzs in ! ) Z , z-+o+ X ( ) e-:r lim 1 + xe-* sin 14 . z-+o X 2.3.38. Is the function given by f(2:) = { r2 - 2'1 differentiable at zero? if z  0, if z=O 2.3.39. Suppose J is n times continuously differentiable on III For a E 1R, establish the equality f(n)(a) =  h  (-l)n-k()/(a+kh»). 2.3.40. Prove the following version of I'Hospital's rule. Suppose f, 9 : (a, b)  IR, -00  a < b S +00, are differentiable on (a, b). Suppose, further, that (i) g'(x):f:. 0 for x E (a, b), (ll) lim g(z) = +00 (-00), z-to+ (ill) lim g (: = L, -00 < L  +00. z-+o+ Then 1im f{x) = L. z-ta+ g(x) 
2.4. Convex Functions 61 2.3.41. Use the above version of I'Hospital's rule to prove the fol- lowing generalizations of the results given in 2.2.32. Let f be differ- entiable on (0, (0), and let a > o. (a) If lim (a/(x) + f'(x» = L, then lim f(x) = . Z + Z + (b) If lim (af(x) + 2v'Xf'(x» = L, then lim f(x) = . Z + Z + Are the above statements true for negative 01 2.3.42. Assume that / is three times differentiable on (0,00) and such that f(x) > 0, f'(x) > 0, f"(x) > 0 for z > o. Prove that if . f'(x)f"'(x) lim )2 = C, c ¥: 1, Z (f"(X) then fun f(x)f"(x) = 1 . z oo (f'(x»2 2 - c 2.3.43. Assume that I is in Coo on (-1,1), and that f(O) = O. Prove that if 9 is defined on (-1,1) \ to} by g(x) = I z) , then there exists an extension of 9 which is in Coo on (-1,1). 2.4. Convex Functions A function f is said to be convex on an interval I c R if (1) f(Ax I + (1 - '\)X2) S '\f(xl) + (1 - >')/(x2) whenever %.,Z2 e I and e (0,1). A CODVex function I is said to be strictly convex on I if strict inequality holds in (1) for Xl ¥: X2. f is COncave on I if -I is COnvex. 2.4.1. Prove that a function I differentiable on an open interval I is convex if and only if I' is increasing on I. 2.4.2. Prove that a function I twice differentiable on an open interval I is convex if and only if f" (x) 0 for x E I.
62 Problems. 2: Differentation 2.4.3. Prove that if a function / is convex on an interval I, then the following Jensen inequality I(AI X l + A2 X 2 +...... + AnXn) S At/(Xt) + A2/(x2) +..... + An/(xn) holds for any points Xl,..., xn in I and any nonnegative numbers h. · · 'n such that At + A2 + .. . + n = 1. 2.4.4. For x, y > 0 and p, q > 0 such that  +  = I, establish the inequality x" yq X y < - + -. - P q 2.4.5. Prove that 1 n n ;  Xk  !! Xk for Xt,X2,... ,x n > O. 2.4.6. Show that if a:F b, then e b - eO eO + e b < b-a 2 2.4.7. For positive x and y, establish the inequality x+y x In x + y In y  (x + y) In 2 · 2.4.8. For Q > 1 and for positive XbX2,.... ,Xn, prove that ( ) 0' 1 n 1 n n LXk < ;; LX" k=1 k=l 
2.4. Convex Functions 63 2.4.9. Let Xl t . . . , X n E (0, 1) and let PI, . . . , Pn be positive and such n that E Pie = 1. Prove that 1:=1 (a) 1 + ( t PkXk ) -'  Ii ( 1 + XI: ) ". , 1:=1 1:=1 Xle n 1 + E PlcXk n ( 1 ) PI, Ie: I < II + XI: . n - 1 - XIc 1 - E PI:XIe 1:=1 1e=1 (b) n 2.4.10. Let x =  E XIc with Xb.",Xn E (0 1 11'). Show that 1:=1 (b) n II sin XIc < (sin x)n, k=1 IT sin X" < ( Sinx ) n. k=! Xle X (a) 2.4.11. Prove that if a > 1, and X.,..., X n E (0, 1) are such that Xl + . . . + X n = 1, then  ( XI: +  ) O > (11 2 + 1)0 . L...J x - 1&0-1 #t=1 Ie 2.4.12. For n > 2, verify the following claim: n 21: _ 1 ( 2 1 ) n < 2- - + II 2 k - 1 - n n.2 n - 1 . k=2 
64 Problems. 2: Differentation 2.4.13. Establish the following inequalities: (a) n 2 1 1 < - + · · · + -, Xl, . . . ,X n > 0, Xl + . . · + Xn - XI Zn (b) 1 01 a < X l ... X " < a l X l + .. . + a n X n 21.+...+9& - n - ZI Z. n for Qk, Xk > 0, k = 1,2,..., n, such that E ak = I, k=1 (c) xr 1 .. . x,. + yf1 .. .y" S (Xl + VI )0'1 . .. (Xn + Yn)O,. n Cor XIc, Vie  0, Qk > 0, k = 1,2, .. . ,n, such that E ale = 1, i=1 (d) ( ) 0'1 m n n m n zfj  n  Zij for XiJ  0, £ti > 0, i = 1,2,...,n, j = 1,2,...,m, such that n Laic = 1. k=1 2.4.14. Show that if / : III -.. IR is convex and bounded above, then J is constant on Jll 2.4.15. Must a convex and bounded function on (a, 00) or on (-00, a) be constant? 2.4.16. Suppose that I : (a, b) -.. IR is convex on (a, b) (the cases a = -co or b = co are admitted). Prove that either / is monotonic on (a, b), or there is e e (a, b) such that fee) = min{f(x): x e (a, b)} and / is decreasing on (a,e] and increasing on [e,b). 
2.4. Convex Functions 65" 2.4.17. Let f : (a, b)  R be convex on (a, b) (the cases a = -00 or b = 00 are admitted). Show that finite or infinite limits lim f(x) and lim I(x) %-'4+ zb- exist.. 2.4.18. Suppose that 1 : ( a, b) .... 1t is convex and bounded on ( a, b) (the cases a = -00 or b = 00 are admitted). Prove that 1 is uniformly continuous on (a, b). (Compare with 2.4.14). 2.4.19. Let 1 : (a, b) .... Ii be convex on (a, b) (the cases a = -00 or b = 00 are admitted). Prove that one-sided derivatives of I exist on (at b) and are monotonic. Prove moreover that the right- and left- hand derivatives are equal to each other except on a countable set. 2.4.20. Assume that I is twice differentiable on R and /, /' and /" are strictly increasing On IR.. For fixed a,b, a < h, let x -+ (x)t x > 0, be defined by the mean value theorem, that is, f(b + x) - f(a - x) = I'(). b - a + 2:c Prove that the function  is increasing on (0,00). 2.4.21. Using the result in 2.4.4 prov Holder's inequality: H p, q > 1 and  + ; = 1, then n ( n )  ( n )  ?: IXiUil  ?: J:cd P ?: IYil q · 1=1 1=1 a=1 2.4.22. Using Holder's inequality, prove tbe Minkowski inequality: H p  1, then .! .!. 1 ( t IXI + Yil P ) P  ( t IXiI" ) l' + ( t IYiI" ) II . .=1 a= 1 -.=1 00 00 2.4.23. Prove that, if a series E a converges, then E 4 also n=1 n= 1 nl converges. 
66 Problems. 2: DifFerentation 2.4.24. For Xi, Yi > 0, i = 1,2,..., n, and p > 1, establish the inequality 111 «Xl +... + Xn)P + (Yl +... + Yn)P).  (xf +yf). +.. .+(x +Y)P. 2.4.25. Prove the following generalized Minkowski inequality: For Xi,;  0, i = 1,2,..., n, ; = 1,2,..., m, and for p > 1, ( t ( t XiJ ) P )   t ( t xfJ ) * · 1=1 J=1 J=1 1=1 2.4.26. Assume that /, continuous on an interval I, is midpoint- convex on that interval, that is, / ( X + 2 Y )  f(x) + 2 f(y) for x,y E I. Prove that f is convex on I. 2.4.27. Show that the continuity is an essential hypothesis in 2.4.26. 2.4.28. Let J be continuous on an interval I and such that !( X;v ) < !(X);!(y) for x,y E I, X:F y. Show that f is strictly convex on I. 2.4.29. Assume that f is a convex function on an open interval I. Prove that / satisfies the Lipschitz condition locally on I. 2.4.30. Let f : (0,00) -. R be convex and let lim f(x) = O. %-.0+ Prove that the function x t-+ ¥ is increasing on (0,00). 
2.4. Convex Functions 67 2.4.31. We say that I is 8ubadditive on (0, (0) if for Xl,X2 E (0, (0), f(xI + X2) < f(x1) + l(x2). Prove that (a) if x r+ Iz) is decreasing on (0, (0), then J is 5ubadditive, (b) if f is convex and subadditive on (0,00), then z r+ I) is a decreasing function on that interval. 2.4.32. Suppose f is differentiable on (at b) and for x, Y E (a, b), z  y, there is exactly one (' such that I(y) - I(x) = j'(). y-x Prove that I is strictly convex or strictly concave on (a, b). 2.4.33. Let I : Ii -. lit be continuous and such that for each d e IR the function gd(X) = f(x + d) - lex) is in Coo on III Prove that f is in Coo on III 2.4.34. Assume that an S ... S a2 S a1, and f is convex on the interval [an, al]. Prove that n n I: f(at+l )ak < I: f(at)alc+l' 1:=1 k=l where l1n+l = a1. 2.4.35. Suppose that / is concave and strictly increasing on an in- terval (a, b) (the cases a = -00 or b = 00 are admitted). Prove that if a < I(x) < x for x E (a, b) and lim I(z) = 1, zo+ then for x, y E (a, b), . In+l(x) -/n(z) lim = 1, noo In+l(y) - In(y) where fn denotes the nth iterate of f (see, e.g., 1.1.40). 
68 Problems. 2: Diff'erentation 2.5. Applications of Derivatives 2.5.1. Using the generalized mean value theorem, show that (a) ., x- l - - < cosx for z  0, 2! (b) x 3 Z - - < sinx for z > 0, 3! (c) Z2 x 4 cos X < 1 - 2! + 4! for x  0, x 3 z6 sin x < x - 3! + 5! for x > 0, (d) 2.5.2. For n E N and x > 0, verify the following claims: x 3 Z5 x 4n - 3 x 4n - 1 (a) z - 3f + 5f -... + (4n _ 3)! - (4n -I)! < sin x x 3 x 4n - 3 x 4n - 1 x4n+l <x- 3! +...+ (4n-3)! - (4n-l)! + (4n+I)!' (b) z2 x 4 X 4n - 4 x 4n - 2 1- 2f + 4! -... + (4n - 4)! - (4n _ 2)! < cosx X2 x 4n - 4 x 4n - 2 x 4n < 1- 2! +... + (4n - 4)! - (4n - 2)! + (4n)r 2.5.3. Let J be continuous on [a,b] and differentiable on the open interval ( a, b). Show that if a > 0, then there are Xl, X2, X3 E ( a, b) such that f'(Xl> = (b + a) J'(Z2) = (tr + ba + a 2 ) /'(X3) . 2X2 3x 2.5.4. Prove the following generalization of the result in 2.2.32. Let / be a complex valued function on (0,00), and let a be a complex number with a positive rea1 part. Prove that if f is differentiable and lim (af(x) + f'(x» = 0, then lim f(x) = o. %+ Z+ 
2.5. Applications of Derivatives 69 2.5.5. Let I be twice differentiable on the interval (0,00). Prove that if lim (/(x) + /'(x) + /"(x» = L, then lim I(x) = L. %+ %+oo 2.5.6. Let / be three times differentiable on (0,00). Does the exis- tence of the limit lim (f(x) + /'(%) + /"(x) + f"'(x» z-++oo imply the existence of lim I(x)? %-1-+00 2.5.7. (a) Let I be continuously differentiable on (0, (0) and let /(0) = 1. Show that if If(x)1  e- Z for %  0, then there is Xo > 0 such that f'(xo) = _e- zo . (b) Let I be continuously differentiable on (1, (0) and let 1(1) = 1. Show that if I/(x)1 <  for x > 1, then there is Xo > 0 such tbat f'(xo} = --:z. o 2.5.8. Assume that f and 9 are differentiable on [0, a] and such that 1(0) = g(O) = 0 and g(x) > 0, g'(x) > 0 for x E (O,a]. Prove that if ;; increases on (0, a], then also f increases on that interval. 2.5.9. Show that each of the equations sin(cosx) = x and cos(sinx) = x has exactly one root in [O,1r/2]. Moreover, show that if Xl and X2 are the roots of the former and the latter equation, respectively, then Xl < X2. 2.5.10. Prove that if I is differentiable on [a, b), lea) = 0, and there is a constant C  0 such that II' (x)1  Clf(x)1 Cor x E [a, b], then f(x) = o. 2.5.11. Using the mean value theorem, prove that if 0 < p < q, then (l+ ; r < (l+ : r for X > o. 
70 Problems. 2: Differentation 2.5.12. Show first that   1 + x for x e JR, and then, using this result, prove the arithmetic-geometric mean inequality. 2.5.13. Show that xy  e Z + y(ln y - 1) for x E IR and positive y. Show also that the equality holds if and only if y = e Z . 2.5.14. Suppose that / : R -.. [-1,1] is in C2 on IR, and that (f(0»2 + (f'(0»2 = 4. Prove that there exists Xo E R such that /(xo) + /"(xo) = O. 2.5.15. Establish the following inequalities: (x+  ) arctanx > 1 for x> 0, 2tanx-sinhx>0 for O<x<, x In x < - for % > 0, x ':/: e, e xlnx 1 x 2 _ 1 < 2 for x > 0, x  1. (a) (b) (c) (d) 2.5.16. DeLide which of the two numbers is greater: (a) e'll or 1r, (b) 2 V2 or e, ( c) In 8 or 2. 2.5.17. Verify the following claims: (a) In (1 + : ) In (1 + : ) <  , a, b, x > 0, (b) (l+ r(I- ;r <1, xeJR\{O},m,neN,m,n > lxl, (c) In(1 + V I +X2) < ! + lnx, X> O. x 
2.5. Applications of Derivatives 71 2.5.18. For x > 0, establish the following inequalities: (a) (b) x 1n(1 +:c) < , V 1 + x (x _1)2 > xln 2 x. 2.5.19. Show that (a) (b) I) 3 I) x. x X. x + 2 - '6 < (x + 1) m(t + x) < x + 2 Cor x > 0, x 2 ID(l + cosx) < In2 - '4 for x E (0,11"). 2.5.20. For x > 0, verify the following claims: (a) (c) (e) e Z < l+xe z , (b) xe f < eX - 1, (d) ( X + 1 ) %+1 < x z 2 -. e Z - 1 - x < x 2 e% 1 eX < (1 + X)I+%, 2.5.21. Show that (e + x)-Z > (e - x)£+z for x E (0, e). 2.5.22. Show that if x> 1, then e z - 1 + lox - 2x + 1 > o. 2.5.23. Establish the following inequalities: () 1 2. r 1r a gtanx+ 3 smx>x lor O<x<'2' (b) (c) z(2 + cosx) > 35inx for z > 0, 5in 2 X 11" COSX < ? for 0 < x < _ 2 - x- 2.5.24. Show that if a> 1, then for 0 S x S 1, 1 2 0 - 1 < ZO + (1 - z)Q S 1. 
72 Problems. 2: DifferentatioD 2.5.25. Show tbat if 0 < Q < 1, then for x, y > 0, (.z+y)O < ZO +yQ. 2.5.26. For Q E (0,1) and z E [-1,1), show that a(a - 1) '2 (1 + .z)0  1 + ax - 8 z. 2.5.27. Prove the following generalizations of the result in the fore- going problem. For B  0 and z E (-I,B], a a(1 - Q) 2 (1 +z)  1 +az- 2(1+B)2 x o a(l - Q) 2 (1+z) > 1+ax- 2{1+B)2 Z (a) if 0 < Q < 1, (b) if 1 < a < 2. 2.5.28. Prove that (a) sinz  z for z e [0, ], sinz> !x + .!.(1f2 -.u2) for x E [0,  2 ). - 1f' 1f'3 (b) 2.5.29. Prove that for z e (0, 1), 1f'z(1 - z) < sin 1f'X  4x(l- x). 2.5.30. Prove that for a positive z and a positive integer n, n k Z '"' X X ( z 1) e-L.J,<-e- . Ic=O k. n 2.5.31. For a positive integer n, find allloca1 extrema of the function ( z2 x n ) f(x} = 1 + x + 2f + · · · + nT e -% · 
2.5. Applications of Derivatives 73 2.5.32. For positive integers m and n, find all local extrema of the function I(x) = xm(l- x)n. 2.5.33. For positive integers m and n, find the maximum value of the function f(x) = sin 2m x. cos 2n x. 2.5.34. Determine all local extremum points of f(x) = x1(1- x)l. 2.5.35. Find the m inimu m and maximwn values of the function f(x) = xarcsinx + v' l- x 2 on [-1,1]. 2.5.36. Find the maximum value of f on R, where f is given by 1 1 f(x) = 1 + JxJ + 1 + Ix -lr 2.5.37. Show that for nonnegative ai, a2, . . . J an the following in- equalities hold: (a) 1 n 1 -  ake- a . < -, n L...J - e k=1 (b) 1 n 4 - a2e-a. <- n L...J k - e2' Ie=! (c) n ( 3 ) " ( 1 n ) IIak e exp 3 Lak · k=l k=l 2.5.38. Determine all local extremum points of the function f(x>= { e-r.T(J2+sin H  :1:#0, o If x = o. 
74 Problems. 2: Differentation 2.5.39. Let { X4 (2 + sin  ) if x  0, I(x) = 0 if x = o. Prove that I is differentiable on Ii and that at zero I attains its proper absolute minim um, but in no interval (-E, 0) or (0, E) is / monotone. 2.5.40. For z > 0, establish the inequa1ities sinh x tanh . h I. h2 < x < x < sm x < 2 sin x. Vs ioh 2 z + cosh 2 x 2.5.41. Using the result in the preceding problem, prove that, if a and b are positive and a  b, then 2 17 b-a a+b Ja 2 +tJ2 l+! <vab< 1nb_lna < 2 < 2. Q b The number L( a J b) = In ::n a is called the logarithmic mean of pos- itive numbers a and b, a  b. (It is also convenient to adopt the convention that L( a, a) = a.) 2.5.42. The power mean of positive numbers x and y is defined by 1 ( xI' +VP ) p M,,(x,y) = 2 if p  o. (a) Show that lim M,,(x,y) = ..flY. p-i'O (So it is natural to adopt the convention that Mo(x,1/) = vxy.) (b) Show that if x  y and p < q, then lvf,,(x, V) < Mq(x, V). 2.5.43. For   1, and for positive x, V, and for an integer n > 2, prove that < .. xn+yn+"'«x+y)n_zn_yn) < z+y .JXY - 2 + "'(2'1 - 2) - 2 . 
2.5. Applications of Derivatives 75 2.5.44. Prove that (a) sin(tanx) > x for x E [O,] , (b) tan (sin x) > x for x E [0, ; ] . 2.5.45. Prove that if x E (0, 1r /2], then 114 < - + 1 - -. sin 2 x - :z;2 1f'2 2.5.46. For x > 0, show that 3x arctan x > . 1 + 2 y l + x 2 2.5.47. Let ak,bk, k = 1,2,...,n, be positive. Prove that the in- equality  (xak + (1- X)bk) S max { n ak. n bk } holds for x e [0,1) if and only if ( t ak -b k ) ( t ak - b k ) > o. k=l ale k=1 bk 2.5.48. Using the result in 2.5.1, show that cosx + cosy S 1 + cos(xy) for z2 + y2 < 1r. 2.5.49. For positive x and y, establish the inequality xll +y > 1. 2.5.50. For an integer n > 2, prove that if 0 < x < nl ' then (I - 2x" + xn+l)n < (1 _ xn)n+l. 
76 Problems. 2: Differentation 2.5.51. Let J be defined by setting x 3 x 4 1 J(x) = x - - + - sin - for x > o. 6 24 x Prove that if y and z are positive and such that y + z < 1, then fCy + z) < f(y) + f(z). 2.5.52. Prove the inequality t(k - nz)2 (  ) Xk(l- x)n-Ic < = . Ic=O 2.5.53. Assume that I is in C2([a,b]), l(a)/(b) < 0 and I' and I" do not change their signs on [a, b]. Prove that the recursive sequence defined by f(xn) Xn+l = X n - f'(x n } ' n = 0,1,2,. ... J where we put Xo = b if I' and I" have the same sign, and Xo = a in the other case, converges to the unique root of the equation f(x) = 0 in (a,b). (This is the so-called Newton's method of approximating a root of the equation / (x) = 0.) 2.5.54. Under the assumptions of the foregoing problem, prove that if M = max{lf"(x}1 : x E [a, b)} and m = min{I/I(X)1 : x E [a, b)}, then M 2 IXn+l - I  2m (x n -), n = 0,1,2,.... , where  is the unique root of fex) = o. 2.5.55. Find sup{2- Z + 2-! : x > OJ. 2.5.56. Let / be infini tely differentiable on [0,1], and suppose that for each x E [0,1] there is an integer n(x) such that I(n(z»(x) = 0.. Prove that I coincides on [0, 1] with some polynomial.. 2.5.57. Show by example that in the foregoing problem the assump- tion that / is infinitely differentiable on [0,1] is essential. Show also tbat the conclusion of 2..5.56 is not true if lim /Cn)(x) = 0 for each n-too X E [0,1]. 
2.6. Strong and Schwarz Differentiability 77 2.6. Strong Differentiability and Schwarz Differentiability Definition 1. A real function defined on an open set A C JR is said to be strongly differentiable at a e A if lun I(x!) - I(X2) = I-(a) (ZI.Z2)"'(O,O) Xl - X2 z 1 Z2 exists and is finite. I-(a) is called the strong derivative of I at Q. Definition 2. A real function defined on an open set A C Ii is said to be Schwarz differentiable at a E A if lim I(a + h) - J(a - h) = /1I(a) h...O 2h exists as a finite limit. I-(a) is called the Schwarz derivative or the symmetric derivative of I at a. The upper (resp. lower) strong derivative 011 at a is defined by replacing lim by lim (resp. lirn) in Definition 1 and is denoted by D-/(a) (resp. D_/(a». The upper and the lower Schwarz derivatives are defined analogously. We denote them by DII/(a) and DIII(a), respectively. 2.6.1. Show that if I is strongly differentiable at a, then it is differ- entiable at a and I-(a) = J/(a). Show by example that the converse is Dot true. 2.6.2. Let / : A -+ R and let A I , A. denote the sets of points at which I is differentiable and strongly differentiable, respectively. Prove that if a E A-is a limit point of A-, then lim I-(x) = lim I' (x) = J-(a) = I' (a). Z-J-O Z-J-O zEA- zEAl 2.6.3. Prove that each function continuously differentiable at a is strongly differentiable at a. 
78 Problems. 2: DifferentatioD 2.6.4. Does the strong differentiability of I at a imply the continuity of I' at this point? 2.6.5. Suppose G C A is open. Prove that I is strongly differentiable on G if and only if the derivative I' is continuous on G. 2.6.6. Prove that if I is differentiable on IR, then it is strongly dif- ferentiable on a residual set, that is, on a set lIt \ B, where B is of the first category in R 2.6.7. Suppose I is continuous on [a, b] and the Schwarz derivative /11 exists on the open interval (a, b). Show that if /(b) > /(a), then there is c E (a, b) such that IB{C) > o. 2.6.8. Let I be continuous on [a, b] and let I(a) = I{b) = O. Show that if I is Schwarz differentiable on the open interval ( a, b), then there are Xl, X2 E (a, b) such that IB(Xl)  0 and f.(X2)  o. 2.6.9. Let I be continuous on [a, b) and Schwarz differentiable on the open interval (a, b). Show that there are Xl,X2 E (a, b) such that r(X2) $ J(b = (a) $ J"(Xl). 2.6.10. Assume that I is continuous and Schwarz differentiable on (a, b). Show that if the Schwar' derivative /- is bounded on (a,b), then f satisfies a Lipschitz condition on this interval. 2.6.11. Suppose that / and /11 are continuous on (a, b). Show that I is differentiable and f'(x) = fll(X) for all X E (a, b). 2.6.12. Assume that / is continuous and Schwarz differentiable on an open interval I. Prove that if IS (x) > 0 for x E I, then f increases on I. 2.6.13. Assume that I is continuous and Schwarz differentiable on an open interval I. Prove that if 1 8 (x) = 0 for x E I, then / is constant on I. 2.6.14. Let I be Schwarz differentiable on (a, b) and let Xo E (a, b) be a local extremum of I. Iust the Schwarz derivative vanish at :to? 
2.6. Strong and Schwarz Differentiability 79 2.6.15. A function I: IR. -+ IR is said to have Baire ' s property if there exists a residual set S C IR on which I is continuous. Prove that if f has Baire's property, then there exists a residual set B such that for x E B, D./(x) = D./(x) and DB I(x) = D-f(x). 2.6.16. Prove that if f has Baire's property and is Schwarz differen- tiable on R, then I is strongly diHerentiable on a residual set. 2.6.17. Let I be Schwarz differentiable on an open interval I and let [a, b) C I. We say that I is unijorrilly Schwarz differentiable on [at b], if Cor any E > 0 there is 6 > 0 such that if Ihl < 6, f(x+h) - f(x-h) 1 8 ( ) 2h - x <E whenever x E [a, b] and x + h, x - ]" e I. Assume that I is Schwar differentiable on I and [a, b) C I. Prove that if there is Xo E ( a, b) such that lim I/(xo + h)1 = +00, and there is XI such that f is locally h-+O bounded in [Xl, xo), then f is not uniformly Schwar differentiable on the interval [a, b]. 2.6.18. Assume that f is continuous on an open interval I cont ainin g [a, b]. Show that / is uniformly Schwarz differentiable .on [a, b] if and only if I" is continuous on [a, b]. 2.6.19. Show by example that in the foregoing problem the assump- tion of continuity of f is essential. 2.6.20. Prove that a function J locally bounded on an open interval I is uniformly Schwarz differentiable on every [a, b] C I if and only if I' is continuous on I. 
Chapter 3 Sequences and Series of FUnctions 3.1. Sequences of Functions, Uniform Convergence We adopt the following definition. Definition. We say that a sequence of functions {In} converges uni- fonnly on A to a function / if for every E > 0 there is an no E N such that n  no implies I/n(x) - /(x)1 < e for all x E A. We denote this symbolically by writing I n  I. A 3.1.1. Prove that a sequence of functions {In} defined on A is uni- formly convergent on B C A to / : B  IR if and only if the sequence of numbers {dn}, where d n = sup{ljn(X) - l(x)1 : x E B}, n E N, converges to zero. 3.1.2. Assume that In  I and gn  g. Show that In + gn =s / + g. A A A Is it true that In . gn ::t f · g? A - 81 
82 Prob lemR.. 3: Sequences and Series or Functions 3.1.3. Assume that fR ::I I, 9n =t 9, and there is M > 0 such that A A 1/(z)1 < M and 19(:1:)1 < AI for all z E A. Sbow that I.. · 9n =t I. g. A 3.1.4. Let {an} be a convergent sequence of real numbers, and let {In} be a sequence of functions satisfying suP{l/n(z) -lm(z)1 : % e A}  Ian - Omit n,m EN. Prove that {In} converges unifonnly on A. 3.1.5. Show that the limit function of a uniformly convergent on A sequence of bounded functions is bounded. Does the assertion hold in the case or pointwise convergence? 3.1.8. Show that the sequence of functions {In}, where { ; if n is even, In(x) =  if n is odd, is pointwise convergent but not uniformly convergent on JIl Find a unifonnly convergent subsequ ence. 3.1. T. Prove the following Cauchy criterion for uniform convergence. The sequence of functions {In}, defined on A, con"wge5 uniformly on A if and only if for every e > 0 there exists an no e N such that m > no implies I/n+m(z) -/m(z)1 < £ for all n E N and all x E A. 3.1.8. Study the uniform convergenoo on [0,1] of the sequence of functions {J n} defined by setting (b) (c) 1 /n(z) = 1 + (nx -1)2 ' r In(x) = Xl + (11% - 1)2 ' In(Z) = zA(l - z), (a) 
3.1. Sequences or Functions, Uniform Convergence 8.3 (d) (e) In(x) = Ilx n {l - x), /n(!r) = n 3 2"" (1 - x)'1 r f) nx" In(x) = 1 + t -nx 1 fn(x) = 1 · +xn (f) (g) 3.1.9. Study the uniform convergence of {In} on A and B when (a) /n(x) = cos,. x(1 - cos n x), A = (0. 1r/2] , B = [7t'/4, .,../2), (b) In(z) = cos" zsin 2n x, A =  B = [O,1r/4]. 3.1.10. Determine whether the sequence {In} converges uniformly on A when (a) 2x !n{x) = arctan  3 ' A = R, x-+n fn(x) =nln (1+ :: ) , A=IR, l+nx /n(x) = nln , A = (0,00), nx /,,(x} = 2 \11 + $2n, A = Ii, !..{x) = \12" + IxJ", A = 1R, /n(:r.) = '; 11 + 1 sinn :r.o.c;:r. A = 1R, In(x) = '1&( \IX - 1), A = [Ita], a> 1. (b) (c) (d) (e) (f) (g) 3.1.11. For a function I defined on [at b] set In(x) = Jn/;EH , z E [n,h], n E N. Show t.hat In =t J. [o.b] 3.1.12. \'erify that the sequence { In}, where In(x) = nsin V4 7t' 2 n 2 + x2, converges uniformly on [O,a], a > O. Does {In} converge uniformly On R? 
84 Problems. 3: Sequences and Series of FUnctions 3.1.13. Show that the sequence of polynomials {Pn} defined induc- tively by 1 Po (x) = 0) Pn+l (x) = Pn(x) + 2 (z - P(x»), n = 0,1,2,..., converges Wliformly on the interval [0,1] to I(x) = Vi. Deduce that there exists a sequence of polynomials converKing uniformly on the interval [-1, 1] to the function x  Ixl. 3.1.14. Assume that I : It -. 1R is differentiable and I' is uniformly continuous on III Verify that n (f (z+  ) - /(Z») -+ /,(z) uniformly on Il Show by example that the assumption of uniform continuity of I' is essential. 3.1.15. Let {In} be a sequence of uniformly continuous functions converging uniformly on R. Prove that the limit function is also uniformly continuous on III 3.1.16. Prove the following Dini's Theorem: Let {In} be a sequence of continuous functions on a compact set K which converges pointwise to a function I that is also continuous on K. If !n+l{Z) $ In(z) for x E K and n e N, then the sequence {In} converges to I uniformly onK. Show by exmnple tbat each of the conditions in Dim's theorem (compactness of K, continuity of the limit function, continuity of In and monotonicity of the sequence {In}) is essential. 3.1.17. A sequence of functions {In} defined on a set A is sald to be equicontinuous on A If for every E :> 0 there exlsts a 8 :> 0 such that I/n(x) - In (z')I < E whenever Ix - zll < 6, x,r E A, and n E N. Prove that if {f n} is a uniformly convergent sequence of continuous functions on a compact set K, then {In} is equicontinuous on K. 3.1.18. We say that a sequence of functions {In}, defined on a set A) converges continuously on A to the function J if for z e A and for every sequence {xn} of elements of A converging to x the sequence 
3.1. Sequences or Functions, Uniform CODvergence 85 {/n(zn)} converges to I(z). Prove that if a sequence {In} converges continuously on A to It then for every sequence {Xn} of elements of A converging to x E A and for every subsequence {In,.}, lim In,. (XI:) = I(x). k-+oo 3.1.10. Prove that if {In} converges continuously on A to I, then I is continuous on A (even if the In are not themselves continuous). 3.1.20. Prove that if {In} converges uniformly on A to the contin- uous function I, then {J n} converges continuously on A. Does the converse hold? 3.1.21. Let {In} be a sequence of functions defined on a compact set K. Prove that the following conditions are equivalent. (i) The sequence {In} converges uniformly on K to I E C(K). (ll) The sequence {In} converges continuously on K to /. 3.1.22. Assume that {In} is a sequence of increasing or decreasing functions on [a, b] converging pointwise to a function continuous on [a,b). Prove that {In} converges uniformly on [a,b]. 3.1.23. Let {In} be a sequence of functions increasing or decreas- ing on IR. and uniformly bounded on III Prove that {In} contains a subsequence pointwise convergent on nl 3.1.24. Show that under the assumptions of the foregoing problem, if the limit function I of a pointwise convergent subsequence {In,.} is continuous, then {In} converges to I uniformly on each compact subset of III Must {In} converge uniformly on Ii? 3.1.25. Show that the limit function of a sequence of polynomials uniformly cODvergent on IR is a polynomial. 3.1.26. Assume that {Pn} is a sequence of polynomials of the form I(Z) = an,p:z:P + Gn'P_lp-l +. - - + G n .1 Z + an.o. Prove that the following three conditions are equivalent: (i) {Pn} converges uniformly on each compact subset of 1R, 
86 Problems. 3: Sequences and Series of Functions (ii) there are p + 1 distinct numbers Q), Cl,..., Cp such that {Pn} converges on {CO, c},.. ., Cp}, (ill) the sequence of coefficients {an.i} converges for i = 0, It 2,.... ,p.. 3.1.27. Prove that if {In} is pointwise convergent and equicontinu- ous on a compact set K, then {in} converges uniformly on K. 3.1.28. Let {In} be a sequence of functions cont.inuous on a closed interval [a, b) and differentiable on the open interval (a, b). Assume that the sequence {J} is uniformly bounded on (a,b); that is, there is an 1\1 > 0 such that IJ(z)1 =5 kl for all n E N and x E (a, b). Prove that If {In} Is pointwise convergent on [a, b], then {In} is uniformly convergent all that intervaJ. 3.1.29. Study the convergence and the uniform convergence of {In} and {/} on A, where sin nx In(x) =  ' A = IR, :r In(X)= } 1)2 ' A=[-l,l]. +n"x (a) (b) 3.1.30. Assume that {In} is uniformly convergent on A to the func- tion J. Assume moreover that Xo is a limit point of A and, begiWling with some value of the index n, Jim In(x) exists. Prove that Z-+ZO Jim Jim In(x) = litn I(x). n-+oo -+ZO z-u:o Prove also that if {In} is uniformly convergent on (a,oo) to I and, be ginnin g with some value of the index n, lim In (x) exists, then z-+oo lim Urn In(z) = llm J(x). n-+oo z-+oo z-tooo The above equalities mean that if the limit on one side of tbe equality exists, then the limit on the other side exists and the two are equal. 3.1.31. Let {In} be a sequence of functions differentiable on [a,b] and such ihat {!n(XO)} converges for some Xo E [,b]. Prove that if 
3.2. Series of Functions, Uniform Convergence 87 the sequence {f} converges unifonnly on [a,b), then {In} converges uniformly on (a. b) to a function J differentiable on [a, b], and f'ex) = llin f(x) for x E [atb]. n-foOO 3.1.82. Fur / : [0, 1]  IR, let Bn{f, x) be the Bernstein polynomial of order n of the function I, defined by Bn(f, x) = 'to ()f (  ) xk(l-x)n-k, Prove that if J is continuous on [0,1], then {Bn(f)} cOIlverges uni- fonnly on (0, 1 J to I. 3.1.33. Use the result in the r()rcgiIlg problem to prove the approx- imation theorem of We-ierslrass. If f : [a, b] -7 Ii is continuous on [a,b), then for every E > 0 there is a polynomial P such that If(x) - P(x)1 < e Cor all x E [a, b). 3.2. Series of Functions, Uniform Convergence 3.2.1. Find where the following series converge pointwise: (a) 00 1 L l+xn ' xt:-l, n=l 00 n  X =f: -1, L.J 1 : :r. n ' n=1 . 00 2n + x n 1 L 1 + 3"xn ' X  - 3 J "=1 00 n-l t1 (1- :l: n )(I- :e n +!)' X  -1,1, (b) (c) (d) 
88 Problems. 3: Sequences and Series of Functions (e) 00 z2"'-1 E 1 _ z2n t a: 1= -1, 1, ,1 (f) E ( n ) Z , n=2 00 Lxl nn , x> 0, n= 1 00 E sin 2 (21f V n 2 + z2). n.=O () (b) 3.2.2. Study the uniform convergence of the following series on e given set A: 00 (a) E G - arctan(n2(1 + x2» ) , A = IR, n=l  In(t + nz) (b) L..J flZR ' A = [2,(0), n=1 00 (c) E n2z2e-n2Izl, A = Ill, n=l 00 (d) E x 2 (1- z2)n-l, A = [-1,1], n=l (e) 00 ,,2 E . q (ZR + :c- n ), A = {z e : 1/2  l:cl  2}, n=1 V n: 00 E2 R sin 3Z ' A= (0,00), n=1 EIn ( l+ n:;n ) , A=(-a,c), a>O. n=2 (f) (g) 00 3.2.3. Show that tbe!W!r1PJ; I; In(x), whPIe In is defined by n=1 In(z) = 0 if 0 S x < 2n1 or 2Rl_l  X  1, In(z) =  if z = 2 J n ' 
3.2. Series of Functions, Uniform Convergence 89 In(x) is defined linearly in the intervals [1/(2n + I), 1/(2n)] and fl/(2n), 1/(2n - I)), is uniformly convergent on [0,1] although the I-test of Weierstrass cannot be applied. 3.2.4. Study the continuity on [0, 00) of the function f defined by 00 J(x) =  «n -l)X: l)(nx + 1) . 3.2.5. Study the continuity of the sum of the following series on the domain of its pointwise convergence: (c) 00 . '" x n sin(nx) .l-J I ' O 7&. n= 00 L n2 n x n , n=1 (b) 00 '" n 2 L..J x , n=O 00 :E lnn(x + 1). n=l (a) (d) 00 3.2.6. Determine where the series E Ixl.Jn converges pointwise, and n=l study the continuity of the SUID. 00 . ( 2 ) 3.2.7. Show that the series E z Sl::l converges pointwise to a n=1 continuous function on III co 3.2.8. Suppose tbat tbe series E In (x), x E A, converges uni- n=l formly on A and that I : A -+ Ii is bounded. Prove that the series 00 E I(z) I n(z) converges uniformly on A. n=l Show by example that boundedness of f is essential. Under what assumption concerning I does the uniform convergence of the series 00 00 E f(x)/n(x) imply the uniform convergence of E fn(x) on A? n=l n=1 3.2.9. Assume that {In} is a sequence of functions defined on A and such that (1) /n(x) > 0 for x E A and n E N, 
90 Problems. 3: Sequences and Series of Functions (2) In (X)  !n+l(X) for x E A and n E N, (3) sup In(x)  o. zEA no;, 00 Prove that L (-I)n+1/n{x) converges uniformly on A. n=1 3.2.10. Prove that the following series converge uniformly on 1R: 00 (_l)n+l (a) L n+x2 ' n=1 00 (_l)n+l (c) L v'ii + COS;li . n=2 (b) 00 (_l)n+l L n+z2+x2 ' n=l 00 3.2.11. Show that if E f(x) is pointwise convergent on A, and if n=1 sup ( f /;(X) ) < 00, zeA n=1 00 00 and E c; converges. then E Cn/n(X) converges uniformly on A. n=l n=1 3.2.12. Determine the domain A of pointwise convergence and the dODlain B of absolute convergence of the series given below. l\1lore- over t study the uniform convergence on the indicated set C. (a) 00 L .!.2 R (3x - 1)", n n=1 00 .!. ( X + 1 ) 8 Ln x ' n= 1 c = [  ,  ] , (b) C = [-2, -I]. 3.2.13. Assume that the functions Jn, Un : A -+ Ii, n e N, satisfy the follo\ving conditions: 00 (1) the series E I/n+l (x) - I,. (x) I is uniformly convergent on At n=J (2) sup J/n(x)1  0, zEA n-+oo 
3.2. Series of Functions, Uniform Convergence 91 n (3) the sequence (Gn(x)}, where Gn{x) = L 911(X), is uniformly k=1 bounded on A. 00 Prove that the series L: /,,(x)gn(x) converges uniformly on A. n=1 Deduce the following Dirichlet test for -unifornl convergence: As- sume that f n, 011 : A  fit, n e N, satisfy lIe follO'win conditions: (1') for each fLxcd x E A the sequence {/n(x)} is monotonic, (2') {/n(X)} converges uniformly to zero on A, 00 (3') the sequence of partial sums of E 9n(X) is uniformly bounded n=1 onA. 00 Then the series L /n(x)Yn(X) converges uniformly on A. n=1 3.2.14. Show that the following series converge uniformly on the indicated set A: (a) (b) (c) (d) (e) (f) 00 n :E(-l)n+1 : , A=[O,l), n=1  Sin(l1X) L n ' A = [6,2ii - 6], 0 < 6 < 1r, n=1 :E oo sin(n2x) sin (nx) A _TO I) , - L:"' n + X" n=1 f sin(nx} :ctan(nx} , A = [6,2'/1' - 6), 0 < 6 < '/1', n=1 00 (_l)n+l..!.., A = [a, (0), a> 0,  n Z n=l 00 -n (_l}n+J .,/ .. ' A = [0,00).  n + X" n=1 3.2.15. Assume that the functions f'h On : A -+  n E N, satisfy the following conditions: (1) the function /1 is bounded on A, 
92 Problems. 3: Sequences and Series of Functions 00 (2) the series E I/n+l{x) - I",(z) 1 converges pointwise on A and n=l sup ( E I/n+l (x) -In{x)l ) < 00, eA n=l 00 (3) the series L On(Z) converges uniformly on A. n=l 00 Prove that E In(z)gn(z) converges uniformly on A. n=1 Deduce the following Abel test for uniform contJergence: Assume that functions In, On : A -+ R, n e N, satisfy the following conditions: (11) for each fixed % e A, the sequence {/n(z)} is monotonic, (2') the uce {In} is u.uiful'wly bowuleU 011 A, 00 (3') E 9n(Z) converges uniformly on A. n=1 00 Then the series E fn(z)On(x) converges uniformly on A. n=1 3.2.16. Show that the following series converge uniformly on the indicated set A: 00 (_1)n+1 (a) :E 2 arctan (nx), A = IR, n=1 n+z 00 (_l)n+l cos  (b)  ,fii n , A = [-R,R], R> 0,  n+cosz 00 (_l}hf n l (c) L vi ,A = [0,00). n=1 n(n+%) 3.2.17. Suppose that I"" n E N, are continuous on A and the series 00 E In(z) converges uniformly on A. Show that if 2:0 E A is a limit n=1 point of A, then 00 00 lim '" In(x) = E fn(xo). Z-'Zo L...J n=l n=1 
3.2. Series of Functions, Uniform Convergence 93 3.2.18. Verify the following claims: (a) (b) (c) (d) (e) 00 ( l ) n+l lim  - zn = In 2, -+ 1- L....., 1 n n= co ( 1 ) "+1 Jim '" - = In2, z-+1 L-i n Z n=1 00 lim (zn - z"'+I) = 1, z-+ 1 - L....., n= 1 00 lim" 1 = 1, z-+o+ L 2 n n z n=1 00 Z2 11'2  E l+n 2 z 2 = 6. n=1 00 3.2.19. Suppose that the series E an converges. Find n= 1 00 lim  lInX n . z-+l- L..J n=1 3.2.20. Assume that the functions In' n E N, are continuous on 00 [0,1] and L I",(z) converges uniformly on [0,1). Show that the series n=1 00 E In(l) is convergent. n=1 3.2.21. Find the domain A of pointwise convergence of the series 00 E e-n.- cos(nz). Does the series converge unIformly on A? n=1 3.2.22. Assume that In : [a,6} -+ (0,00), n e N, are continuous 00 and /(z) = E In(z) is continuous on [a, b]. Show that the series n=1 00 E In(z) converges uniformly on that interval. n=1 
94 Problems. 3: Sequences and Series of Functions co 3.2.23. Suppose that 2: In (x) converges absolutely and uniformly ,a=1 00 on A. lYlust the series E I/n(x)1 converge uniformly on A? n=1 3.2.24. Assume that In' n E N, are monotonic on [a,b]. Show that 00 if E /n(x) absolutely converges at the endpoints of [a,b], then the n=1 00 series L fn(x) converges absolutely and uniformly on the whole [a, b]. R- l 00 00 3.2.25. Suppose that L: ltLr converges. Prove that L za" con- n=1 n=J verges absolutely and uniformly on each bounded set A that does not contain an, n e N. 3.2.26. For a sequence of real numbers {an} 1 show that if the Dirich- 00 let serie., I: ,. converges at z = XO, then the series converges uni- n=1 Cormlyon [xo,oo). 3.2.27. Study the uniform convergence on ]Ii of the series E oo sin{n2z) x ") . n" n=1 3.2.28. Assume that Int n E N, are differentiable on [a, b].IVloreover, 00 00 assume that E J n(z) converges at some Xo e [a, b] and E / (x) con- n=1 n=l 00 verges uniformly on [a, b]. Show that E In(x) converg.s uniformly n=1 on [a, bJ to a differentiable function, and (  In (x) )' =  /(X) for x E la, b). 00 3.2.29. Show that f(x) = E nz:l is differentiable on III n=l 
3.2. Series of Functions, Uniform Convergence 95 3.2.30. Show that the function f ( x ) = > coo('II) 1 +n2 n=1 is differentiable on [ : , 1 ;f ] . 00 3.2.31. Let. f(z) = E (-1)"+1111 (1 + ; ) Cur;c e [0,(0). Sbuw tlaaL n=1 / is differentiable on [0, (0) and calculate /'(0), /'(1), and Urn f'(:c). z-.oo 3.2.32. Let 00 1 I(x) = E(-l)'l+l-arctant x E JIl n=1 Vii Vii Prove that f is continuously differentiable on lIt 3.2.33. Prove that the function f(x) =  sin(-nx 2 } x E  L.J 1 + -n 3 ' ..=, is continuously differentiable on IR. 3.2.34. Let 00 I(x) = E Jji(tanx)", x E (-,). n=l Prove that I is continuously differentiable on (-  ' 7) . 3.2.35. Define 00 -nz f(x) = E Ie 2 ' x E [0,(0). ,1::0 + 11 Prove that I E C([Otoo» and / E COO (0, 00) and /'(0) does not exist. 3.2.36. Show that the function 00 Ixl J(x) = E 2 + 2 n=l X n is continuous on !?:.. Is it differentiable on ti? 
96 Problems. 3: Sequences and Series of Functions 3.2.37. Prove that the Riemann (-function defined by 00 1 ("(x) =  - L..J n Z n=1 is in 0 00 (1,00). 3.2.38. Assume that f e Coo ([0, 1]) satisfies the following conditions: (1) I  0, (2) f(n)(o) = 0 for n = 0) 1, 2)  .. , 00 (3) for a sequence of real numbers {an}, the series E an/(n)(x) n=l converges uniformly on [0,1]. Prove that 1im n!Cln = O. n-tOQ 3.2.39. For x E Ii let In(x) denote the distance from x to the nearest rational with the deno minato r n (the numerator and denominator do 00 not need to be co-prime). Find all x E R for which the series E In{z) n=l converges. 3.2.40. Let g{z) -izi for:e e [-1,1] and extend the definition of 9 to all real z by setting 9(x + 2) = 9(x). Prove that the Weierstrass function I defined by 00 ( 3 ) R fez) ::  4 g(4 n z) is continuous on R and is nowhere differentiable. 3.3. Power Series 00 3.3.1. Show that, given the power series E an(x - zo)n, there is n=O R E [0,00] such that (I) the power series converges absolutely for Ix - xo I < R and diverges for Ix - xol > R, (2) R is the supremum of the set of the r e [0, (0) for which {Ianlrn} is a bounded sequence, 
3.3. Power Series 97. (3) 1/ R = lim Viani (here t = +00 and c!o = 0). n-.oo 00 R is called the radius of convergence of E un(z - :to)". n=O 3.3.2. Determine the domain of convergence of the power series given below: 00 00 2 n (a) Ln 3 :t n , (b) L ,:tn, I n. n=1 n= 00 2 n 00 (c) L n 2 xn , (d) L(2+(-1)n)n:t n , n=1 n=1 co ( 2+(-1)n r . 00 (e)  5+ (_l)n+l zn, (f) L 2nxn:l , n=1 00 co ( 1) (-1)-n 2 (g) L 2,2 :rAt t (h) L 1 + n :era . n= 1 n=1 3.3.3. Find the domains of convergence of the following series: (a) 00 (x _ 1)3n L 2nn3 ' n=1 00 4 n L n zn(l-x)n, n=1 (b) f 71 ( 2X+l ) ft n=1 n + 1 x t  (n!)2 n (d)  (2n), (3: -1) , 00 n 2 (f)  (arctan;) . (c) (e) 00 L Jji(tanx)n, n=1 00 00 3.3.4. Show that if the radii of convergence of E on:c n and E bnz n n=O n=O are R 1 and R2, respectively, then co (a) the radius of convergence R of 2: (an +bn):c n is min{RI, R 2 }, n=O if RI #: R 2 . What can he said ahout R if R I = R2? 00 (b) the radius of convergence R of E anbn:t" satisfies R 2: RIR2. n= O Show by example that the inequality may be strict. 
98 Problems. 3: Sequences and Series of Functions 00 3.3.5. Let R. and R2 be the radii of convergence of L anx'i and n=O 00 L bnx" t respectively. Show that n=O (a) if R 1 , R2 e (0,00), then the radius of convergence R of the power series 00 ,1lJl '1 L.J;;-x , 1&=0 n bnO, n=O,1,2,...., satisfies R < &. - R2' (b) the radius of convergence R of the Cauchy product (see, e.g., 1,3.6.1) of the givcn series satisfies R > min{Rl,R:!}. Show, by example, that the inequa1ities in (a) and (b) can be strict. 00 3.3.6. Find the radius of COD vergence R of E anx n t if n;=;O (a) there are Q and L > 0 such that lbn lann Q I = L, rI-POO (b) there exist positive a and L such that IiID lann"1 = L, n-t>oo (c) lim lann!1 = L, L E (0,00). n-too co 3.3.7. Suppose that the radius of convergence of E anx n is R and n= O o < R < 00. Evaluate the radius of convergence of: (a) 00 L2 n a n z n . 0=0 00 nn L ,anZ n , O n. n= (b) oc L nnanxn, n=O 00 L a;x n . n=O (c) (d) 3.3.8. Find all power series uniformly convergent on IR. 3.3.9. Find the radius of convergence R of the power series co x2n+1 t; (2n + I)!! and show that its sum J satisfies the equation f'(x) = 1 + xf(z}, x E (-R,R). 
3.3. Power Series 99 00 s.. 3.3.10. Show that the series E (n)! converges on I! and the sum f n=O satisfies the equation f"(x) + I'(x) + f(x) = e Z , z e III 3.3.11. Let R > 0 be the radius of convergence of the power series 00 n L an xn and let Sn(x) = E akx k , n = 0,1,2,... . Show that if / is n=O k=O tbe Ulll ur the eries and if:co e (- R, R) is such that Sn (xo) <: / (xo), n = 0,1,2,..., then f'(xo} i= O. 00 3.3.12. Let {Sn} be the sequence of partial swns of E an and let n=O Tn = n+Si"+s.. . Prove that if {Tn} is bounded" then the power 00 00 00 series E ('n Xn , E Sn xn and L (n + l)T n x n converge for Ixl < 1, 1&=0 n=O n=O and 00 00 00 L "n X " = (1 - x) L Sn x " = (1 - X)2 L (n + l)Tn xn . n=0 n=O n= 0 00 3.3.13. Let f(x) = L :£2" t Ixi < 1. Prove that there is an ..J > 0 n=O HUcl1 that 1\1 1/'(x)1 < 1 -lxi ' Ixl < 1. 00 3.3.14. Prove the following Abel theorem. If E an converges to L., n=U then 00 (1) E anx n converges uniformly on [0,1), n=O 00 (2) IiDt E allx n = L. z...l- ra=O 3.3.15. Prove the following generalization of the Abel theorem. H 00 {Sri} is the sequence of partial sums of E an and the radius of con- n=O 00 vergence of the power series J() = E a"x" is 1, then n=0 Jin1 Sn  linl f(x)  lim J(x) < lim Sn. J1OO z-+I- z-+l- noo 
100 Problems. 3: Sequences and Series of Functions 3.3.16. Prove the Tauber theorem. Assume that the radius of con- 00 vergence of the power series fez) = E Bnz n is 1. H lirn nan = 0 n=O n-+oo 00 and lim I(x) = L, L E IR, then E an converges to L. -+1- n=O 3.3.11. Show by example that in the Tauber theorem the assumption 1im nCn = 0 is essential. n-+oo 3.3.18. Suppose that {an} is a positive sequence and the radius of 00 convergence of I(x) = E anz n is 1. Prove that lim I(x) exists and .n=1 z-U- 00 is finite if and only if E an converges. n=l 3.3.19. Prove the following generalization of the Tauber theorem. 00 Assume that the radius of convergence of / (x) = E anx n is 1. If n=0 lim al +  + ... + nan = 0 and Jim I(z) = L, L E R, n-too n %-1-1- 00 then the series  CAn converges to L. n=O 00 3.3.20. Assume that the radius of convergence of I(x) = E CLnXR is n=O 00 1. Prove that if E na converges and Jim f(x) = L, L E R, then n=l z-.l- no E an converges and has sum L. n=O 3.3.21. Assume that an, b n > 0, n = 0,1,2,...., and the power se. 00 00 ries I(x) = L anx n and g(x) = E bnx n both have the same ra- n-O n-,O dius of convergence, equal to 1. lvloreover, assume that lim I(z) = :.:-+1- lim g(z) = +00. Prove that if lim  = A e [0,00), then also z-+l- n-+oo ft lirn &l = A. z-+l- 9('ZT 3.3.22. Prove the following generalization of the result in the fore- 00 going problem. Assume that the power series f(x) = E anx n and n=O 
3.3. Power Series 101 00 g(z) = E bnx R both have the same radius of convergence, equal n=O to 1. Moreover, assume that Sn = ao + 41 + · · .. + an and Tn = 00 be + 6 1 + . .. + b n , n e N, are positive and both series E Sn and n=O 00 E Tn diverge. If Urn ¥- = A E [0,00), then lim 9 /: = A. n=0 noo · zl- 3.3.23. Show by example that the converse of the above theorem fails to hold. Namely, the fact that lim 9 /= » = A does not imply the z...1 - existence of Jim . n...oo .... 3.3.24. Let the radius of convergence of the power series j(x) = 00 E Clnz n with nonnegative coefficients be 1 and let lim j(x)(l-z) = n=O %-.1- A E (0, co). Prove that there are positive Al and A2 such that AI n < 8n = ao + al + . · · + 4n  A 2 n, n E N. 3.3.25. Prove the following theorem of Hardy and Littlewood. Let 00 the radius of coDvergence of the power series J(:£) ;.;; E anzn with n=O nonnegative coefficients be 1 and let 1im j(x)(1- x) = A E (0,00). %--t 1- Then Ii Sn - A m -, n-.nn '11 where Sn = Go + 01 + · . . + an. 00 3.3.26. Let the radius of convergence or f{x) = E 4nxR be equal n=O to 1. Prove that if the sequence {nan} is bounded and lim fez) = z....l - 00 L, L E Ii, then the series E an converges and has sum L. n=0 00 3.3.27. Let the radius of convergence of J(z) = E anz n be equal n= U to 1. Prove that if lim (1 - x)f(x) exists and is different from zero, zJ- then {an} cannot converge to zero. 
102 Problems. 3: Sequences and Series of Functions 3.4. Taylor Series 3.4.1. Assume that / is in COO([a, b». Show that if all derivatives fen) are uniformly bounded on [a, bI, then for x and Xo in [a, b], 00 / (n) ( ) J(x) = L r Xo (x - xo)". n=O fl. 3.4.2. Define f(x) = { e-.;.s  x  0, o If % = o. Does the equality 00 I (n) ( o ) /(3:) = L r x n =0 11,. n- hold for x :F O? 00 2 3.4.3. Define j(z) = E c()g z) , x e IR. Show that J is in COO(JR) n=O and the equality /(x) = f: /(n)o) x" O n. n= holds only at x = o. 3.4.4. Show that If Q E IR \ N and Ixl < 1, then  a( 0 - I} . . . (0 - n + 1) (l+x)Q=l+ L I x n . J n. n= .!'his is called Newton's binomial Jonnula. 3.4.5. Show that for Ixl < 1, I I - 1 - ! (1 _ 2 ) _  (2n - 3)!! (1 _ 2 ) " x - 2 x L.J ( 2n)!! x. n -"J -... 
3.4. Taylor Series 103 00 3.4.6. Show that if the power series E unx n has positive radius of ,,=1 00 convergence R and J(x) = E an xn for x E (-R,R), then f is in n=1 COO(-R,R), and fen) (0) an = " n = 0,1,2,... . n. 3.4.7. Prove that if Zo is in the interval of convergence (-R, R), 00 R > 0, of the power series f(x) = E anz n , then n=O 00 I(n)(xo) I(x) = L 1 (x - xo)n Cor Ix - %01 < R -izol. O n. n= 00 00 3.4.8. Assume that y: anx n and >: bnx n converge in the same in- n=O n=0 terval (- R, Rl. Let A be the set of all x e (- R, R) for which 00 00 L anx n = L bnz n . n=O n=O Prove that if A has a limit point in (- R, R), then an = 6n for n = 0, 1, 2, . . . . 3.4.9. Find the Taylor series of I about zero when (n) /(3:) = sin z3, 3: E JR, (b) f{x) = sin 3 X, X E Ii, (c) fez) = sin % COS 3x, % e IR, (d) f(x) = sin 6 x + cas 6 z, x e Ii, (e) f(x) =  In   : ' :e e (-1,1), (f) f(x) = In(1 + z + Z2), x E (-1,1), 1 (g) f(x) = 1- 5x + 6x 2 ' z E (-1/3,1/3), e:I: (h) I(z) = 1 ,:te(-l,l). -x 
104. Problems. 3: Sequences and Series of Functions 3.4.10. Find the Taylor series of the following functions I about the point x = 1: (a) (b) (c) (d) I(x) = (x + l)e% t x E 1R., e Z I(x) = -, x :F 0, x cosx I(x) = t X  0, z lnx I(x) = -, % > o. z 3.4.11. For Ixl < 1, establish the following equalities: ( ) . _  (2n-l)!! 2n+l a arcsmx-x+  (2n)!!(2n+l)% , 00 1 arctan x = L(-l)n x2n+l. n=O 2n+1 Using the above identities, show tbat 7r 1  (2n - 1)!! "6 = 2 +  2 2n +1(2n)!!(2n+ 1) and (b) 00 : =  (-I}n 2nl . 3.4.12. Find the Taylor series for I about zero when 1 (a) f(x) =xarctanx- 2 ln!! + x2), x E (-1,1), (b) j(z) = xarcsinz + V l- x:l, :e e (-1,1). 3.4.13. Find the sum of the series OC> (_l)n+l  n(n+ I)' 00 (_1)8 L n 2 +n-2 ' n=2 f (-I}R(2n -I)!! J (211)!! t (a) (c) (e) (b)  (-I)R n  (2n+ I)!' co (_I)n-l L n(2n -I) ' n=1  3 n (n + 1) L-, r · O n. n= (d) (f) 
3.4. Taylor Series 105 00 2 3.4.14. Find the sum of the series E «(;:!!) (2x)2n for Ixl < 1. n=l 3.4.15. Using the Taylor formula with integral remainder (see, e.g., 2.3.4) prove the following theorem of Bernstein. Assume that I is infinitely differentiable on an open interval I and all its derivatives fen) are nonnegative in I. Then I is real analytic on I; that is, for each Xo E I there is a neighborhood (xo - T, Xo + r) C I such that 00 j(n)(xo) f(x) = E , (x - xo)n for Ix - xol < r. O n. n= 3.4.16. Suppose that f is infinitely differentiable on an open interval I. Prove that if for every :1:0 E I there are an open interval J C I with Xo E J, and constants C > 0 and p > 0 such that r IJ(n)(x)1 < C for x E.I, then 00 j(n)( ) I(z) == E .x o (z - ZO)R for :z: E (zo - p,Zo + p) n.J. =0 n. R- 3.4.17. Assume that f is real analytic on an open interval I. Show that for every Xo E I there are an open interval J, with Xo E J c I, and positive constants A, B such that n' IJ(n)(x)1 < A for x e J. 3.4.18. Apply the formula of Faa. di Bruno (see, e.g., 2.1.38) to show that for each positive integer n and for A > 0, '"' k! k ( n-l L-i k Ik I. uk f A = A 1 + A) t 1. 2. n. where k = k 1 +k2+.. .+k n and the sum is taken over all kl'1... ,kn such that k 1 + 2k2 + . . . + nk n = n. 
106 Problems. 3: Sequences and Series of Functions 3.4.19. Let It J be open intervals and let I : I --Jo J and 9 : J --Jo IIi be real analytic on I and J, respectively. Show that h = 9 0 I is real analytic on I.. 3.4.20. Let J be in Coo on an open interval I and (-l)njCn)(x) 2= 0 for x E I and n e N. Prove that f is real analytic on I. 3.4.21. Apply the l01'wula oC FaA ill Bl'W10 to pl0ve that, COI' eacb positive integer Rt (-l)kk! ( l ) kl ( 1 ) k2 ( 1 ) "" ( ! ) L k 1 !k 2 ! . . . k n !   .. .. ..  = 2( n + 1) n + 1 ' where k = kl +k2+." .+kn and the sum is taken over all k}, k2'..'" k n such that. k 1 + 2k2 + . . .. + nk n = n, and () = Q(Q-1).Q-k+l) . 8.4.22. Assume that I is real analytic on an open interval I. Prove that if I' (xo)  0 for an Xo E I, then there are an open interval J cont :tinin g Xo and a real analytic functIon 9 dcfin(.,(} on an open interval K containing j(xo) and such that (9 0 f)(x) = x for x E J and (/0 g)(z) = z for x E K. 3.4.23. Prove that if I is differentiable on (0,00) and such that f-1 = I', then J is real analytic 011 (0,00). 8.4.24. Prove that there is only one function I differentiable on (0,00) and such that f-l = /'.. . 3.4.25. Prove that the only function satisfying the assumptions of the foregoing problem is f(x) = axc, where c = 1+ 2 ,/5 and a = cl-c. 3.4.26. Apply the result in 2.3.10 to show that for x E (0,2), 00 1 ( x ) 2n+l In(l + z) = 2 L 2n + 1 2 + x .. n=O 3.4.27. Let l'vlp(x,V) and L(x,y) denote the power mean and the logarithmic mean of positive numbers x and y (see, e.g., 2.5.41 ,and 2.:».42 for the definition!;). Show that jf p  l7 then L(x,y) < kfp(x,y) for x,y> 0, x f; y. 
3.4. Taylor Series 107 3.4.28. Show that, in t.he not.at.ion of 3.4.27 t if p < , then there exist positive numbers x and y for which L(X,1J) > lvlp{x,y). 3.4.29. Show that, in the notation of 3.4.27, if P  0, then L(x,y»lvlp(x,y) for x,y>O, x-y. 3.4.30. Show that, in the notation of 3.4.27, if p > 0, then there exist positive numbers x and y for whir.h L(x,y) < J\;/,,(z,y). 
. Solutions 
Chapter 1 Limits and Continuity 1.1. The Limit of a Function 1.1.1. (a) Since Ix cos  I 5 lxi, the limit is O. (b) For x > 0, l- < x [] :$1 and for x < 0,1 =s x [] < I-x. Therefore lim x [;] = 1. Z--i>O (c) As in (b), one can show that the limit is equal to : . (d) The limit does not exist because the one-sided limits are dif- ferent. (e) The limit is equal to  (compare with the solution of I. 3.2.1). (f) We bave lim cos (i cosx) = liu sin (i(l + cosx)) z-+o sin(sin x) Z--i> sin(sin x) . ( 2 :r ) = Iim sIn 1r COS 2' %.-.0 sin(sin x) I . sin ( 'iT 8in2  ) = 1m - zo sin (sin x) . z 2 . z % Ii sIn 2 Sll. 2 COO 2 = m7r. · z-+O 2cos ; sin (2 sin  cos ; ) = O. sin ('iTsin 2 i) 11' 8in 2 : 2 - 111 
112 Solutions. 1: Limits and Continuity 1.1.2. (a) Huppose that lim J(z) = ,. Then, given e > 0, t.here j!i& 0 <  <  z - such that (1) If(y) -II < e if 0 < 11/1 < o. Note also that if 0 < Ixl < 6, then 0 < Ivl = I sin:cl < Ixl < 6. Thus, by (1), If(sinx) -11< e, which gives lim f(sinx) = I. z...o Now suppose tbat lim f{sinx) = I. Given E > 0, there exists z...o O<6<isucbthat (2) I/(sinz) -- II < c if 0 < Izi < 6. Now if 0 < Ivl < sin 6, then 0 < Ixl = I arcsin 1/1 < 6 and by (2) we get 1/(1/) -II = I/(sinx) -II < E. This means that lim fez) = I. z-.o (b) The implication follows immediately from the definition of a limit. To show that the other implication does Dot hold, observe that, e.g., lim[lzJ] = 0 but lim[z] does not exist. %....0 Z-.O 1.1.3. Clearly, f(x) +   2. Hence, by assumption, given E > 0, there is 6 > 0 such that 1 o  I(x) + I(z) - 2 < E for 0 < Ix I < 6. This condition can be rewritten equivalently as (1) o  (f(z) -1) + ( f) -1) < E or (2) 0 ::: (f() -1) (1- f» ) < 10. Squaring both sides of (1) and using (2). we get 2 ( 1 ) 2 2 (/(x) -1) + f{:c) -1  E + 2e. Consequently, (f(z) _1)2 S £2 + 2£. 
1.1. The Limit or a Function. 113 1.1.4. Suppose that lim J(x) exists and is equal to I. Then .on :t-tc account of our condition, we get 1 +  = 0. whicb implies I = -1. Now we show that lim f(x) = -1. To this end observe that there is z-ta «5 > Osuch that f(x) < 0 for z E (a-6,a+6)\{a}.Indeed,ifinevery deleted neighborhood of a there were an Xo such that f(xo) > 0, then we would get J(xo) + I(o)  2, contrary to our assumption. Since fez) < 0, the following inequality holds: I/(x) + 11 < j1(X) + II tX) I · 1.1.5. There is M > 0 such that If(x)1 < M for z E [0,1]. Since J(ax) = bf(z) for z E [0, ]J f(a 2 x) = 1J2 f(x) for z E [0, ]. One can show by induction that l(a R 3:} = b R 1('1') for 3: (; [0. -In]. n E 1\1. Therefore (*) I/(x)I < M for xE [Ol]' REN. On the other hand) the equality f(ax) = bJ(x) implies f(O) = 0, which together with (*) gives the desired result. 1.1.6. (a) We have x2 (1+2+3+...+ []) =%2 1+ !rh] []. It follows Crom the definition of the greatest intger function that if 0 < Ixl < 1, then !(I-lxD < z2 ( I + 2 + 3 +... + [ 2- ]) < !(l + Ixl). 2 Ixl - 2 Consequently, the limit is . (b) As in (a), one can show that the limit is k(I:;-I) . 
114 Solutions. 1: Limits and Continuity 1.1.7. Since P is a polynomial with positive coefficients, for z > 1 we get P(x) - 1 < [P(x)J < P(x) P(x) - P([x]) - P(x - 1) . Thus lun  = 1. ;e-too P((ZJ) { (-l)n f(x) = o Now if f(z) > cp(:c), then <,o(x)  J(x} = (/(x) + 1(2x» - /(2x) < (/(x) + J(2x» - cp(2x), 1.1.8. Consider J : Ilt --). R defined by setting if x = 2 ) n = 0, 1, 2,3, . . . , otherwise. which gives lim f(x) = o. z-tO 1.1.9. (a) Consider, for example, I: a -t Ii defined as follows: if x= 1 n= 1,2,3,..., otherwise. { (-1)" J(x) = o (b) H I(x)  Ixl a and l(x)f(2x)  Ixl. then Q Ixl Ixl Ixl < f(x) < 1(2x) < 1 2x 1 Q . Since  < Q < 1, we see that Urn I(x) = o. - z-tO 1.1.10. We have 9:} = lim z: = Jim ) = g(l). z-too t-too . 1.1.11. It follows from lim Ill-x » = 1 that for any n EN, ,z-too ;z . 1(2"x) . ( j(2"x) J(2'.-IX) /("lX» ) lim = lim .. · = 1. z-too f(x) zoo f(2n- 1 x) f(2n- 2 x} J{x) Assume that J is increasing and c  1. Clearly, there is n E N U {OJ such that 2 fJ < C < 2"+1. Hence by the monotonicity of 1 we obtain j(2 n x) :5/(a;) :5/(2n+l)1 which gives lim f(cx) _ - 1  1 lor c > . -too J(x) - 
1.1. The Limit of a Function. 115 In view of the above, if 0 < c < 1, then lim I(a) = lim j(t) = 1. %-+00 f(x) t-+oo f( t) 1.1.12. (a) Note first tbat if a > 1, thcn lim a Z ==: +00. Indeed, given M > %-+00 0 , a Z > k/ if and only if z > Iln  . To see that lim : + "1 = +00, n-+oo write no;l = (J+71I))n and observe that by the binomial fonnula (1 + (a - 1»" > n(n.,-I) (a - 1)2. Thus, given N, there is no such that :1 > N whe';ever n > no. Now for x > no + 1, set n = [z). Then 0; > nO + "l > N, which gives lim  = +00. z-+oo (b) Clearly, lim  = +00 for a < O. In the case when Q > 0, we %....00 get  = (  ) a = (  ) Q , :cQ x x where b = a-!- > 1. By (a), fun If = +00. Consequently, z-+oo Urn  = lirn ( !C ) Q = +00 :r:-too zOo %-+00 X for positive Q. 1.1.13. It follows from the foregoing problem that Jim c = o. .,-1'00 Substituting y = In x yields lim I: = o. %-+00 · 1.1.14. \Ve know that lim a-k = fun a--k = 1. Suppose first that n.....oo n-l' 00 a > 1. Let E > 0 be given. There exists an inteer 11{) such that n > no implies 1 - E < a-!- < a Z < a  < 1 + E for Ixl < !.. n Therefore lim a Z = 1 for a > 1. H 0 < a < 1, it follows from tbe z-.o above that fun a Z = lim 1 = 1. z-tO z-+o (1/a)Z 
116 Solutions. 1: Limits and Continuity The case a = 1 is obvious. To show the continuity of the exponential function z .... a Z , choose xo E IR arbitrarily. Then lim a Z = lim azoaz-zo = a Zo lim a" = a Zo . z-tzo %-t%0 ,,-to 1.1.15. (a) Since (see, e.g., 1, 2.1.38) fun (1 + *)ft = e, given e > U, there n-too is no such that if z > no + 1, and if n = [x], then ( 1 ) n ( 1 ) Z ( 1 ) n+l e - E < 1 + n + 1 < 1 + x < 1 + n < e + E. (b) We have ( I ) Z ( 1 ) -31 lim 1+- = Urn 1-- z-oo x 1f+CO II ( 1 ) 11-1 ( 1 ) = Iim 1+ 1+. 1f+CO II - 1 y - 1 Hence the required equality follows from (a). (r.) In view of (a) and (b) we get Urn (l+x)t = Um ( 1 + _ ,, 1 ) " = e %-.0+ 1/-.+00 and lim (1 + z)!- = lim ( 1 + 1/ 1 ) II = e. %-t'O- 1'-+-00 1.1.16. It is known that (see, e.g., I, 2.1.38) 0 < In (1 +  ) < k, n e N. Moreover, given & > 0, there is no such that noJ. < s. Consequently, if Ixl < , then -E < - 1 < In ( 1 - 1- ) < In(t + x) < In ( 1 +  ) < ..!.. < E. no-I no no no Hence lim m(1 + x) = o. To prove the continuity of the logarithmic z-tO function take an Zo E (0,00). Then Urn lnz = lim ( InZO + In =- ) = lnxo + lim Iny -+zo z-u:o %0 . 11-+ 1 = Inxo + limIne! + t) = Inxo. t....o 
1.1. The Limit of a Function. 117 1.1.17. (a) By the result in 1.1.15 and by the continuity or the logarithmic function (see 1.1.16), lim In(1 + x) = lim In(l + z)!- = IDe = 1. z-+o x zo (b) Note first that the continuity of the logarithmic function with base at a > 0, a  1, follows from the continuity of the natural logarithm function and from the equality loz = :: . So, by (a), 11m lo(l + x) 1 = oga e . z-.o Z Set 'Y = a Z - 1. Then lim a Z -1 = lim y = 1 = ma. z-.o z u...o lo(y + 1) lo&ae (c) Put'Y = (1 + z)a - 1. Clearly, x tends to zero if and only if y tends to zero. Moreover, (1 +z)O -1 y In{I +1/) y £tln(l +x) z - In(l + y) · :c = In(l + y) :c . This and (a) give lim (I+z)O-1 = Q. zo z 1.1.18. (a) Set y = (In x)!. Then In y = In:nz2) . I a . Hence, by 1.1.13 and by the continuity of the exponential function, lim (In x)!- = 1. z-.oo (b) Set 'Y = :r;&inz. Then my = &Z . zInz. By 1.1.13, lim In Urn -lnt 0 z :c= = . z-+o+ t-++oo t Again by the continuity of the exponential function, we obtain lim zSin z = 1. z-+o+ (c) Setting 11 = (005 x) .ID\ . , we see that In In(cosz) cosz-l y- . - COS % - 1 sin 2 z · 
118 Solutions. 1: Limits and Continuity NQw, by 1.1.17 (a), lim(cosx) ei: 2 . = e-. %-+0 (d) For sufficiently large, e 1 2f $ (e - 1). S e. Since lim 2! = 1 (see 1.1.14), the limit is e. z-+oo (e) We have lim (sinz)tf. = eO, where z-,o+ 1 . In fiin % + In _ 1:_ nSlnz_ lim -Z %_ 1 a- wn - - . %-100+ In z z-.o+ in % The lost cquolity follows from the continuity of the logarithmic function (see 1.1.16). 1.1.19. (a) We have sin 2% + 2 arctan 3x + 3x2 . sin 2z+2 arctan 3%+3z 2 lim =hm %2 = Z-IoO In(t + 3x + 8in 2 X) + xe Z z-.O D(1+3z+sin z! + eZ z because, by 1.1.17(a), lim !!!.(I+3;r+sin 2 zt = 3. z-.o Z (b) By 1.1.17{a) we get lim 21ncosx = lim In(l- sin 2 x) = 1. z-.o -z2 -+o -x 2 Henr.e lim In cos z = _1 . .. --.o Z 2 2- (C) We have v l-e -. =.,i l-cos % lim vI - e- Z - vI - cos x I . Vi 1 = 1m r;;;;; =. -tO+ vs inx z-+O+ V  (d) We have Jim (I + z2)CO' Z = eO t where %-.0 a = lim In(1 +z2) = lim x 2 = 0, z....o tan 9: Z-JoO X because, in view of 1.1.17(a), lim InUtz2) = 1. z-+o z 
1.1. The Limit of a Function. 119 1.1.20. (a) Observe first that 2In tan ...!!... In ( cos;J 1 .. - 1 ) 2z+1 _  z z (1) . By 1.1.16 and 1.1.18 (d), lim In(x -1) = Urn In(x -l)th = Jim !n(e" -l) = 1. %-+00 In X %-+00 11-+ 00 Hence (2) In ( 3 1 fl. - 1 ) In 2 1 .. fun COS b+T = lim cos  %-+00 X z-+oo X _ lim -21ncos 2::1 . %-+00 X Next, by 1.1.18 (e), lim -21nCOS2:1 = Jim -21nsin'2(2:+1) _ lim 2In!(2+I} . z-+oo X Z-+OO X %-+00 X The last limit is 0 (see 1.1.13). This combined with (1) and (2) implies that the limit is 1. (b) We have Jim x ( In ( 1 +  ) -In  ) = Jim In (1 + ) %-+00 2 2 %-+00 1. Z = lim In (1 + y) = 2, 11-+ 0 1I wber tbe la.5t equality bs. a C()UellCe or 1.1.17 (a). 1.1.21. Put hex) = !1. Then Urn g(x) In/(x) = lim (o:g(x) Inx + g(x) lnb(x» z-+o+ z-+o+ = lim ag(x) lnz = 1'. Z-IoO+ 
120 Solutions. 1: Limits and Continuity 1.1.22. By 1.1.17(a), Jim g(x)lnf(x) = Jim g(x) ln(f}? )-It 1) (f(x) -1) = 'Y. z-tO -t0 Z - 1.1.23. (a) Apply the result in 1.1.21 with g(x) = x, a = 1/2 and fez) = 25in vz + vx sin!. z and use the equality lim x In,;x = 0 (see, e.g., 1.1.13). The z-tO+ limit is 1. (b) Put f(x) = 1 + xe--:S sin .;. and g(x) = e . and Dote that Jim g(x)(f(x) - 1) = O. Thus, by 1.1.22, the limi1 z-ioO is 1. (c) As in (b) one can show that the limit equals ef. 1.1.24. No. For a. fixed positive and U'l'ational a, coJl8idet, the tUllC- tiOD defined by f(x) = {  if z = na, n e N, otherwise. This function satisfies our assumption. Indeed, if a  0 and a+k = lIa for some k, n e N, then there are no other k', n' e N such that a + W = n'a. If there were, we would get Ie - W = (n - n')a, a contradiction. Clearly, lim fez) does not exist. z-+oo 1.1.25. No. Consider the function defined by f(x) = {  if z = n V'2, n E N, otherwise. The limit lim f(x) does not exist, although the function enjoys the z-+oo property given in the problem. In fact, if a > 0, and for some k, n e N 
1.1. The Limit of a Function. 121 we have ak = n v'2t then there are no other k' J n' E N for which all = n' DV2. H there were, we would get k n ..'-n - = -2-;;1 k' n' , a contradiction. 1.1.26. No. Consider the function defined in the solution of the fore- going problem. To see that the function satisfies the given condition, suppose that a and b are positive and a + bn = .m V'2t a + bk = li/2 for some n. m, k, I E N such that n :; k, m :; I. Then (1) nl.v2 - mk V'2 m v'2 - 1.v2 a= . b= . n-k n-k If there were p, q e N such that p-:; n, p ':F k and q :F m, q :F I, and a + bp = q, then in view of (1) we would get m(p - k) V'2 + l(n - p) = q(n - k), a contradiction. 1.1.21. Fix e > 0 arbitrarily. By assumption there is 6 > 0 such that I/(x) - f(x)1 II < E, whenever 0 < Ixl < 6. Hence, for 0 < Ixl < 0, . fez) 1 = lim f(x) - J (z) x n-+oo Z < iIiii   1/ () - J ()I - n...oo L....J 1 I IP I k- l 2'-1 -" _ n+l 1  J.!.. L 2 k - 1 F. = - k=l 
122 Solutions. 1: Limits and Continuity 1.1.28. Put lim (f(x + 1) - f{x» = I and set :t-too lV n = sup f(x) and m n = inf !(x). zE(n,n+l) ze(n.n+l) The sequences {M n } and {mn} are well defined for n > [a] + 1. By the definition of the supremum t given E > 0 there is {x n } such that Zn E: [n, n + 1) and f(zn) > M n - E. Then /(X n + 1) -f{x n ) - E < 1\tf n + 1 -lV n < !(Zn+l) - f(Xn+l -1) + e, and consequently, I - E S lirn (Mn+l - M n )  lim (A-l n +l - AtI,.,,) < I + E. n-+oo n-t>oo Since E > 0 can be arbitrarily chosen, Urn (Mn+l-k1",) = I. In much n....oo the same way one can show that lim (mn+l - m n ) = I. It follows "'--'00 from the Stolz theorem that (see also, e.g. J I, 2.3.2) I . Mn lim mn I 1m -- - n-tOQ n - n-t-oo n + 1 - · Hence, given e > 0, there is no such that for n > .no, (*) mn -E < - ,.< e and n+l M n - E < - -I < E. n It follows from the above that if I > 0, then f{x) > 0 for sufficiently large x. Therefore if n z = [x), then ffl n21 < J(x) < Mna . +1- x - n,:t Now, by (*), we see that for x > no + 1, -e < mna -I < f(x) -I < Mn:tl -I < €. nz+l - x -n z For I < 0, one can show that mn:a < f{x} < },tIn. n z - X -n z +l 
1.1. The Limit of a Function. 123 and proceed analogously. In this way the assertion is proved for l:f:. o. To show that. our assertion is also true for I = O. put kIn = sup 1/(x)l. As above, one can find a sequence {xn} such that ze[n,n+l) IJ(zn + 1)(-IJ(zn)l- E < 1\l n + 1 - A.l n  I/(x n +l )I-I/(zn+l -1)1 +£ and show that Jim Al n = O. Since I /() I < 1\-[n for z E [n. n + 1). n-+oo n  n we get lim fez) = o. Z-foOO :f 1.1.29. For n > [a] + 1 set m n = inf f(x). By the definition :r:e[n.n+l) of the infimum, given e > 0. there ic; a sequence {Zn} such that !en E [n, n + 1) and m n < f{zn) < m n + E. Then /(Xn+l) - /(Zn+l - 1) < mn+l - m n + E. This, in turn, implies Jim (mn+l - ffl n ) = 00. By the Stolz theorem n.... 00 (see also I, 2.3.4), Jim n. = +00. If z E [n, n+ 1), then ,,:t > : + "1 ' n-foOO which gives Jim IZ) = +00. :':"'00 1.1.30. Using notation introduced in the solution of Problem 1.1.28, one can show that lim Mn+l - kIn = lim mn+l - m n = l. n-foOO uk n-foOO nl: Now by the Stolz theorem (see, e.g., I, 2.3.11), 1 6 l\1n 1 1 - Mn+l };In 1m = 1m n-.co nk+l k + 1 n....oo n k and lim. ffl n _ 1 Ii mn+l - m n n....oo nk+l k + 1 n.... n k . To prove Our o.sscrtion it if; enough to apply the rcnsoning analogouG to that used in the solutions of the two preceding problems. 1.1.31. Set Urn J:) = I and note that the function x H- In(/(x» %-'+00 satisfies the assumptions of Problem 1.1.28. Therefore we obtain 1lm In(f(-r.)) = In I. Hence Z-foOO  Urn (f(x»t = e 1nl = I. z-++oo 
124 Solutions. 1: Limits and Continuity 1.1.32. No. Consider the function defined by I{x) = {  if z = , n = 1,2,..., otherwise. 1.1.33. No. Let us take the function defined as follows: I{x) = {  if z = n  , n == 1,2,. · · I otherwise, and proceed as in the solution of 1.1.25. 1.1.34. Given € :> 0) there is 6) 0 <  < 1 J stirn thRt if 0 < 1%1 < J then I/(x(; - [  ])) I < E. Now take n e N so large that  < 6. For 0 < 8 < nI ' set z = In .. Then 1 1--1- 1-8 1 = n+l < = Z < _. n+ 1 n n n Thus n <  < n + 1 and [] = n. Consequet1YJ x(;- [  ]) =x(-n) =1_ 1 n 8 n=s. Finally, if 0 < 8 < n:\:r, then If(s)1 = If (x ( - [])) I < E. For 8 < 0, one can proceed analogously. 1.1.35. (a) Assume that f is monotonically in creasin g on (a, b). H {xn} is a decreasing sequence convergent to Zo, then {/(zn)} is also mono- tonically decreasing and bounded below by I(zo). Thus (see, e.g., I, 2.1.1), lim I(:£n) = inf j(x n ). Clearly, n-too nEN inf f(xn)  inf f(x). neN z>ZO Moreover, given x > Xo, there is an n such that X n < z, and consequently, f(zn)  I(x). Hence inf f(xn}  inf fez). nEN z>ZO 
1.1. The Limit of a Function. 125 In this way we have proved that if {Zn} is monotonically decreas- in to Xo, then lim I(x n ) = inf I(z). n-too Z>ZO Now assume that {zn} is a sequence convergent to Xo and such that %n > Xo. Then (see, e.g., I, 2.4.29) {zn} contains a mono- tonically decreasing subsequence {X nit}. In view of the above, lim f(xn/e) = inf fez). Jc-.oo z>zo H the sequence {zn} contained -3 subsequence {%nJr} such that lim f(zn..):1= inf fez), then we could find a monotonic subse- koo z>zo quence of it not convergent to inf I(x), a contradiction. This :1:>%0 implies that lim fez) = inf I(z). Z-fozt Z>ZO It is worth noting here that the above analysis shows that to de- termine a one-sided limit it is enough to consider only monotonic sequences. The same reasoning applies to the other equalities in (a) and (b). (c) Assume that I is monotonically increasing. Since I(z)  f(zo) for x > xo, f(xt) = inf fez) > f{xo). Likewise, one can show .z >o that I(x;) = sup I(z) :S I(xo). z<zo 1.1.36. (a) It follows from the solution of the foregoing problem that J(t) :S /(z-) S I(x) whenever a < Xo < t < x. If x .... xt, then t -. zt, and therefore I(xt) = lim I(t) < lim I(z-) C-+Zo z-+zt and iiiii f(z-)  f(xt> = lim I(z). z-tzt -.z: Consequently, lim J(z-) = j(xt). Z-fozt (b) This follows by the same reasoning as in (a). 
126 Solutions. 1: Limits and Continuity 1.1.37. Necessity of the condition follows immediately from the def- inition of a limit. Indeed, if Jim I(x) = I, then given e > 0, there is G  > 0 such that the relation 0 < Ix - al < 6 implies I/(x) -II < ; . Consequently, I/(x) -/(x')1 =:; I/(x) -II + I/(x') -II < e. Now we show that the ondition is sufficient. Suppose that it is satis- fied and / does not have a limit at a. Take {xn} such that lim X n = n-.oo a, Zn 1= a and {/(zn)} does not converge. Consequently, {/(x n )} is not a Cauchy sequence. On the other hand, since lim X n = a, there is n-.oo no such that if fi, k  no, then 0 < IXn -al < 0 and 0 < IXk -al < o. It follows from the assumption that I/(xn} - /(ZI:) I < e, a contradic- tion. In an entirely similar manner one can show that in order that the finite limit lim /(:&) exist the following condition is necessary and z-+oo sufficient: for every e > 0 there exists M > 0 such that x, x' > M implies I/(x) - 1(x')1 < E. 1.1.38. Let {Xn}, x n i: a, be an arbitrary sequence converging to a. It follows from the definition of the limit of a function at Q. tbat lirn /(x n ) = A. Set Yn = /(xn). Since I(x) i: A in a n-+oo deleted neighborhood of a, /(x n ) =F A for sufficiently large n. Hence Urn g(Yn) = B, or equivalently, Urn g(f(xn» = B. This means that n-+CX) n-+oo liw y(J(:&» = B. z-+a 1.1.39. Consider the functions / and 9 defined as follows: fez) = { Inz g(y) = { v II if x =  J n = 1,2, ... , otherwise, if 11 = 0, otherwise. Then g(/(x» = { sin:t) SlDZ if x =  1 n E N, or x = k1r J k e Z, otherwise, 
1.1. The Limit of a Function. 127 and lirn /(x) = 0 and lim g(y) = 1, but Urn g(/(x» does not exist. zo uo zo 1.1.40. By the periodicity of x  fez) - x, /(x + 1) = f(x) + 1. ConsequentlYJ for any integer n, f(x + n) = f(x) + n, x E TIt Since each real x can be written as the sum of its integral and fractional parts (that is x = [x] + r, where 0 < r < 1), we get (*) l(x) = fer) + [x]. The monotonicity of / gives /(0) S fer) :5 /(1) = /(0) + 1 for 0  r < 1. One can prove by induction that IR(O)  fR(T)  IR(O) + 1 for 0  1" < 1 and n E N. Therefore, 1"(0) < In(r) < j"(O) +!. n - n - n n These inequalities prove our assertion in the case when 0  x < 1. Moreover, by (*), J"(x) = In(r) + [x], which implies that the asser- tion holds for the other x e IR. 1.1.41 [6, page 47]. Observe first that x + /(0) - 1 < [x] + f(O) = f([x))  /(x)  f(1 + (x]) = f(O) + [x] + 1  x + /(0) + 1. Now we show by induction that for n E N, (1) z + n(j(O) - 1) =s In(x) =s x + n(/(O) + 1). Fix n arbitrarily and assume that (1) is true. Then, 88 in the solution of 1.1.40, we get 1"+1 (x) = /(/n(x» = f([jR{x)] + r) = (In(z)] + fer)  In(z) + 1(1)  x + n(/(O) + 1) + /(0) + 1 = x + (n + 1)(f(O) + I), 
128 Solutions. 1: Limits and Continuity where r = In(z) - [/n(x)]. This proves the right inequality in (1). In much the same way one can prove the left inequality. Again by induction, we will prove that (2) jR(m,.-I)(o) :S np  jnmp(O), n E N. For n = 1 tbe inequalities fonow from the definition of m". Suppose that they hold fOl' an arbit.rarily fixed n. Then /(n+l)m,,(o) = /m"(/nm,,(O»  jm"(O+np) = jm,,(o) +np p+np. Likewise, j(n+l)(m,-I)(O) = jm,-l(jR(m p -l)(o»  jmp-I(O + np) = np + r.- 1 (0)  np+ p. Thus the inequalities (2) are proved. Any positive integer n can be written as fl = km" + q, where o < q < mJJ. By (1) and (2) we obtain kp+ q(/(O) -1)  /Il(kp)  JIl{/lcmp(O» = In(o) = /9+ k (jlc(m p -l)(0» :s /9+ k (kp) S kp+ (q + k)(1 + /(0», which implies. ( 3 )  + q(/(O) -1) < 1"'(0) < kp + k + q (1 + /(0». n n - n -n n Since lim  = ,, and lim '* = 0, the required inequality is a n-+co p »-+00 consequence of (3). 1.1.42 [6, page 47]. Note that by 1.1.40 it is enough to show that lim IAO) exists. If /(0) = 0, then the limit is o. Assume now that 8-+00 /(0) > O. Then either for any positive integer p there is an integer m such that Im(o) > p, or there is a positive integer p such that jm (0) < p for all mEN. In the latter case {/n(o)} is a bounded sequence 
1.2. Properties of Continuous Functions 129 and, consequently, lim /ft!O) = O. In the first case lim m" = 00, n-+oo ""'00 where mp is defined as in 1.1.41. Passage to the limit as p -+ .00 in the inequalities given in 1.1.41 shows that li.m;!!: exists, and p-+oo mp consequently lim /no) also exists. R-+oo In the case where f (0) < 0 one can prove an inequality similar to (2) in the solution of the foregoing problem,-and proceed analogously. 1.2. Properties of Continuous Functions 1.2.1. The function is discontinuous at each Xo :F 'Irk, where k e Z. Indeed, if {xn} is a sequence of irrationals converging to Xo, then lim f(xn) = O. On the other hand, if {zn} is a sequence of ratio- n-roo naIs. converging to :to, then, by the continuity of the sine function, lim f(zn) = lirn sin IZnl = sin Izol i: O. Similarly, one can show n-+oo n-+oo that f is continuous at k'lf' with k e Z.. 1.2.2. As in the solution of the foregoing problem we can show that J is continuous at -1 and at 1 only. 1.2.3. (a) Observe first that if {xn} converges to x, with Zn = t where Pn e Z and qn E N are relatively prime, and Zn ;/; x, n E N, then lim qn = OO SO, if x is irrational and {xn} ic; as above, nfX) then lim f{xn) = lim _ q l = 0 = f(x). If {Zn} is a sequence n.-.oo n'-'oo ft of irrationals converging to x, then lim f(zn) = 0 = f(x). This noo means that f is continuous at every irrational. Likewise, one can show that 0 is a point of continuity of f. Suppose now that x :F 0 and x = i, where p and q are co-prime. If {x n } is a sequence of irrationals converging to x, then Jim f(x n ) = 0  n-+oo f(x). Consequently, f is discontinuous at every rational different from zero. (b) Suppose x E 1R \ Q and Jet {zn} be a sequence of irrationals different from x approaching x. Then Jim f(zn) = lim IZnl = n-roo n-rOO Ixl. H {x n } is a sequence ofrationaIs approaching x, then, by the 
130 Solutions. 1: Limits and Continuity remark at the beginning of the solution of (a), 1 . I( ) 1im xnqn 1D1 X n = 1 = x. n-too n-tCCl qn + This means that I is continuous at every positive irrational and discontinuous at every negative irrational. Similarly, one can show that 1 is continuous at zero. Now let 0 -F 3; =  (p, q co-prime). Then p (np+ l)q+ 1 X n = - . q (np + l}q converges to . Note that the numerator and denominator of X n are relatively prime. Therefore, . . /( ) _ Ii (np + l}pq + p _ E,J. P lUl X n - m f) - r . n-too n-too (np + l)q'" + 1 q q + 1 Thus the function is discontinuous at every rational different f zero. 1.2.4. Let I e C([a,b]) and let Zo be a point in la,b]. Given e : there is 6 > 0 such that if x E [a, b] and 0 < Ix - :to I < 6, 1 II (x) - 1 (xo) I < E. Now the continuity of III at Xo follows from obvious inequality II/(x)I-I/(xo)1I < Ij(x) - j(xo)l. The function given by { I for x e Qn [a,b], I(x) = -1 for z e [a,b] \ Q, is discontinuous at each point in [a, b], although 1/1 is a constant function and therefore continuous on [a, b]. 1.2.5. In order that / be continuous on IR, a necess and sufficient condition is that lim f(x) = 1im fez) and lim j(z) = lim f{x) z-t-2n- z-+2n+ %-+(2n-IJ- z-+(2n-l)+ for each n e Z. This gives b n + 1 = an and "n-l = b n -1. Consequently, by induction, an = 2n+ao and b n = 2n-l+ao, ao E III 
1.2. Properties of Continuous Functions 131 1.2.6. Since the function is odd, we will study its continuity only for x > O. Clearly, f is continuous at each x #: ,;;i, n = 1,2,... . Now suppose that n = k 2 , where k is a positive integer. Then lim f(x) = n lim sin 1(% = 0 %-tok+ z-.k+ and lim f(z) = (n - 1) lim sin 1rX = o. z-.k- z-tot- Hence the function is also continuous at every n = k 2 . If n E N is not a square of an integer t then lim fez) = n lim sin 7rX = nsin(1rvn) %-to..JRT z-t v-n+ and lim f(x) = (n - 1) sin(7ry'ii). z-+,fR- WP. r.nndnrle that. ! it; (1ir.ontinun1l at x = +yIn, wherp. n  1:2. 1.2.7. We get { I if x E [, 1), f(x) = n + (x - n)R if x E [n, n + 1), n E N. Consequently, the function is continuous at each x 1: n, n E N. Moreover, lim f(x} = Urn f(x) = n = fen). So, f is continuous zn+ z-tn- on [i,oo). Now we show that f is strictly increasing on (1,00). Clearly, f is strictly increasing on each interval [n, n + 1). H Xl E [n - 1, n) and X2 E (n, n + 1), then /(X2) - f(xl) = (X2 - n)n + 1- (xJ - n+ l)n-l > (X2 - n)n > O. It thE'n follows that I(X2) - !(Xl) > 0 for 2 E [m, m + 1) and Xl E [n, n + 1), if m > n + 1. 1.2.8. (a) We have 1 if z > 0, I(x) = 0 if x = 0, -1 if x < O. 
132. Solutions. 1: Limits and Continuity 1 -1 The function is discontinuous only at zero. (b) By the definition of I, I(z) = { r  z  0, '% if x < o. The function is continuous on III (c) We get fez) = lim 1n(e n +xn) = lim n + In (1 + (x/e)n) . n-too n n-+oo n Consequently, { I if 0 < x < e, J() = In if > x z e. 1 I I I . o e. The function is continuous on [0,00). 
1.2. Properties of Continuous Functions 133 (d) J(z) = max{4,z2, }. The function is continuous on IR \ to}. . , .,  , II , , \ , , , \ I , , , , , I \ , \ I \ , \1 /\ 1 "  , , , '\J .-.... "- . " -2 .1 .J. 2 - II It I' " , " I I . ' , I . ' , ' , ' , I , I , I ,I ,\ I , I " v " ,," ..... 1. 1 :z 2 (e) f(x) = max{1 coszl, I sin xl}. Clearly, J is continuous on III I -I i i K 1.2.9. Let T > 0 be a period of f. By the continuity of f on [0, T) there are z* E [0, T] and zit E [0, TJ such that j(x*) = in! j(z) and ZE[O,T] J(z*) = sup j(z). The desired result follows from the periodicity ze[o,TJ of f. 1.2.10. Since P is a polynomial of even degree, we obtain fun P(x) z-+oo = lim P(z) = +00. Therefore for any At! > 0 there is a > 0 such z..-oo that if Ixl > a, then P(z) > M. Let xo e [-a, a] be such that P(xo) = inf P(x). zt:(-c,o) If P{xo)  lvI, then we can set z* = xo. If P(xo) > M, take b > 0 such that P(z) > P(xo) whenever Izi > b. By continuity there exists 
134 Solutions. 1: Limits and Continuity :1:* E [-b, b] such that P(x lr ) = inf P{x). zE{-6.6] To prove the second assertion, observe that Iim IP(x)1 = 1im IP(x)1 = +00 z-+-oo z....oo and proceed analogously. 1.2.11. (a) Set ( { 2x - 1 if :c E (0, I), I x) = o if x = 0 or x = 1. (b) For n E N, put { I 232 ft - 1 } An = 0, 2 n ' 2 n ' 2 n ' · . . , 2 n . n-I 00 andBl = AI, Bn = An\ U A" = An\An-l- Clearly, U A" = k= 1 k=) 00 U Bk o Define / as follows: k= 1 { 00 o if x e [0, 1] \ U Ak. I)= k=l 2 1 " - 1 if Z E Bn t n E N. For any a and b, 0 5 a < b < 1, inf I(x) = -1; -1 is Dot zelo.b) attained by J on [a, b]. 1.2.12. Observe first that (1) w/(xo,Ot)  W/(xo, 02) whenever 0 < 6 1 <. Assume that lim w/(xo,6) = O. Then, given E > 0) t.here if) no .> 0 6...0+ such that w/(xo,6) < e if 6 < 00. Consequently, if Ix - xol < 6 < 6 0 , then I/(x) - l(xo}1 < E, wbich proves the continuity of f at Zo. Assume now tbat / is continuous at Xo- Then, given E: > 0, there is 60 > 0 such that Ix - xol < 6 0 implies I/(xo) - /(x)1 < . Hence, in view of (1), if 0 < 6 < 60, then w,(xo,o)  WJ(X, 6 0 ) < E, 
1.2. Properties of Continuous Functions 135 and consequently, lim W/(x o , 6) = O. 6-.0+ 1.2.13. (a) Let:co E [a, b] and E > 0 be chosen arbitrarily. It follows from the continuity of 1 and 9 that there is 6 > 0 such that if z E [a, b] and l:.c - Xo I < 6, then I{xo) -E < f(x) < f(xo} +E and g(xo) -e < g(x) < g(xo) +£. Hence (1) hex) < min{f(xo) + e, g(xo) + e} = min{f(xo), g(xo)} + e = h(xo) + e and f(x) > f(xo)-e  II(xo)-£ and g(x) > g(xo}- > h(xo)-e. Consequently, (2) h(x) > h(xo) - E. Thp. r.ontinl1it.y of h at o follows from (1) and (2). In much the same way one can prove that H is continuous on [a, b]. (b) As in (a) one can show that ma:x{/11/2,/3} and min{/hf2,/3} are continuous on [a, b]. Continuity of I (ollows from I() = /1() + 12(:1:) + /3 (x) - max{/S(X)'/2(X),/3(X)} - min{!1 (x), 12(X), 13 (x) }. 1.2.14. Since! is continuous, the fun(.1.ions m and M are well de- fined. Let Xo be in [a, 6] and let c; > O. By the couLiuuiLy ur I, t.he is 6 > 0 such that sup I/(xo + h) - l(xo)1 < E. Ihl<6 It follows Crom the definition of m that (1) m(xo + h) - m(xo} = inf fee) - in! ICe) < o. (E[a.zo+h] (Ela.zoJ 
136 Solutions. 1: Limits and Continuity Observe that if the first infimum is attained at a point in [a, %0], then the equality holds in (1). So, suppose tbat Xh e (xo, Xo + h) and m(xo + h) = inf /(e) = /(Xh). (E(o,zo+h) Then for Ihl < 6, m{;z;o + h) - m(xo) = /(Zh) - in! /(t;)  !(;Z;h) -/(zo) > -e, (E[o.%o] because IZh - zor S )hl < 6. We therefore have shown that m is continuous at each Zo in [a, b]. The same argument can be applied to show that M is continuous on [at b]. 1.2.15. Since I is bounded, the functions m and M are well defined and bounded. Moreover m is decreasing on (a, b] and M is increa - on [a, b). For Xo e (a,6), by 1.1.35, lim ''''(;1;) = inf rn(') 2: 1n(xo). %-+%0 <E(o,zo) If inf m«() > m(xo), then there is a positive d such that (E(O.zo) in! m({) = m(xo) + d. (':(0 , %0) Thus, for each (e (a, xo), m«() = inf fez) 2: m(xo) + d, a < :r« and consequently, f(x)  m(xo) + d for every x e [atzo), a contra- diction. So, we have proved that 1im m(x) = m(xo). The continuity %-+zo from the left of M can be proved in precisely the same manner. 1.2.16. No. Consider the following function: 2 if X e [0, 1), j(z) = 1 if X e [1,2), 3 if x E [2,3]. Then m* is not continuous from the left at Xo = 1, and M* is not continuous from the left at Xl = 2. 
1.2. Properties of Continuous Functions 137 1.2.17. Set lim I(x) = I. .Then, given E > 0, there is M > a BUch z-+oo t.hat I/(x) -II < F: for x > M. Thn!; if x > lv', thEm. 1-£ < J(x) < l+f:. Obviously, since f is continuous, it is bounded on [a, M]. 1.2.18. Suppose that lim X n = Q. By the continuity of I, for every noo E > 0 there exists 6 > 0 such that (1) If(x) - l(a)1 < E for Ix - al < 6. It {ollows from tbe definition of limit inferior that there is a sequence {xn.} for which IZnlr - al < 6 beginning with some value ko of the index k. Now by (1) we get IJ(x n .,) - J(a)1 < E for k > ko. We therefore have shown that !ill! J(xn) < f( lim x n ). n-too 8-+00 We now show by example that this inequality may be strict. Take f(x) = -:x, z E JR, and X n = (-1)", n E N. Then -1 = .!ill! f(xn) < J( !ill! x n ) = 1. n-too n-+oo In an entirely similar manner one can show that lim f(xn) 2: f( 1im zn). n....oo n-+oo The same example can be used to show that this inequality may also be strict. 1.2.19. (a) It has been shown in the solution of the preceding problem that for any bounded sequence {xn} and for any continuous function / the following inequalities hold: fun f(xn) < f( Urn Xn) n-too n-too and nm f(xn)  f( lim x n ). n-+oo n-+oo Put Urn Xn = a. Then there exists a sequence {x n ,.} such that n-+oo (1) J (x n ,) < I(a) +E 
138 Solutions. 1: Limits and Continuity (see the solution of the foregoing problem). Clearly, for suffi- ciently large n we have Xn > a -  . Hence by monotonicity and continuity of f we obtain I(zn}  I (a-  ) > I(a} - e. Combined with (1), this gives Urn J(Zn} = I( Urn Zn). n-.oo n-too (b) The proof of this equality runs as in (a). 1.2.20. Apply 1.2.19 to -I. 1.2.21. Note that 9 is well defined and increasing on ]It (a) By Problem 1.1.35, (1) g(z;) = sup g(x)  O(xo). %<%0 Suppose that O(xo) < g(xo). Then there is a positive d such that g(x o ) = g(xo) - d. Consequently, for every x <. xo, sup{t: J(t) < x} < g(xo) - d, or equivalently, t < g(xo) - d if I(t) <;&. Thhi iwpli tha.t t  g(xo) -d if f(t) < Xo, which gives g(xo) = sup{t : j(t) < xo} < g(xo) - d, a contradiction. (b) The function 9 may be discontinuous, as tbe following example shows. If x for x < 1, for 1:S x :S 2, for x > 2, f{x) = -x+2 3;-2 then { X for x < 0, g(x) = - 2 + x for z > o. 
1.2. Properties of Continuous :Functions 139  2 f 1.2.22. We know that the set {m + n# : m, n E Z } is dense in 1R (see, e.g., I, 1.1.15). Thus, given an x E Ii, there is a sequence { mk + nl: R-} convergent to ';'2 . By the periodicity and continuity of 1 we get 1(0) = lirn f(ml:T2 +nkTt) = I(x). k-looo Let Tl and T2 be two incommensurate numbers and let w = {x E 1R: x = rT 1 + ST21 s, t e Q}. Define f by setting { l. f(x) = 0 for xeW, for x E 1R \ w. Then Tl and T 2 are periods of f. 1.2.23. (a) Assume that Tnt n e N, are positive period') of / such that lim Tn = O. By the continuity of / t given Zo E R and E > 0, n-+oo there exists 6 > 0 such I/(x) - /(xo}1 < E whenever Ix - xol < o. Since lirn Tn = 0, there exists no for which 0 < Tno < . Then at n-+oo least one of the numbers kTno with k E Z belongs to the interval (xo - o,xo + 0). Conse<!uently, I/(xo) -/(0)1 = I/(xo) - f(kTnu)1 < £. It then foIlows from the arbitrariness of E > 0 and Xo e JR that I is constant, contrary to our assumption. 
140 Solutions. 1: Limits and Continuity (b) The Dirichlet function defined by setting { I if :c e Q, I(x) = 0 if x e R \ Q, is periodic. Every rational number is its period. Therefore a fundamental period does not exist. (c) Assume that the set of all periods of I i not dn in nt Then there exists an interval ( a, b) which does not contain any period of I. As in (a), one can show that there is a period T and an integer k such that kT E (at b). A contradiction. 1.2.24. (a) Let Zo E lIt be a point of continuity of I. Since I is not constant, there is %1 ¥: xo such that /(:1:1) :/= I(xo). Hthere were no minimal positive period of I, there would exist a sequence {Tn} of positive periods of 1 converging to zero. Take 0 < E < I/(ZI) - 1(2"0»)' It follows from the continuity of / at %0 that there is 6 > 0 such that (1) I/(x) -/(zo)1 < £ whenever Ix - zol < 6. Since lim Tn = 0, there exists an index no for which 0 < Tno < n-+oo . Thus at least one of the numbers kT no' k E Z, belongs to the interval (zo - Zl - 6, Zo - Xl + 6). Consequently, XI + kT no e (zo - 6, Zo + 6) and, by (1), I/(zl) - /(zo)1 = I/(zl + kT no) - /(%0)1 < E. A contradiction. (b) This is an immediate consequence of (a). 1.2.25. Let Tl and T 2 be positive periods of f and g, respectively. Suppose that I  g. Then there is %0 such that I(zo) f; g(zo), or in other words, (1) I/(zo) - g(xo}1 = M > o. For 0 < £ < !f, there is 6 > 0 such that (2) I/(xo + h) - /(zo)1 < £ whenever Ihl < 6. 
1.2. Properties of Continuous Functions 141 By the assumption lim (f(x) - g(x» = 0, there is a positive integer z-t>oo k such that, if x  xo + kT2' then I/(x) - g(x} 1 < E. Consequently, for any positive integer m, (3) I/(zo + kmT 2 ) - 9(Zo + kmT 2 ) I < E. By (2), (3) and the periodicity of f and 9 we get I/(xo) - g(xo)1 = If(xo) - f{xo + kmT 2 ) + f{xo + kmT 2 ) - g(xo + kmT 2 )1 (4) S If(xo) - I(xo + kmT2)1 + I/(xo + kmT2} - g(xo + kmT 2 )1 = I/(zo) -f(%o + kmT2 - nT1)1 + If{xo + kmT 2 ) - g(xo + kmT 2 )1 < E + E = 2E, whenever (5) ImkT2 - nTll < 6. However, since 2e < M, (4) would contradict (1) if there were mEN and n e Z satisfying (5). On the other hand, if  is rational, (5) is obviously satisfied for some integers m and n. If  is irrational, then (5) is also satisfied (see, e.g., I, 1.1.14). 1.2.26. (a) Set I(x) = sin x and g(x) :; :Ii - [x] for x e III Then 1 and 9 are periodic with fundamental periods 211" and 1, respectively. Therefore no period of / is commensurate with any period of g. Put h = J + g. If h were periodic with period T, then we would get sinT + T - [T] = 0, sin(-T) -T - (-T] = O. Consequently, (T - [Tn + (- T - [-T]) = 0, which would imply T -[T] = o. This would mean that T is an integ, a contradiction with sin T = O. (b) [A. lJ. KudriaSov, A. S. MeSeriakov, Mathematics in SChool, 6 (1969), 19-21 (Russian)] Let Q, /3 and 'Y be such real numbers that the equality aa + b{J + cry = 0, where a, b, c E Q, holds if and 
142 Solutions. 1: Limits and Continuity only if a = b = c = O. Such numbers do exist. One can take, for example, Q = 1, {J = ,f2 and "I = ,;3. Define W = {all: + bp + C')': a, b, c E Q}. Consider two functions J and 9 defined by setting ( X = { -b - c - fil +  if x = aa + bJ3 + Li E W, I) 0 if z rt w, ( x ) = { a + c + (},2 - t? if :£ = act + bfj + C"'f E W, 9 0 if x rt w. Note that each number Ter, T e Q\ {OJ, is a period of I and each number s{3, 8 E Q\ {OJ, is a period of g. We will show that these funt.1.ions do not have any other periods. H T is a period of I, then J({3 + T) = J({3), and because /(13) = -2, we get P + T E W. Consequently, T E W. Therefore T = TQ + 8/3 + t1 with some r, s. t E Q. Since J(T) = /(0). wP ohblin -H -I. - 8 2 +  = OJ or equivalently, (8+t)(1+s-t) = o. \\7e now show that l+s-t  O. Indeed, if 1 + 8 - t = 0, then T = TO + s{3 + (1 + 8)1. Using (1) f(x + T) = f(x), with a: = -1, we .get -8 - 8 - 8 2 + 8 2 = 1 + 1, or 8 = -1. .Therefore T = ra -{3. Now substituting x = (J into (I) yields J(Ter) = J({3), and consequently, 0 = -1 - 1, a contradiction. Thus we have proved that 1 + 8 - t :f: o. It then follows that s + t = O. Hence T = Ter + s/3 - 87. The task is now to show tbat 8 = o. To this end we take x = 'Y in (1), and we get -8 + S -1- 8 2 + (8 _1)2 = -1 + 1, which implies s = O. In an entirely similar way one can show that the only periods of 9 are thosc mentioned above. So no period of f is commensurate with any period of g. Note now that h = f +g is given by the fonnuIa { a - b + a 2 - 6 2 if x = tlQ + bfJ + C"( E W t h ( a: ) - o if xW. As above, one can show that the only periods of h arc the numbers t1, where t E Q and t 1: o. 
1.2. Properties or Continuous Functions 143 1.2.21. Suppose that h = / + 9 is periodic with period T. Since * fi Q, we see that :. tI. Q or f:; (j. Q. Assume, for example, tbat /r tJ. Q. By the periodicity of h we get j(x + T) + g(x + T) = hex + T) = hex) = J(x) + g(x) for x E III Therefore the function H defined by setting H{x) = f(x + T) - f(x) = g(x) - g(z + T) is continuous and periodic with two incommensurate periods T. and T2. By the result in 1.2.22, H is constant. This means that there is c E JR. such that I(x + T) = I(x) + c for x E R. Suppose that c ¥- o. Substituting z = 0 and then $ = T in the last equality, \ve get j(2T) = j(T) + c = /(0) + 2c. One can show by induction that j(nT) = I(O)+nc, which contradicts the boundedness of I (see, e.g., 1.2.9). Hence c = 0 and T is a period of J. Consequently, T = fl.Tl with some n E Z, a contradiction. 1.2.28. The proof of this result is a modification of that presented in the solution of the preceding problem. Assume that T 1 is the fundamental period of f. As in the solution of the foregoing problem one can show that the function H given by the Cormula H{x) = I(x + T) -f(x) = g(x) - g(x + T) is identically equal to zero. Therefore T is a common period of J and g, a contradiction. 1.2.29. Suppose, for example, tbat f is monotonically increasing. Let Xo he a point. of njont.illility of /. Ry thp. rP.8ult in 1.1.35. j(xt)- j(x;) > O. This means that / has a simple discontinuity at xo. With each such point :1:0 we can associate an interval (I (x;), j(xt». It follows from the monotonicity of / and from the result in 1.1.35 that the intervals associated with different points of discontinuity of / are dhsjoint. Takiug one rational number from each interval, we obtain a one-to-one correspondence between the set of points oC discontinuity of j and a subset of Q. 1.2.30. Since f is uniformly continuous on [0, 1], given e > 0 there exists no e N 5uch that. COl. 2n > Ito and f(l1" k = 1,2,..., 2.7£ w have 1(  )_/( kl ) <e. 
144 Solutions. 1: Limits and Continuity Thus if 2n > 110, then 1 2n ( k ) 182nl = - ""(-l}kf - 2n f;: 2n < - 2. Moreover, 1 2n+1 ( k ) n 1 I S 2n+ll= 2n+l EC-l)'f 2n+l ' < 2n+l E + 2n+l ' /(1)1. k=l . It then follows that lim ! (-l)t I ( k ) = o. n-too n L.J n k=1 1.2.31. As in the solution of the foregoing problem, Dote first that J is uniformly continuous on [0,1]. Hence, given E > 0, there is no e N such that if n > no and k = 0, 1,2, .. . . n, then k () -I e: 1 )1 <S. Consequently, for n > no, 5n =  t<-I)t ( n ) / (  ) 2k=O k n =   <_I)t(n k 1) (I () _I ( k: 1 )). Therefore e n-l ( n-l ) e ISnl < 2" E k = 2 . k=O 1.2.32. Put M = lim sup fez) and m = lim inf f(x). Suppose r-too z > r r-too z > r that M > m. trhen there is a real number k such that M > k > m, and there exists a satisfying / (a) > k. By the continuity of / there exists b> a such that j(t) > k for all t E [a, b]. 
1.2. Properties of Continuous Functions 145 Take p = baba . Then   i + 1 whenever x > p. Indeed,  _ = = x ( :. - ! ) = x > 1. a b a b p- Therefore there is a positive integer no between  and ;; that is, i > no > i, or equivalently, a:S :0 :5 b. By assumption, lex) = I ( no';) > I (;f;;) > k for all.:t  p, which contradicts the definition of m. Consequently, m = M, which means that llm f(x) exists and is finite or infinite. zoo 1.2.33.. Let f be convex on (a,b) and a < s < u < v < t < b. It follows from the geometric interpretation of convexity that the point (u, /(u» lies below the line passing through (8, /(8» and (v,/(v». This means that (1) leu) S 1(8) + I(V = (8) (u - 8). Similarly, the point (v,/(v» is below the line passing through (u, feu» and (t, J(t»). Thus (2) f(v)  J(v.) + f(t) - /(u) (v - u). t-u Inequalities (1) and (2) give /(8) + J('U) -/(8) (v _ s) < f(v) < feu) + J(t) -/(u) (11 - u). u-s t-u It follows from these inequalities and the squeeze law that, if {v n } is a sequence converging to u from the right, then lim J(v n ) = J(u), n-t-oo which means that lim J(x) = feu). Likewise, lim fez) = f(u). %u+ Z-i'U- Thus the continuity of / at any point u in (a, b) if) proved. The follo\ving example shows that the assertion is not true if an interval is not open: f{x) = { 0:l:2  z E [0,1), .2 if x-I. 
146 Solutions. 1: Limits and Continuity 1.2.34. It follows from the uniform convergence of {III} that, given E > 0, there exists no such that 1 I/n(x) - l(x)1 < 3 E for n  no, x E A. Fix a E A. By the continuity of I no at a there is 6 > 0 such that 1 l/no(:C) - /no(a)1 < 3 t: whenever l:c - al < d. Thus I/(x)- l(a)1 S I/no(x)- I(x) I + I/no (x)- /no(a)1 +1/'JQ(a)- l(a)1 < e. 1.3. Intermediate Value Property 1.3.1. Let / be defined on [a, b) by setting { Sin-L if a<x<b I(x) = Z-Q. - , o if x = o. ] a b -1 Clearly, J enjoys the intermediate value property on [a, b] but it is discontinuous at o. We now construct a fun(:tion enjoying the intennediate value property and having infinitely many points of discontinuity. Let C denote the Cantor set. Recall that the Cantor set is defined as fol- lows. We divide the interv-c:1l [0, 1] into three equal parts, remove the 
1.3. Intermediate Value Property 147 intenraJ (1, i), and denote by E 1 the union of the intervals [0, i] and [. 1]. At the second stage we remove the open nriddle thirds of the relnaining two intervals and set E 2 = [o,] u [,] u [ : '] U [ : ,1]. Proc:(ling analogously, we rp.lllove .at th{& nr.h st:lgP. t.hp union of thp. open middle thirds of the remaining 2,.-1 intervals and we denote by En t.he union of 2° closed intenws, each of length 3- n . Then X) C = .n Ell. ,.,:: 1 Note that if (Ui, b i ), i = It 2,... , is the secluencc of removed intervals, then 00 C = [0,1] \ UCai,b;). I=J Define the function 9 b}- setting { o if xEC g(x) = 2(Z-tli) 1 if ( b ) . 1 2 b. -Gi - x E 6i, i , & = , ,.... It follows from the construction of the Cantor set t.hat each interval [a, b] C [0, 1] contains an open subinterval disjoint with C. Indeed, if (a, b) is free of points .of C, then (a, b) is one of the removed intervals (ai, bi) or its subinterval. IT th(re is x E (a, b) n C, then there are liEN and k E {O, 1,2,..., 3 n - I} such that x E [b. j:3: 1 ] C (a. b). Then the open middle third of [ 3':. t  I ], which in fact is One of the intervals (ai, b i ), is an open subinterval free of points of C. The function 9 is discontinuous at each x e C, and it [ollows CrOID the above that 9 enjoys the intermediate value property. 1.3.2. Let Xo E (a, b) be arbitrarily fixed. It follows from the mono- tonicity of / that sup J(x} = I(xo)  f(xo) < f(xri> = inf J{x) BSZ<;.f.O zo<$6 (see, e.g., 1.1.35). Suppose no\v that f{xo) < J(xt). 
148 Solutions. 1: Limits and Continuity Then there is a strictly decreasing sequence {x n }, xn E (xo,b], con- verging to Zo and such that lim I(xn) = I(X6). Since 1 is strictly ra-tQU increasing, f(x n ) > f(xt) > f(xo). By the intermediate value prop- erty, there is x' E (XOt x n ) such that I(x') = I(xt). Then inf I(x)  in! I(z) = fez'). ZO<Z<%' zo<b On the other hand, by the strict monotonicity of I, in! f(x) < zo<z<z' J(x'), a contradiction. So we have proved that I(xo) = I(zt). The equalities I(x(;) = I(xo), I(a) = I(a+), and j(b) = I(b-) can be proved in an entirely similar mMln er. 1.3.3. The function 9 defined by g(x} = I(x) - x, x E [0,1], is continuous, and 9(0) = 1(0)  0, and g(l) = 1(1) - 1 < O. Since 9 has the intermediate value property, there exists Xo e [0,1] such that g(xo) = O. 1.3.4. Consider the function hex) = I(x) - g(x), x E [0, b], and ob- serve that h(a) < 0 and h(b) > o. By the intermediate value. property there is Xo E (a, b) such that h(xo) = O. 1.3.5. Define the function 9 by setting g(2:) = / (z +) -/(2:). Then 9 is continuous on IR. g(O) = I(  ) - f(O). and g( ; ) = 1(0)- I( t). Thus there is Xo E [0, f] for which g(zo) = o. 1.3.6. Put m = min{f{zl),..., f(zn)} and M = max{/{xl)'..., f(xn)}. Then m < .!. (I (XI) + 1(%2) +... + f(xn» < M. -n - Consequently, there is Xo E (a, b) such that I(xo) = .!. (f(Xl) + f(X2) + . . . + f{x n » · n 
1.3. Intermediate Value Property 149 1.3.7. (a) Set I(x) = (1 - z) cos - sin 3:. Then I{O) = 1 and /(1) = - sin 1 < o. Therefore there is Xo e (0,1) satisfying /(xo) = O. (b) It is well known that (see, e.g., 1.1.12) Urn e-:rIP(x)1 = 0 and lim e-ZIP(x)1 = +00. %OO %- Consequently, there is an Xo E IR such that e-ZOIP(xo)1 = 1. 1.3.8. Let us observe tbat sgnP(-a,) = (-1)' and sgnP(-b,} = (-1)1+1, 1= 0,1,... ,n. By the intermediate value property, there is a root of the polynomial . P in every interval (-b" -ad, I = 0,1,... tn. 1.3.9. No. Consider, for example, I and 9 defined as follows: I(x) = { sin z 1 a  a < x  b, o If z = a, and { -sin %:a if a <  S b, g(z) = . 1 If x = a. 1.3.10. Set g(x) = I(x + 1) -/(x), x e [0,1]. Then g(l) = 1(2) - /(I) = -g{O). Hence there exists 2:0 E [0,1] such that 1(%0 + 1) = /(xo). So, we can take X2 = Xo + 1 and XJ = Xo. I I I . o %. lX:z ? - 
150 Solutions. 1: Limits aod Continuity 1.3.11. Consider the fWlction 1 g(x) = I(x + 1) -j(x) - 2 (j(2) -j(O», x E (0,1], and apply the reasoning analogous to that used in the solution of the preceding problem. 1.3.12. Define the function 9 by the formula g(x) = f(x + 1) - lex) Cor x E [0,71 - 1]. H g(O) = 0, then 1(1) = 1(0). So suppose, for exarnple, that g(O) > o. Then /(1) > 1(0). If alo I(k+ 1) > J(k) for k = 1,2,..., n -1, t.hen we would get 1(0) < f(l) < /(2) < . .. < fen) = 1(0). A contradiction. It then follows that there is a ko such that g(ko) > 0 and g(ko + 1) S O. Since 9 is continuous, there is Xo E (ko, ko + 1] for which g(xo) = O. Consequently., I(xo + 1) = f(xo). Analogous reasoning can be applied when g(O) < o. 1.3.13. The function / can be extended on rO, 00) so as to have period n. The extended function is denoted also by I. For an arbitrarily fixed k E {1,2,...,n -I}, define g(x) = I(x + k) - f(x), x  o. Now we show that there is Xo e [0, kn] such that g(xo) = o. Indeed, if g(O) = 0, then Xo = O. So suppose, for example, that g{O) > O. H also gU) > 0 for all j = 0, 1,2, . . . t 1m - k, then we would get f(O) < f(k) < f(2k) < ... < I(kn) = f(O). A contradiction. It then follows that there. is jo such that g(jo) > 0 and g(jo + 1)  O. Since 9 is continuous, there is Xo E (jo, jo + 1) for which g(xo) = O. Consequently, I(xo + k) = I(xo). Suppose first that Xo e [(1- l)n,ln - k] for some 1 < I < k. It then follows from the periodicity of f that f(xo) = j(xo - (I - l)n) and f(xo + k) = J(xo - (1- 1)1£ + k). Therefore we can take XI: = Xo - (l- l)n and xi = XO - (/- l)n + k. If Xo E [in - k, in], then Xo + k e [In, (I + l)n), 
1.3. Intermediate Value Property 151 and \ve have I{xo - (I-l)n) = f(xo) = I(xo + k) = I(xo -In + k). So we can take Xk = Xo - (I - l)n and x = Xo -In + k. It is not true that for any k E {I, 2, . . . , n - I} there are x" and x such that X/c -  = k and I(x/c) = f(x). In £3.(.1;, it is enough to consider the function J(X) = sin (x) for x E [0,4]. 4 It is easy to see that J{x + 3) #:- f{x) for all x e [0, 1]. 1.3.14. The following solution of this problem is due to our student Grzegorz Iichalak. Without loss oC generality we can assume that 1(0) = I(n) = O. The case on = 1 is obvious. So suppose that n > 1. We will consider first the case where 1(1) > 0. f(2) > 0,..., J(n - 1) > o. For k = 1,2, . . .,11 - 1, we set 91c(X) = I(x + k) - I(x). The function 91: is continuous on [0,11-k], and by asfiumptioD 9,,(0) > 0 und 9k(n k) < O. Consequently, there is Xi E [0, n - k] such that 9t(XIc) = 0, or in other words I(xk + k) = f(xlt). This shows that the assert.ion is true in this case. In an entirely similar manner we can see that it is also true if /(1) < 0, /(2) < 0,..., /(1& - 1) < O. Suppose now that /(1) > 0 (resp. 1(1) < 0), the numbers 1(1), /(2),... t fen - 1) are distinct and different Crom zero, and there is m, 2 < m < n -1, with f(m) < 0 (rcsp. f(m) > 0). Then there are integers kit k 2t .. . , k. 
152 Solutions. 1: Limits and Continuity between 1 and n - 2 such that 1(1) > 0,/(2) > O,...,/(k 1 ) > 0, l(k 1 + 1) < O,/(k. + 2) < 0,. . . ,/(k2) < 0, . . . I(k. + 1) < O,/(k. + 2) < 0,..., I(n - 1) < ° (or /(k. + 1) '> O,/(k. + 2) > 0,.. .,/(1) -1) > 0) (resp. 1(1) < 0,/(2) < 0,..., I(k.) < 0, ... ). Now reasoning similar to that in the proof of the first case shows that there are k 1 solutions in [0, kl + 1],  - k 1 solutions in [kl, k2 + 1], and so on. Clearly, in this case all these solutions must be distinct and therefore the assertion is proved. Finally, consider the case where there are integers k and m, o < k < m < n, with I(k) = f(rn). Suppose also that the numhnt l(k),/(k + 1),..., I(rn - 1) are distinct. It follows from the at that there are m - k solutions in the interval [k, m]. Next define { I(z) if 0 < x < k, II (x) = f(z + m - k) if k < x < n - (m - k). Clearly, II is continuous on [0, n - (m - k)] and 11 (n - (m - k) 11 (0) = 0. If 11 (0),11 (1),.. .,11 (n - (m - k) -1) are distinct, theJ the first part of the proof we get n - (m - k) solutions which toge' witb the m - k solutions obtained above give the desired result some of the numbers 11 (0),11 (1), . .. ,11 (n - (m - k) - 1) coin( the procedure-can be repeated. 1.3.15. Suppose, contrary to our claim, that the equation I(x) =- 9(X) has no solutions. Then the function h(x) = f(x) - g(x) would be either positive or negative. Hence O:F h(f(x}) + h{g(x» = 1(/(x» - g(/(x» + /(g(x» - g(g(x» = IZ(x) - 92(X). A contradiction. The following example shows that the assumption of continuity is essential: {  j(x) = 0 if xER\Q, if z E Q, 
1.3. Intermediate Value Property 153 g(x) = {  if x E Ii \ Q, if. xeQ. 1.3.16. Assume, contrary to our claim, that there are X1,X2 and Z3 such that Xl < X2 < Z3 and, for example, /(ZI) > /(X2) and /(X2) < /(X3). By tbe intermediate value property, for every u such that !(X2) < u < min{J{X1),J(X=)} there are 8 E (X"X2) and t E (X2, X3) satisfying /(8) = 1l = Jet). Since 1 is injective, 8 = t, contrary to the fact that Xl < 8 < X2 < t < Z3. 1.3.17. It follows from the result in the foregoing problem that / is either strictly decreasing or strictly increasin. (a) Suppose that I is strictly increasing and there is :to such that /(xo)  Xo. Let, for example, /(xo) > Xo. Then In(zo) > Xo, contrary to our assumption. Similar arguments apply to the case /(xo) < xo. (b) If / is strictly decreasing, then /2 is strictly increasing. Since jn(x) = x, we get f2n(x) = X, which means that the nth iteration of /2 is the identity. Therefore, by (a), 12(x) = x.. 1.3.18. Note that / is injective. Indeed, if I(xl) = l(z2), then -XI = /2 (XI ) = j2(X2) = -X2. Hence Xl = %2. It follows from 1.3.16 that if I were continuous, then it would be either strictly increasing or strictly decreasing. In poth cases 1 2 would be strictly increasing. A contradiction. 1.3.19. As in the solution of the foregoing problem, one can show that / is injective on a. Analysis similar to that in the solution of 1.3.16 shows that 1 is either strictly increasing or strictly decreasing. In both cases f2k, keN, is strictly increasing. Consequently, the integer 11 in the condition JU(x) = -x has to be odd. 1£ J were strictly increasing, In also would be strictly increasing, which would contradict our condition. So, 1 is strictly decreasing. Moreover, since I(-x) = J(/R(x» = /n(j(x» = -/(x), we see that lis an odd {unction (and so Is every IteratIon of I). Now we will show that f(x) = -x, x E lIt Suppose that there is an Xo such that Xl = I(xo) > -Xo, or in other words, -Xl < xo. 
154 Solutions. 1: Limits and Continuity It then follows that X2 = I(x) < I(-xo) = - J(xo) = -x) < xo. One can show by induction that if Xi = !(Xk-J), then (-l)"xn < XOt which contradicts our assumption that X n = In(xo) = -Xo. Similar reasoning applies to the case where J(xo} < -Xo. Hcnc:p. J(x) = -z for all x E JR. 1.3.20. Suppose t.hat / has a discontinuity at _ rfhen there exists a sequence {xn} convergent to x such that {f(x,,)} does not converge to I (x). This means that there c..xists E > 0 such that for ever)' kEN there is nk >.k for which I/(xn,,) - l(x)1 > e. So l(x nJr )  j(:£) + e > I(x) or l{xmJ $ f{x) - e < I(x). Assume, for example, that the first inequality holds. There exists a rational number q 8uch that J(;I:) +e > q > J(:I:}. Thu6 J(:r.n,,) > q > J(z} for kEN. By the intermediate value property of J there is ZI: between x and XIIi such that /(Zk) = q, which me.ans Zk E J-l({q}). Clearly, Jim Zt: = x. Since J-l({q}) is closed, x E ,-I({q}), and therefore k-+oo I( x) = q. A contradi(:tion. 1.3.21. To prove our theorem it is enough to consider the case when T > O. Set g(x) = J(x + T) - lex). Then there are two pos.;bilities. (I) There exists Xo > a such that g(x} is positive (or negative) for all x > 70- (2) There is no such Xu. In case (I), if, for example,!J is positive on (xo, (0), then the sequence {f(xo + nT)} is Inonotonically increasing. Since I is bounded, the. following limit exiSts and is finite: lim I(xo + nT) = lirn /(xo + (n + l)T). n-too R-tCO Therefore one can take X n == xo+nT. In case (2), by the intermediate value property of U, for every positive integer n > a there is X n > n such that g(xn} = O. 
1.3. Intermediate Value Property 155 1.3.22. Set +2 if - 3 < x <-1 - - , g(x) = -x if - 1 < x < 1, x-2 if 1<x < 3. and define f by the formula f(x) = g(x - On) + 2n for 6n - 3 < x  6n + 3, 11 E Z. 3 2 .J _____ ____________ 8 9 The function I has the desired property. There is no continuous function on 1R that attains each of its val- ues exactly two times. Suppose, contrary to this claim, that I is suc.h a function. Let XI, X2 be such that /(Xl) = !(X2) = b. Then I(x) =f:. b for x '# X., X2. SO either f(x) > b for all x E (Xl' X2) or I(x) < b for all X E (Xl,X2). In the former case there is one Xo E (Xl,%2) such that /(xo) = max{f(x): x E [Xl, X2]}. Indeed, if there were more points at which f at.t.ains its ma..ximllm on [Xl, X2], then there would be values of I assumed more t.han two tiDies in [Xl, X2]. Consequently, there is exactl)- one point x (outside the interval [Xh%2]) such that c = J(xo) = f(x) > b. Then the intermediate value property of f implies that every value in (b, c) is attained at least three times. A .contradiction. Analugous reasuning call be appJid t.u th case wh1I I(x) < b for x E (Xl, X2). 1.3.23. Assume that f is strictly monotone on each interval [ti-J, t;), where i = 1,2, ... , n and 0 = to < it < · · - < t n = 1. rl'he set Y = {j(td: 0 < i < n} consists of at most 1& + 1 el- elnents Yo, YI,.. - ,Ym. We can assume that Yo < Vt < ... < 11m. Put ZU = Yi, 0 5 i < In, and choose Z}'Z3,...,Z2m-1 so that 
156 Solutions. 1: Limits and Continuity Zo < %1 < %2 < %3 < . ... < Z2m-l < Z2m" Let Xi = {x e [0,1] ; f(x) = Zit}, X = Xu UX 1 U...... UX 2m = {Xh X 2,...... ,XN}, and let 0 = Xl < X2 < ..... < XN = 1. For 1 =::; j :S N, let kj denote the only element of the set to, 1,2,. . . , 2m} for which f{:£j) = Zits" Then kl and kN are even and kj - kj+l = :1:1, 1 < j < N. It then follows that the number N of elements of the set X is odd. Consequently, one of the sets Xi = /-l(Zi) has an odd number of elements. I , I I . " '" 4 ------ .--1---;--- -----r-1- I I I I I I I, I. ..' . ..' , I 'I' . -.' _ ______.___J__-J------L--- . I . I I. ..' 1'1'1 I. I." I. i · L I -:- I.J" --r---r--- ---'---I--- ---r-,--r- . . I ..' . ,.1' I , I I. I . " I', I . I . I I . ""1 ____._-.--------.---L t I ; I : I : I: I : r I XIS o I A2 .\] XI AS A6 X7 Xa X9X,OXUXazX.:f1 · :0 ------------------------------------------- ---- .1.3..24. We first show that there are at most countably many pr&r-- local extrema of I. Indeed, if Xo E (0,1) and /(xo) is a proper local maximum (minimum) of /, then there exists an interval (p,q) C [0,1] with rational endpoints such that I(x) < I(xo) (f{x) > I(xo» for z  Xo and x E (p, q). Consequently, our assertion follows from the fact that there are countably many intervals with rational endpoints. Since there are at most countably many proper local extrema of I, there is a y between 1(0) and 1(1) which is not an extremal value of I. Assume that 1(0) < 1(1) and put /-1 (Y) = {Xl, X2,... ,x n }, where Xl < X2 < ... < Xn. 1oreover, set %0 = 0 and Xn+l = 1. Then the function x t-+ 1(%) - Y is either positive or negative on each interval (Zi, Zi+t), and signs are different in the adjoint intervals. Note that 
1.3. Intermediate Value Property 157 the function is negative in the first interval and positive in the last one. Therefore the number of these intervals is even. Consequently, n is odd. 1.3.25-. Define the sequence {xn} by setting X n = In(xo). H tere is a term of this sequence which is a fixed point of I, then {xn} is constant heginning wit.h some value of the index n. Thus it. converges. H there is a term of this sequence which is its limit point, then by assumption, the sequence is as above. So it is enough to consider the case where no term of the sequence {xn} is its limit point. Suppose, contrary to our claim, that the sequence is not convergent. Then a = Jim X n < b = Urn Xn. 11-+00 n-tOQ Let Xio E (a, b). Since xko is not a limit point of {xn}, there is an interval (c, d) c ( a, b) which docs not contain any other term of the sequence. lvIoreover, there are infini tely many terms of the sequence in each of the intervals (-00, c) and (d,oo). H there are no terms of the sequence in (a, b), then we can take c = a and d = b. Now we define a subsequence {xn.} of {xn} in such a way that x nlt < c and Xn.+l > d for kEN. Therefore, if 9 is a limit point of {xn.}, then 9 < c and leg) > d. This contradicts our assumption that each limit point of the sequence is a fixed point of I. 1.3.26 [6]. By the result in 1.1.42, we know that lim J"O) = o(J) n-foOO exists. We will now show that there i$ Xo E [0,1] such that /(xo) = Xo +o:(f). H fez) 2:: x+o:(f) +e for all x E [0,1] and for some E > 0, then, in particular, /(0)  o(J) +e. We shall show by induction that for n E N, fn(O)  n(o:(f) +e). Indeed, setting r = 1(0) - [!(O»), we get /2(0) = /(/(0» = /([/(0)] + r) = [/(0)] + fer)  [f(O)] + r + o(f) + e = /(0) + a(/) + e > 2(0(/) + e). Similar arguments can be applied to prove that /"(0) > n(o(/) + e) implies In+l(o)  (n + 1)(0(f) + E). Now observe that if In(o) > n(o(/) + e), then er(/) > a(/) + E, a contradiction. In an entirely 
18" Solutions. 1: Limits and Continuity similar manner one can show that if J(x} < x + o:(f) - E for all x E [0,1] and Cor some e > 0, then a(J) $ o.(f) - E. A contradiction again. Consequently, by the intermediate value property there is Xo E [0,1] such that F(xo) = f(xo)-xo = o(f). In particular, if a(/) = 0, then Xo is a fixed point of f. On the other hand, if Xo is a. fixcd point of I, then o(J) = lim /11(;';0) = o. n -+ 'Xt ,.. 1.3.27. Let A = {x E [0,1] : f(x) > O}, s = inf A, and h = f + g. Since II is decreasing, we get h(s) 2: h(x)  g(x) for x E A. Since 9 is continuous, this implies that h(s) > yes). Consequently, 1(8)  o. It follows from our assumption that g(O} > h(O) > h(s) > g(s). By the intermediate value property of g, there exists t E (0,8] such that gCt) = h(s). Then h(t) > h(s} = g{t), whic:h givP.s J(t) > O. By the definition of s, we have t = s, which implies that g(s) = h(s), or equivalently, /(8) = O. 1.3.28. Note first that I is not continuous on III If I were continuous on IR. then by tbe result in 1.3.16 it would be strictly monotone, for example, strictly increasing. In this case, if f(xo} = 0, we would get J(x) > 0 for x> ::co, and f(x) < 0 for x < xo, which would contradict tbe assumption that J maps R onto [0,00). Similar analysis may be used to show that J cannot be strictly decreasing. Consequently, J is not continuous on IR. Suppose now, contrary to our claim, t.hat I has a finite num- bers of points of discontinuity, say, Xl < X2 < - -. < Zn.. Then I is strictly monotone on every interval (-00, Xl), (X),Z2),". , (x n , (0). Consequently, by the intermediate value property of I, !({ -00, Xl», l«xJ, 2:2), -.. ,/«x n , (0» are pairwise disjoint open intervals. Hence [0,00) \ (/((-OO,ZI» U  /«XI;,XI;+1» U f((Zn, 00») has at least n + 1 elements. Qn the other hand, the onI)- elements of Ii \ ( -00,:1:1) U U(Zk,:l:k+l) U (Zn, 00» ) k=1 
1.3. Intermediate Value Property 159 are XI, X2,..., Zn. Therefore f cannot be bijective, a contradiction. So, we have proved that J bas infinitcl}. many points of discontinuity. 1.3.29. We show that if I is a subinterval of (0,1) witb nonempty interior, then /(1) = [0,1). To this end, note that such an I contains a subinterval ( 2O ' t ) . So it is enough to sho\v that f ( ( 20 , : )) = [0, I). Now observe tbat if:r E (0,1), then either x - olr:!O with some m d ( -L .i.:!:! ) . h . .. - 0 1 2  1 If - m an no, or x E 21&0 , 2"0 WIt some J, 3 - , ,..., -. z - 2110 1 then J(x) = 1 and the value of J at the middle point of ( 20 ' :; ) is also 1 N if ( ...L ill ) f' · th tl . I ( I: k+l ) · · ext x E 2"0 , 2"0 lor some J, en lere 15 x E 2no , 2 n o such that f(x) = /(z'). Indeed, all numbers ill ( 2O '  ) have the sarne first no digits, and we can find x' in this interval for which all t.hc rClnaining digits are as in the binary expansion of X Since n Ea; 11m i= 1 n-.oo n n E ai lln l i=no+1 n-too 11. - no , we get fez) = I(x'). Consequently, it is enough to show that 1«0,1» = [0,1], or in other \vords, that for every y E [0,1] there is x E (0,1) such tbat J(:e) = y. It follows from the above that 1 is attained, for example, at x =  . To show that 0 is also attained, take x = .al a2 . .. t where { I if i = 2 k , k = 1, 2, . . . , Ui = 0 otherwise. Then k f(x) = lim 2 1: = o. k-t'X;; To obtain the value y = , \\.b(1.re p and q are co-prime positive 'I integers, take x = . 00 . . . 0 11 .. . 1 00 . . . 0 . .. 1 '-v--"'-v-'"''-v-'"' q-p p q-p where blocks of q - p zeros alternate with blocks of p ones. Then 1(:1:) = lim  == l!. q . Now our took .is to show that every irrational k-+CXa ... y E [0,1] is also attaiucd. It is wcll known (see, e.g., I, 1.1.14) that there is a sequence of rationals , \\here each pair of positive integers 
160 Solutions. 1: Limits and Continuity Pn and qn is co-prime, converging to y. Let x = . 00 .. . 0 11 .. . 1 00 .. . 0 ... t  .......  91-PI PI Q2-P2 where ql - PI zeros are followed by PI ones, then q2 - P2 zeros are followed by P2 ones, and so OD. Then I( ) I . PI + 1>2 + .. . + Pn 1 . Pn x = 1.Dl = lD1 - = v. n-.oo ql + CJ2 + . · · + qn n-.oo qn Since lim qn = +00, the second equality follows immediately from n-+oo the result in I, 2.3.9 or £rom the Stolz theorem (see, e.g., I, 2.3.11). 1.4. Semi continuous Functions 1.4.1. (a) Set sup inf{f(x) : x e A, 0 < Ix - zol < 6} = o. Assume first 6>0 that a is a real number. We shall show that a = lim f(x). By Z-+ZO the definition of supremum, for every 6 > 0 (i) inf{f(x): :e e A, 0 < Ix - :Co 1 < 6} < a, and for every E > 0 there is 6- such that (ii) w{f{x): x e A, 0 < Ix - %01 < 6.} > a - E. By (n), (iii) fez) > a - e if 0 < Ix - zol < 0- - Now let {x n } be a sequence of points of A different from zo- If the sequence converges to %0, then, beginning with some value of the index n, 0 < IXn - ol < 6-. Therefore f(zn) > a-E. If {J(Xn}} converges, say to'll, then we get II  a - E. and consequent.1y, lim I(x)  a. To show that also lim f(x)  a, we will use (i). Z-+ZO %-+%0 It follows from the definition of infimum that, given el > 0, there 
1.4. Semicontinuous Functions 161 exists x. e A such that 0 < Ix. - xol < 6 and J(x.) < a + £t. Taking 6 =  .. we get a sequence {x} such that o < Ix: --xol <.!. and j(x:> < a + CI. n Combined with (ill), this gives a - e < f(x) < a + et. Without loss of generality \ve may assume {f (x;)} is convergent. Theo its limit is less than or equal to a+£t. It follows from the arbitrariness of £1 > 0 that lirn f(x) < a. z-t-zo H a = +00, then, given /vI > 0, there is 6. such that iof{f(x) : :z: E A, 0 < Ix - xol < tS-} > M. Hence if 0 < Ix - 2:01 < 6-, then f(x) > !vI. Consequently, if {Xn} converges to XO, then, beginning \vith some value of the index n, f(:en,) > M_ Thus lim /(x,,) = +00, which means that n-ioOQ lim /(x) = Urn f(x) = +00. Finally, if a = -00, then for any z-t-zo z-t:z:o 6> 0, inf{f(x) : x e A, 0 < Ix - xol < 6} = -00. Therefore there is a sequence {x} convergent to Zo such that fun /(x;) = -00, which gives Jim f(x) = -00. n-i-OQ :t-i-ZO (b) The proof runs as in (a). 1.4.2. The result is an immediate consequence of 1.1.35 and the pre- ceding problem. 1.4.3. It follows from the result in the preceding problem that, given E > 0, there is 6 > 0 such that o  yo - iuC{J(;c) : ;c e A, 0 < Ix - xol <: 6} <. By the definition of infimum this is equivalent to the conditions (i) and (ii). By 1.4.2 (b), ii = lim f(x) if and only if for every e > 0 the - Z-'zo following two conditions are satisfied: (I) There is 6 > 0 such that f(x) < ii + E for all z E A from the deleted neighborhood 0 < Ix - %0 I < 6 
162 Solutions. 1: Limits and Continuity (2) For every 0 > 0 there is x' E A from the deleted neighborhood 0< Ix' - %01 < 0 for which J(x') > U - e. 1.4.4. (a) B)I' 1.4.2(a), lim f(x) = -00 if and only if for any 6 > 0 Z-I'ZO inf{j(x) : x e A, U < Ix - xol < 6} = -00. This means that for any 6 >.0 the set {fex) : x e At () < Ix - %01 < 6} is unbounded below, which gives the desired result. (b) The proof runs as in (a). 1.4.5. Let {6n} be a monotonically decreasing sequence of positive numbers converging to zero. It follows fram 1.4.2(8.) that 1= Hln ine{/(x): x E A, 0 < Ix - xoJ < 6n}. noo For a reali, this is equivalent to the following two condit-ions (1) For n E N there exists k,. E N such that 0 < Ix - %0 I < 6k implies f{x) > 1-  for k > k n . (2) For n E N there exist k,. > n and Xk" E A such that 0 < IXI: - %01 < OleA and I(Xk n ) < I +  . Conse(luently, there exists a sequence {Xk q } convergent to Xo such that lim f{Xk a ) = I. n-too If lint J(x) = -00, then, by 1.4.4(a), for any n e Nand 6 > 0 z-tzo there is X ra e A such that 0 < IXn - xol < 6 and f{x n ) < -no Therefore lim X n = :'1:0 and lim I(x,.) = -00. n-.oo nOQ H liTn /(z) = +00, then the existence of {Xn} follows inlDledi- z....zo ately from the definition. 1.4.6. Thp result follows jmmP.CIiau1y from It 1.1.2 anct from 1.4.1. 1.4.1. It is enough to apply It 1.1.4 and 1.4.1. 
1.4. Semicontinuous Functions 163 1.4.8. Note tbat (1) (2) inf (f(x) + g(x» > inf f{x) + inf g(x), zeA zEA zEA Stlp(/(x) + g(x»  sup f{x) + sup g(x). zeA ;rEA zEA Indeed, for x E Ar f(x) + o(x) 2: il1f f(x) + illf o(x), zEA :l:EA which implies (1). Inequality (2) can be proved analogously. We fU1)t J1ow that (3) Jim J(:c) + lirn g(x)  !!ill (f(x) + g(x». :l:-+ZO z-+zo z-+zo By (1), we get inf{f(x} + g(x) : X e A, 0 < Ix - xol < 6} > inf{/(x): x E A, 0 < Ix-xol < 6} + inf{g(x) : z E A, 0 < Ix - xol < 6}. Passage to the linlit  6  0+ and the result in 1.4.2(a) give (3). The inequality (4) Iin1 (f(x) + g(x» < iii'i1 f(x) + iIiii g(x) z-tzo 2:-.Zo z-u:o can be proved in an cntirely sinwar manner. Furthermore, it follows from Probleul1.4.6 and (3) that lim f(x) = !!!!! (/(x) + O(x) - g(x» Z-+ZD -+ZO  !!ill (/(x) + g(x» + Jim (-g(x» z-+zo Z-+ZO = fun (f(x) + g(x» - iIiii 9(x). z-t-zo z-t-zo One can prove, in much the same way, that lirn f(x) + iIiii g(x) < iiiii (f(x) + g(z». z-t-zo Z-+Zo z-t-zo 
164 Solutions. 1: Limits and Continuity To show that the inequalities can be strict, consider tbe functions defined as follows: { sin! if % > 0, fez) = 0 z if x < 0, { 0 if x  0, g(;c) = sin! if x < 0 z . For %0 = 0 the given inequalities are of the form -2 < -1 < 0 < 1 < 2. 1.4.9. Observ first that if f and 9 are nonnegative on A, then (1) (2) inf (/(z) . g(z»  inf f(x). inf g(z), zEA zEA zeA sup(f(:t) · g(:c»  sup fez) .. sup g(x).. zEA zEA zEA Th .n8L of tbe pl"OOr l'UI15 as in the solution of tbe fOl"egaing problem. To see that the given inequalities can be strict, consider the functions given by setting f(x) = { 2 8in 21;' +1 if if : Z > 0, x $0, { 3 if %  0, g(x) = I if z < o. SiD 2 -!- +1 For Xo = 0 the given inequalities are of the form t < 1 <  < 3 < 6. 1.4.10. We have 1im fez) = iiiii f(x) = lim fez). So, by 1.4.8, Z-'zo z-+-zo z-+zo lim I(x) + lim 9(a;)  lim (/(x) + g(x» S  J(x) + lim g(x). z-+zo z-+zo %-+-zo %-+zo :I:-I>ZO Therefore !!m (f(x) + g(x» = lim f(z) + fu!! g(x). Z-+ZO -+ZO %-+zo The other equalities can be proved analogously. 
1.4. Semicontinuous Functions 165 1.4.11. H " = I or " = L, then the assertion follows immediately from 1.4.5. So assume that" e (I. L). Then by 1.4.5, there exist sequences {x'n} and {x:} both converging to a and such that lim f(x') = l and lim f(x") = L. n-+oo n n-too n It then follows that f(x'n) <" < f(x) beginning with some value of the index n. Since f is continuous, it enjoys the intermediate value property. Hence there is Xn in the interval with the endpoints z and x' for which f(xn} =. Since {x} and {z'} converge to a, the sequence {xn} does also. 1.4.12. The function is continuous at every kn with k e Z (see, e.g., 1.2.1). Clearly, - { sinxo Jim lex) = %-+ZO 0 if sinxo > 0, if sinxo < 0, d { 0 if sin Xo > 0, lim J(x) =. if . < 0 %-4zo sm Xo sm Xo _ . Consequently, f is upper semicontinuous on the set (Qn U (2k1r, (2k + 1)11'») u (a \ Q) n U[(2k -1)lI',2k1r J ) keZ kez and lower semicontinuous on (Qn U«2k -1)1I',2k1r» ) U (iii \ Q) n U[2k1r,(2k+ 1)11'] ) . kEZ kez 1.4.13. We have { -1 liiii f(x) = 0 , Z-+:l:O 0 if Zo < -lor Xo > 1, if Xo E [-1,1], and { o lim f(x) = 2 Z-U:o Xo -1 if Xo E [-1,1]. Thus f is upper semi continuous at each irrational in (-00, -1 )U(I, 00) and at each rational in the interval [-1,1]; f is lower semi continuous if %0 < -1 or :Co > 1, 
166 Solutions. 1: Limits and Continuity at each rational in (-00, -1] U [1, (0) and at each irrational in the interval (-1,1). 1.4.14. The function f is continuous at zero and at each irrational (see, e.g., 1.2.3). Assume tbat 0  %0 = i, where p E Z and q e N are co-prilue. Then J(xo) =  and Jim f{x) = 0 < _ '1 1 . Hence J is .. z-tzo upper m1conti.nuous on III 1.4.15. (a) The function I is continuous at zero and at each positive irra- tional (see, e.g., 1.2.3). Assume that Xo is a negative irrational. Then lim J(x) = Ixol = J(xo). Therefore f is upper semicon- z....zo tinuous at zero and at each irrational. H:to =  > 0, then lim f(x) = ; >  = f(xo). This weans that I is lower semi. z-tzo continuous at eu<>.h positive rational. If Xo =  < 0, then - p P linl J(x) = -- > = J{xo) %-'zo q q+ 1 and lim J(x) = p < Ii = J(xo). Z-foZo q q + 1 So J is neither upper nor lower semicontinuous at negative ratio- nals. (b) Note that for x E (0,1]7 !!!!! J(t) = -x < J(x) < % = iiffi J(t). t-.z t....z Thus f is neither upper nor lower semicoDtinuous on (0 7 1]. 1.4.16. (a) If Xo E A is isolated ill A, then the a.c;sertion is obviously true. If :to e A is a limit point of A, then the assertion follows from the fact that { a lim I(x) if a > 0, -:-- z.....zo Urn (I./(x) =. . o£-'zo a hm J(x) if a < o. %-+%0 
1.4. Semicontinuous Functions 167 (b) Assume that Xo is a limit point of A and, for example, J and 9 are lower selnk,ontinuous at Xo. Then, by 1.4.8, lim (f(x) + g(x»  !!!!! fex) + Urn g(x) > I(xu) + 9(Xo). z-tzo ;l:-ioZO z-+zo 1.4.17. Assume, for example, that the In are lower senucol1tinuous at xo. Since sup /n  /n for n e N, we get nEN 1im sup fn(x)  lim In(x)  In(XO) for n E N. zzo nEN z....zo Consequently, lirn sup In (x) > sup /n(ZO). zzoneN neN 1.4.18. It is enough to observe tbat if {In} is an increasing (resp. de- creasing) sequence, then lim !n(x) = sup In (x) (resp. lim /n(x) = n....oo 'JeN n-too in£ /n(x» (see; e.g., I, 2.1.1) and use the result in the foregoing prob- nEN lern. 1.4.19. By 1.4.1 we have /J(x) =max{/(z), nm f(z}} :-foZ = inf 5Up{J(Z) : z E A, Iz - xl < o} 6>0 = lim sup{f(z): z E A, Iz - xl < 6}. 6.-.0+ Similarly, 12(x) = lim inf{J(z): Z E A, Iz - xl < o}. 6-.0+ Hence 11 (x) - 12(x) = lim sup{f(z): Z E A Iz - xl < 6} 6-+0+ - Urn inf{J(u): U E A, lu - %1 < 6} 0-+0+ = lim sup{f{z) -f(u) : z,u e A, Iz - xl < 6, lu -xl < 6} 6-.0+ = lim sup{lJ(z) - f(u)1 : z,.u E A.lz - xl < 6, lu - xl < o} 6-+0+ = o,(x). 
168 Solutions. 1: Limits and Continuity 1.4.20. Let x be a limit point of A, and let {xn} be a sequence of points in A converging to z. Set 6n = Ixu -zl+. Then, Iz-znl < 6n implies Iz - xl < 26n. Consequently, see the solution of the preceding problem, /2(:£") = lirn inf{/(z): z E A, Iz - x,,1 < 6n} n-too  inf{j(z) : z E At Iz - xii <: 6k}  inf{j(z) : z E A, Iz - xl < 2 d k}. Passage to the limit as k -+ 00 gives Urn f2(Xt) > /2(X). It then k-.oo follows that lim 12(z) > 12(x), and therefore the lower semicontinuity z-+z of /2 is proved. In an entirely similar manner one can show that /1 is upper Senllcontinuous. Now, by the result in the foregoing problem. oJ(x) = 11 (x) - 12(x), which together with 1.4.16 proves tbe UJ semicontinuity of OJ. 1.4.21. We will prove our statement for lower semicontinuous ft tions. Assume first that the given condition is satisfied. Then a < f(xo} there is 6 > 0 such that I(z) > a whenever Ix - zol <  {Zr.} is a sequP.Dce of points in A converging to Xo. then IXn -xol for sufficiently large n. Hence f(xn) > a, which implies Urn j(x n....oo a. Because of the arbitrariness of a we get lim f(x) > f(xo). r z....zo assume that I is lower semicontinuous at Xo and, contrary to claim, the given condition is not satisfied. Then there is a < I such that for any n E N there exists X n E A for which (zn - :rol < 6n =  and f(xn)  a. Thus the sequence {xn} converges to Xo and Urn f(xn) S a < f(xo}t a contradiction. n-too 1.4.22. Suppose that for every a E ]It the set {x E A: f(x) > a} is open. Let Xo be an element of A and take a < /(xo). Then there is 6> 0 such that (xo -6,xo+6) C {x E A: f{x} > a}. It then follows by the result in the foregoing problem that I is lower semicontinuous. :)uppose now that / is lower semicontlnuous on A. We shall show that the set {x E A : fex)  a} is closed in A. Let {xn} be a sequence of points in this set converging to z. Then I(xn) < a, and 
1.4. Semicontinuous Functions 169 consequently, I(x)  lim I(xn} < a, which impUes that z is also an n-too element of {:c e A ; J{:c)  a}. So we have proved that this set is closed, or equivalently that its complement is open in A. 1.4.23. Assume that 1 is lower semicontinuous on Ill, and set B = {(x,y) E JR2 : y  I(x)}. Our task is to show that B is closed in Ji2. Let {(Xn,Yn)} be a sequence of points in B converging to (xo,Yo). Then Yo = lim Yn  lim I(x n )  fu!! f(x)  I(xo). n-tOO noo Z Hence (Xo, Yo) E B. Assume now that B is closed and f is not lower semicontinuous at an Xo E R. Then the set Be: = {(x,y) E JIt2 : y < f(x)} is open in Ji2 and there exists a sequence {x n }, X n t: Xu, converging to Xo and such that y = lim I(:e n ) < /(:£0). Take 9 such that'll < 9 < I{zo). Then n-tooo (xo,O) is in B C . Hence there is a ball centered at (xo,g) contained in BC. This means that for sufficiently large R, (xn, g) are in Be:, or equivalently, 9 < I(xn). Therefore 9 < !It a contradiction. Recall that 1 is upper semicontinuous on IR if and only if - / is lower semicontinuous on IR. So J is upper semicontinuous on R if and only the set {(x,y) E JR2 : !I =:; f(x)} is closed in a 2 . 1.4.24 [21]. We show first tbat f is lower semicontinuous if anc1 only if the functiun g(x) =  arctan/(x) is luwer sewicuntwuuus. To thiH end we use the characterization given in 1.4.20. Suppose that f is lower semicontinuous. To prove that 9 is also lower semicontinuous it is enough to show tbat for every real a the set B = {x E A : ; arctan/(x) > a} is open in A. Clearly, if a  -1, then B = At and if a  1, then B = 0. H lal < 1, then B = {:c E A = I(x) > tan ( ia)}; so it is open by assumption. Suppose now that 0 is lower semicontinuous. Then {x E A : O(x) >  arctan a} is open for every real Q. Consequently, the set {x E A : f(x) > a} is open. For n e N, a E At define &Pa,n by set.tiug fPo.n(x) = o(a) + nix - ai, :c e 1R, 
170 Solutions. 1: Limits and Continuity and put On (X) = inf A CJ'/1.n(X). BE Obviously, Yn(X) < 9n+l (x) for x E Ii and gn(X) < «Pz,n(X) = g(x) for x E A. Hence for each x e A the sequence {On (x)} is convergent. Now we show that the functions Un are continuous on 1ll Indeed, for ,;'C' E 1R, ICPo,n(x) - V'a,n(x')1 S nix - x'i. It then follows that 'Po,n(X') - nl:t - %'1 < <Pa.n{X) $; <Pa.n(X') + nix. - x'i. C01J:)t,ut:uLly, 9n(X') -7&lx - z'l < Yn(x) < 9n(x') + nix - x'l, and therefore continuity of OR is proved. It follows from the above that for:c e A, Jim 9n(Z) < 9(:Z:)' Our tnsk is to show that lim g",(:z:) > )\-700 n-a.oo g(z). Let x E A and let Q < g(x). Since 0 is lower sernicontinuous at z, there is D > 0 such that g(a) > Q if Ix - al < 6. Hence (1) CPu.n(X) 2= g(a) > Q Cor Iz - al < o. On the other hand, (2) CPa,R(X) > -1 + no for Ix - 01 2= 6, whir.h r.omhinP.CI with (1) givp.s gn(X) = l «Pa,n(x) 2: min{Q, -1 + n6}. Therefore On(z) 2= Q for sufficiently large n, and consequently we get Jim 9n(Z) > (t. Finally, upon parn-,age to the limit os (t ) g(z), we ,,-too obtain Jim g",(x) 2= 9(X). n-+co 
1.5. Uniform Continuity 171 1.4.-25. It follows from t.he theorem of Baire (see the foregoing prob- lem) that there are a decreasinK sequence {f n} and an increasing sequence {On} of continuous functions converging on A to f and 0, respectively. Set CPI (x) = /1 (x), 1jJ. (x) = min{c,ol (X), 9. (x)), . . . CPn(X) = max{Wn-I(X),/n(x)}, 1/ln(x) = Inin{CPn(x),gn(x)}. Then {fPnl is decreasing, because the inequalities 1Pn < tpn and f n < CPn hnply CPn+I = max{Wn,fn+l}  max{1,9,ufn}  max{fPrufn} = CPn. Similarly, one can show that {1b,.} is increasing. Observe now that the sequences of continuous functions {fPn} and {Wn} both converge, say to cp ;)ncl ,/,, rp..pp.r.t.ivp.ly. One rAn show that. rp(x) = max{ ?b(x) , f(x)} and 1,9(x) = m1n{rp(x),g(x)} (see, e.g., I, 2.4.28). So if cp(x) :F'ljJ(x) for some x, then cp(x) = I(x); and since f(x) < g(x), we have also t/J(x) = /(x), a contradiction. Consequently, tbe sequences {c,on} and {1Pn} have a common limit, say h, such that f(x) < hex) < g(x). By 1.4.18, h is lower and upper semicontinuous, lltnce continuous. 1.5. Uniform Continuity 1.5.1. (a) The function can be continuously extended on [0,1]. Therefore f is uniformly continuous on (0,1). (b) Note that for n E N, 1( 21r ) -/( 2n1r\ ) =1 although 1 27r - 2n+.;. 1 can be arbitrarily small. Consequently, the function is not uniformly continuous on (O,I). (c) Since there exists a continuous extension of f on 10,1], the func- tion f is uniformly continuous on (0,1). 
172 Solutions. 1: Limits and Continuity (d) We have 1 (iik) -I ( In(:+ 1» ) I = In - (n+ 1)1 = 1 and I I':" - In(+l) I n o. Hence f is not uniformly continuous on (0,1). (e) Since lim e- = 0, the function can be continuously extended z-tO+ OD [O,IJ. Thus I is uniformly continuous on (0,1). (f) The function is not uniformly continuous on (0,1) because f ( .-!..- ) - f ( 1 ) = e + e n+.. > 2 n (;; N. 2n1r 2n1T + 1r J (g) To see that the function is not uniformly continuous on (0 .., note that I (in) -/( enl ) -1. (h) Observe that J () -I ( 2n 1 ) 1 1 =OOS- 2 +cos 2 1 --t 2. n n+ n-+oo So, the function is not uniformly continuous on (0,1). (i) As above, one can show that the function is not uniformly I tinuous on (0,1). 1.5.2. (a) We will show that 1 is uniformly continuous on [0,00). Indeed, in view of the inequality I VXi - JX2I < V IZl - x21 Cor 3:1, X2 E [0,00) we have IXI - X21 < 6 = e 2 implies IVXi - VX21 < E. (b) Note that j(2n1r) -I ( 2n1r + .!. ) 1 -. 21T. n n-too So, f is not uniformly continuous on [0, (0). 
1.5. Uniform Continuity 173 (c) Since I sln 2 Xl -sln 2 x21 = 1 sIn Xl -slnX21.1 SlnXl +slnx21  21 X l -x21, the function is uniformly continuous on [0,00). (d) The function is not uniformly con tinuous o n [0, 00) because J( ,fiii1;) - J ( V2 mr+ ;) = 1 although 1 2n1r - 2 n1r + ; I  o. n....oo (e) The function is not uniformly continuous on [0,00). Indeed, it follows from the continuity of tIle logarithm function that linn -In(n + 1)1 = In ( 1 + ! ) -+ o. n n-.oo Moreover, I/(ln n) - j(ln(n + 1»1 = 1. (f) One can show , as in (d), that the function is not uniformly con- tinuous on [0, 00). (g) Since 1 . ( . ) . ( . ) 1 21 . sin XI - sin %2 1 I SID SlDXl - SIn smX2 :5 sm 2 :5 Xl - X2 , f is uniformly continuous on [0,00). (h) Note that j1(2mr + 27J - !(2mf>1 . (2 .1 1.1 ) . 1 = sm n1i'Sln-+-sln-  sm . 2n1i' 2n1r 2n1i' n....co Consequently, the function is not uniformly continuous on [0,(0). (i) Observe that I sin VXi - sin v%"1 _ 2 . v'Xi - v'X2 v'Xi + v'X2 1 < I r::: J:::t - sm 2 cos 2. - Y%l - y%21- Now reasoning as in (a) proves the uniform continuity of I. 
174 Solutions. 1: Limits and Continuity 1.5.3. We will show that lirn I(x) exists. By uniform continuity, %-+Q+ given F. > n therp. exigt.s 8 "> 0 such that. 1/(9:1) - !(x2)1 < F. \vhenevef IXI - $21 < 6. Clearly, if a < XI < a + 6 and a < X2 < a + 6, then IXI - x21 < 6. It follows from the Cauchy theorem (see, e.g., 1.1.37) that the left-hand limit of / at a exists. In an entirely similar manner one can show also that the right-hand limit of f at b exists. 1.5.4. (a) It follows directly from the definition of uniform continuity that the sum of two uniformly continuous functions is also uniformly continuous. (b) IT f and 9 are uniformly continuous on a finite interval (a, b), then by the result in the foregoing problem the functions can be continuously extended on [a, b]. Thus I and 9 are bounded on (a, b). Consequently, the uniform continuity of 19 on (a, b) follows Croln the inequality 1/(x1 )g(XI) - /(x2)0(x2)1 < I/(xl)lIg(xl) - g(x2)1 + ly(x2)11/(xl) - /(x2)1. On the other hand, the functions I(x) = g(x) = x are uniformly continuous on [a, 00) but f(x)g(x) = x 2 is not uniformly contin.. uous on this infinite interval. (c) By (b), X -4 f(x) sin x is uniformly continuous On (a, b). The func:tion need not be uniformly continuous on [a, (0), as the ex- ample in 1.5.2(b) shows. 1.5.5. (a) Given E > O. there are J :> 0 and 6 2 > 0 sl1ch that I/(Xl)- f(h) 1 < ! if 0 < b - Xl <  and 1/(x2) - /(b)1 <  if 0  X2 - b < . Setting 6 = min{61'), we get (1) I/(Xl) - /(x2)1 < E if I X I - x21 < . For XI, X2 E (a, b) or Xit X2 E [b, c), (1) is clearly satisficd with some positive 0 > o. 
1.5. Uniform Continuity 175 (b) No. Let A = N and B = {n +  : n e N}, and consider the function I defined by J(xJ = {  if x e A, if z E B. 1.5.6. II ! is .constant, then it is uniformly continuous on III H J is a nonCOlli»1.ant periodic fWlction, then its fundamental period T exists (see 1.2.23). Clearly, / is Wliformly continuous on each interval [kT y (k + l)T), k E Z. So, as in the solution of 1.5.S(a), one can show that I is uniformly continuous on III 1.5.7. (a) Set lim lex) = L and Jim lex) = I. Then, given E > 0, there Z+OO Z- is .4 > 0 such that I/(x) - LI <  for z  A and I/(z) -II <  for x $ -il. This ullplies that if Xa,X2 e [A, (0) or %1,X2 e (-00, -.4], then I/(xJ) - !(x2)1 < E. Obviously, f is uniformly continuous on [-A, A]. Finally: as in the solution of 1.5.5(a) one can sho\',. that / is uniforrnly continuous on IR. (b) The proof runs as in (a). 1.5.8. It is enough to apply the result in the foregoing problem. 1.5.9. lim j(z) need not exist. To see this consider the fun(:tion in z-too 1.S.2(c). rrbe llinit limo f(x) exists (see 1.5.3). Z-.OT 1.5.10. Assume that I = (a, b) is a bounded interval and, e.g., J is Inonotonically increasing: Then, as in 1.1.35 J one can show tbat lim f(x) = inf J(x) and 1im J(x) = sup I(z). Consequently, z-.o+ zE(a.b) ;r:-t>b- zE(a,6) I can be continuously extended on [a, b). So it is uniformly continu- ous on (a, b). H the interval I is unbounded, then the limits lim I(x) z-t>oo and/or linl f(x) exist and are finite. By 1.5.7 I is uniformly con- z-+-oo tinuous on I. 
176 Solutions. 1: Limits and Continuity 1.5.11. No. The following function is uniformly continuous on [0,00) but the limit lim I(z) does not exist: oo x for x e [0, 1], -x+2 for x e [1, 2], /(:») =: . . . z - n(n + 1) for Z E [n(n + 1), (n + 1)2], -z + (n + l)(n + 2) for z E [en + 1)2, (n + l)(n + 2)], . . . 3 ---------------------------------. . . . . . . . . . . 1234 S 6 9 1.5.12. Let e > 0 be nrbitrarily:&xed. Choose 6 > 0 so that for x, X'  0 Ix - x'i < 6 implies I/(z) - /(x')1 < . Let Xl,X2,... ,Xi be points in the interval [OJ 1] such that for any :c e [0, 1] Lhe i:s x, for which l:c -.d < o. Since fun f(Xi + n) = 0 n-+oo for i = 1,2,..., k, there is no such that I/(xi + n)1 <  for n > no and for i = 1,2,..., k. Suppose x > no + 1 and set n = [x]. Then there is Xi such that Ix - (n + xi)1 < 6. It then follows that 1/(x)1  I/(x) -/(zi + n)1 + I/(xi + n)1 < E:. 1.5.13. By uniform continuity of Ion [1,00) there exists 6 > 0 such that I/(x) -/(x')1 < 1 if Ix - %'1 :5 6. Any x  1 can be written in the form x = 1 +n5 +r, where n e NU {OJ and 0 S r < 6. Hence 1/(x)1 < 1/(1)1 + I/(z) - /(1)1 < 1/(1)1 + (n + 1). 
1.5. Uniform Continuity 177 Dividing by x gives 1/(:Ii) I < 1/(1)1 + n + 1 < 1/(1)1 + 2 = At/. x - l+ncS+r - 0 1.5.14. As in the solution of the foregoing problem, we find 6 > 0 such that if z = n6 + r.. then for any u  0 I/(x + u) -f(u)1 < n + 1. Therefore If(x + u) - Je u)1 < n + 1 < 2 = k/. x+l - l+no+r - 0 1.5.15. Assume {xn} is a Cauchy secluence of elements in A; that is, given 6 > 0 there is no E N such that IXn - Xml < 6 for R,m 2: 1l(). By uniform continwty of /, given E > 0 there is 6£ > 0 such that I/(x n ) -/(xm)1 < E if IXn - xml < 6£. Thus {/(x n )} is a Cauchy sequence. 1.5.16. Assume, contrary to our claim, that I is not uniformly con- tinuous on A. Thus there is E > 0 such that for any positive in- teger n there exist X n and x in A such that IXn - :c1 <  and I/(xn) -/(x'n)1  E. Since A is bounded, there is a convergent sub- sequence {xn,,} or {x n }. It follows from the above that the sequence {xl n .} is convergent to the same limit. Thus the sequence {z,,} with terms X nl ,XI 'X n2 ' Z2' . . . J X n ., x. ' . -. is convergent, and therefore it is a Cauchy sequence. But I/(xn..) - /(x.)1  £, and so {/(Zk)} is not Cauchy. A contradiction. The boundedness of A is essential. To see this, consider the function I(z) = Z2 on (O,oo). 1.5.17. The necessity of the condition follows immediately from the definition of uniform continuity. Now assume that the given condition is satisfied and f is not uniformly continuous on A. Then there is e > 0 such that for any positive integer n there exist Xn and Yn in A such that IXn - Vnl <  and I/(xn} - /(Yn)  e, a contradiction. 
178 Solutions. 1: Limits and Continuity 1.5.18. No. Define J by setting 1 2' 1- n 2 I(x) = n 1 x-n+- n n+2 ( 1 ) + 2 - t 1l+1)(n-l ) x-n- n ii ---------------- ') t t 1- ] -----",------""---'-..- 1 I , I s for z E (0,2], for x = n, n > 2, for x = n +  , n  2, for x E (n, n +  ), n  2, for x E (n + *, n + 1), n > 2. 2 2  3 3+1 44+ 1 4 s The function f is continuous on (0.00), lim J(x) = 0 and lim J z-too z.-.O+ =  . It then follows from 1.5.7 that J is uniformly continuous - (0,00). But lim J (n + ) = 2. n-tco I(n) 1.5.19. By the continuity of J at zero, given c > 0 there is 6 > 0 such that I/(x)1 < E. for Ixl < 6. Hence the subadditivity of 1 implies that, for x e R and It I < 6, /(x + t) - I(x) < f(t) < f:: and J(x) -/(x + t) ::; !( -t) < c. Consequently, I/(x + t) -/{x)1 < c., which proves the uniform conti- nuity of J on III 1.5.20. Observe that wI is mODot.omcally int.Tecu.ing on (Ooo). Thus (see 1.1.35) Urn CIJ/(o) = inf wJ(o) > 0.. 6...0+ 6>0 
1.5. Uniform Continuity 179 H Urn wf(6) = 0, then given E > 0 there is 6 > 0 such that w/(6) < E. 6O+ Consequently, if IZl - .2:21 < 6, then If(z1) -f(zz)1  w/(6) < €. This means that J is uniformly continuous on A. Assume now that 1 is uniformly continuous on A. Then given E > 0 there is 6 0 > 0 such that 1/(3;1) - /(x2)1 < E for )X1 - z21 < 60. Hence 6rg+ w/(6)  W/(Do)  £. The arbitrariness of e > 0 yields lirn wJ(6) = O. 6-.0+ 1.5.21. Clearly, it is enough to prove that (b) implies (a). Let £ > 0 be arbitrarily fixed. Since 1 9 is continuous at zero, there is 6 1 > 0 such that Ixl < 6 1 implies If(x)g(x) - I(O)g(O)1 < . Thus if IXII < 1 and I X 21 < 6., then If(x) )g(XI) - l(x2)g(x2)1 < €. Fur IZII  6. we have I/(xl )g(X1) - /(x2)0(z2)1 Ig(zl )1  Ix. I I X lll/(Xt) - /(x2)1 + 1/(z2)lIo(zl) - 0(x2)1. Consequently, I/(xl)9(Xl) - /(x2)g(x2)1 19(zl )1 < lXI' (li x al/(xl) - I X 21/(x2)1 + 1/(X2)lI z 2 - XJ I) + 1/(x2)lIo(xl) - g(x2)1. This combined with the result in 1.5.13 gives If(x. )y(x.) - f(x?)y(x?)1 < kIUx.ll(xl) - Ix?I/(x?)1 + !vI Lixi - x21 + Llo(Xl) - g(x2)1. where M = sup { I)I : Ixl  6 1 } . L = m8X {sup{lj(X)I: Ixl :S 6d,sup { IXIIIx)1 : Ixl > 6. }} . 
180 Solutions. 1: Limits and Continuity Thus the desired result follows from the uniform continuity of g(z) and Ixl/(x) on R. 1.5.22. Suppose that! is uniformly continuous on I. Then, given E > 0, there is 6 > 0 such that (i) IXI - X2) < 6 implies I/(xl) - !(x2)1 < E. We will prove that, given E > 0, there is N > 0 such that for every Xl, %2 e I, Xl :F %2, (ii) I(xl) -/(x2) 1 > N implies I/(xl) -/(x2)1 < e. %1 - %2 Clearly, this implication is equivalent to 1/(%1) -/(%2)1  e implies I /(Xl) -/(x2)  N. Xl - %2 By (i), if 1/(ZI) - /(%2)1  €, then I X 1 - %21 > D. Without loss of generality we may assume that Xl < X2 and I(xl) < I(X2). Since l(x2) -/(xl)  e, there are TJ E [e,2e] and a positive integer Ie such that /(X2) = I(xl) + k'IJ. Now the intermediate value property of I on the interval [Xl, X2] implies that there are Xl = ZO < %1 < ... < Zk = X2 for which I(z;} = l(zl) + iTJ, i = 1,2,..., k. We have If(z;} -/(zi-1)1 = " 2= E, so IZi - Zi-ll  D. Thus IXI - x21 2= k6. Setting N = z;., we obtain 1 /(%1) - /(X2) < k'1 = !!. <  = N. Xl - X2 - k6 6 - 6 Assume now that (ii) is satisfied. Then, given € > 0, there is N > 0 such that I/(x]) -/(x2)12= e implies I /(xl) -/(x2) I  N. XI - %2 Consequently, I/(xl) -/(x2)1 :> £ implles IXI - %21 2 -N. This means that (i) is satisfied with 6 = N. 
1.6. Functional Equations 181 1.6. Functional Equations 1.6.1. Clearly, the functions 1(3;) = ax are continuous and satisfy the Cauchy functional equation. We show that there are no other continuous solutions of this equation. Observe first that if I satisfies (1) f{x + y) = f(x) + fey) for x, y E Ii. then f(2x) = 2f(x) for x E IR. One can show by induction that for n E N, (2) l(n3;) = nJ(3;). If in (2) we replace x by : ' \ve get (3) J (;) = ;J(x). If r = E, where p, q E N, then (2) and (3) imply q (4) J(rx) = I ( : x) = pi G x) = : J(x) = r/(x). It follows from (2) that. 1(0) = o. Comhineo with (1») this giv{as o = /(0) = f(x) + I( -x), or in other words, I( -x) = - I(x). Thus, by (4), we get -r/(x) = I(-rx) = - I (rx) for any negative rational r. Since for any real Q there is a sequence {r n} of rationals converging to a, and since I is continuous, by (4) we get /(ox) = I( Urn TnZ) = lim /(TnX) = lim Tn/(x) = a/(x). R-+OO n-+oo R-too Setting % = 1, gives 1(0) = 0/(1). Consequently, I(z) = ax, where a = /(1). 1.6.2. (a) We will show that if I is continuous at at least one point and satisfies the Cauchy functional equation, then it is continuous on :Ill So, the assertion follows from the preceding problem. Clearly, if f satisfies the Cauchy functional equation, then equalities (2)- (4) in the solution of 1.6.1 hold. We first show that the continuity of 1 at an Xo implies the continuity at zero. Indeed, if {Zn} is 
182" Solutions. 1: Limits and Continuity a sequence converging to zero, then {Zn + %o} converges to 2:0. Moreover, it follows from the equality I(zn + xo) = /(;n) + I(xo) and from the continuity of 1 at Xo tbat Urn I(zn) = 0 = 1(0). noo Now if  is any rea] numher anr1 {xn} convp.rges t.n x.. then {x n -x} converges to zero. The equality f(xn - x) = I(xn) -/(:£) and continuity of j at zero imply lim I(xn} = j(x}. n-too (b) We first show that if J satisfies the Cauchy functional equation and is bounded above on the interval ( 0, b), then it is bounded on every interval (-E,E), e > o. To this end, consider the function g(X) = I(x) - I(l}x, z e 1R. Clearly. D satisfies the Cauc.hy functional equation. and it follows . from the solution of 1.6.1 that g(r) = 0 for r E Q. For x e. (-e,e), one call find a rational r such that x + r E (a,b). Then: g(x) = g(x) + g(r} = 9(x + r) = f(x + r) - 1(1)(x + r), which. implies that 9 is bounded above on (-E, e), and consequently, so. is f. Since fe-x) = -lex), / is.al50. bounded below on (-e,e).. Now our task is to show that f is continuous at zero. Let {x n } be a sequence converging to zero, and choose a sequence {r n} 4 of rationals diverging to +00 so that Urn x"r n = O. Then the n-+oo sequence {1/(rnxn)1} is bounded above, say by k/, and I ( 1 ) 1 /vI I/(xn)1 = f -rnX n = -IJ(rnxn)1 < -. r n r n Tn Hence lim I(x n ) = 0 = 1(0). So our assertion follows from (a). n--tooo (c) Asswne, for example, that 1 is monotonically increasing. It fol- lows from (2)-(4) in the solution of 1.6.1 that for - < x <   _.!. f(l)  J{x) < .!. /(1). n n Thus 1 is continuous at zero, and our claim follows from (a). 
1.6. Functional Equations 183 1.6.3. Observe that f(x) = f2 (  ) > O. H f attained zero at an Xo, then in view of f(x + y) = f(x)f(y), f would be identically zero, which would contradict I( 1) > O. Thus f is positive on Ii and the function g(x) = In/(x) is continuous and satisfies the Cauchy functional equation. It follows from 1.6.1 that g(x) = ax, where a = g(l) = In f(l}. Hence fez) = b Z , x E Ii, with b = J(I). 1.6.4. For x, y E (0,00), choose i, s e fit so tbat:c = e t and V = elf. Define 9 by tbe formula g(t) = feet). Then g(t + s) = get) + .q(s) for t.,s E lit, and, by 1.6.1, get) = at. Thus f(x) = alnx = lox, where I b = eii . 1.6.5. As in the solution of-the foregoing problem, for x, y E (0,00) we choose t, s E IR so that x = e t , y = e B . Next we define 9 by seting g(t) = J(e l ). Then J satisfies the given equation if and only if get + s) = g(t)g(s) for tt s E R. It follo\vs £raIn 1.6.3 that g(t) = at. Hence j(x) = a1u;s: = X b l where b = In a. 1.6.6. IT f is continuous on 1R. and f(x) - fey) is rational for rational x - V, then g(:c) = f(x + 1) - f{x) is continuous and assumes onJy rational values. It follows from the intermediate value property that 9 is constant. Let f{x+ 1) - J(x) = q, q E Q. If 1(0) = r, then f(l) = r+q, and by induction, I{n) = nq+T, n E N. Since J{x) = I(x+ l)-q, we get 1(-1) = -q + r., and by induction, I( -n) = -nq + r, n E N. For a rational p =  , the function f{x + p) - f(x) is also constant. Let I( + p) = J() + ij. As bove onp r,an show t.hat j(kp) = kij + r for keN. In particular, I(n) = 1(11&p) = mij +T. On the other hand, J{n) = nq + r. Hence ij =  q and I ( ;:J = :a q + or. Since p can be arbitrarily chosen, I(x) = qx + T fOT x E Q. The continuit.y of / implies that / is defined by this formula for all z E III 1.6.7. Observe that J{O) = o. IVlorcover, for x E R we get f{x) = -f(qx) = f(q 2 x) = -f(f/x). One can show by induction that f(x) = (-l)R f(qn x ). Letting n -+ 00 and using the continuity of I at zero, we see that f(x) = o. Thus only the identically 7ro function satisfies the given equation. 
184. Solutions. 1: Limits and Continuity 1.6.8. We have 1(0) = 0 and /(z) = - f GZ ) + z = f ( G) 2 z) - Z + x. One can prove by induction that for n E N, (( 2 ) R ) ( 2 ) n-1 2 l(z)=(-IYf 3 x + (_I)n-1 3 x+...- 3 x+:t. We now pass to the limit as n -+ 00 and use the continuity of I at zero, and get f(x) = : x. 1.6.9. H in the equation we put y = 2x, we get 1 ( 1 ) 1 1 ( 1 ) 1 1 fey) = 2 1 2 Y + 2 2 Y = 2 2 / 2i Y + 2i Y + 2iY. One can prove by induction tbat 1 ( 1 ) 1 1 1 fey) = 2R f 2R"Y + 22n Y + 2 2 (n-l) Y +... + 2jY. Letting n -+ 00 and using the fact that 1(0) = 0 and / is continuom at zero, we conclude that I{y) = iy. 1.6.10. Set 1(0) = c. Putting y = 0 in the Jensen equation, we get f ( ! ) = I(x) + 1(0) = I(x) + c 2 2 2. Hence fez) + fey) = J ( :c + Y ) = f(x + y} + c 2 2 2' which ves, f(x) + /(y) = f(x+y)+c. Now set g(x) = f(x)-c. Then 9 satisfies the Cauchy equation (see 1.6.1). Therefore g(x) = ax, or in other words, I (x) = ax + c. 1.6.11. We will first show that I is linear on every closed subinterval [0 t.8] of ( (lJ b). Ry the .Tp.np.n p.qntion t f(a + (P - a») = f(a) +  (f(P) - j(a». 
1.6. Functional Equations 185 Furthermore, ( 1 ) ( Q+ Q+,, ) I 0+ 4 (P-O) =1 2 2 = ! /(0) + 1 f ( Q + P ) 2 2 2 1 = I(a) + '4(/(P) -/(0» and I( a + (.8 - a») = IGp +  (a + (P - a»)) 1 1 ( 1 ) = -f«(3)+-f a+-(p-Q) 222 3 = 1(0) + '4(/(13) -/(a». Now we prove by induction that I (a + 2: (.8 - a») = I(a) +  (/(.8) - I(a» for k = 0,1,2,3,... t 2 n and n e N. Ass umin g the equality to hold for m S n, we will prove it for n+ 1. Indeed, if k = 21,1 = 0,1,. -., 2 n J then, by the induction hypothesis, I (a+ 2:+ 1 (P -0») = / (0+ (P -0») I = /(0:) + (f{{3) - j(a» k = /(0.) + 2n+l (/(p) - j(Q». Similarly, if k = 21 + Itl = 0,1,... ,2 n -1, then 1(0+ 2':1 (8-0» = I(Ho+ 2Ll(8 -0»+  (0+ #-0») 1  1 -  1  1  := - / a + (8 - 0.) + - / a + -=..Ja - 0.) 2 2 n - 1 2 2D"" k = 1(0) + 2n+l (!({3) - I(Q». 
186 Solutions. 1: Limits and Continuity Since the numbers 2 form a dense set in [0,1], the continuity of / implies that J (0: + t(j3 - a» = f(o:) + t(J({3) - /(0» for t E [0,1]. Setting z = Q + t({J - 0) gives I(x) = J(a) + /(13) /(0:) (x _ Q). {3-0 Now observe that under our hypothesis one-sided limits of I at a and b exist. Indeed, for example, we have lim fey) = / ( z + b ) _ J(x) with x E (a,b). u-+6- 2 2 2 Clearly, ex) (a, b) = U [an, .on], n=1 where {an} is a decreasing sequence of points in (a, b) converging to a, and {Pn} is an increasing sequence of points in this interval converging to b. Thus for x E (a, b) there is no E N such that x E [O:,u.8n] for all n  no. It then {ollows that I(x) = f(on) + J(l3n) - J(Ct n ) (x - an). /3n - O:n If we let fl 4 00, WE' get I(x) = I(a+) + J(b-) - J(a+) (x - a). b-a 1.6.12. For:c e R, set X n -1 %1 = X and Xn+l = 2 1 n = 1,2,3,... . Then lim X n = -1 and J(xn) = J(2Xn+l + 1) = J(X o +1), n E N. nm Hence f(x) = I(zo). Letting n -+ 00, we see that I(x) = 1(-1). Thus only constant functions fulfill our assumptiogs. 
1.6. Functional Equations 187 1.6.13. Note that g(x) = J{x) - ix2 is continuous on IIi and satisfies the Cauchy functional equation (see 1.6.1). Thus g(x) = g(l)x, which gives f(x) - ; x 2 = (f(l) - i) x for x E nt 1.6.14. By assumption, 1(-1)=/(-) =f(-D =...=1(0). lvloreover, for t =F 0, -1, - , -  , ... we have l(t)=/( t:l ) =1( 2t:l ) =1( 3t1 ) =.... Since lim t ' + " 1 = 0, the continuity of f at zero implies that I(t) = n...oo n 1(0). So the only solutions of the equation are constant (unctions. 1.6.15. No. In fact, there are infinitely many such functions. For a E (0,1) let 9 be a strictly decreasing and continuous transformation of [0, a] onto [a, I). rrhen J, defined as ( ) { ,g(x) for x e [0, aJ, f x = 9- 1 (x) Cor x e (a, 1], where g-1 is the inverse of g, enjoys the desired property. 1 ------.----;' , , , " , , , , , , ,. a a ----- ,'. , . " I , I " . , . 1.6.16. Suppose, contrary to our claim, that there is Yo E III such that Ig(yo}J = a> 1. Set ]vI = sup{lf(x)1 : x E IR}. By the definition 
188 Solutions. 1: Limits and Continuity of the supremum, there exists Xo e R for which I/(xo)1 > If. By assumption, I/(xo + yo) I + I/(xo - yo)1  I/(xo + Yo) + I(xo - Yo)1 = 21/(xo}lIg(yo}J > 2a = 2M. . a Hence I/(xo + Yo)1 > }.II or I/{xo - Yo)1 > M, a contradiction. 1.6.17. Note that g(z) = I(z)e- Z satisfies the Cauchy functional equation. It follows from 1.6.1 that f(x) = axe Z . 1.6.18. By assumption, /(0) = 0 and 1(2z) = (/(X»2. By induction, /(z) = (/ ()r = (/ ()( =... = (/ ()f.. Hence 1( ; ) = 2 \1/ (3:). If lex) > 0, then upon passage to the limit as n -+ 00 we get 0: a contradiction. Thus only the identically zero function satisfies given equation. 1.6.19. Replacing z by %1. 1 in ( z - 1 ) lex) + 1 z = 1 + x (i) gives ( X-I ) ( -1 ) _2X-1 1 +/ 1 - .. x :;c- x Next replacing x by Z--:l in (i), we get ( -1 ) -2 (ill) f z _ 1 + fez) = ; - 1 . Adding (i) to (ill) and subtracting (il) from tbe sum yield x-2 2z-1 2/ ( z) = 1 + x + 1 - · .x- z (ll) Hence Z3_Z2_1 f(x) = 2x(z - 1) . 
1.6. Functional Equations 189 One can easily check that this function satisfies the given functional equation. 1.6.20. For real x and y, define {xn} as follows: X2k-l =:c and X21: = y, k = 1,2,... . Then the equality j(C- Jim x n ) = C- lim j(xn) n-+oo n-+oo implies I ( lim nx + n y ) = lim nf(:r) + nf(y) , n-+oo 2n n."oo 2n which means that f satisfies the Jensen equation I ( Z;U ) = . As in the solution of 1.6.11, one can show that (i) J (z +  (y -z») = fez) +  (f(y) - J(x» for k = 0,1,2,3,... ,2 n and n e N. For t E [0, I], one can find a sequence {} convergent to t. Since every convergent sequence is also Cesaro convergent (to the same limit), the sequence with the terms X n = x + (y - x) converges in the Cesaro sense. By (i) the sequence {f(xn)} converges to lex) + t(f(y) - f(x». Consequently, f(x + t(y - x» = lex) + t(f(y) -f(x». It follows from 1.2.33 that f is continuous on lIt Combined with 1.6.10, this shows that f(x) = ax + c. 1.6.21. Since/(2x-/(x» = x and jisaninjection, we get /- 1 (x) = 2z -f(:£). Thus (i) fez) - x = x - I-I(x). For Xo e [0,1], define the sequence {xn} recursively by X n = f(xn-J). It follows from (i) that X n - X n -l = X n -1 - X n -2. Therefore X n = 2:0 +n(xl -xo). Since IX n -zol  1, we have IXI -zol <  for n E N. Consequently, J (xo) = XI = Xo. 1.6.22. We will show that the only continuous solutions of the given equation are the functions f(x) = 7n(x - c). If g(x} = 2x - I!:) , then 9 is continuous and (i) 9(9(x» = 2g(x) - x for x e fit 
190 Solutions. 1: Limits and Continuity Thus 9 is a one-one function. Indeed, if g(Xt) = g(X2), then we get g{g(Xl» = g(g(x:z», which gives Xl = X2. By the result ill 1.3.16, 9 is either strictly increasing or strictly decreasing on B. We will show that the former case holds. By (i), (ii) 9(9(X» - 9(X) = g(x) - x for x E III If 9 were strictly decreasing, then for ZI < %2 we would get 9(::CJ) > g(X2), and consequently g(g(Xl» < g(g(X2». On the other hand, (ii) gives g(g(X1» - g(X1) = g(Xl) - Xb g(g(X2» - g(X2) = g(X2) - X2, a contradiction. It follows from (i), by induction, that gR(X) = ng(x) - (n - l)x for n  1, where gn denotes the nth iteration of g. lIenee liD) O"%)  g() - z. n-t Ioreover , (iii) gR(X) - gR(O) = 71(g(X) - X - 0(0) + x. Thus, letting n  00 and using the monotonicity of g, we get g{x)  x + y(O) for x < 0, g(x) > z + g{O) for z > 0, (1) which in turn gives g(R) = III So the inverse function 9- 1 is defined on nt Replacing in (i) :e by g-1(g-1(y» we see that g-l(g-I(y» = 2g- 1 (y) -y. Sine g-1 satisfies (i), one ccw show by the sam methpd that g-n(y) _ 9- n (O) = n(g-l(y) _ y _ g-I(O») + y. Next, upon passage to the limit as n -t 00, we get (as above) g-l(y) < Y + g-I(O) for y < 0, g-l(y) > 11 + 9- 1 (0) for y > o. We now show that g-1 (0) = -9(0). Replacing x by g-1 (y) in (ll), we obtaiu (2) g(y) -1/ = Y - g-1 (y), which gives g-I(O) = -g(O). 
1.6. Functional Equations 191 Assume, for example, that 0(0) 2: o. Then g(x) > 0 for x > O. By (2) with 11 = g(x) > 0, we see that x > g(x) + g-1 (0) = g(x} - 9(0). 'fhus by (1), for x > 0 we get f}(X) = x + g(O}. Since g-1 (0) < 0, we have 0-l(y) < 0 for y < 0, and as above one can show that g-I(y) = Y + 0- 1 (0), which means that O(x) = x + g(O) for x < O. Thus g(x) = 3: + 9(0), or equivalently, f(x) = m(x - 9(0» for z E III 1.6.23. It is easy to verify that the given IWlctions satisfy the desired conditions. Now we will show that there are no other solutions. H in the equation (i) I(x + y) + I(y - 2') = 2f(x)f(y) we put x = 0 and a y such that f(y) "# 0, we get /(0) = 1. Taking y = 0 in (i), we see that f(x) = f( -x), which means that f is even. Since f is continuous and j(O) = 1, there exists an interval (0, c] on which the function is positive. We con5ider two Ca5e5: J(c)  1, and fee) > 1. In the first case there exists 9, 0 :S () <  , such that fee) = cos9. Now rewrite (i) in the form f{y + x) = 2f(x)f(y) - fey - x). .Application of this e<!uation with x = c, y = C, and x = c, y = 2c gives j(2c) = 2cos 2 9-1 = cos29 auu f(3c) = 2cos8cos28-cos8 = cos38, respectively. One can show by induction tbat f(nc) = cosnD. Now applying (i) with x = y =  gives . .. (f ()) 2 = f(O) ; fee) = 1 + ;058 = 0052 e ) . Since f () and cos ( ) are positive, the last equation implies that J () = CDS (£) t and recursively I ( c.. ) - cos ( .f" ) for n E N. H we start with the equation I (nc) = cos n9 and repeat the above proce. dure, we obtain f ( tllC ) ( 1nO ) - =cos - 2" 2" for m,n EN. Thus f (ex) = cos Ox for x =  . Since the set of nUDlbers or the form  , 1n. n E Nt is a dense subset of R+ , the continuity of f implies tbat .. 
19 Solutions. 1: Limits and Continuity J(cx) = cos9x for x > o. Since J is even, the equality holds also for negative z. Finally, J(z) = cosaz with a = . In the case where f(e) > 1, there is 9 such that f(c) = cosh9. To show that j(z) = cosh(ax), reasoning similar to tbe above can be usd. 1.6.24. H we put z = tanh 'U, Y = tanh v, then X + y tanh'll + tanh v anh( ) = =t u+v. 1 +X1/ 1 + tanhutanhv Therefore the function g( u) = I (tanh 'U) satisfies the Cauchy func- tional equation (see 1.6.1) and is continuous on IR. Consequently, g(1£) = au. Hence I(z) = laIn ! for Ixl < 1. 1.6.25. Assume that P is not identically zero and satisfies the equa- tion. Set Q(z) = P(l - :e). Then Q(l - x) = P{x), and the given equation can be rewritten as Q{(l - X)2) = (Q(l - X»2 or (i) Q(x 2 ) = (Q(x»2 for x e ill If Q is not a monomial, then it is of the form Q{:c) = axle + x m R(x), where a 1= 0, m > k > 0, and R is a polynomial such that R(O) 1= o. For such a Q, by (i), ax 21t + x 2m R(:c 2 ) = a 2 x 2k + 2az k + m R(x) + z2m R2(x). Equating coefficients of like powers. we conclude that Q(x) = axle, a -:/: 0, and that a = 1. Consequently, P(x) = (1 - x)k with kEN U {OJ. Clearly, the identically zero function also satisfies the given equation. 1.6.26 [Sa Kotz, Amer. Math. Monthly 72 (1965), 1072-1075]. For simplicit.y of notation, we win write fm{x.) instp.rI of (J(Xi»m. If in the equation (i) ( 1 n ) 1 n f - Lxi = - Ljm(Xi) n . I n. 1 J= .= we put Xi = C, i = 1,2, . . . , n, we get (li) I(c m ) = jtn(c). 
1.6. Functional Equations 193 In particular, 1(1) = Im(l}, which implies J(I} = 0 or J(I} = 1; or f(l) = -1 in the case where m is odd. Likewise, f(O) = 0 or 1(0) = It 1(0) = -1 if m odd. Putting c = x ':' , x > 0, in (ii), we get / (x) = , :,. (x). .1. Replacing Zi by z;n in (i) and using the last equality, we obtain (ui) ( 1 n ) 1 n .1. 1 n I - LXi = - Llm(x;"') = - L!(xd. n, } n, } no . a= a= a= In particular, for Z3 :; Z4 = ... = X n = 0, ( Xl +X2 ) 1 1 n-2 I = - /(Xl) + - /(X2) + 1(0). n n n n H in (ill) we put X2 = X3 = ... = X n = 0, and replace Xl by Xl + X2, we get 1 ( Xl + X 2 ) = ! J(XI + X2} + n - 1 / (0). n n n Consequently, I(xl + X2) = f{xl) + J{X2) - /(0). So, the function 9(X) = f(x) - 1(0) satisfies the Cauchy functional equation and is continuous at at least one point. By the result in 1.6.2, 0(7) = ax for :r.  O. Thus I(x) = ax + b, where a = 1(1) -/(0), b = 1(0). It follows from the above that b = 0 or b = 1; or additionally, if m is odd, b = -1. So, the only possible values of a are -2, -1,0,1 or 2. One can easily verify that I(x) = 0, J(x) = 1, J(x) = X, and, for odd m, j(z) = -1, J(x) = -x are the .only solutions. 
194 Solutions. 1: Limits and Continuity 1.6.27. If /satisfies the given condition, then for any real a, b, b ':F 0, 1(0 + b) = !«ab- 1 % + %)(%-1b» = !(ab- 1 % + %)J(%-16) = (!(ab- 1 z) + l(z»/(z- l b) = j(a) + j(b). Hence 1(0) = 0 and f(-x) = -f(x). Moreover, fen) = nf(l) for any integer n. If I is not identically zero, then there is c such that I(e) :f:. O. But /«(;) = /(1)/«(;), so /(1) = 1. H z :F 0, then 1 = /(:c)J(z-I), and consequently, 0 "# f(x) = (/(x- 1 »-1. It follows from the above that for integers p and q #- 0, l(pq-l) = /(P)f(q-l) = /(P)(/(q»)-l = pq-l. Note that for x > U we have j(x) = (f{ VX»2 > U. 'l'hus if y - x > 0, then I{y - x) = I(y) - J(x) > O. This means that f is strictly increasing, and f(x) = x if x E Q. It then follows that I(x) = x for xE1ll 1.6.28. A function f of the form (i) J(x) =g(x) -g G ), where 9 is any real function on IR \ {OJ, sat,isfies the given functional equation. On the other hand, if f satisfies the given equation, then f(x) = fex) -I () , 2 which means that I is of the form (i). 1.6.29. Observe that if f satisfies the given functional equation and if we set g(x) =  (/(x) + f G ) ). hex) =  (J(x) - J C ) ) · then the functions 9 and h have the following properties: (i) g(x) =g G ) and (ii) hex) = -h G), hex) + hex:!) = 0, h( -x) = hex). 
1.8. Functional Equations 195 Now note that if 9 and h satisfy (i) and (ii), then ! = 9 + h satisfies the given functional equation. So our aim is to find functions 9 and h. As in the solution of tbe foregoing problem, one can show that all functions satisfying (i) are of the form g(z) = 1:(:1:) + I: (;) t where Ie is any function defined on lit \ {O}. To find functions h. observe 6rst that (ii) implies that h(l) = O. Now for z > 1 set h(z) = a(1n1n%). Then 8 satisfies the functional equation s(lnlnz) + a(ln(2lnz» = 0, which can be rewritten in the form a(t) + a(ln2 + t) = 0 for t E Iil This means that a can be any function 8uch that set) = -,(In 2 + f) (note that a is periodic with period 21n 2). Tbere are infinitely many such functions, e.g., one can take set) = cas :at . Next we extend the function h onto (0,1) by setting h(z) = -h () , and then onto (-00,0) by setting h(-:z:) = h(z). 1.8.30 [8. Haruki, Amer. Math. Monthly 86 (1979), 577-578). If in the given equation we replace z by % + II and JJ by % - JJ. we get (1) /(z + II) - 9(:1: -1I) = ,p(z). 211 Now replacing'll by -y in (1) gives /(z: - 'II) - gex + y) = 4>(:r:). -2y Consequently, for u, II E . we get 1 tJ>(u + 'U) + ,p(u - v) = 2Y(!(u + t1 + l/) - 9(u + v -II) + I(u - II + y) - 9(1.1 - V - 7/» = (/(u+V+1/)-g(U-V-II» 1 + 2Y(/(u - (v - JI» - 9(1£ + (11 - II»). 
196 Solutions. 1: Limits and Continuity Thus 1 q,(u + v) + tJ>(u - v) = 2ij(2(v + y)t/>(u) - 2(v -II)(U» = 2t/>(u). If we set B = U + v and t = u - tI, then this can be rewritten in the form ,p(s) + ,pet) = A ( 8+ t ) t e R. 2 'I' 2 ,8, Let A : Jlt -+ R be given by A(s) = "'(8) - t/>CO). Then A(O) = 0 and A(t) + A(s) = t/>(s) + I/>(t) - 2"'(0) = 21/> e ; t ) - 21/>(0) = 2A e;t ). Putting t = 0 gives A(s) = 2A (t). Next, replacing 8 by 8+t, we get ( s + t ) A(s + t) = 2A 2 · . (2) Tills and (2) imply (3) A(s + t) = A(s) + A(t). Thus equation (1) can be written in the form (4) lex + 1/) - 9(X - y) = B + A(z), 211 where B = q,(0) and z t-+ A(z) is a function satisfying (3). If in (4) we pu 'IJ = % and IJ = -XI respectively, then we obtain /(2x) = g(O) + 2Bz + 2zA(%) and g(2x) = 1(0) + 2Bx + 2zA(z). Replacing 2z by x and using that fact tbat A(s) = 2A (i) , \\'E! get 1 1 I(z) = 9(0) + Bz + 2 xA (z)t g(z) = 1(0) + Bz + 2 xA (z). Substituting these equations into (1) and applying (3), we arrive at 9(0) - f(O) + 2By + :tA(y) + yA(z) = 4>(z). 2y 
1.6. Functional Equations 197 Setting % = 1, we find that A(y) = dy + /(0) - g(O). where d = 2tf>(1) - A(I) - 2B. Since A(O) = 0, we have 1(0) = g(O). Hence A(z) = cU and I(z) = g(z) = /(0) + Bz + Idz2. It is easy to check that f(z) = g(z) = or + bz + c and q,(z) = /'(x) = 24% + b satisfy the given functional equation. 1.8.31. The set R can bE" regarded as a vector space over Q. A Hamel 6a6i3 for It over Q is a maximal linearly independent set. There exists a Hamel basis H that contains 1. Thus each z e lit can be represented in a unique way 88 z = E wh(z)/., heR where only finitely man y coefficients Wh(Z) e Q are different from zerO. Consequently, for %,lI e It, Z + II = E W,.(z + y)h = E (W,.(:t) + wh(y»h, hEH heH which implies Wh (x + y) = Wh (x) + w" ell). So. in particular, I = WI satisfies (a). We will show that it has the other properties also. Note that wI(l) = I, because 1 = 1.1 and 1 e H. Now we show that WI (x) = % for % e Q. By the additivity of WI t 1 = wI(l) = WI G + i +... + i) = QWI G) · Hence WI (D =i. It then follows, by additivity again, that WI (i) = i for p,q e N. Moreover, WI (0) = 0, because 0 = 0 · 1 and 1 e H. Thus 0= WI (0) = WI (i + (-)) = WI () +WI ( -) . 
198 Solutions. 1: Limits and Continuity or, in other words, WI ( -n = -. So we have proved that WI (x) = x Cor % e Q. Finally, we show that WI is not continuous. H it were, we would get WI (z) = z for aU z e R. This would contradict the fact that WI assumes only rational values. 1.7. Continuous Functions in Metric Spaces 1. '1.1. We will show first that (a) => (b). Let F be a closed set in Y. Then, if a sequence {Xn} of elements in 1- 1 (F) is convergent to z, then I(zn) e F, and by continuity of I, I(zn) -+ I(x). Since F is closed, I(x) E F, or in other words, % e l-l(F). So we have prove that I-I (F) is closed. To prove that (b) => (c) it is enough to note that every ope subset G of Y is the complement of a closed subset F, that is, G : Y \ F. Then, we have 1-I(G) = X \ 1-1 (F). We will now prove that (c) => (a). Let Xo e X and £ > be arbitrarily fixed. By assumption, the set 1-1 (By(J(ZO),E» j open in X. Since %0 is an element of 1-1 (By(/(Xo),E», there j 6 > 0 such that Bx(zOt6) C 1-1 (By (/(zo), e» . Therefore we hav / (Bx(xo, 6» c By(/(xo),E), wbich means that I is continuous 8 %0. So we have proved that the first three conditions are equivalent. Next, we sbow that (a) => (d). 'Ib this end, take Yo e I(A). By the definition of tbe image of a set under I, there is Zo e A such that /(xo) = 110- By continuity of I at xo, given E > 0 there is a baU Bx(xo, 6) such that I(Bx(zo,6» c Byb/o,E). Since Xo e At we see that Bx(xo, 6) n A  0. Thus o i= I (Bx( xo I 6) n A) c By (r/o, E) n I(A), which means 1/0 e I(A). 
1.7. Continuous Functions in Metric Spaces 199 To show that (d) => (e), s et A = /-1 ( B). Then I{I l(B» C 1(1 l(B» C B. Hence f-l(B» C l-l(B). To end our proof we show that (e) ==* (b). H F is closed, then F = F . By (e), /- 1 (F) C J- 1 {F), which means that I-I (F) is closed. 1.7.2. Let B(X) denote the family of all Borel subsets of X, that is, the smallest a-algebra of subsets of X containing aU open sets. Denote by B the family -of sets BeY such t.hat f- 1 -(B) E B(X). Then B is a u-a1gebra of subsets of Y. Since f is continuous, it follows from the foregoing problem that the inverse iUlage of any open set is open. Consequently, B contains all open subsets of Y . Hcnce B(Y) C B, which uuplies Lhat if B E 8(Y), I,hu /-1 (B) E B(X). 1.7.3. Let X = Y = JR. be endowed with the usual Euclidean metric d(x,y) = Ix - yl. Define f(x) = sin1ix and F = {n +  : 71.  2}. Then F is closed in the metric space X, bec:nuse it contains only isolated points. On the other hand, f(F) { . 1r . 1i . 1(' } = SID 2 ' - sm 3 ' sm 4 ' ... is not. closed in Y because it does not contain zero, \vhich is its accu- mulation point. Let X and Y be as above and define J(x) = x(x - 2)2 and G = (1,3). Then J(G) = [0,3). 1.7.4. If Yn E I(F), then '!In = /(x n ), where X n E F, n = 1,2,3,... - H F is compact in X, then there exists a subsequence {x n ,,} of {x n } converging to an x E F. By the continuity of J, {Vn..} defined by Yn" = /(x n ,,) is a 5ubse{IUenCe of {Yn} converging to f(x) E I(F). SO the compactness of I(F) is proved. 1.7.5. Let {x n } be a sequence of elements in Fl U F2 U ... U Fm converging to x. Then there is at 1ea:s1 one set Fi containing a subse- quence {x n ,\:}. Consequently, the sequence {Xn} can be decoDlposed 
200 Solutions. 1: Limits and Continuity into finitely many subsequences in such a way that every subsequence is contained in one set F.. Since F i is closed and f is continuous on F., J(x n .) = J'F (Zn,)  I'F, (x) = 1(%). It then follows that {J(zn)} is decomposed into finitely many subsequences converging to I(z), which means that {J(Zn)} converges to J(x). To see that the statement does not hold in the case of infinitely many sets, consider F, defined as follows: Fa = {O}. F i = {t}, i = 1,2,3,.. . . The function given by I(x) = { I for x e Fi. i = 1.2,3 p .., o for z e Fo, is continuous on each F., i = Otl,2,3,.., but is not continuous on 00 tbe set U F.. i=O 1.7.6. Let %0 e U G, be arbitrarily chosen. Then there is to E  teT such tbat zo e G to . Since G ,o is open and the restriction of f to G, i continuous, given E > 0 there is 6 > 0 such that if z e B(xo,6) C GIG then J(z) = J'Gro (x) e B (/lGro (x o ),£) = B(J(zo)tE), which mean tbat J is continuous at Xo. 1.7.T. Assume that for every compact A C X, JtA is continuow If a sequence {Xn} of elements in X converges to z, then the se A = {Z,ZIt2"2,Z3,".} is compact in X. So, J(xn) = J,A(Zra) - J'A (x) = J(z). Tbus I is continuous on X. The other implication i obvious. 1.7.8. The continuity of J-I is equivalent to the condition that J(G) is open in Y for each open G in X. fiG is open in X, then Ge = X\G, as a closed subset of the compact space X, is compact. By the result in 1.7.4, J(GC) = Y \ J(G) is also compact, and therefore closed. This means that f(G) is open. To show that compactness is an essential assumption, consider J : (0,1) U {2} -+ (0,1] given by J(x) = z for x e (0,1) and 1(2) = 1. Obviously, I is a continuous bijection of (0,1) U {2} onto (0, 1]. Since /-I(z) = Z for z e (0,1) and J-I(I) = 2, tbe inverse function is not continuous on (0,1]. 
1.7. Continuous Functions in Metric Spaces 201 1.7.9. Let d) and d 2 be metrics for X and Y, rf2\.spective1y. By con- tinuity of I, given E > 0 and x E X there exists 6(x) > 0 5uch that (1) d1(y,x) < 6(x) implies d2(f(y),f(x» < i. Since the family of the balls {B (x, c5(x») : x E X} is an open cover of a compact space X. there is ;.I. f1nitp. 11hcovp.r (2) {B (Zi,O(Zi») : i= 1,2,...,1I}. Set 6 = l mint 6(Xl), 6(X2), . . . , 6(x n )} and take x and y in X such that dl (x, y) < iJ. Since family (2) is a cover of X, there exists an i E {I, 2, ..., n} such that d 1 (x, Xi) < l6(x;). Then 1 d1(y,Xi) < dl(Y,X) + d1(x, Xi) < 6 + 2 c5 (Xi)  6(Xi). Consequently, by (1), (f(x), f(y» < d 2 (f(x), f(xi» + d 2 (f(xi), f(y» < E. 1.7.10. F01" o,X E X and yEA, dist(x, A)  d(x, y) :5 d(x, xo) + d(xo,.y). Thus dist (x, A) < d(Xt :to) + dist (xo, A). Hence <list (, A) - dist (xo, A)  d(xtxo). Likewise, dist (xo, A) - dist (x, A)  d(x, xo). Consequently, Idist (x, A) - dist (xo, A)I S d(x, xo), and therefore J is uniforInly continuous on X. 1.7.11. H the set f(X) were not connected, then there would exist nonemptYt open and disjoint sets G 1 and G 2 such that G 1 U G2 = I(X). Continuity of I implies that /-l(G j ), i -:- 1,2t are open. Clearly, they are nonempty and disjoint and their union is X, a con- tradiction. 
202 Solutions. 1: Limits and Continuity 1.7.12. Let d) and  be metrics for X and Y , respectively. Assume that f is continuous at Xo e A. Then, given e > 0, one can fin4 6 > 0 such tbat f(x) E B(f(xo),e/2) whenever z e B(xo,6) n A. Consequently, d 2 (!(x),!(y» < e for x,y E B(zo, D) n A. It then follows that o/(xo) = o. Conversely, if o/(xo) = 0, then given E > 0 there is o£ > 0 such that 0<6 < OE implies diam(j(A n B(zo,6») < e. Hence d1{x,xo) < 6 implies (f(x)., f(xo»  diam (/(A n B(zo,6))) < E. 1.7.13. Set B = {x E A : o/(x) > E} and let {xn} be a sequence of points orB converging to Xo. Since B C A, Xo E A . Therefore o/(xo) is well defined. l\tloreover., for any {} > 0 there is n e N such that B(xn,6/2) C B(xo,6). Hence diam(f(A n B(xo, 6»)  diam(f(A n B(xn, 6/2»)  o/(x n ) > e. It then follows that 0/{xo)  c., or in other words, Xo E B. 1.7.14. By the result in 1.7.12 the set C of points of continuity of / is equal to the set on which the oscillation vanishes. Put Bn = {z EX: o/(z) < } . It follows from the foregoing problem that the Bn are open in X. On the other hand, 00 C = n Bn' n=1 that is, the set of points of continuity of I is of type f}6. It then follows that the set X \ C of points of discontinuity of J is of type F(1 in X. 1.7.15. Consider the function defined by (compare with 1.2.3(a» o if x is irrational, /(:&) - 1 if:e - (J, i if x = :., p E Z, q E N, and p, q co-prinle. 
---...r'I......... ............. 1.'1. Continuous Functions in Metric Spaces 203 1.'1.18 [8. S. Kim, Amer. Matb. Monthly 106 (1999), 258--259]. Let A be of type '11 in II, that is, 00 U F n = A. n=1 where the F n are closed. Without loss of generality we can assume that F n C F n+l for n e N. Indeed, it is enough to replace F n by FI U F2 U ... U F n. U A = R, then, for example, I(z) = XQ(z) is discontinuous at each z e R. If A '# - then we define a function 9 by setting { E t- if 2: e At g() = nEK o if z e nt \ A, where K = {n: z e Fn}, and we put J(:z:) = g(:z:) (XQ(:Z:) - i) · First we show that each point of A is a point of discontinuity oJ" /. Indeed, if z E A 0 , then every neighborhood of z contains a point at which the sign of f is different from the sign of I(z). If z e BAn A, then fez) ¥: 0 and every neighborhood of % contains a point at which I vanishes. Since A = A O U(8AnA), the function 1 is discontinuous on A. Our task is nOW to show that 1 is continuous on R\A. We have fez) = 0 if z  A. If a sequence {Zl:} converges to % and %Ic e A, then tor each n there is a k n such that Zic j F n for Ie  kn. (If there were infinitely many %i in some Fn. then z would be also in Fn.) Consequently, for k 2: kn. 1 1 1 g(Zi) S 2n+1 + 2n+2 +... = F' which means that lim g(Xt) = 0 = g(z). Ic-+oo 1.'1.1'1. No. Every function defined on a discrete metric space is . contmuous. 1.'1.18. Assume first that z e 8A = A n X \A. Since each ball B(z, 6) contains points of A and points of X \A, we get OA(Z) = 1. 
... - 204 Solutions. 1: Limits and Continuity Assume now that OA (x) > o. This means that for every 6 > 0. SUp{tA(X) - XA(II)I : JJ e B(x,6)} = 0XA (%,6) > o. Consequently, each ball B(x,6) must c ontain points of A and points of X \ A. Hence x e 8A = A n X \ A. Clearly, if A is both open and closed, then 8A = 8. Therefore, by 1.7.12, XA is continuous on X. Conversely, if XA is continuous on X, then 8A = 0. Now we show that A c A. If not, there is % E A \ A C X \ A C X \ At a contradiction. One can show in an entirely similar manner that X \ A is also closed. 1.1.19. For % e A and 6 > 0 we have 0/(%,6) = sup{d 2 (!(x),!(II» : 11 e B(x,6)}  sup{d 2 (!(z),/(II»: 11 E AnB(z,6)} +sup{d2(f(x),f(1I»: 11 e (X \ A) nB(z,6)}. Thus o/(x, 6)  SUp{d2(gl (=-=),91 (II» : 11 E AnB(z,6)} + sup{d 2 (g1 (Z)t !I2(II» : 11 E (X \ A) n B(z,cS)} S 0'1 (z,6) + sup{d 2 (gl(X),!/2(Y» : JJ e (X \ A) n B(z,6)}  0'1 (x, 6) +sUp{tt.J(91()t92(Z» +(!12(x),92(Y»: 11 e (X \A) nB(z,6)}  0'1 (x, 6) + d 2 (91 (z),!/2(z» + 062 (x, 6). Since 91 and 92 are continuous, we get, by 1.7.12, (1) o/(x) S (gl(x),92(Z». Now our task is to show tbat for % e A, (2) 01(x)  d2(91 (Z),92(Z». Let {cS n } be a sequence of positive numbers converging to zero. Since A 0 = 0, the set X \ A is dense in X. Thus each ball B(z,6 n ) contains 
1.7. Continuous Functions In Metric Spaces 205 a point fin of X \ A. Consequently, sup{d2(/(%),/(1I» : 11 e B(%,6 n )}  sup{d 2 (gl (Z), 92 (y» : II e B(%,6 n ) n (X \ A)}  (g1 (z), 92(1/..». This combined with the continuity of!/2 implies lim 8Up{(/(z),/(lI» : 11 e B(z,6 n )}  (91(%),92(Z», n"'oo which in turn gives (2). It fonows from (I) and (2) that the desired equality bolds for z e A. In an entirely similar manner (using the density of A) one can show that this equality holds also for % e X \A. 1.7.20. Assume that {In} is a sequence of functions continuous on X and such that I(z) = lim I,,(z). For £ > 0, put ft-l'OO P m (£) = {z eX: I/(z) -/m(%)1  e} 00 00 and G(E) = U (Pm(£»o. We will prove that C = n G(1/n) is m=1 n=1 the set of aU points of continuity or I. We show first that if I is continuous at %0, then %0 e C. Since I(z) = lim In(z), there is an ""'00 m such that E 1/(2:0) -/m(zo)1 S 3. It follows from the continuity of I and 1m at Zo that there exists a ball B(%Ot 6) such that for % e B(Zo,6), £ £ I/(z) -f(:to)1  3 and I/m(z) -fm(zo)1  3. Consequently, I/(z) - Im(%)1  £ if z e B(Zo,6). This means tbat Zo e (P",(E»O c G(e). Since £ > 0 can be arbitrarily choseD, we see that Zo e C. Now if 00 %0 e C = n G(I/n), ft=1 then, for any £ > 0, %0 e G(e/3). Thus there is a positive integer m such that:l:o e (Pm(E/3»o. Consequently, there exists a ball 8(%0,6) 
206 Solutions. 1: Limits and Continuity 8uch that if z e B(zo, 6), then E I/(z) -/m(z)1 S 3. Since 1m is continuous, this shows that I is continuous at %0. Now our task is to prove that X \ C is of the first category. To this end, define F m(£) = {z eX: I/m(z) -/m+lc(z)1 S e for all keN}. The continuity of In, n e N, implies that Fm(e) is closed. Since 00 /(z) = Urn In(z), z e X, we see tbat X = U F met) and F m(E) C n....oo m=J Pm(e). Consequently, 00 U (F m(£»O C G(E). m=1 Now note that for any F C X the interior of F \ F O is empty, because (F \ FO)O C FO \ ( )O = 0. Moreover, if F is closed, then F \ po is closed and therefore F \ FO is nowhere dense. Since 00 00 X \ U {Fm(£»O C U (Fm(E) \ (Fm(E»O), m=1 m=1 00 the set X \ U (Fm(e»O is of the first category. Moreover, since m=J 00 X \ G(e) C X \ U (F m(E»O, the set X \ G(E) is a1so of the first m=1 category. Finally, observe that 00 00 X \ c = X \ n G(l/n) = U (X \ G(l/n». n=1 n=1 Therefore the set X \ C of points of discontinuity of I is of the first category. 1.7.21. We win use the notation from the solution of the preceding problem. We have 00 00 X \ G(l/k) eX \ U (Fm(l/k»O C U (Fm(l/k) \ (Fm(l/k»O). m=1 m=1 ,.._ -1-.....- -' .._&_-1_1
1.'1. Continuous Functions in Metric Spaces 207 Hence 00 co 00 U (X \ G(I/k» C U U (Fm(l/k) \ (Fm(l/k»O). k=l 1c= 1 m=l Sot X \ C is a subset of the union of countably many closed and nowhere dense sets (their complements are open and dense in X). It then follows tbat C contains the intersection of count ably many open and dense- sets. By the theorem of Baire, C is dense in X. 1. '1.22. For £ > 0 put Fi = {O} U n {% > 0 : IJ (!) 1 5: E } . k = 1,2.3,... . n ' n Since I is continuous, the sets are closed (see. e.g.. 1.7.1). By hy- pothesis, U F. = [0,00). According to the theorem of Baire, at least l one or tbe sets F. has a nODcmpty interior. Consequently, there exist a> 0,6> 0, and Ie E N 8uch that (a - 6,a + 6) c F.. Without loss of generality we can assume that 6 S i. If 0 < z S 6 and n = [i] , then a-6 S a-z < nz Sa < a+6. and n k. Thus nz e Fit and, by the definition of Fie, J(z:) = II ( ) I $ E. which implies lim fez) = o. z-tO+ 1.'1.23. Define F n as follows: Fn = {z eX: 1/(%)1 S n for aUl e F}. It follows from the continuity of 1 that the F n are closed. By hy- potheses, for every z e X there is a positive integer Rz 8uch that 00 I/(z)1 S n.z for aUl e :F. Thus z e F n.. Consequently, X = U F n. n=l Since (X. d.) is of the second category, there is an F no with noncmpty interior. Let G = F:o. Therefore I/(z)1 no for every I e F and each z e G.
208 Solutions. 1: Limits and Continuity 1.7.24. We know that J ( Dl Fn) C DJ J(Fn). Now we show that if f is continuous, then D /(Fn) C / ( f\ Fn). (XI Let yEn I(F n). Then, for any positive integer n, y E f(F n), or n=1 in other words, y = f(xn} with an X n E F n. By Cantor's nested set 00 . theorem, n Fn = {xo} for some Xo e X. By the continuity of I, n=l y = lim f{x n ) = I(xo}. Thus y e J ( n Fn ) . . n-+oo n=1 1.7.25. }tor ,",vex we have d(/culu) = sup{ld 1 (u,x) - d 1 (v, x)1 : x E X} 5 d 1 (u,v). Moreover, d(fUtfv) = SUp{ldl(U, X) - dl (v, x)1 : x E X}  Id 1 (u,u) -dt(u,v)1 = dl(U,V). 1.7.26. Assume first that X is a compact metric space and that I : X -t R 15 continuous. Then, given E > Q and x E X, there is 6% > 0 such that I/(Y) - f(x)1 < e for Iy - xl < 6%. Since the family {B(x,o;r;}, x E X} is an open cover of X, there is a finite sub cover B{X),OZI),B(X2,OZ2)'''. ,B(xn,ozn). Therefore for x E X there exists i e {1,2,...,n} such that x e B(Xi,ozJ. It then follows that I/(x)1 < I/(x) -f(x;}1 + 1/(xi)1 < E + max{f(xl), /(xz),... t f(xn)}, which proves the boundedness of I on X. Assume now that every real function continuous on X is bounded, and suppose, contrary to our claim, that X is not compact. Then one can find a sequence {xn} of elements in X that does not contain any 
1.7. Continuous Functions in Metric Spaces 209 convergent subsequence. Then F = {Zn : n e N} is closed in X. The function I given by I(x,,) = n is continuous on F. According to the Tietze extension theorem, there exists a continuous extension of / defined on all of X. So, we have constructed a continuous and unbounded function, a contradiction.. 1.7'.27. First we prove that (a) implies (b). So assume that (a) holds, let Jim p(zn) = 0, and suppose, contrary to our claim, that {Zn} n-+oo does not contain a convergent subsequence. Then there is a sequence {Yn} of .elements in X such that lirn d 1 (zn1f1n) = 0 and 1In F %n n-+oo for n E N. If {,In} contains a convergent subsequence {1IR.}, then by lim d 1 (%n't1ln.) = 0 the sequence {zn.} is also convergent. Thus i-+oo . {fin} does not contain any r.onvergent subsequence.. It then follows that no term of the sequences {zn} and {1In} is repeated infinitely many timeS. Therefore there is a strictly increasing sequence {Ri} of positive integers such that the infinite sets FI = {Xn. : keN} and F2 = {lIn, : kEN} are closed and disjoint. According to the Urysohn lemma, there is a continuous function J : X -.. lIt such that I is one on FI and zero on F2.. Thus I/(zn.,) -/(,In.,)1 = 1 and lim d 1 (zn.,lIn.) = 0.. '-'00 Hence I is continuous but not uniformly continuous on Xt which contradictS (a). To show that (b) implies (a), denote by A the set of limit points of x. By (b) every sequence of elements in A has a subsequence converging to an element in A. Therefore A is compact. If X  A.. then for 61 > 0 put 6 2 = inf{p(z) : Z E X, dist(z,A) > 6)}.. We will show that  > o. If 62 = 0, then there is a sequence {z,,} of elements in X such that Urn p(zn) = 0 and dist(ZRt A) > 6 1 . By (b), {zn} nco has a subsequence converging to an element in A, a contradiction.. Let J : X -. R be r.ontinuous and let E > 0 be arbitrarily fixed. Then for z E A there is 6% > 0 such that if d 1 (z,y) < 6 z , then I/(x) - l(y)1 < !E. Since A is compact, there are %1,... ,Xn e A such that A CUB ( Zit 6",. ) · k= 1 
210 Solutions. 1: Limits and 'Continuity Let 6 1 = k min{ZI'''. ,6 z1l J and 8 2 > 0 be as above. Put 0 = min{6a,02} and let :£,y E X be such that dl(X,y) < o. H dist(x,A) > 6 1 , then p(x} > 6 2 , SO d 1 (x, y) < 0  62 only if x = y. Then, obviously, If(x) -f(y)1 < E. H dist(x, A) < 6., then there is an a E A such that d 1 (x, a) < 6.e It follows from the above that there is k e {I, 2,. . . , n} for which dl (a,xk) < i6z.,. (:onsequently, 1 d 1 (y, Xk) < d. (y, x) + d 1 (x, a) + d 1 (a, Xk) < 6 + 6 1 + 3 6,r. < 6z,.. Hence 1 1 I/(x) -f(y)1  I/(x) -f(Xk)1 + I/(Xk) -f(y)1 < 2 E: + 2 e = £. This proves the uniform continuity of f on X. 1.7.28. It is well known, see, e.g, 1.7.9, that evp.ry function contin- uous on a compact metric space is uniformly continuous. \Ve claim that if X is compact, then each set {z € X : p(z) > }, E: > 0, is finite. On the contry, suppose that there is an e > 0 for which the set {x EX: p(x) > e} is infinite. Since the family of balls {B(X,E) : x E X} is an open cover of X, it has a finite subcover, which contradicts the fact that p(x) > E for infinitely many x. Assume now that every real continuous function on X is uni- formly continuous and that every set {x EX: p( x) > E J is finite. We will sho\v that X is cOlnpact. Let {xn} be a sequence of points in X. If a term in the sequence is repeated infinitely many timcs, then obvi- ously there is a convergent subsequence. If not, then lim p(xn} = 0, n-roo because the sets {x EX: p(x) > e} arc finite. By the result in the foregoing problem, {xn} contains a convergent subsequence. 1.7.29. It is enough to c9Dsider X = [0,1] U {2} U {3} U {4} U ... equipped with the usual Euclidean nletric dl (x, y) = Ix - ul. 
Chapter 2 Differentiation 2.1. The Derivative of a Real Function 2.1.1. (a) We have { X? if x  0, f{x) =  if .., x < o. Hence { 2 ,.. if x > O J f'(x) = :2x if x < 0, because f(O) = Jim h 2 -; 0 = 0 = f(O). hO+ (b) We get I { 2 if z > 0, I () = _ I if 0 .f=i x < . _ -z Since J ' ( ) 1 . Vii - 0 +O=un h =+00 h-.O+ and I ' (0) - 1 . V -It - 0 - - 1m - -00, - hO- h the derivative of f at zero does not exist. - 211 
212 Solutions. 2: Differentiation (c) /'(z) = R1I'sin(21rz) for :z e (n, n + I), n e Z. Moreover, for neZ, J' (n) = Iim nsin 2 (1I'z) - 0 = Iim nsin 2 (1I'z - m) = 0, + z-+n+ Z - n z-+n+ Z - n I(n) = Iim (n - 1) sin 2 (1I'z) - 0 = O. z....n- Z - n It then follows that /'(z) = 71'[zJ sin(271'z). (d) It follows from (c) that /'(z) = (zsin 2 (wz»' - ([z] sin 2 (1I'%»' = sin 2 (7I'z) + 71'(z - [z» sin(211'z). (e) I'(z) =  for z F o. (f) J'(z) = zv':iJ-l if Izi > 1. 2.1.2. (a) Since logz 2 = m-!, we get J'(z) = _ In2 = _ Iogz 2 .logze . z ln 2 Z Z (b) As in (a) we show that J ' ( ) -tanzlnz-lnCOS% Z = 2 lnz 1 = - tan:elog e - -IOg.OO8Z .logz e. z 2.1.3. (a) Clearly, , % _ { 1-J if Izi < 1, I ( ) - 1 if Izi > 1. We will now check whether tbe derivative exists at :a: = 1 and at z = -1. We have /.'(1)= Iim j+-f =!, + z-+l+ %-1 2 I(I) = lim arctanz -  = arctan'(l) = !. 2....1- z -I 2 
2.1. The.Derivative of a Real Function 213 So /'(1) =  . We have also / ' ( 1) - Jim arctan x +  _ ' ( 1) _ 1 + - - -arctan - -- z-+-l+ X + 1 2' _l!: +  + !:. 1(-I) = 1im 4 2 4 = +00. z....-I- X + 1 Therefore /' (-1) does not exist. (b) We have f'(x) = { 0 2xe-z2 (1- x 2 ) if if  Ixl < 1, Ixl > 1. Moreover. 1_! /' (1) = lim e e = 0, + %-+1+ z-1 x 2 e- z2 _! 2 1(I) = lim e = (x 2 e- z )' _ = o. z-+l- x-I Ix - 1 Since I is even, 1'(-1) = O. (c) Observe that f is continuous at zero. Moreover, t 1.. t tr I' (0) = lim arc an z - 2' = Jim - '2 + -+o+ X t-t · - ....!....- l' tan t = Jim ( t -  ) tant = -1 t-+ f - 2 and I(O) = lim arctan (-) -  = lim t - f Z-I>O- X t.... :. - _-L it tant = - lim ( t - 1r ) tan t = 1. t-+ i - 2 Thus the function is not differentiable only at zero. 2.1.4. Note first that /'(0) = Jim x2lcos I = o. z....o Z 
214 Solutions. 2: Differentiation Clearly, for x i:- 2n11 neZ, f'(x) exists. For X n = 2n1 t n = 0,2,4,..., we get x'J cos !: ( 11' ) ' f(xn)= lim Z = x 2 cos- I =1f', z-+zt x - X n X X = X n J ' ( ) Jim _%2 COS  ( ') 'Ir ) ' _ X n = = -x" COS- = -'lr. Z-JoZ;' x - %n X Ix = X n Similarly, if X n = 2';+1 1 n = 1,3,5,... ,then I.+(x n ) = 1f' and f!...(zn) = -'lr. Since J is even, I is not differentiable at X n , n e Z. 0.5 0.4 0.3 0.2 0.1 0.2 0.6 0.8 I 0.4 2.1.5. (a) Since f must be continuous, we get c = 0 and a + b = 1. Since I(O) = 4, 1'+(0) = h, we get b = 4 and a = -3. It is easy to verify that for such a, b and c the function f is differentiable on R (b) a = d = -l,b = O,e = 1. (e) b=c=l,a=O,d=i. 2.1.6. (a) For x #; 0, n 1 _ e(n+I)z el:%= . L...., 1 - e Z 1: =0 Differentiating both sides of this equality, we get E n ne{n+2)Z - (n + l)e(n+l)z + e% ke kz - - (1 - e z )2 . k=O 
2.1. The Derivative of a Real Function 215 (b) Differentiating both sides of the equality E<_1)k ( 2n ) e iZ = (e Z _1)2n k=O Ie n times, we obtain E<-l)kkne"" e:) = «eO' _1)2n)(n l · k=O To calculate (e Z - 1)2") (n) at zero we set g(x) = ,.: - 1 and note that the nth derivative of (g(z»2n is a sum whose every term contains a power of g(x) at least of order n (compare with 2.1.38). So, the nth derivative of z ..... (e Z - I):ln at zero is o. Consequently, E<_1)kkn ( 2n ) =0. k=O k (c) Differentiating the equality n sin  sin (n+J>.!. Esin(kz)= 2. % :I , z217rtleZ, t=1 sm 2" we get n nsin Z sin $2n+I>,! - sin 2  Ekcos(kx) = 2' 2. 2 2 ;r 2 t Z  2111', I e Z. i=1 sm 2 For 2: = 2111', n 1 Ekcos(h) = 2n(n+ 1). i::::l 2.1.7. Put f(x) = al sinz +B2sin2x +... + Qn sinnz . Then lal + 2a2 + .. · + nOnl = If'(O)1 = lim 1 /(%) -/(0) I z-tO % = Um (z) . sin % I = lim I fez) I  1. z-+o sm x z .1:-+0 sin % 
216 Solutions. 2: Differentiation 2.1.8. (a) We have lim xl(a) - al(x) = lim (x - a)J(a) - a(f(x) -/(a» Z-+G X - a Z-+G X - a = I(a) - a/' (a). (b) As in (a) we have Jim J(x)g(a) - J(a)g(x) z-+a Z - a = lim (f(x) - I(a»g(a) - J(a)(g(x) - 9(a» z-+a x - a = J/(a)g(a) - I(a)g'(a). 2.1.9. (a) Since I is continuous at a and I(a) > 0, we see that / (a + *) > 0 for sufficiently large fl. Moreover, ince J is differentiable at a, the function x  In(/(z» is also. Consequently, 1 lim In ( I (a + ) ) it = lim  In I (a +) -ln/(a) 0-+00 I(a) n-+co n 2 * = o. (In l(x»'lx = a = o. Hence lim ( f (a+ ) ) !- = 1. 0-+00 I(a) (b) As in (a) we get I . ( /(X» ) l a.- I ao . In I(x) -In I(a) x - a f'ea) lim in = lim . = a. -+a I(a) z-+a z - a lnx -In a I(a) 2.1.10. (a) By 2.1.8(b) with g(x) = x n J an /e x ) - xn /( a ) Jim = _na D - 1 J(a) + aD/'ea). Z-+G z - a 
2.1. The Derivative of a Real Function 217 (b) lim I(x)ez - 1(0) = lim l(x)e Z - 1(0) z z-+o I(x) cosx - 1(0) z-+o x I(x) cosx -/(0) 1 = (J(z)e"')' tz = 0 (f(x) casz)llz = 0 _ I' (0) + 1(0) - /'(0) (c) n n (I (a+;) + I (a+) +...+ 1 (a+;) -k/(a») = lim ( /(a+*) -/(a) +2 /(a+) -/(a) n-+oo.!. .1 n n k f (a + : > - I(a» ) +...+ I.: n = (1 + 2 + ... + k)l'(a) = k(k; 1) I'(a). I' (d) For kEN, Iim I (a+) - f(a) = f(a). n-+oo  This implies that given E > 0 there is no such that ifn > no, then /'(a) - e < I ( a+  ) -/(a) < /'(a) + E n 2 n 2 n 2 n 2 n 2 for k E {I, 2 1 . . . , n}. Summing over k, we obtain n( 1) f'ea) _ n(;; 1) £ <  (I (a+  ) - lea») n(n + 1) / , ( ) n(n + 1) < 2n 2 a + 2n 2 E. It then follows that the limit is !/'(a). 
218 Solutions. 2: Differentiation 2.1.11. (a) We have Jim ( n+l)m+(n+2)m+...+(n+k)m -kll ) n-too n m - 1 = lim (n + l)m - n m + en + 2)m - n m + ... + (n + k)m - n m n-too nm-l ( (1 + ! ) m _ 1 (1 + .1 ) m_ 1 (1 + i ) m - 1 ) = lim n + 2 n + . . . + k n n-too.!. 1. 1£ n n n k(k + 1) = m. 2 Compare with 2.1.10(c). (b) By 2.1.10(c), lirn In ( (a+*)n{a+*)n.''(a+)n ) = k(k+l) .!.. R-too a nk 2 a Thus ... lim (a + ) n (a + ) n . . . (a + ) n n-..oo ant .("+1 ) =e 2 0. (c) Note that lim In (( 1 +  ) ( 1+  ) ... ( 1+ na )) n-too n 2 n 2 n 2 = lim ( In ( l+  ) +1n ( 1+ 2a ) +".+ln ( 1+ ))  w   = lim ( In ( ! +  ) + .. .. + In ( ! + .!!. ) - n In ! ) . n-+co a n 2 a n 2 a It then follows by 2..1.10( d) that lim (( 1 +  ) ( 1 + 2a ) ... ( 1 +  )) = e i . n....oo n 2 n 2 n 2 
2.1. The Derivative of a Real Function 219 2.1.12. We bave ; (/(z) + / (i) + / (i) + ... + / (i)) = llm ( /(Z) - /(0) + / (i) -/(0) +... + f (I) -/(0» )  Z Z Z = (l++i+...+i)f(O). 2.1.13. (a) If fez) = zm, m e Nt then Jim /(zn) -f(zn) _ Jim z:' - z::' _ m-I _ / ' ( ) - -ma - a_ n-+oo Zn - Zn n-+oo Z" - Zn (b) Consider the function given by { %2 sin ! fez) = 0 :e if Z  0, if Z = o. For 2 % - n - 71'( 4n + 1) 1 and Zn = - 2n7r we bave Jim fez") - /(zn) = _! 1: 0 = "(0)_ n-+oo Zn - Zn 'II" On tbe other hand, if { zi sin 1 if z ¥ 0, g(x) = 0 z . - f z=O, and {Zn}, {zn} are as above, tben lim g(xn) - g(Zn) = -00. n-+oo Zn - Zn 2.1.14. By hypothescs, f(zn) - f(zn) /(:l: n ) -lea) Zn - a f(zn) -lea) 0 - Zn - . + . - , Zn - Zn Zn - a %n - %n Zn - Q Zn - Zn wbere O a-Zn 1 < <, :l: n - Zn o < Zn - a < 1 Zn - Zn 
220 Solutions. 2: Differentiation and a - Zn Zn - a + =1. X n - Zn X n - Zn It then follows that I(xn) - I(zn) X n - Zn is between /(Xn) - f(a) and f(zn) - lea) Xn - a Zn - a By the squeeze law for sequences, lim /(x n ) -f(zn) = f'(a). 8-+00 Zn - Zn 2.1.15 [W. R. Jones, M. D. Landau, Amer. Math. Monthly 76 (1969), 816-817]. (a) Note first that / is continuous only at 1. If {xnJ is a sequence of rationals different from 1 converging to 1, then lim f(xn) -1 = 1im (xn + 1) = 2. n-+oo Xn - 1 oo If {Zn} is a sequence of irrationals converging to 1, then lim I(xn) - 1 = Jim 2 = 2. n-+oo Zn - 1 n-+oo Thus /'(1) = 2. Clearly, / is one-to-one on (0,2). The inverse function /-1 is defined on (0,3) except for the rationals with irrational square roots. This means that there are no interior points in the d om8- in of 1-1. So, (/-1 )'(1) CAnn ot be defined. (b) Note first that 1 is defined on (0, 2)UB, where B C (2, 7/2). Note also that the restriction of / to (0,2) is a function defined in (a). Thus 1'{I) = 2. Since J(B) = AJ the range of 1 contains (0,3). However, (/- 1 )'{1) does not exist, because each neighborhood of 1 = /(1) contains images under f of points in (0,2) and images of points in B. Consequently, even the limit of 1-1 at 1 does not exist. 
2.1. The Derivative of a Real Function 221 2.1.16. By a theorem of Liouville (see, e.g., J.C. Oxtoby, Measure and Category, Springer-Verlag, 1980, p. 7), any algebraic surd % of degree k is badly approximable by rationals, in the sense that there exists M > 0 such that Ix - i I > M. for all rationals i. Consequently, I () - f(x) l. _ X  Mlaql. q It then follows by assumption that /'(x) = o. It is worth noting here that if, e.g., at} = 2- Q , then f is differen- tiable at each algebraic surd. 2.1.17. Let P(x) = a(z - X1)(X - X2)... (x - x n ). Then 11 P'(xJ:) = a II (Xi - Xj), k = 1,2,... ,n. j=l j;l=k The identity to be proved, Q(x)  Q(Xk) P(z) = f=: P'(Xt)(x - XI:) , is equivalent to Q(z) = t Q(x,,)P(z) I i=1 P'{Xi)(X - XIc) which, in turn, can be written as n n (x - Xj) n ;=1 °.J.k Q(x) = L Q(XIc) '; · Ie;::! n (Xi - Xj) ;=1 j#:k Since Q is a polynomial of degree at most n -1, it is enough to prove that this equality holds at n different points. Clearly, the equality holds at x = XI:, k = 1,2,. ..,n. 
222 Solutions. 2: Differentiation In particular, if Q(x) == I, then n 1 n 1 = L P'(x ) JI(X-Xj). 11:=1 11: )=1 jt Equating the coefficients of x"-I, we get n 1 L P'( ) = 0 for n  2. 1:=1 Xt 2.1.18. Apply the result in the foregoing problem with (a) P(x) = x(x + 1)(x + 2} ... (x + n) and Q(x) = n!. (b) P(x) = x(x + 2)(x + 4} · ... (x + 2n) and Q(x) == n!2 R . 2.1.19. Clearly, the derivative of 1/1 exists at each x such that /(:1;)  O. Moreover, if f(x) = 0 and f'(x) = 0, then 1/1'{x) = O. 2.1.20. There is a neighborhood of x where each of the functions lie does not change its sign. Consequently, Ilkl is differentiable at x and we have Cl 1!t.1)' (x) = ( In Ii Ifj:I ) ' (x) = t 1!t.1'(x) . n I lie I k=l. 1.=1 lit (x)1 k=l Our proof ends with the observation I/kl'(x) = sgn(/k(x»f{x). 2.1.21. Apply the result in the preceding problem with II: replaced by £;. 2.1.22. (a) Clearly, I and III are continuous only at z = O. Moreover, f'(O) = 1 and 1/1'(0) does not exist (compare with 2.1.19). (b) f and III are continuous only at Xle = *, k = 2, 3, . . . . It is easily - verifiable that j'(Xk) = 1, and that Ifl'(Xt) does not exist. 
2.1. The Derivative of a Real Function 223 2.1.23. Let e > 0 be chosen arbitrarily. By tbe definition of J{xo), (1) (/(xo) - c:)(x - xo) < f(x) - f(xo}  (/(xo) + e)(x - xo) for x > Xo sufficiently close to Xo. Likewise, (2) (/:(xo) - e)(x - xo) > I(x) -f(xo) > (f(xo) + e) (x - xo) for x < Xo sufficiently close to Xo. Continuity of I at Xo is an imm e- diate consequence of (1) and (2). 2.1.24. Since f(c) = max{/(x) : x e (a, b)}, we have I(x) - I(c) < 0 for x E (a, b). Therefore f'- (c) = fun f(x) - f(c) > o. z-t>c- X - C Similarly, I(c) :S o. H !(eo) = min{f(x) : x E (a, b)}, then we get f.+(eo) > 0 and I (Co)  o. 2.1.25. Clearly, the assertion is true if 1 is constant. Suppose, then, that f is not constant. Without loss of generality we can assume that I(a) = I(b) = o. Then, for example there exists Xl E (a, b) Cor which l(x1) > o. Let k be a real number such that 0 = I(b) < k < j(X1). Set c = sup{x E (Xhb) : I(x) > k}. Then f(x) < k for x E [c,b]. lvloreover, there exists a negative-valued sequence {h n } onvergent to zero and such that I (c + h n ) > k. Since I!... exists, / ' ( ) = 1 - f(e + h n ) - I{c) < 0 - C 1m h - . n-+oo .n So, we have proved that inf{J(x) : x E (at b)} < O. In an entirely sinlilar Inanner one can show that sup{j:.(x) : x E (a,b)} > o. It is worth noting here that an analogous result can be obtained for f. Namely, jnf{f(x) : X E (a,b)} < 0 < sup{!(x) : x E (a, b)}. 
224 Solutions. 2: Differentiation 2.1.26. To prove the assertion we apply the above result to the aux- iliary function Z t-+ fez) - 1(6) -f(a) (z - a). b-a A similar assertion can be proved for f, that is. int {/(z) : z e (a, b)}  I(b = :(a)  sup{/(z) : z e (a, b)}. 2.1.21. By the result in the foregoing problem, int{/(z) : z e (z,z + h)}  I(z + hl-/(z)  sup{/(z) : z e (x,.1: + h)} for z e (a,b) and 0 < h so small that z + h is in (a,b). Since J'- is continuous on (a,b), upon passage to the limit as h --+ 0+ we get J(%) = J!.. (x). 2.1.28. It follows from the result in the preceding problem that such a function does not exist. 2.1.29. By assumption, I vanishes at at least one point of an open interval ( a, b). Set e = inf{x e (a, b) : f(x) = O}. Then f(c) = O. Since /'(a) > 0, we have I(z) > 0 for z e (a,c). Moreover, since f'(e) exists, f'(c) = lim I{e + h) -f{e) = lim fCe + h) S o. h-+O- h h-+O- h 2.1.30. Clearly, (1 + x2)f'(x) = 1, which implies (1 + z2)/"(x) + 2xf' (x) = o. Using induction one can show that (1 + z2)/(n){z) + 2(n -1)xf(n-l)(x) + (n - 2){n _1)/(n-2)(x) = 0.. H we take z = 0, then by induction again, we get 1(2m)(o) = 0 and /(2m+l)(O) = (-1)m(2m)!. 2.1.31. The identities can be established easily by induction. 
2.1. The Derivative of a Real Function 225 2.1.32. (a) Apply the Leibniz formula (J(x)g(x»(n) =  ()/(n-t)(x)g(")(X) and the identity (a) in the foregoing problem. (b) Apply the Leibniz formula and the identity (b) in the foregoing problem. 2.1.33. It is easy to see that if :z: > 1, then fez) > 0, f'(z) > 0 and I"(z) < O. Now differentiating (f(%»2 = z2-1 n times, n  3, and using the Leibniz formula, we get 2/(x)/(n)(x) + E ()j<t)(x)/(n-t)(x) = O. The desired result can be obtained by induction. 2.1.34. We have 2n J2n{X) = In(l + z2n) = Lln(z - WI:), k=1 where Wi = COS (2";;.1)" + i sin (2i;':).!. . Hence ")n IJ:n)(x) = -(2n -I)!  ( 1 )? . L.J Z - Wi ..n 1c=1 Putting x = -1, we get 2n 1n)(_I) = -(2n-I)! L (I 1 )2n ' t=l + Wi An easy calculation shows that ( ) 1 2n ( ) k j!2n) ( -1 ) = i 2n - 1 .  -1 . ..n 22n L.J 2n ( 21e-l ) 11' 1e=1 COS 4n 
226 Solutions. 2: Differentiation Since IJ:n) (-1) is real, we see that fJ:n) (-1) = O. 2.1.35. Denote by L(z) and R(x) the left and right side of the iden- tity to be proved. Clearly, Land R are polynomials of degree n+ 1 and L(O) = R(O) = o. So, it is enough to show that L'(z) = R'(z), 2: e III We have n pet) (0) L'(z) = L , zit = P(z), i=O k. n p(Ir) ( ) n p(k+I) ( ) R'(2:) = L(-l)t I x. zt + L(-l)t z x t + 1 k=O k. k=O (k + 1)1 p<n+I) ( ) = P(z) + (_1)" z z"+1 = P(z). (n + I)! 2.1.36. There is a neighborhood of zero where f is positive. Thus ( , /'(x) 1 n ( ) In /(z») = /(z) = 1 _ lX +... + 1 _ n:Z: = 9 X · Hence /'(z) = /(%)9(X) and /'(0) = 1 +2+.. .+n > O. Moreove ( ,1 \1 ) (1) (i) ( ) _ ., " I . . . A n 9 X - I. (1 _ lZ)'+l + + (1 _ n%)i+l · By the Leibniz fonnula, i-I ( ) /(k)(x) = L k -: 1 g(i) (X)/(k-I-.) (:1:). i= 0 'I In view of (1), it then follows by induction that /(")(0) > 0, Ie e N. 2.1.31. We will proceed by induction. For n = 1 the equality is obvious. Ass umin g the equality to hold for Ie  n, we will show it for 
2.1. The Derivative of a Real Function 227 n+ 1. We have (_I)n+1 (Xn/ G) yn+1) = (_I)n+1 ((xn/ G))') (n) = (-I)n+1 n (xn-ll G)) (nl - (_I)n+1 (xn-2/, G )) (n) = _ n fen) ( ! ) _ (_l)n-l ( X n - 2f1 ( ! )) (n>. zn+l X :& Moreover, (-W- I (xn-2 l' G )) (n) = (_I)n-1 ( (n-2 I' G )) (.HI)'. The induction hypotheses applied to J' with k = n - 1 gives .;nj!n) G) = (_I)n-1 (xn-2 l' G)) (n-l). Consequently, (_I)n+1 (xn/G)ynH) =- x:+1 /(R) G ) - (.;n/(R) G))' = ] f(n+l) ( ! ) . xn+2 X 2.1.38. The proof of this well known formula presented here is based on S. Roman's paper [Amer. A1ath. lvIonthly 87 (1980), 805-809]. Although methods of functional analysis are applied, the proof is elementary. Linear functionals L : 'P -+ IR defined on the set 'P of all polynomials v..ith real coefficients will be considered. Let (L, P{x» denote the value of L at the polynomial P{x). Let Ak be a linear functional such that (A le II ) _ r.r , x - n.U'I,k, where { I if n = k, 6n.k = 0 if n  k. 
228 Solutions. 2: Differentiation It is worth noting here that the value of Ai at x" is (xn)f : . Let 00 E akA k , ale e IIi, denote the linear functional defined by k;::O (  akAk,p(Z») =  ak(Ak,p(z». Since (A", P{z» = 0 for almost all k, there are only finitely many nonzero terms in the sum on the right side of this equality. The task is now to show that if L is a linear functional on 'P, then (1) L = f: (L,zk) A k . 1e =O k! Indeed, for n > 0, (f (L k7 ) Ak,z,, ) = f (L k 7 k ) (A k , x") = (L,x"). k=O k=O Since L and At are linear, we get (L,P{z» = (  (L ic 7 k ) A k , P(z) ) for any polynomial P, which proves (1). According to the fact men- tioned above that A" at zn is (ZB)(k)lx = 0' it seems natural to define the operation on Ai by setting A' Ai = AA:+j. In view of (1) this operation can be extended to the operation defined for any L, M : 'P  J;t as follows LM = f: (L, ) Ak f: (M :) Aj = f: c,.A", A:=O k. ;=0'. n=O where =  (L,zlc) (M,zn-k) = .!.  ( n ) (L k )(M n-A: ) en L.J k l ( _ k) 1 I L., k ' z , x · k=O · n . n. 1e =O 
2.1. The Derivative of a Real Function 229 Hence by (1), (2) (LM,3: n ) = t (  ) (L,Zk)(M,ZR-k). k=O Using induction, one can show that (L 1 ... Lj,ZR) (3) - t k 1 ! !. kj! (L Io 3:'" )(L2,3:"2)... (L j , 3:"1). k1....,kj=O kl+...+I:;=R Now define the formal derivative L' of L by (AO)' = 0, (A k )' = kA k - 1 for Ie eN and L' = t (L k 7") kA"-l. k=l Now we show that for any PEP, (4) (L', P(x» = (L, zP(x». Clearly, it is enough to show that «(Ak)' ,XR) = (Ak,xR+I). We have «(A k )' ,zn) = (kAk-l,xR) = kn!6n.-1 = (n+l)!On+l,k = (Ak,:c n + 1 ). To prove the Faa. di Bruno formula, set h n = hCn)(t), gn = g(n) (t), In = I(R) (u)lu = g(t). Clearly, hl = /19h h 2 = 1192 + /2gr, hs = /193 + /239192 + /3gf. It can be shown by induction that (5) n h n = E Ik 1 n,k(g.,g2'." ,gn), 1:=1 
230 Solutions. 2: Differentiation where I n ,"(9h92," .,9n) is independent of Ij,; = 0,1,2,..., n. To determine In.t(91t92,...,9n), choose J(t) = eot, a e JIl Then lie = a'eo g (') and h n = (eOg(t»(n) . It follows from (5) that (n) n e-a,(t) (eOg(t») = E a t ' n .Ic(9h92,... ,9n). 1e=1 (6) Put BnCt) = e-ag(t) (eal(t») (n) , n  o. It then Collows by the Leibniz formula that Bn(t) = e-og(t) (ag l (t)eO,(t)) (n-I) n-I ( n -1 ) ( ) (n-Ic-I) (7) = a · e-o,(t) E Ie g"+1 (t) eo,(t) n-l ( 1 ) = a En; g"+1(t)B n -f:-I(t). t=o For an arbitrarily fixed tel, set Bft = Bn(t) and define functional! L and M on P by (L, zn) = Bra, (M, x") = 9n. Then (L, 1) = Bo = 1 and (AI. I) = 90 = get). Moreover, by (1), L = f: B: A" and M = f: Af:. '=0 k. 1:= 0 k. Now (7) combined with (2) and (4) yields (L,xn) = a E ( n  1 ) (kl,z'+l)(L,zn-l-k) Ic=O k =a E (n; 1) (M ' ,x'HL. xn-l-f:) k=O = a(M'L, :z:"-I). So, (L', Z"-I) = a(AI'L, :z:n-l). or in other words, L' = altt'L. rfhis Cormal differential equation has solutions of the form L = ceG(Af-,o), where c is a real constant. By tbe initial value condi- tion, 1 = Bo = (L,I) = (ceo(Af-IO), 1) = c. Hence L = eo(Af-,o). It 
2.1. The Derivative of a Real Function 231 then follows tbat co . B.. = (L,z") = (eO(AI-IO),x") = E i!(CM - !1o)t,zn) '=0 00' n , =E; E _ n.. , <M-90,zll)(M-!JO,zb)...(M-90,:z:1') 1..-'" h_ J l =O 'I!" - 'i. -=u 1......,.- JI +-"+i.=n 00" n I =  !.. k! '" "I"  ", 9i,9h'" 9j" £J  31 32 - - · Ji- k=O J.......=1 Ja+".+i.=n Equating the coefficients of 0' in (6) gives , n I ( ) n.  9JI 9j2 9J. n.t 9t, 92,. .. ,9n = L ! L.J "71 - "71 · - · -=t  j j . _ 1 31, 32- Jit. I...', .- JI"+J.=n n! n k! ( 9. ) '1 ( 92 ) '2 ( 9n ) '. = ki E k l f - - · SRI II 2f' .. fif · t.....,i.. -0 It, +...+I:..=i i, +2i2+...+ni. -n Finally,   n n! ( 91 ) " ( 9n ) '. L.J Jt 1 n,t(9h .. ',9n) = L.J Jt E k l !.., k"! If .. 'Ri · '=1 '=1 "1....,i.=O lei +...+". It Ie, +2":t+...+nk. =n which ends the proof.. 2.1.39. (a) We have { 2 -:'J- J'(z) = -:e. because (see, e.g., 1.1.12) if z  0, if z = 0, -* Urn e · = O. z-+o Z 
232. Solutions. 2: Differentiation It then follows that I' is continuous on III Moreover, for z =F 0, J"(x) = e--:Z ( .! _ 2.3 ) . x 6 x 4 Again by the result in 1.1.12. it can be shown that 1"(0) = O. Consequently, I" is also continuous on 1i. Finally, observe that I(n)(x) = { e-* P ()  z 10, o if z = 0, where P is a polynomial. Therefore for every n E N, j(n) is continuous on _.. (b) As in (a) one can show that g(n)(o) = 0 for n e Nt and that 9 is in Coo (JR). (c) The function is a product of two functions Ih/2 E COO(R). 1J deed, 11 (x) = g( - a) and 12(Z) = g(b - x), where 9 is definE in (b). 2.1.40. We have I"(x) = g'(j(x»/'(x) = g'(/(x»9(j(3:», j'Il(Z) = 9"(/(X»(9(/(x»2 + (g'(/(z»)2 g (/(z». Therefore I" and /'" are continuous on (a, b). One can show by iJ duction that I(n), n > 3, are sums of products of derivatives gCI:) (I k = 0, 1,2, . . . . n - 1. Consequently, they are continuous on ( a, b). 2.1.41. H Q  O. then I"(x) = -fJI'(x) - 'Y/(z) . a Consequently, /,,'(x) = -Pf"(z) -'Y/'(x) = (p2 - -YCk)/'(:) +-y{Jf(z) . Q a One can show by induction that the nth derivative of j is a linear combination of 1 and J'. H Q = 0, then {J i: 0 and I'(x) = =; j(x). By induction once . agam. n I(n)(z) = (-l)n/(z). 
2.2. Mean Value Theorems 233 2.2. Mean Value Theorems 2.2.1. The auxiliary function h(z) = eO% f(x), z E [a, b), satisfies the conditions of Rolle's theorem. Hence there is Xo e ( at b) such that 0= h'(xo) = (af(zo) + j'(xo»e QZo . Consequently, a/(xo) + /'(xo) = O. 2.2.2. The function h(z) = e 9 (z) f(x), x e [a,b], satisfies the condi- tions of Rolle's theorem. Therefore there is Xo E (a, b) such that 0= h'(xo) = (g'(xo)f(%o) + f'(xo»e9(zo). Hence g'(xo)f(xo) + I'(xo) = O. 2.2.3. Apply Rolle's theorem to the function hex) = I) , x E [a,b]. 2.2.4. Take hex) = J2{x) - x 2 t X E [a, b], in Rolle's theorem. 2.2.5. Apply Rolle's theorem to the function hex) = : , x E [a, b]. 2.2.6. Note that the polynomial Q(x) = ao x R + 1 + al x R + · · · + an:c n+l n satisfies the conditions of Rolle's theorem on the interval (0,1]. 2.2.7. The function hex) = an lnn+l x+-. -+ a 2 ln 3 x+ ln2 x+ lnx n+ 1 3 2 l' satisfies the hypotheses of Rolle's theorem. x E [I, e 2 ], 2.2.8. By Rolle's theorem, between two real zeros of the polynomial P there is at least one real zero of P'. Moreover, each zero of P of order k (k > 2) is a zero of P of order (k - 1). Thus there are n - 1 zeros of pi, counted according to multiplicity. 2.2.9. By Rolle's theorem applied to f on [a, b] there is c e (at b) such that J'(c) = O. Next, by Rolle's theorem gain applied to I' on [a,c], we see that there is Zl e (ate) C (a,b) such that f"(zl) = o. 
234 Solutions. 2: Differentiation 2.2.10. .pply the reasoning similar to that used in the solution of the foregoing problem. 2.2.11. (a) Set P(z) = x 13 + 7Z3 - 5. Then P(O) = -5 and lim P(x) = +00. Z-P By the intermediate value property there is at least one positive root of P( z) = O. IT there were two distinct positive roots, then Rolle's theorem would imply P'(zo) = 0 for some positive Zoo This would contradict the fact that P' (x) = 0 if and only if z = o. Finally, observe that P(x) < 0 Cor x < o. (b) Consider the function /(z) = G f + G f -1. Then /(2) = O. If J vanished at another point, then by Rolle's theorem its derivative would vanish at at least one point, which would contradict the fact that /' (x) < 0 for all x e R. 2.2.12. We will proceed by induction. For n = 1 the equation OIXO'I = 0 does not possess zeros in (O,oo). Assuming that for an arbitrarily chosen n E N the equation 01XQ1 + a2xQ2 + . . . + a,.x Qn = 0 has at most n - 1 roots in (0, 00), we consider the equation QIZO'I + a2x Q2 +... + anxO n + a n + 1X On+1 = 0, which can be rewritten as 41 + l12X02-01 +... + Bn+lXQa+1-01 = o. H the last equation had more than n positive roots, then by Rolle's theorem the derivative of the function on the left side of this equal- ity would have at least n positive zeros. This would contradict the induction hypotheses. 2.2.13. Apply the last result, replacing :z; by e Z . 
2.2. Mean Value Theorems 235 2.2.14. Clearly, F(a) = F(b) = 0, and F is continuous on (a,b]. Moreover, F is differentiable on (a, b) and f'ex) g/(X) II' (x) F'(x) = det J(a) yea) h(a) . f(b) g{b) h(b) By Rolle's theorem there is Xo E (a, b) such that F'(xo) = O. Taking g(z) = x and hex) = 1 for x E [a, b], we get I'(xo) 1 0 F'{xo) = det J(a) a 1 = 0, J(b) b 1 which gives J(b) - I(a) = J'(xo)(b - a). So, we have derived the mean value theorem. To get the generalized mean value theorem it is enough to take hex) = 1. 2.2.15. It follows from the mean value theorem that there exist Xl in (0,1) and X2 in (1,2) such that /'(Xl) = f(l) - f(O) = 1 and f'(X2) = f(2) -/(1) = 1. Now the assertion follows from Rolle's theorem applied to J' on [Xl, X2]. 2.2.16. Since I is Dot a linear fun(:tioD, there is c E (a, b) such that f(c) < f(a) + I(b) - I{a) (c-a) or f(e) > J{a)+ /(b) -fea) (c-a). b-a b-a Suppose, for example, that fee) < f(a) + f(b} - I(a) (c - a). b-a Then I(e) - I(a) f(b) - J(a) d f(e) - /(b) /(b) -f(a) < an > . c-a b-a c-b b-a Consequently, the assertion follows from the mean value theorem. Analogous reasoning can be applied to the case where f(e) > f(a) + f(b = (a) (e - a). 
236 Solutions. 2: DifI'erentiation 2.2.17. Suppose first that Xo   . Then either [O,xo] or [xo,I] has length less than l. Suppose, for example, that this is [Xo, 1]. By the mean value theorem, -1 = /(1) - /(:co) = f'ee), l-xo l-xo and consequently, If'(c)1 > 2. Suppose now that Xo =  and that I is linear on [O,]. Then /(x) = 2x for x E [0,  ] . Shtce I' (  ) = 2, there is Xl > t such that /(Xl) > 1. In this case the assertion follows from the mean value theorem applied to I on [Xl, 1]. Finally, suppose that I is not linear on [0,  ] . If there is X2 E (O, t) such that I (X2) > 2X2, then to get the desired result it is enough to apply the mean value theorem on (0, X2]. H /(X2) < 2X2, then one can apply the mean value theorem on [X2' tJ. 2.2.18. Applying the generalized mean value theorem to the fun tiODS X H- lc; and X to? ; on [a,b], we see that 6J(a) _ aJ(b) Ib) _ Ia) ;r1/'(z-/(Zlt , b _ a = ! _ 1. - _ = J{Xl) - xII (Xl). b G Zl 2.2.19. By the mean value theorem, for Xl, Z2 e [0,(0), 1 lln(l + Xl) -!n(1 + x2)1 = 1 IXI - x21 < I X I - x21. +xo Likewise, IIn(l +) -In(l +)I = 12 Xo 2 1 x l - x21 < I X l - x21 +xo and 1 1 arctan Xl - arctanx21 = 1 0) I X I - %21 < I X 1 - x21- +xo - 2.2.20. Fix Xo e (a, b). Then for every x E (a, b) (by the mean value theorem) there is c between Xo and x such that I' (x) - I' (xo) = /"(C)(X - xo). Hence 1/'(x)1 < Mix - xol + 1/'(xo)1  lYf(b - a) + 1/'(xo)l, 
2.2. Mean Value Theorems 237 which means that /' is bounded. It then follows (as in the solution of the foregoing problem) that f is uniformly continuous on {at b). 2.2.21. Consider the function x t-+ arctan/(x). By the mean value theorem, for a < Xl < X2 < b, X2 - Xl > 1r, we have 1/'(xo)1 I arctan/(x2) - arctan/(xt)1 = 1 + f2(xo) (Z2 - Xl). Hence, If'(xo)1 1r  1 + 1 2 (XO) (X2 - %1), and consequently, If'(xo>J < 11" < 1. I + /2 (xo) - 3;2 - Xl 2.2.22. We have I' (xo) arctan f{x'J) - arctanl(xl) = 1 + J2(xo) (X2 - XI) for a < XI < X2 < b. By (ll), arctanf{x2) - arctanf(xt) > -(Z2 - Xl). Letting X2 -? b- and Xl -. a+ and using (i), we see that -1f'  -(b - a). 2.2.23. By the mean value theorem, I!...(b) = lim 1(6 + h) - f(b) = lim I'(b + Oh) = A. h- h hO- 2.2.24. Since f'(x) = O(x), there are M > 0 and Xo E (0 , 00) such that If'(z») < Mx for X > %0. By the mean value theorem, I/(x) - f(xo}1 = If'(xo + 8(x - xo»I(z - %0)  1\.-[(xo + 6(x - zo»(x - xo) S l\tfx(x - zo) < Mz 2 for x > Xo. 
2.38. Solutions. 2: Differentiation 2.2.25. The result follows CrOID Rolle's theorem applied to the aux- iliary function  ( hW-hW ) h(x) = Li /k(X) - /1c(a) - (gJ:(x) - Ok (a» (b) _ () '. 1  a 2.2.26. Assume first that J is uniformly differentiable on [a, b]. Then for any sequence {h n } converging to zero such that h n =F 0 and z + h n e I for :z; e [a, b), the sequence of functions { /(Z+hh-/(%) } is uniformly convergent on [a, b) to /'. By the result in 1.2.34, I' is continuous on [a, b). Assume now that I' is continuous on la, bJ. By the mean value theorem, for x E [a, b), x + h e I, fex + h) - f(x) _ /'(x) = f'(x + fJh) - f'(x) h with some 0 < (J < 1. It then follows by uniform continuity of I' on la, b] that I is uniformly differentiable. 2.2.21. Since f is continuous on [a, b], it is bounded; that is, there is A  0 such that If(x)1 S A for x e [a,b]. By assumption, Ig'(2:)1 :::; 1 IA lg(z)l. Now let [c, d] be a subinterval of [a, b] whose length is not greater than  =  and such that gee) = O. For an Xo E [c,d], we get Ig(xo) - g(e) I = Ig(xo)1 = (2:0 - e)lg'(xl)1   Igdl . By repeating the process t one can find a decreasing sequence {Xn} of points in [c l dJ such that 1 1 Ig(xo)1 ::; 2 10(%1)1 :S ... :S lg(Xn)1 :S .... Consequently, g(xo) = O. To end the proof it is enough to decompose [a, b] into a finite number of subintervals with lengths less than or equal to i-. It is worth noting here that the assumption of continuity of I on [a, b) can be replaced by its boundedness on [a, b]. 
2.2. Mean Value Theorems 239 2.2.28. By the generalized mean value theorem, /(2z) _ ful 2%1 1 z = I«() - <f'«(), 2z - Z where z < , < 2x. Hence 1(2z) _ I(x) = 5- ( J'{() _ !«(» ) . 2x x 2x ( This implies that o < II'({)I < 21 f) _ f) I + f>j. Upon passage to the limit as x --. 00 we obtain the desired result. 2.2.29. The result is a direct consequence of 1.6.30. 2.2.30. By assumption, (1) I'(px + qy) = I'(qx + py) for x =F y. IT p :/: qt then I' is a constant function. Indeed, if I'(xl) 1: 1'(X2), then, taking p p-l and p-l p x = 2p _ 1 XI + 2p _ 1 X2 Y = 2p _ 1 Xl + 2p _ 1 X2, we have XI = p.£ + {l- p)y and X2 = py + (1- p)x, \vhich contradicts (1). So we have proved that if p :F q, then I is a linear function. If p = q = %, then by the result in the foregoing problem, I is a ... polynomial of the second degree. 2.2.31. For [a, b) C I, assume, for example, that I'(a) < 1'(6). Let A be a number such that f'(a) < .,\ < f'(b). Consider the function given by setting g(:c) = f(x) - Ax. Then g'(a) < 0 and g'{b) > O. Consequently, 9 attains its minllnum on [at b] at an Xo in the open interval (a, b). Therefore g'(%o) = 0: or in other words, f'(xo} = "\. 
240 Solutions. 2: Differentiation 2.2.32. (a) Given e > 0, let a > 0 be such that I/(z) + /'(x)1 < E for z  a. Then by the generalized mean value theorem there is { E (a, z) such that ez fez) - eO I(a) = f() + I'(). e;%-e,G Thus I/(z) -/(a)eO-zJ < £11 - eO-Zit which gives I/(z)1 < If(a)le a - z + Ell - eo-zi. Consequently, I/(z)1 < 2E for sufficiently large z. (b) Apply the generalized mean value theorem to z t-+ e,fi fez) and z t-+ e,f%, and proceed as in (a). 2.2.33. It follows from the hypotheses tbat tbe function z t-+ e-:Z: /(z) has at least three distinct zeros in [a, b). Hence by Rolle's theorem its derivative z t-+ e-Z(f'(z) - fez» has at least two distinct zeros in this interval. This in turn implies that the second derivative has at least one zero, which means that the equation e-Z(/(z)+ f"(z)-2/'(z» = o has at least one root in [a, b). 2.2.34. Observe first that Q(z) = F(x)G(x)t where F(z) = P'(z) +zP(z) = e-';' (e P(z) r. G(z) = zP'(z) + P(z) = (xP(z»' . Let 1 < lJt < 42 < ... < an be zeros of the polynomial P. By Rolle's theorem F has n -1 zeros, say bhi = 1,2,...,n -1, and G has n zeros, say CiJ i = 1,2,. . . J n. We can assume that 1 < al < b 1 < 112 <  < · .. < b n -l < an, o < Cl < al < C2 < Cl2 < ... < en < an. If bi :F Ci+l for i = 1,2,. . . J n - 1, then the polynomial Q has at least 2n - 1 zeros. Now suppose that there is i such that  = Ci+l = r. Then P'(r) + rP(r) = 0 = rP'(r) + Per). Hence (r 2 -1)P(r) = O. Since r > 1, this gives Per) = 0, a contradiction. 
2.2. Mean Value Theorems 241 2.2.85. Let Xl < %2 < ... < X m be zeros of P. It follows from the hypotheses that P'(%m) > 0 and P'(Xm-l} < 0, and P'(Zm-2) > 0,... . Moreover, we see that Q(xm) < 0, Q(Xm-l) > 0,... . If m is odds then Q(Xt) < o. If m is even, then Q(%l) > o. Consequently, by Rolle's theorem, Q has at least m + 1 real zeros when m is odd, and at least m real zeros when m is even. Now we show that all real zeros of Q are distinct. Since all zeros of P are real and distinct, (p'{Z»2 > P(X)PIl(Z) for z E Il Indeed, since P(x) = am(x - Xl)(Z - X2)... (z - zm), we see that for % #: %j, j = 1,2, . . . , m, P'(x) m 1 P(x) = L x-x. " j=l J Hence m 1 P(x)P"(z) - (P'(Z»2 = _P 2 (z) H (z- Zj)2 < O. Moreover, for Z = Zj, (P'(Zj»2 > 0 = P(Zj)P"(zJ). Thus the inequality (p'(z»2 > P(X)PIl(X) is proved. Consequently, P(x)Q'(x) = P(x)(2P(X)P'(X) - P"(x» = 2P'(%)(p2(Z) - P/(%» + 2(p'(%»2 - P(z)P"(z) > 2P'(x)p2(x) - (p'(z»2. This means that (1) P(x)Q'(Z) > 2P'(x)Q(x), which shows that all zeros of Q are of the first order. If 'lit and 'lI2 are two consecutive zeros of Q, then Q' ('lI1) and Q' (Y2) have distinct signs. Then by (1) P(Yl) and P(Y2) also have distinct signs. Therefore between two consecutive zeros of Q there is at least one zero of P. Thus, when m is odd, if Q had more than m + I real zeros, then P would have more than m real zeros, which would contradict the hypotheses. Similarly, when m is even, if Q had more than m real 
242 Solutions. 2: Differentiation zeros, it would have at least m + 2 real zeros, and consequently P would have more than m real zeros. A contradiction. 2.2.36 [G. Peyser, Amer. lvIath. Monthly 74 (1967), 1102-1104]. Let us observe that if all zeros of a polynomial P of degree n are real, then by Rolle's theorem all zeros of P' are real and lie between the zeros of P. Thus P' is of the form given in the problem. We will prove only the first assertion, because the proof of the second one is analogous. Clearly, P(x) = Q(x)(x - an). Hence (*) P'(z) = Q'(z)(x - an) + Q(x). It is enough to consider the case ai < 0;+1. Suppose, for example, that P(x) > 0 for x e (ai,oi+l). Then Q(z) < 0 for x E (oi,ai+l). Moreover, it follows from (*) that Q'(x) < 0 for x E (ai,Ci), and Q'(e;) < O. Consequently, d i > Ci, which ends the proof of the first assertion. 2.2.37 [G. Peyser, Amer. Math. Monthly 74 (1967), 1102-1104]. A surne that Bn-l < an and e > O. Clearly, S(x) = P(x) - ER(x), where R(x) = (x - a2) · . . (x - an). Suppose that, e.g., P(z) < 0 for z E (an-ban). Then also Sex) < 0 and R(x) < 0 for x E (an-I, an). Since (1) S'(x) = P'(x} -eR'(x), we see that S'(Cn-I) = -ER'(Cn-I). It follows from the foregoing problem that R'(Cn-l) > O. By (1) we have S'(en-I) < o. Since 8' changes its sign from negative to positive at a point in tbe interval (an-I, an), we see that In-l > Cn-i. The other assertion can be proved in an entirely similar manner. 2.2.38 [G. Peyser, Amer. Iath. Monthly 74 (1967), 1102-1104]. Set "(x) = (x - ai)i(x - 0.+1). Hi = 2,3,..., n - 1, then W'(x) = 0 for x = aj and for iai+l + a; ai+1 - aj x = C = i + 1 = ai+l - i + 1 · 
2.2. Mean Value Theorems 243 If i = 1, then W' vanishes only at c. Applying the first result in 2.2.36 (n - i-I) times, and Dext tbe first result in the foregoing problem (i-I) times with E equal to a,-altOi -lJ2t... ,Oi-ai-l, successively, we arrive at lJ.+I - a. Cj < c = °i + l - · 1 · - t+ To get the left inequality one tan apply the second parts of the last two problems. 2.2.39. Observe that, by the IDcan value theoln, for every z in (0, 1/ K) n [0, 1] we bave 1/(z)1  K zl/(zl)1  K 2 ;cz,I/(x2)1  ...  KRXXI .. · %n-II/(xn)I, where 0 < x.. < Zn-1 < ... < %. < %. Thus I/(z)1 S (Kz)nl/(zn)l. Since I is bounded, it then follows that I(z) == 0 on [0,1/ K]n[O, 1]. If K  1, one can show in an entirely similar manner tbat fez) = 0 on [1/ K, 2/ K]. Repeating the process finitely many times gives I(z) = 0 on [0,1]. 2.2.40. For %. e Jl and :1:3 e J 31 / (Ic-I) ( Z3 ) - / (k-I) ( Zl ) = 1(1c) () Za - XI with some C E (ZII%3). Hence mt(J)  1 (1/(lr-I)(za)1 + I/(k-I)(Zl)1) %3 - XI S (IP-I)(Zdl + I/(A;-I)(XI )1). Taking the infimum over all ZI e Jl and .1:3 e Js gives the desired inequality. 2.2.41. We win proceed by induction on k. For k = 1, the inequality can be concluded from the mean value theorem and CrODI the fact that 1/(z)1  1. Assume that the iuequality to be proved bolds for an arbitrarily chosen positive integer k. Then by the result in the 
244 Solutions. 2: Differentiation foregoing problem we get 1 mk+l (J)  2 (ml;(J 1 ) + mk(JS» 1 ( 1 '('+1) I: 1 '('+1) Ie ) < :\;' :\f2 2 k +):f2 2 k '(i+1) t ( 1 1 ) = 2 2  2 + 2 · Putting 1 = 3 = 2(:;1) and 2 = kl J we obtain 2 Ci+1Y.+2) (k + 1)1;+1 fflk+t(J)  k+1 · 2.2.42. We have p(p -l )( ) (p )' (p + I)! 2 n! n-..u..l Z = -1 .llp-l + IIp+lZ +...+ anx I'T. 2! (n - p + 1)! It follows from Rollets theorem that between two consecutive real zeros of P there is exactly one real zero of pl. Consequently, the polynomial p(p-l) bas n - p + 1 distinct real zeros and pep) has n - p distinct real zeros. As we have already mentioned, between two consecutive real7ros of pCP-I) there is exactly one real zero of pep). Suppose, contrary to our claim, that Clp-l and apor1 have the same sign. Without loss of generality, we can assume that both are positive. Then there is E > 0 such that p(p-I) is decreasing in (-e,O) and is increasing in (O,e). Clearly, p(p)(O) = O. If there were no other zeros of p(p), then we would get p(p-l) (x) > p(p-l) (0) > 0 for x :F 0, a contradiction. If p(p) has nonzero zeros, then denote by Zo #: 0 tbe nearest to zero. So between 0 and 2:0 there is a zero of p(p-l). On the other hand, p(P-l)(X) > ° on an open interval with ends 0 and :Co, a contradiction. 
2.3. Taylor's Formula and L'Hospital's Rule 245 2.3. Taylor's Formula and L'Hospital's Rule 2.3.1. Note that for n = 1 the formula follows immediately from the definition of f'(%o). Now for n > 1 put ( /'(xo) I(n) (xo) ) rn(x) = I(x)- /(zo) + l! (x - xo) + .. · + n! (x - zo)n . Then rn(xo) = r(xo) = ... = rn)(xo) = O. By the definition of the nth derivative, rn-l)(z) = rt n - 1 ) (x) - rn-l)(xo) = Tn){XO)(x - xo) + o(x - xo). Consequently, rh n - 1 )(x) = o(z - xo). Using the mean value theorem, we get rn-2){z) = rn-2)(x) - r"-2)(xo) = rn-l)(e){x - xo), wbere r. is a. point in an npE'n interval with the endpoints :r. and Xo. Since Ie - xol < Ix - xol, we see tbat rn-2) (x) = o«x - xO)2). By repeating the process n times we obtain Tn(X) = o«x - xo)"). 2.3.2. For Xt Xo E [a, b), set ( r) » ) rn(z) = f(x)- f(xo) + l! (x - xo) +... + n! (x - %o)ft · Without loss of generality we may assume that x > xo. On [x0 1 x] define the auxiliary function cp by cp(z) = 1(:1:) - (/(Z) + f'i) (z - z) + ... + I(:Z) (z - Z)R) . We have (1) cp(Xo) = rn(x) and tp(z) = O. Moreover, 'P' exists in (xo,x) and / (n+l) ( z ) (2) </,'(z) = - I (x - z)n. n. By the generalized mean vdlue theorem, cp(x) - rp(xo) _ (,O'(c) - 1 t/J(x) -1/1(%0) tJI(c) 
246 Solutions. 2: Differentiation where !/J is continuous on (xo, x] \\oith non vanishing derivative on (x, xo). This combined with (1) and (2) gives . .,p(x) -.,p(xo) j(n+l) (c) n rn(x) = 1/J'(c) . n! (x - c) · Taking 1/J{z) = (x - z)P and writing c = Xo + 9(x - xo), we arrive at ( ) _ /("+I)(XO + 6(:& - xo» (1 9) R+I-P ( ) n+l r fI x - , - x - Xo . n.p 2.3.3. Note that these results are contained as special cases in the foregoing problem. Indeed, it is enough to take (a) p = n + 1, (b) p= 1. 2.3.4. Integration by parts gives I(x) - I(xo) = 1 z f'(t)dt = [-(x - t)f'(t>J: o + 1 :1: (x - t)f"(t)dt. ZO %0 Hence J(x) = J(xo) + J'to) (x - xo) + 1 :1: (x - t)f" (t)dt. . ZO To get the desired version of Taylor's formula it suffices to repeat the above reasoning n tiDIes. 2.3.5. For n = 1, 1 % 1 '2 1 z R2(X) = j(2)(tl)dt 1 dt 2 = (f'(t2) - jl(XO» dt 2 %0 zo zo =jlx) - J(xo) - (x - XO)J/(XO). To derive the forolu1a 1 induetion can be used. 2.3.6. By Taylor's fonnu)a, with the Lagrange form for the remainder (see, e.g.., 2.3.3 (a», ";1 + x = 1 + !z - !z1. + (1 + 8x)-l x 3 2 8 3!8 
2.3. Taylor's Formula and L'Hospital's Rule 247 with some 0 < (J < 1. Consequently, 31x l 3 v'2lxl3 1 ' v I + x - ( 1 + 1% - 1%2) 1 < = < _ 1 % 1 3 . 2 8 - 4$( i) I 4 2 2.3. '1. Applying Taylor's formula with the Lagrange form for the remainder to I(x) = (1 + z)Q t we get (1 ) 0 I Q(er - 1)(1 + 8Z)0-2 -- +% = +0%+ 2 z- with some 0 < (J < 1. Now it is enough to observe that Q(Q -1)(1 + 6%)0-2 2 >0 for 0>1 or 0<0, and a(a - 1)( + 8x)0-2 < 0 for 0 < Q < 1. 2.3.8. By Taylor's formula, f(x) -f(O) _ /'(0)% + ll"(8 1 (z»x 2 g(x) - g(O) - 9'(0)% + 19"(8 2 (x»X 2 · On the other band, the mean value theorem yields /'(8(%» _ /'(0) + 8(:1;)/"(9 3 (x» o'(8(x» - g'(O) + 6(%)9"(6.. (x» · Using the above equalities and continuity of J" and g" at zero, one easily checks that 1im Sex) = !. z....o+ z 2 . 2.3.9. (a) By Taylor's Connu1a t 1(0) = J(z: + (-x» = J(x) + I':) (-z) + J'x) (_x)2 j(n) (x) n /(n+I)(% - 8 1 z) n 1 +...+ n! (-3;) + (n+ 1)1 (-z) +. Taking 6 = 1 - 8. gives the desired equality. (b) Observe that J ( lZ ) = J ( z - lZ ) . and proceed 88 in (a). 
248 Solutions. 2: Differentiation 2.3.10. We have ( z :& ) ( :& ) I' () ( Z ) 1(2n} (  ) ( z ) 2n I(z) =J 2' + 2 = J 2" + If '2 +... + (2n)! 2 /(2n+l) ( + 8 1 i) ( X ) 2R+l + (2n + I)! 2 · Similarly, /(0) =/ ( Z _ Z ) = I ( ! ) _ JI () ( ! ) +... + /(2n) (I) ( 2: ) 28 2 2 2 I! 2 (2n)! 2 _ 1(2n+l) ( - 92) ( ! ) 2n+l (2n + I)! 2 · Subtracting the above equalities yields I(z) =/(0) + I' () () + J<3) () ()3 ... 2 (2n-l) ( X ) ( = ) 2n-1 + + (2n _1)! J 2 2 j(2n+l) (j + 9 1 ) + f(2n+l) (i - 8 2 f) ( ! ) 2R+1 + (2n + 1) r 2. Since the derivative enjoys the intermediate value property (see, e.g., 2.2.31), the desired result follows. 2.3.11. Apply the foregoing problem with f(x) = In(! + z), z > 0, and observe that the odd derivatives of I are positive for positive z. 2.3.12. By Taylor's formula with the Peano form for the remainder (see, e.g., 2.3.1), (a) fun I(z + h) - 2/(z) + f(x - h) h-tO h 2 = lim { lex) + hf'(x) +  J"(x) + o(h 2 ) ,,-.0 h 2 _ 2/(z) -/(z) + hl'l- Jt.1"(z) + o(h 2 ) } = I"(z). 
2.3. Taylor's Formula and L'Hospital's Rule 249 (b) Jim I(z + 2h) - 2/(x + h) + f(x) h-..O h 2 _ Ii h2 J"(Z) + o(4h 2 ) - 2o(h 2 ) - f " ( ) - m 2 - z. h-.O h 2.3.13. As in the solution of the foregoing problem one can apply Taylor's formula with the Peano form for the remainder. 2.3.14. (a) It follows from Taylor's formula that for z > 0, n zk zn+l n zk  = L kf+ ( l)! e'l z > L kf" k=O n + k=O (b) For z > 0 we have Z2 3: 3 Z4 Z6 1 3;2 z3 x 4 In(l+z) = 2:- 2 + 3 - 4 + 5 (1 + 8 1 z)5 > %-2+3-4. Similarly, for x > -1, x =F 0, x 2 Z3 x 4 1 :£2 x 3 In(l+z) =z- 2+ 3 - 4" (1 + 8 2 z)4 < z- 2 +3 6 (c) Applying Taylor's formula to the function x ..... VI + x, we get 11117 v I +x = 1 + -x - _x 2 + -x 3 - -(1 + 81 Z )-2Z. 2 8 16 128 1 I 2 1 3 < 1 + -z - -2: +-x 2 8 16 and ..j 1 1 2 1 ( 8 ) _!1 3 1 1 2 1 + z = 1 + -x - -z + - 1 + 2:1: 2 Z > 1 + -z - -z 2 8 W 28. 
250 Solutions. 2: Differentiation 2.3.15. By 2.3.1, "," ",n+ 1 l(x+h)=/(x)+hl'{x)+.. -+- j(n) (Z)+ fC n + 1 )(X)+O(Ji n + 1 ). n! (n + I)! On the other hand, h n - 1 h n j(z+h) = f{z)+hf'(x)+...+ (n _l)! /(n-l)(x)+ n! j C n)(x+8(h)h). Subtracting the first equality from the latter, we get f(n) (x + 9(h)h) - f(n)(x) I{n+l) (x) o(h) h = n+ 1 + h- Consequently, /(,,+I)(Z) o(hl D ( h ) - n+ 1 + h - f(Dt(z+9(h)h)- jCD)CZ) · 1) ( /.. ) /. The desired result follows from the fact that j(n+l)(x) exists and is different from zero. 2.3.16. For 0 < z < 1, (1) Z2 1(0) = f(x - x) = f(x) - I'(x)x + f"(x - D1x)2"' and for 0 < z < 1, 1(1) = I(x + (1 - z» (2) (1 Z}2 = fez) + 1'(x)(I- z) + 1"(:£ + 6 2 (1- x»  . Note that (1) implies 1/'(1)1 < -i, and (2) implies If'(O)1   . lVlore- over, subtracting (2) from (1) gives I'(x) = (f"(X-91X)r-I"(X+l12(l-x»(1-X)2) for 0 < x < 1. Hence A A II' (x) I < "2 (2x 2 - 2z + 1) < 2' 0 < x < 1. 
2.3. Taylor's Formula and L'Hospital's Rule 251 2.3.17. (a) For z E [-c, c], (1) f(e) - f(x) = l'(x)(e - x) + l"(x + O (e - x» (e - x)2 and f( ) I( } f ' ( )( ) J"(X - 9 2 (c + x» ( ) 2 -c - X = - X c + X + 2 c + X . Hence f'(x) = /(c) - f(-c) 2c (c - x)2 J"(x + Dt(e - x»-(c + :1:)2 J"(x - 8 2 (c + x» 4c Consequently, If'(x)1 < k: o + (e + z2)  . (b) By (1) in the solution of (a), for x e [-c, c), we get f ' ( ) = f(c) - f(x) _ f"(x + 9t h ) l x h 2 &, where h = c - x > O. Thus 1 /'(x)1 $ 2 + !M 2 h. Now taking h = 2 /  gives If'(x}1 < 2 yA1 o lvf2 , which in turn implies tbat M 1  2 ,jl v[oAf 2 - 2.3.18. The inequality Ml < 2 y [\,10]\1[2 is proved in the solution of 2.3.17 (b). The equality is attained, for example, for J defined by { 2x2 - 1 if - 1 < x < 0, f(x) = z2-1 - f 0 < +J I _ X < 00. Indeed, we have },I/o = 1 and .kJ 1 = M 2 = 4. 2.3.19. For h > 0 and x E Ii, h 2 f(x + 1&) = J(x) + f'(x)h + I"(x + 9h)?: and h 2 f(x - h) = I(x) - f'(x)h + I"(x - 9 t h)2- 
252 Solutions. 2: Differentiation Consequently, 1 h /'(:c) = 2j;(/{:C + h) - f(x - It» - '4(/"(:c + 8h) - f"(x - 8 1 11.», which implies Mo h If'(x)1  T + 2 M2 for h > o. To get the desired inequality it suffices to take h = V2. 2.3.20. For p = 2 the result is contained in the foregoing problem. Now we proceed by induction. Assuming the assertion to hold for 2,3,.. _ ,p, we will prove it for p + 1. We have /(p-l) (x + h) = j(P-l)(X) + f(P}(x)h + j(P+l){a; + 8h)£ 2 and /(P-l)(:c _ h) = jCP-1){X) - j(p) (x)h + jh*l)(x -.8 1 h) h 2 . 2 Hence f(P){z) = .!..(f(P-l)(z + h) - f(P-I}(x - h» 2h h - 4 (f(P+l) (x + 8h) _/(1I+ 1 )(x - 8 1 h». Consequently, I f (p) ( )1 < kf"-l h M a; - h + 2 1'+1, Te:OOng h = 2  : gives M"  ..j2 M p - 1 Mp+l- By the induction hypothesis for k = p - 1, upon simple calculation, we get ( 1 ) M < 2f.kl.;:}:r M;;tr p - 0 p+l- h> O. So the inequality is proved for k = p. Now we will prove it for 1  k  p - 1. By the induction hypothesis, . ,. .(p-.) 1-- - M" .2 2 Mo P M; , 
2.3. Taylor's Formula and L'Hospital's Rule 253 which combined with (1) gives M 2 11(p.a-lI) .I'l-ptr M "1 Ie :5 2 ..cVlo ,*1 · 2.3.21. Suppose If" (x) 1 S M (M > 0) for x E (0,00). By Taylor's formula for z, h e (0,00), h 2 f(z + h) = f(x) + f'(x)h + f"(x + Bh)2. It then follows that 1/'(x)1  I/(:r: + ' -f(:r:)1 + " . Since lim f(x) = 0, given e > 0 there is an %0 such that -+oo 1/'(x)1   + h for x > :Co,k > O. Taking h = v2f:1 we get 1/'(x)1 :5 v2E M, x> 3:0, which means that lim f'{x) = O. z-too 2.3.22. For z > 0, 1 f(x + 1) = f(x) + f'{x) + 2 f"() with some  e (x,% + 1). Hence z 1 X x/'(x) = z.+ 1 (x + I)/(x + 1) - x/ex) - 2. . /"(). Consequently, lim 3:/'(3:) = o. %-.+00 2.323. For u, z e (0,1), u > x, by Taylor's formula we get 1 feu) = f(x) + J'(z)(u - x) + 2/'/()(U - x)2 with some ( e (x, u). Taking u = x + e(l - x), 0 < E < i, we obtain 1 feu) -/(z) = eel - x)/'(x) + 2 e2/"(x + 8e(1- x»(1 - :.:)2 
254 Solutions. 2: Differentiation with some 8 e (0,1). Upon letting %.-. 1- we see that (1) o = Urn ( 1 - z)/'(3:) + _ 2 1 £/"(z + 8£(1 - %»(1 - %)2 ) . -+ 1- By the definition of the Ibnit, if EI > 0, then 1 (1- z)I/'(z)1 S £) + 2el/"(z + 8£(1 - %»1(1 - %)2 1 kle  EJ + 2 (ge -1)2 for % sufficiently close to 1. Since E > 0 can be chosen arbitrarily, (1 - :r)ll'(x)1  £), \vhich means that lint (1 - :r)/'(x) = 0. -+1- 2.3.24. We have 1 (";6 ) =/(0)+ 1"1) e;o r and I( O;b ) =/(6)+ J"2) e;" r. witb some ZI e (0, a6 ) and %2 e ( tI ' b) . Hence 1/(6) - l(a)1 = e; a r 1J"(X2) -/"(xl)1 s e ; o r 1J"(e)l. where 1/"(c)1 = max{l/"(zi )1, 1/"(z2)1}. 2.3.25. It follows from Taylor's Connula that 1 = 1(1) = ! /"(0) + 1"'(zl) and 0 = I( -1) = ! 1"(0) _ 1'''(%2) 2 3! 2 3! with some %1 e (0,1) and %2 e (-1,0). Thus /"'(ZI) + 1"'(Z2) = 6, which implies 1"'(zl)  3 or 1'''(:£2)  3. It is worth noting here that the equality is attained! for example, for I(z) = (r + r). 
2.3. Taylor's Formula and L'Hospital's Rule 255 2.3.26. Write (1) I(t) = J(z) + (t - z)Q(t). Dif£ertJotiating both sides of this equality wit.h respect to t gives (2) J'(t) = Q(t) + (t - :r:)Q'(t). Substituting %0 for " we get (3) f(z) = /(xo) + (% - xo)/'(xo) + (z - ZO)2Q'(ZO). Differentiating (2) witb respect to t, next taking t = ZOt and aided by (3), we obtain 1 1 J(z) = /(zO) + (%-zo)/'(zo) + iJ"(:tO)(z-zO)2+2(z-%o)3Q"(zO). To get the desired equality it is enough to repeat the above procedure n times. 2.3.21. By Taylor's forlDula given in 2.3.1, I(Yn) = /(0) + 1'(O)Yn + o(Yn), I(x,,) = 1(0) + /'(O)%n + O(xn). Hence (I) /'(0) = f(Yn) -f(zn) _ O(lI,.") - o(xn} . 'IIn - %n 1/n - Zn (a) Since X n < 0 < IIn, we see tbat OClIn) - o{zn) < 10(Yn) 1 + 10(%n)1 < 10(Y'I)1 + 10(%n)l . 'Un - Zn - lIn - Zn Un - X n - 1/n -Zn Consequently, I - o(Yn} - o(xn) 0 1m = I noo 1/n - Zn which -along witb (1) shO\vs that linl Dn = /'(0). ,,oo (b) By (1) it is enough to sho\\ that lim 1f;)-:(z.) = o. \Ve have n-+oo " - '" 1 - o{Yn) - o(%n) 1m n-too y,. - Xn = lim ( O(lIn) . Yn _ o(%n} . X n ) = 0, n-+oo 'IIn 'IIn - Zn Zn IIn - z,. 
256 Solutions. 2: Differentiation where the last equality follows from the oundedness of { lie } 1/.. -z. and { "..%.. }. (c) By the mean value theorem, Dn = /'(6 n ), where Zn < 8n < IIn. Tbe desired result follows from the continuity of I' at zero. 2.3.28. Observe first that P is a polynomial of degree at most m. Differentiating the equality m+l ( ) (l-lI)m+l = E m+l (-l)'=yt, k=O Ie we get (I) -em + 1)(I-lI)m = E (m; I) (_I)kkyk-l. k= 1 TakiDg 11 = 1 gives 0= E ( m+l ) C_I)kk. 1=1 Ie It follows from this equality that p(m- 1)(0) = O. Next, differentiat- ing (1) and then putting II = I, we see by (2) that p(m-2)(0) = o. Continuing this process, one can show that pU)(O) = 0 for j = 0, 1, 2, .., m - 1. Moreover, p(m) (0) = 0, because (2) ,,+1 ( 1 ) O=(l-l)m+l=E m+ (-l)k. k=O k Now it follows from Taylor's formula that P(z) == o. 2.3.29 [E. I. Poft'ald, Amer. Math. Monthly 97 (1990), 205-213]. We will need the following mean value theorem for integrals. Theorem. Let / and 9 be continuous on [a, b], and let 9 have con- stant sign on [ab]. Then there is  E (a, b) such that [!(Z)g(Z)d:1: = !({) [ g(z)d:1:. 
2.3. Taylor's Formula and L'Hospital's Rule 257 Proof. Set m = min{f(z) : z e [a, b)} and M = max{J(z) : % e [a, b]}. Assume. e.g.. that g(z) > o. Then mg(z) S /(z)g()  M g(z). Integrating both sides of this inequality yields m 1& g(z)dz  1& /(z)g(z)dz  M 1& g(z)dz. Hence m < J: I(z)g(z)dz < M. - J: g(z)d% - Since J enjoys tbe intermediate value property on [at b). tbe assertion follows. Now following E. I. Poffald, we start tbe proof of the given for- mula. By Taylor's formula with integral remainder (see, e.g., 2.3.4). len) ( :& ) = I(n) (0) + /(n+l)(o) :c n+l n+l + (m 1(,*2)(t) ( z _ t ) dt. J o n + 1 Hence / ' ( 0 ) J (n-l) ( 0 ) I(n) ( -L ) /(0) + z + . . . + zn-l + n+l z" 11 (n - I)! nl = 1(0) + /'(0) z +... + /(n-l) (0) Z_1 11 (n - 1)1 + ( /(n)(o)+.t<n+l)(O) z + (rn/(n+2)(t) ( z -t ) d' )  n + 1 J o n + 1 n! = 1(0) + 1'(0) z + . . . + I(n+l) (0) zn+l 11 (n + I)! + (m Jln+2)(t) ( Z _ t ) dt. J o n + 1 n! 
258 Solutions. 2: Differentiation On the other hand, by Taylor's formula with integral remainder, f(x) = 1(0) + 1'(0) x + ... + I(n. 1)(0) x R + 1 I.! (n + I)! + 1 ("Z j(n+2) (t)(x _ t}n+ldt. (n + I)! 10 Consequently, f(x) - ( /(0) + /'(0) % +... + jln-l)(o) %n-l + ftn) (ntr) zra ) I! (n - I)! n! 1 1 % = /(n+2) (t)(x - t)n+J dt (n + I)! 0 _ r iitr f(n+2)(t) ( X _ t ) dt 10 n + 1 n! . = .!.. 1 --+i f(n+2)(t) ( Z - t)n+l _ ZR ( :c _ t )) dt n! 0 n + 1 n + 1 + 1 1 % j(n+2)(t)(x - t)n+ldt. (n+ I)!  -+1 Consider 9 defined by (t) = (x - t)R+J _:en ( Z _ t ) 9 n+l n+l for t E [0, x). It easy to check that g'(t) > 0 for t E (O,x) and g(O) = o. Thus 9 Is positive on the open interval (O,z). By tbe mean value theorem for integrals (proved at the beginning of the solution), we get 1 m jCn+2)(t> ( {x-t)n+l _:r.n ( x -t )) d1 o n+l n+l r .1 = j(n+2) ({I ) 10 g(t)dt and f: f(n+2) (t)(% - t)n+1dt = /(n+2)(&) /" (z - t)n+1dt. _+1 ;tr 
2.3. Taylor's Formula and L'Hospital's Rule 259 It fonows &om the above tbat 1(%) - ( /(0) + /'(0) z +... + /(n-l l (o) zn-I + /(n l (nil) zn ) 11 (n - I)! n! = 1.. /(n+2)({I) r atr g(t)dt + 1 /(n+21({2) f z (x - t)n+ldt. nt 10 (n+ I)! m Setting 1 m l z (z - t)n+1 A. = 0 g(t)dt and >'2 = tit m n+l we see that zn+2n At + A 2 = 2(n + 1)2(" + 2) · By the intermediate value property, l(n+2)(1) j;;tJ g(t)dt + l(n+2)(2) fZ J;e-t)-+1 dt o  n+l = / (n+2) (  ) z-+2n  , 2( n+l) 2( n+2 ) where  is between (I and . Consequently, fez) _ ( /(0) + /'(0) z + ... + /(n-l) (0) Zn-l + /(n l (nTI) zn ) It (n - I)! nl _ ! / (n+2) ( ) Zn+2n _ n / (n+2) ( ) Zn+2 - n! ( 2(n + 1)2(" + 2) - 2(" + 1) ( (n + 2)r To end the proof it is enougb to set 9 = . 2.3.30. It follows from tbe assumption and Taylor's formula applied to I(n) that I(n)(zo + B(z)(z - zo» = /(n)(zo) + j<n+p)(ZO +81(Z)(Z - zo» (8(z)(z - zo))". p. 
2f30 Solutions. 2: Differentiation Hence /'(xo) I(n) (xo) j(x) = I(xo} + I! (x - %0) + . u + n! (x - xo)n l(n+I')(xo + lJ 1 6(x)(x - XO» ( _ ) n+ p (8( » 1' + I , :z; xo x. n.p. On the other hand, !'(xo) I(n) (:co) n f(x) = /(xo) + I! (x - xo) + . .. + n! (x - xo) I(n+p)(xo + 6 2 (x - xo» n+p + ()' (x - xo) . n+p. Upon putting the last two equalities together, we get j(n+p)(xo + 8 1 8(x)(x - xo» I(n+p)(xo + 82 (x - xo}) n!p! (8(x»" = (n + p)! . Since j(n+ p ) is continuous at 3:0 and j{n+p)(xo)  0, upon passage to the limit as 3: -+ Xo, we obtain (n;p)! = n:p! :r:1.!.o (9(3:»1'. Note that if p = 1, then we get the result in 2.3.15. 2.3.31. By Taylor's formula, (1) I*J [*] ( ) b I(kz) = t; I'(O)kz +  1"(9kz)k2x2 = 1'(0» )*] ([*] + 1) +1](:1;), 2 where 1 [*] fJ(x) = 2" L j"(8kx)k 2 x 2 . k=:1 Since I" is bounded in a neighborhood of zero and (2) [k2= [*]([*]+1)(2[*]+1) , k=l 6 
2.3. Taylor's Formula and L'Hospital's Rule 261 (2) implies that 1im 1}(x) = O. Now (1) shows that %-.0+ [*J f ' ( O ) lim L j(kx) = . o+ '=1 2 2.3.32. It follows from the Bolzano- Weierstrass theorem (see, e.g., I, 2.4.30) that the set of zeros of J bas at least one limit point, say p, in [c, dJ. Clearly, 1(P} = O. Let {Xn} be a sequence of zeros of I converging to p. By Rolle's theorem, between two zeros of f there is at least one zero of I'. Therefore p is also a limit point of the set of zeros of f'. Since I' is continuous, I' (P) = 0, and inductively, j(k) (P) = 0 for kEN. Consequently, by Taylor's formula, f(x) = f(n)(p + 8(x - p» (x _ p)" n! with some 9 E (0,1). Since sup{l/(n)(x») : z E (a,b)} = O(n!), there is M > 0 such that I/(x)1 sAt/Ix - pin for sufficiently large n. So, if :& E (a, b) and Ix - pi < 1, then f(x) = o. 2.3.33. As in the solution of the foreoin1( problem, one can show that 1(1c)(O) = 0 for keN. By Taylor's formula, I(n) ( Ox ) I(x) = x n , n E N. n! Since given x, lim  = 0, we see that J() = 0 for z e III n-too 2.3.34. (a) 1. (b) -e/2. (c) lIe. (d) 1. (e) e-I. 2.3.35. To show that lim ( I (  )) % = lim e ZID /( *) = e-';', Z-Io+OO -IX %-++00 
262 Solutions. 2: Differentiation one can use Taylor's formula with the Peano form for the remainder (see, e.g., 2.3.1), which gives :£2 f(x) = 1- 2' + 0(2=2), and next apply 1.1.17(a). One can also use I'Hospita1's rule in the following way: lim zlnf ( -!!:. ) = lim In/(aVt) = lim af'(aVi} %-++00 .;x f-tO+ t t-+O+ 2Vtf(a-/i} . a21"(a) a 2 - lint - -- - fO+ 2J(aVt) + 2aVtJ'(a../i) - 2 · 2.3.36. Assume first that a > 1. Then by I'Hospital's rule, z ]'"' Cr: = » )! = o. H 0 < a < 1, then ( z 1 ) .1 lim a- --I %-++00 x(a -1) -. 2.3.37. (a) Since z :;::; does not .exist, one cannot apply I'Hospital's rule. Clearly, the limit is 1/2. (b) We are not allowed to apply l"Hospital's, because the derivative of the function in the denominator vanishes at  + 2n7r J n e N- On the other hand, it is easy to show that the limit does not exist. (c) To find the limit of l(x)9(Z) when x -4 0+ it ic; enough to deter- mine lim In (:) . However, this limit CTln ot be calculated using z....o+ II. l'Hospital's rule because the assumption that the limit of the quo- tient of derivatives exists is not satisfied. It is shown in 1.1.23(a) that tbe limit is 1. (d) The limit is 1 (see, e.g., 1.1.23(b». However, to find it one CAnn ot apply I'Hospital's rule. 
2.3. Taylor's Formula and L'Hospital's Rule 263 2.3.38. We have -----L- _ 1 _ 1 L-I- _ 1 _ 1 n :zTIi2 2 2 -1 2 li mU+t) , 2 m = m z-+O :& t-.o In(J+tl . 2 = lim In 2(2t - 2ln(1?+ t) - t ID(! + t» t-.o 2t In- (1 + t) ln2 = - 12 ' where the last equality can be obtained applying l'HospitaI's rule several times in succession. Hence I' (0) = - i · 2.3.39. As in the solution of 2.3.28, one can show that  ( Ie ( n ) { 0 if T = 0,1, .. . , n - 1, L....i -1) k r = L.- k n! if r = n. --O Now to get the desired equality it Us enough to apply l'lIO:ipital's rule n times successively. 2.3.40. Assume first that lim g(x) = +00 and L e III By (ili), z-.o+ given E > 0, there is aI, such that for z E (a,al)' f'(x) (1) L - E < g'(z) < L + £. Since g' enjoys the intermediate value property, (i) implies that !/ has the same sign on (at b). Consequently. 9 is strictly monotonic on (a, b). For z,y e (a,al)' x < V, by the generalized mean value theorem, fex) - fCy) J'(xo) 9(.1:) - g(y) - 9'(%0) with somc :l:o C (:c,y) C (a,a!). Fix'll for n moment. Then by (1), lMful L-e < 9(Z) - 9W" < L+£. 1 - !lUll 9{Z) Thus if, Cor examplc, 9 is strictly dccrcooing in (a,b), then (L - E) ( 1- g(y» ) + J{y) < f(x) < (L + E) ( 1- g(y» ) + f{y) . g(x) g(x) g(x) 9(z) g(x) 
264 Solutions. 2: Differentiation Letting %  a+ gives L - e < Urn fez) < L + E: - z-to+ g(x) - , which ends the proof in this case. The other cases can be proved analogously. Note that a similar result holds for the left-hand limit at b. 2.3.41. (a) By PHospital's rule (see 2.3.40), we have lim J() = lim e GZ f(x) = lim eG%(af(x) + f'(x» = !. Z-++OO %-++00 ea z z-++co ae 4Z a (b) Likewise, Jim (z = Jim e°V% fez) = Jim e<>./i (f'(z) + 27; f (z) ) ""T"'" f ) ""T"" ,r.; ......T.... e..". 2 z = lim !(a/(x) + 2v'Xf'(z» =. %-++00 a a To see tbat statements (a) and (b) are not true for negative a, consider the functions f(%) = e- OZ and f(x) = e-o,ji t respec- tively. 2.3.42. Using ItHospita1's rule proved in 2.3.40, we get ( LJrl ) ' . ( f'(x) ) . % - J"(%} . J'(x)f'''(x) 1im 1- =lim =llm =c. z-+oo xf"(%) z-+oo z' z-+oo (I" (X»2 Hence Jim f'(x) = 1- c. %-+00 3:/"(3:) By assumption, we therefore see that c  1. Clearly, if c  I, then . %/"(x) 1 Iim = . z-+oo f'(x) 1- c (1) Now we prove that. lim I(x) = +00. By Taylor',; formula we have z-+oo h 2 J(% + h) = J(z) + f'(x)h + J"«() 2:' h> O. 
2.3. Taylor's Formula and L'Hospital's Rule 265 Hence I(z + h) > f(x) + I'(z)h. Letting h -) +00 shows that lim I(z) = +00. Now, by l'Hospital's rule again, %-+00 Iim xl' (x) _ lim I'(x) + xl" (x) _ 1 1 - - + , -+oo f(x) 1:-+00 /'(x) 1 - c which combined with (1) yields lim f(x}/"(x) = lim xj"(z) . f(x) = 1 . 2:-t00 (/'(x»2 -too I'(x) x/'(x) 2 - c 2.3.43. For X:F 0, by the Leibniz formulas we get (1) n ( ) ( ) (n-i) g(n>(z) = E  j(£o)(z)  nil = "(_l)"-k j(k) (x) . L., k! zn+l-i 1c=O Set g(O) = 1'(0). Then an application of l'Hospital's rule gives g'(O) = llm g(x) -1'(0) = lim f(x) - :1'(0) z-tO X z-tO z- _ f'(x) - /'(0) /"(0) =bm =. z-tO 2x 2 Thus g'(O) exists. Now we show that g' is continuous at zero. By (1) and l'Hospital's rule t lim g'(x) = lim j'(x) - g(x) = lim xl'(x) - f(x) z-tO z-+O X z-tO x'Z = lim zJ"(x) = '(0). z-.o 2z 9 So, 9 is in Cl(-I, 1). Now we proceed by induction. Suppose that g(n)(o) = I(::(O) and 9 E e"(-I, 1). Then by (1) and }'Hospital's 
266 Solutions. 2: Differentiation rule we obtain (n) ( ) (n) ( 0 ) g(n+1) (0) = lim 9 x - 9 z-+O Z n E (-l)n-kifjCk) (z)xk - xn+lg(n> (0) = lim k =O z-tO X"+2 E (-l)n-1: :: j(l:) (X)Xk _ xn+l /C:+(O) Ii k= 0 = m z-+O x"+2 n = lim { nl(-l)n J I(X) + :;1 (_l)n-k iffCI:+l) (x)zk z-+o (n + 2}Xn+l n E (_l)n-k (t ! 1)f J(k)(x)xk-l - znf(n+l) (0) } + «=1 (n + 2}Xn+l n { n!( -1)n f'(x) + L: (_l)n-k : f(k+l) (X)X k Ii k=1 = m z-+O (n + 2)xn+l n-l E (_l)n-l-kif/(k+l)(x)x i - x n j(n+l) (0) } + 1:=0 (n + 2)Xn+l _. x n (j(n+l)(z) _ j(n+l)(O» _ /(n+2)(0) -lim -. z-+O (n + 2)Xn+l n + 2 Our task is now to show that g(n+l) is continuous at zero. By (1) and l'Hospital's rule again, n+l L: (_I)n+l-1: (nl)! j(l:} (x) xl: k. lim g(n+l)(x) = Iim 1:=0 z-tO z-+O xn+2 - - 
2.4. Convex Functions 267 nt 1 (-1 ).+1. ("+1 )! f(kH) (x )x" + "t l ( -1 rl::;}..+1 )1 f(k) (x )xk- I full ,,=0 k= 1 z.-.o (n + 2)xn+1 / (n+2) ( ) = lim x = 9(n+l) (0). z-+O n + 2 Summing up the above, we see that the extended 9 defined above is - 00 (n) /(,,+1'(0) m C on (-1,1), and 9 (0) = n+1 ,n = 0,1,2,... . 2.4. Convex Functions 2.4.1. Assume that I is convex on I. Then for Xl < X < X2 we have (1) f(x) - f(Xl) < f(X2) - l(x1) Z - Xl Z2 - Xl (see (1) in the solution of 1.2.33). On the other hand, since X2 - x X - Xl X = Xl + X2, X2 - Xl X2 - Xl we see that X2 - X X - Xl f(:r)  !(Xt) + !(X2), X2 - Xl X2 - Xl and consequently, /(Z2) - /(Xl) < l(x2) -f(x) . 3:2 - Xl - 3:2 - X This combined with (1) gives (2) f(x) - I(xl) < !(X2) - f(x) . Z - Xl - X2 - X Upon passae to the limit as x -+ xt we see that /'(Xl> < f(Z2) - /(x 1 ) . X2 - XI Similarly, letting X -. x; in (2) yields f'(X2)  /(X2) - /(:&1) . X2 - Xl Consequently, f'(X1)  1'{x2), which shows that I' is increasing. 
268 Solutions. 2: Differentiation Assume now that /' is increasing on I. Let Xl <:e < %2. By the mean value theorem, I(x) - I(xl) = 1'(1)' X-XI 1(%2) - I(x) = I'({,.), %2- Z where %1 < l < X < {2 < %2. Therefore by the monotonicity of I' we get inequality (2). Now we show that (2) implies the convexity of I. To this end set % = 1 + (1- '\)%2, where XJ < Z2 and  E (0,1). Then X e (XhX2), z - %1 = (1- '\)(X2 - %1) and X2 - x = A(x2 - Xl). So (2) gives I(z)  A/(xt) + (1 - ).)/(X2). It is worth noting here" that (2) is in £act equivalent to the cODvexity of J. Note also that if I' is strictly incr easin g on I, then I is strictly convex on I. 2.4.2. It suffices to observe that the condition /"(x)  0 for % e I is equivalent to the fact that /' increases, and to apply the. result in the foregoing problem. 2.4.3. We proceed by induction. The condition with n = 2 is the definition of convexity of / on I. We therefore assume the inequality to be proved is true for some n  2 and show that it is also true for n + 1. Let AI t A2 t . .. . An, An+l be nonnegative and such that Al + '\2 + ... + An + An+l = 1. Since '\nzn + An+1Xn+l can be written in the form ('\n +An+l) ( A,,;AfI+l X n + ).."':;:;+1 %"+1) I by the induction hypothesis we get I(>'I X I + >'2:1;2 + ... + >'n+l Z n+l)  Al/(Xl) + '\2/(Z2) ( An An+l ) +... + {).n + '\n+l)1 '\0 + "0+1 Zn + An + AO+l 3: n +l · Now we only have to apply the definition of the convexity of f to the last s urnm and. 
2.4. Convex FUnctions 269 2.4.4. Since In"(z) = -, the function % .... IDz is concave on (0, (0). Thus ( zP Ill ) 1 1 In - + - > -lnzP + -In II' = In(zy). p q -1' q 2.4.&. Since z t-+ Inz is concave on (0,00), we get ( Xl %2 %n ) 1 In - + - + ... + - > - (lnzl + lnz2 +... + lnzn) n n n-n 1 = -In(ZI · Z2.. .z,,). n 2.4.8. The function z H  is strictly convex on II. J (b,e b ) (a,ea) Q b If, for example, a < b. tben the area under tbe graph of II = e;Z from z = a to z = b is less than the area of tbe trapezoid with vertices (0,0), (6,0), (a, eO) and (6, e b ). Therefore 1 " eO + e" e. - e a = a blt < (b - a) 2 · 2.4.'1. Consider the function given by /(z) = zlnz, z > o. Then I"(x) =  > o. Thus / is convex. Consequently, z+lI ln z +II <!. 1n !l ln 2 2 - 2 z + 2 'II. 2.4.8. Use the fact that % .... ZOl, Q > 1, is convex on (0, (0). 
270 Solutions. 2: Differentiation 2.4.9. (a) The function lex) = In (k + 1) is convex on (0,00), because f"(x) > ° on that interval. So the result follows from the Jensen inequality (see, e.g., 2.4.3). n Observe that if PI: = * for k = 1,2,.. . , n, and if E ZIc = 1, then Ie=} we get the inequality given in I, 1.2.43(a). (b) It is enough to apply the Jensen inequality to the function ( 1 + X ) f(z) = In 1 _ x ' 0 < x < 1. n Observe that if PI: =  for k = 1,2,. . . ,n, and if E X/c = I, then k=l we get the inequality given 1,1.2.45. 2.4.10. (a) Define f(x) = In(sinx) for x E (0,11'). Since f"(x) = - 81:2 z < 0, we see that f is concave on (0,11"). Now it suffices to apply the Jensen inequality (see, e.g., 2.4.3) to -f. (b) Consider the function defined by f{x) = In(sinx) -lnx, x E (O,1r), and observe that I" (x) = - 81n z +:z < 0, and apply the Jensen inequality (see, e.g., 2.4.3) to - /. 2.4.11. Note that the function f(x) = (x +  r is convex on (0,00), because I"(z) = a (z+ ;)0-2 (a-I) (1- x )2 +  (z+  )) > o. By Jensen's inequality (see, e.g., 2.4.3), Q ( n 2 :1 f = 1 n 1 - L Xic + n n 1 .\- 1 ii L" X/C k=1 1 PI ( 1 ) 0  - L Xi: + - · n k=1 x" 
2.4. Convex Functions 271 Hence  ( 1 ) a. (n 2 + 1) a L." XI: + -"> 1. k=l Xk - n a - 2.4.12. Applying Jensen's inequality- to x H> -In X, X> 0, we get 1 ( 22 - 1 2 3 - 1 2 n - 1 ) ; In 1 + In 2 + In 2 2 + . .. + In 2 n - 1 ( 1 ( 22 - 1 2 n - 1 ) ) ( 2 1 ) <In - 1+ +...+ =In 2--+ . - n 2 2 n - 1 n n2 n - 1 2.4.13. (a) Applying Jensen's inequality to f(x) = ,x > 0, we obtain 1111111 1 I 1 < -. - + -. - +... + _.-. -Xl + -X2 + · · · + -Xn - n Xl n X2 n X n n n n Hence n 2 1 1 1 < -+ -+...+-. 3:1 -I- 3:2 -I ... I 3: n - 3:1 3:2 3:n (b) By Jensen's inequality applied to f(x) = -lnx, X> 0, we have In (Xfl X;2 ... x.) = OllnXl + 02lnX2 +... + On lnx n < m(alXI + 0'2 X 2 +... + QRZn). Consequently, (1) xf1 X2 2 · · · x:" $ 0IXl + 02X2 + · · · + 0nXn. H in (1) we replace Xk by -:;, we get the left inequality. (c) H one of ZJe or Yle is zero, then the inequality is obvious. So we can assume that XI:, Yk > 0 for k = 1,2,. . . ,n. Then the inequality in question can be rewritten as ;)e01 ;)eQ2 ;)eQ" + Y at Y Q2 "110ft "' 1 "' 2 ... "' n 1 2 .0.. .:I n ) ( < 1. (Xl + YI 01 X2 + 112)0 2 · · · (Xn + Yn)Qft 
272 Solutions. 2: Differentiation Now by (b) we get ,..01 ,..Q ,..0. + Y Q1 Y Cl:a Y OIi  1  2 ... .&. " 1 2 ... n (Xl + 1/1 )01 (X2 + Y2) Cl 2 · · · (Xn + lIn)o.. Xl X n VI < at + . · . + an + Qt - + + + Yn + .. . + Qn = 1. Z'n + Yn (d) The inequality follows from (c) by induction on m. 2.4.14. Suppose, contrary to our claim, that I is not constant on III Then there exist Xl < 3:2 such that /(Xl) < /(Z2) or f(xl) > /(X2). Let x be such that $1 < X2 < x. Then we have 1(:1:2) = / ( X - %2 xl + 3;2 - Xl X ) $ X - X2 /(Xl) + Z2 - Xl f(x). x - XI X - Xl X - Xl X - Xl Thus (1) /(x)  x - Xl /(X2) _ x - X2 I(Xl). X2 - Xl %2 - Xl If /(Z2) = /(Xl) + A with some positive A, then (1) implies that I(x) > A :211 + l(x1)t contrary to the assumption that I is bounded above. Similarly, if /(ZI) > l(x2), then l(x1) = A + l(x2) with some A > o. Now taking X < Xl < 2:2 we get x? -X f(x)  A - + l(x2), X2 - Xl again contrary to the assumption that I is bounded above. 2.4.15. No. It is enough to consider I(x) = e-%, X E (a,oo); and lex) = e Z , X e (-oo,a). 2.4.16. lIppOSP. that J iJ:t not monotonir.. Then there arp. t1 < Xl < :1:2 < X3 < b such that l(x1) > /(X2) and /(Z2) < /(Z3), or /(ZI) < /(:&2) aud /(:£2):> /(:&3). Since I is convex, l(x2) :s max{/(X1), /(Z3)}. So the latter case cannot hold. The continuity of f (see, e.g., 1.2.33) implies that there 
2.4. Convex Functions 273 is c e [XhZ3] such that fee) = min{f(x) : x e [XhX3]}. By the convexity of I we see that I{xt) <; max{f(x),f(c)} for x e (a,xl). Consequently, since f(e)  f(x.), we have f(Xl)  f(x). It then follows tbat if x, y E (a, c], then . x < Y < Xl implies f{y) < max{f(x),f(Xl}} = f(x), . x < Xl < Y implies fey) < max{f{xl), J(e)} = J(Xl) =5 f(x), . Zl < x < y implies I(y)  max{/(z), I(e)}  fez). So we have shown that J is decreasing on (a, c). In an entirely similar manner one can show that f increases on [c,b). 2.4.17. This is an immediate consequence of the result in the pre- ceding problem. 2.4.18. Since f is bounded, by the foregoing result, one-sided limits of f at a and b exist and are finite. Consequently, the assertion follows from 1.2.33 and 1.5.7. 2.4.19. Let Xl < X2 be two points in (a,b). Then for a < Y < Xl < 3; < %2 we have (see (1) and (2) in the solution of 2.4.1) (*) fey) -f(xl) < f(x) - /(Xl) < 1(%2) -/(x1) . Y - Xl - X - Xl - X2 - Xl This means that the function z H- /(Z)-/(ZI) is increasin g and %-%. bounded below in (%1, b). Consequently, the right-hand derivative f (Xl) exists and (**) I' (Xl)  /(X2) - /(X 1 ) . + X2 - x. Now note that for Xl < X2 < t < b, J(X2) - J{Xt) < J{t) - J(X2) - t ' %2 -ZI -X2 which gives 1(:£2) - /(Xl) < I' (X2). X2 - XI + This combined with (**) shows that f+(Xl) S f...(X2). To show that the left-hand derivative exists and is increasing on (at b), sim- ilar reasoning can be applied. Ioreover, it follows from (.) that for 
274 Solutions. 2: Differentiation %1 e (a,b) we have 1'-(%1)  1(zl). Recall that, by (2) in the solu- tion of 2.4.1, if 2:1 < % < %2, then fe%) - /(Xl) < /(X2) -f(%) . X-Xl - X2-% This implies that I(zl)  1!..(z2). Su mmin g up, we get 1(ZI) S I(:Z:I) S /(:Z:2) S 1(:C2) Cor :1:) < :1:2. rrms shows that if one oC the one-sided derivatives is continuous at I point in (a, b), then both derivatives are equal at this point. Since j monotone function can bave only a countable number of discontinu lties (see, e.g., 1.2.29), the one sided derivative.s are equal except OJ a countable set. An analogous statement is also true for concave functions. 2.4.20. Since I' is strictly increasing, the inverse function (/')- exists and {(x) = (/')-1 ( /(b + x) -/(a - z» ) . b-a+2z It then follows that e is differentiable on (0,00). Upon differentiating the equality given in the problem, we obtain (1) 1'(6 + %) + I'Ca - z) - 2/'() = 1"(E)'(z). 6-0+23: Now note that I"  0, and since /" is strictly increasing, I' is strictly convex (see, e.g., 2.4.1). Hence (see the figure below) (6-0+2% /'(6 + %) ; /'(a - %) > 1: 2 J'(t)dt = /(6+%)-/(0-%). 
2.4. Convex Functions 275 f' (b+x) ------------ / /) , ,. '-'. '4  .. b+x Thus J'(b + x)  J'(a - x) > J'({). Consequently, by (1), we see that '(x) > 0 for x > o. n 2.4.21. Without loss of generality we may assume that E Ix;1 > 0 i=J n and E IYil > o. By 2.4.4 we get i=1 p q IXil J. (  IXiI P )' bJil < .!. .l-p (  IYil') · IXil ( t IXil  &=1 ) 1 +- q IYil 1 C IYd, · . Upon su mmin g both sides of this inequality from i = 1 to i = n, we obtain n E IXiYil i=l ( t IXiIP ) t ( t IYilq ) t -=1 1=1 P q n <l L p i=1 Ix. I ( n ) * i Ixil p 1 n +-L q i=l IYII ( t IYil'l )  &=1 
276 Solutions. 2: Differentiation n n 1  Izd' 1  III, I' 1 1 = - n + - n = - + - = 1. p E Izd P q E Ifld' p q i=1 t=1 2.4.22. For p = 1 the inequality is obvious. For p > 1, let q be such that; + t = 1. So, q = #i. Hence n n E 1%( + "tiP = E 1%. + "tll z , + 11,1,-1 i=1 i=1 n n  E Iz.lIz, + y.I,-1 + E Ifldlzi + fldp-l 1=1 '=1 S ( t IZd' )  ( t Iz, + ",IC'-1)9 ) t 1= 1 1=1 1 1 + ( t 1"'1' ) . ( t Iz, + "d C Jl-I)9 ) · t=1 1=1 = ( (Izd')  + (I"'I') ) (IZ, + "d' f Consequently, ( t Iz, + ",I' )  s ( t IZd' ) t + ( t I",IP ) t . '=1 i=1 a=J 2.4.23. By Holder's inequality we have N ( N ) i ( N ) '   ;;W · 2.4.24. Set 81 = %1 + %2 + ... + Zn, 82 = 111 + 112 + · .. + IIn and S = (sf +)t. Then sP = sf +  = (Zlar 1 + 1IISr- 1 ) + (%2sf- 1 + 112-1) +... + (Znar 1 + Yn-l). 
2.4. Convex PUnctlons 277 It then follows by Holder's inequality that S"  (xf + yf)j(sf + )t + ( + JI:) (sf + )t + ... + (+)(sf + )t =sf (zf +yf) + ('+) +...+( +v.:)). which implies the desired inequality. 2.4.25. Set 3f = ZfJ and S = (sf)  · Then by Holder's inequality, ft n m m n sP = E 8.ar- 1 = E E :J:'Jsf- t = E E Z'Jsf"1 .=1 1=1 :J=1 J=l i=1  f: ( t zfJ )  ( t sf ) I!jl = 51>-1 f: ( t zfJ )  · J=1 .=1 t=l j=1 i=1 which implies the desired inequality. 2.4.26. Let z, fI E I and assume that % < JI. For n = 0, 1,2, . .. , set Tn = { : i = 0,1,... ,2 ft }. We show by induction tbat for n = 0,1,... and for 8 e Tn, 1«1- 8)% + 8Y)  (1- 8)/(z) + 8/(y). Clearly, if n = 0, then 8 = 0 or 8 = 1, and therefore the above in- equality is obvious. Assuming the inequality to hold for an arbitrarily chosen n e {O,l,...} and for II e Tn, we will prove it for n + 1. Sup- pose that B e Tn+l. Clearly, it suffices to consider the case 8 fJ Tn- Since tbere exist (, fJ e Tn such that 8 =  I we see that (1- 3)Z + 811 = (1- (; '1 ) z + {;'1 1/ (1 - () + (1 - '1) ( + '1 = 2 z+ 2 11 (1- ()z + J/) + «1 - '1)% + '11J) - - 2 . 
278 Solutions. 2: Differentiation By the midpoint-convexity of J, 1«1- s)x + sy) < 1«1- {)x + {y); 1«1- 1t):I; + ' m ) . By the induction hypothesis, 1«1 8):1; I BY)  (! =-Q I(x) + {fey)  (1- 'l)/(:I;) + 'If(y) = (1- .'l )f(x)+ {f} f(Y) = (1- s)f(x) + sl(y). Let t be an arbitrarily chosen point in [0,1]. Since the set 00 T = U Tn n=O is dense in [0,1], there is a sequence {sn} of points in T such that t = Urn 8n. Hence, by the continuity of /, n-+oo /«1- t}x + ty) = Jim J(1 - sn)x + snY) n-+oo < Jim «1- 8n)J(X) + BnJ(y» n-.oo = (1- t)/(z) + tl(y). 2.4.21. There are functions J : R  Ii which are additive and not continuous (see, e.g., 1.6.31). If 1 is such a function, then Cor any zeIR, /(x) = 1 ( ;. + ; ) = / ( ; ) + / () = 2J () · Thus J ( ; ) = J(x). Consequently, Cor x,y E R, f ( $ + Y ) = / ( X + Y ) = ! /(x) + ! I(Y) = lex) + J(y) 2 22 2 2 2. If / were convex on JR, then it would be continuous (see, e.g., 1.2.33), a contradiction. 
2.4. Convex Functions 279 2.4.28. Suppose, for example, that x < y. For t E (0,1) set z = (1- t)x + tJJ. Then z < z < y and there are a e (x,z) and be (z,y) such that z = tJb . Analogously, there are t a E (0, t) and tb E (t,l) such that a = (1 - ta)x + tay and b = (1 - t6)X + thY. Since z = Gb , we have t = ill ;t& . We know, by the result in 2.4.26, that 1 is convex on I. Now we show that 1 is strictly convex. We have f«l - t)x + ty) = I(z) < I(a) + f(b) 2 _ 1((1 - ta)x + tay) + /«1 - t6)X + thY) 2 < (1 - ta)f(x) + ta/(y) + (1 - tb)f(x)x + thf(y) - 2 = (1 _ to ; t6 ) I(z) + to ; t 6 / (y) = (1- t)/(x) + tf(y). 2.4.29. Since 1 is continuous on I (see, e.g., 1.2.33), it is locally bounded. Let Xo be in I and let  > 0 be so small that the interval [xo - 2e, Zo + 2e] is contained in I. By the local boundedness of 1 there is M > 0 such that (1) I/(x}1 < !vI for x E [xo - 2e, %0 + 2e). Take two distinct points Xl and %2 in [xo - e, Zo + e]. Then %3 = %2 + }if (X2 -Xt) is in [Xn - 2E.%n + 2E) and E IX2 - zll X2 = Xl + X3. I X 2 -XII +E IX2 -xII +e Since J is convex, we see that e Ix"J - zil f(X2) < I I I(Xl) + I - I I(X3). X2 - XI + E X2 - Xl + E 
280 Solutions. 2: Differentiation Consequently, /(X2) - /(Xl) $; I 1:1:2 - i E (/(:1:3) -/(Xl» X2 - XI :S 1:1:2 - zil i/(xa) - /(x1)1, e which combined with (1) show5 that l(z2) -/(zl) S 2 lz2 - xII. Finally, since the roles of XJ and %2 can be interchanged, we get 1/(x2) - /(xl)1 < lx2 - xII. 2.4.30. Let Xl < X2 be chosen arbitrarily in (0,00). H 0 < z < %1, then %2 - Xl Xl - X Xl = X + X2. Z2 - Z %2 - X It follows from the convexity of f that I(Xl) S %2 - XI 1(%) + %1 - Z I(X2). %2 - X Z2 - Z Upon passage to the limit as x  0+, we see that XI /(Xl)  $2 !(Z2). 2.4.31. (a) It follows from the monotonicity of x t-+ / that for Zl,X2  0, /(Xl +:1:2) = XI /(Xl: X 2) +X /(XI +:1:2) $; /(Xl) + /(:1:2). Xl X2 Xl + X2 (b) Suppose that 0 < a < b, and set p = i and q = 1- p. By the convexity and subadditivity of J we see that /(6) = /(pa + q(a + b»  plea) + q/(a + b)  p/(a) + q(/(a) + /(b» = lea) + (1 - ) /(b). Thus I(b) < /(a) . b - a 
2.4. Convex Functions 281 2.4.32. Assume, contrary to OUf claim, that I is neither strictly COD- vex nor concave. Then there are points Q < 13 in (a, b) such that the line through (0,/(0») and ({3t 1«(3» meets the graph of f at a point (")',1("1», where 0 < "1 < 13. By assumption, there are unique (1 E (0,,,) and (2 E (",{3) such that I('Y) -/(0)  /'({1) and 1(/3) -/("() -- /'«(1,). 7- 0 P-1 Since (Q,/(o», (P, f(P» and ("(,/('Y» are collinear, we see that I' «(I) = f' «(2), which contradicts the hypothesis. Jf(P} --------------------- I(a) a  1 Y 2 f3 2.4.33. Note first that the so-called Din; derivatives D+ f(x) = liiii f(x + t) - f(x) t-tO+ t ' D-/(x) = liiii f(x + t) - f(x) , t-tO- t D+f(x) = 1im I(x + t) - f(x) , t-+O+ t D_/(x) = 1im I(x + t) - f(x) '-+0- t always exist (finite or infini te). Moreover, since 911 is differentiable, we get (see, e.g., 1.4.10) D+ f(x + d) = D+ f(x) + g(z), D+/(x + d) = D+f(x) + g(z), D-/(x + cl) = D-/(x) + g(x), D_/(x + cl) = D_/(x) + g(x) for x e III So each of the Dini derivatives of I at x + d is a translate of the corresponding derivative at x. Next, if a < b are arbitrarily chosen, we set m = (/(b) -/(a»/(b - a) and define F(x) = I{x) - m(x - a). Then F(a) = F(b) = I(a), and therefore F attains its 
282 Solutions. 2: Differentiation maximum or minimum value on [a,b] at some point c E (0,6). We may assume without loss of generality that F(c) is the maximum of F. So if c + t e (a,b), then F(c + t)  F(c), Or in other words! f(c+ t) - f(c) S mt. This implies that D+ f(c)  m < D_/(c). Since each of the Dini derivatives of I at z is a translate of the corresponding derivative at c, we see that D+ I (x) S D_/(x) for all x. H j is concave on [ti, b), then f is diHerantiable except on an at most countable set (see, e.g., 2.4.19). It then follows that I is differentiable on (a, b). If I is not concave on [a, b]! then j attains also its minimum value on [a,b] at some point in the open interval (a,b). Then, as above, one can show tbat D- I(x} $ D+f(z) for all z. Consequently, D+ f(x) S D-f{x)  D- /(x)  D+/(x) for all x. So the differentiability of j on Ii is proved. Our task is now to show that I' is continuous. Assume! contrary to our claim, that there is Xo at which I' is not continuous. Then thero is a sequeDCQ {Z,.} converging to Zo such that {I' (zn)} converges to /'(xo) +r with some r -:F 0, or {/'(zn)} is unbounded. In the latter case, we can find a sequence {Yn} such that {/'(Yn}} diverges, say, to +00. Then f(x;) - (Yn) = f'(1In) + 0(1), 0- n and upon passage to the limit as n  00, we get I' (xo) = +00, a contradiction. In the former case, since I' enjoys the intermediate value property (see, e.g., 2.2.31), there is a sequence {Yn} such tbat f'(Yn) = I'(xo) + r/2. Clearly, we can assume that the sequence approaches Xo from one side, say, from above. By the hypothesis, for every x we can find such  sequence with the same r. Indeed, since z = Xo + (z - xo) = XO + d and j'(zn + d) - g(zn) = j'(Zn) = /'(xo) + r = 1'(xo + d) - g(xo) + r, upon passage  the limit, lim f'(zn + cl) = j'(X) + T. n-.oo By the intermediate value property of I' again, there is {in} such that f'(zn) = /'(:&) + r/2.. No\v we construct a sequence {Xn} as follows. Let x be arbitrarily chosen and let XI be such that x < 
2.4. Convex Functions 283 :1:1 < :r + 2- 1 and /'(Xl) = f'(x) + r/2. Next let %2 be such that J < %2 < %1 + 2- and J'(2) = J'(I) + r/2. Continuing this procedure J we gel a set}uenc(! {x,.} \\1th Xn <:: Xn+1 < X n + 2- n and 1'(x n +1) = f'(xn) + ; . Hence the sequence converges, say, to o. Moreover, it follows Crom the last equaUty that I'(x n ) = /'(%1) + (1& - 1)r{2. Consequently, (J'(x n )} diverges to +oc Or -00, contradicting differentiability of / at o. This ends the proof or the continuit). of J'. Thus I' satisfies the same hypothesis as J t so it tQO is r.ontinuously differentiable, alld by Induction so are all derivatives. 2.4.34. For 11 = 2 we get tbe obvious eCluaJity. So, assume that n > 2 and that {an} is a decreasing sequcnce. Set n-I Sn = (/(an)il! -/(fll )a,..) + E(/(n)lJk+J -/(Olr+l )Ok). k=1 Our task is to show that Sn > O. Since J is convex! we see that ( On -.U'*l 01 - 0" ) I(a n ) = I tJa + On't-t a. -Un+1 'Ill - "rHo.! .  Uft - "a+1 !(tll) + a, - Q., J(Unot-J). ('1 - 11;"+1 (11 - U"...1 HCllce. (a,*1 - o,)J(o.,) + (0" - Da.i.t)!(O,.) + (al - Q")!(a-s+J)  o. which :1I11!aI1S thut 5"+1" - Sn  O. CODSeqwmtl)., Sn  53 = o. 2..4.35 L Kl1amta, A. Smojdor. Amer. Mach. 1onthJy 7.1 (l.g67). 401.J. Since J 1s:strittly blcreasing nod II < /(7) < %, we gel o < Jn! (z) < In(z) < oJ: Cor n E N and z E (a, b). Con&eqUcntl).w the :;eqUeDCC (/"(z)} converges to an I (the case , = -co is possilJle). Now we show that 1 = a. To Uii5 end, recaU Coot, by Ltl¥. result In l.233r eucl1 Iii b continuuus. SQ. if I e (0 1 .6). then W would gL /(/)  /- ( 11m In() ) = U,.. J"-t-1 ( l = I, ,,-.  11-' 
284 Solutions. 2: Differentiation which would contradict the assumption J (x) < x for x E (at b). There- fore l = a for each :& E (a, b). By 2.4.19 the right-hand derivative of 1 exists and is decreasing on (a, b). So, if a < tl < to < 6 , then (see the solution of 2.4.19) (l) f+{to) 5 I{t) - {(to) 51+(h). 1 - 0 Taking to = In(x) and tl = r+l(x), we get I' (/n(x» < In+2(x) _/n+l(z) < I' (/n+l(x». + - In+l(z) - /n(x) - + It then follows from lim I(x) = 1 that if:-tG + . r+ 2 (x) -In+l(x) Jim = 1, R-tOO In+1(x) - In(x) and consequently, for keN, lim I"+k+l(x) _/n+k(x) n-too IR+l (x) - In(x) . Ie-I IR+i+2(x) _ In+i+1 (x) = n !! In+i+1(x) -/n+i(x) = 1- Since 1+ decreases, the equality lim J(z) = 1 implies I(z) S 1. Z-Ioo+ Thus by (1) the function x t-+ I(x) - x decreases on (a,b). Since I(v) -v < 0, (2) I(u) - u 1 (v) _ v  1 for v < u, U, v E ( a, b). Assume that a < y < x < b, and put u = I"(x) and v = I"(g). Then /n+l(X) - In(z) 1"+1(y) -I"(y)  1. On the other hand, there is keN such that I'(z) < y < x. This implies that fR+Ic(x) < 11I(y). Since the function x t-+ j(z) - x de- creases, we see that r+ 1 (y) - IR(y)  /n+k+1(z) - fn+"(x). 
2.5. Applications of Derivatives 285 Consequently, /"+1 (x) -/n(x) /h+l (x) - In(z) 1< < - In+l(y) - /n(y) - JR+k+l(X) _ In+lc(z) ' which combined with (2) gives the desired equality. 2.5. Applications of Derivatives 2.5.1. (a) Applying the generalized mean value theorem to the functions 2 lex) = 1 - cosx and g(x) = T' we get 1- cosz sinD 1 = <  (J 2 Cor z =F O. (b) For z > 0, consider f{x) = z - sin x and g(x) = . The general- ized mean value theorem combined with (a) shows that x - sin x 1 - cos (J 1 ;,t!l - 0 2 <, 3!" 2f which implies that the inequality holds true. Note that for nega- tive x we have sinx < z - . (c) Apply the generalized mean value theorem to the functions x2 z4 1(3:) = cosx -1 + 2' g(x) = 4f on the interval with the endpoints 0 and %) and use (b). (d) Apply the generalized mean value theorem to the functions x 3 f(x) = sinz - x + 3ft and use (c). x 5 g(x) = sr' x 2= 0, 2.5.2. Use induction and reasoning similar to that in the solution of tbe foregoing problem. 
286 Solutions. 2: Differentiation 2.5.3. Applying the generalized mean value theorem to the functions I and g(x) = X, g(x) = z2 and g(x) = z3 in succession, we see that 1(6) - I(a) /'(Xl) - b-a 1 t I(b) - lea) _ /'(%3) IJ3 - 4 3 - 3z · I(b) -/(a) _ 1'(X2) - t b2 - a 2 2%2 2.5.4. Set I(x) = II (x) + i/2(X) and Q = a + ib, a > o. It foUows from lim (o/(x) + /'(x» = 0 that z....+oo (1) Urn (all (z) + I; (x) - b/ 2 (x» = 0 ....+oo and (2) lim (aI2(z) + f(z) + b/ l (x» = o. ....+oo Observe now that oz+-i!Z' /( ) lim eib% /(z) = lim e x z...+oo -++oo e O % = lim eGZ(/1 (z) cosbz - /2(X) sin bz) z-++oo e OZ . li e dZ (/2 (x) cos bz + 11 ex) sin bx) +. m . z-++oo ea z Using I'Hospital's rule (given in 2.3.40), by (1) and (2) we get lim eQZ(/I(x)cosbz -/2(x)sinbz) z-++oo e GZ = lim cosbz(a/l(z)+ /:(z)-bI2(x»-sin bz(a/2(x) + /(x)+b/l(x» z.... a =0. Similarly, one can show that lim eOZ(/2(z)cosbx+ /t(x)sinbx) = o. ...+oo e QZ So, we see that Iim e i6z I(z) = 0, which immediately implies that z-++oo lim I(x) = o. z-++oo 
2.5. Applications of Derivatives 287 Finally, let us remark that the proved result can be generalized as follows. If 1im (ol(x)+ f'(x» = L, then lim lex) = L/o. Indeed, Z+ %+ in this case we have Urn (o.(f(x) - Llo.) + (f(x) - L/o)') = 0 and, z-++ by what we have proved, Jim (f(x) - L/a) = O. z-+-+ 2.5.5. Take 01 = ! - i and a2 = ! + i. Then f(x) + f'(x) + f"(X) = 02 a l/(X) + (02 + ol)f'(x) + f"(x) = o2(alf(x) + J'(x» + (alf(x) + f'(x»'. So, by tbe result in the foregoing problem (see the final remark in the solution), we get Urn (al/(x) + !'(x» = Lfa2 and Urn /(x) = z-+-+oo Z+ L/(020.1) = L. 2.5.6. No. Consider, for example, the function f(x) = cosx, a: > O. 2.5.7. (a) Set g(x) = f(x) - e- z , x > O. Then g(O) = 0, g(:c)  0 and lint g(x) = 0. If g(x) = 0, then f'(x) = -e- Z for x E (0,00). -+oo . So, suppose that there is a > 0 such that yea) < o. Then for ufficieutly large x, a.y x :> /, W have g(;c) > g("). Co quently, 9 attains its minim um vdlue at some ZQ in (0, A-f). Thus g'(xo) = O. (b) Apply reasoning analogous to that in (a). 2.5.8. We have ( f(:£» ) ' = g'(:c) ( f'(X) _ f(x) -f(O» ) . g(x) g(x) g'(x} g(x) - g(O) By tbe generalized mean value theorem, ( f(X» ) ' = o'(x) ( fl(x) _ 1 1 (8» ) , g(x) g(x) g'(x) g'(9) where 0 < (J < x  a. Since f'lg' monotonically increases, we see that ( /(X» ) ' > 0 for x > o. g{x) 
288 Solutions. 2: Differentiation 2.5.9. Setting f(x) = sin(cosx) - z, we see that /(0) = sin 1 and f(1r/2) = -1r/2. The intermediate value property implies that there is an ZI E (0,1f/2) such that /(ZI) = O. Since f/(x) < 0 in (0, 1f/2) , there are no other zeros of f in this interval. In an entirely similar manner one can show that there is a unique root in (0, 1f /2), say X2, of the equation cos(sinz) = z. 1 0.8 X.X 2 1 Moreover, we have ZI = sin(COSZl) < COSXlt X2 = cos(sinz2) > COSZ2. Therlore %) < %2- 2.5.10. Suppose, contrary to our claim, that there is Xl E (a, b] such that /(XI)  O. Then the continuity of f implies that f(x) i: 0 for Z E (a, (J). We may assume, for example, that fez) > ° for Z E (a,p), /( a) = 0, ex  a, and f({J) > O. Then by the mean value theorem, for 0 < E < P - 0:, Ilnf(P) -In/(o +£)1 = I n (.8 - a - £) < 0(.8 - a - e). Letting E -+ 0+, we obtain +00 < C({J - a), a contradiction. 2.5.11. Let 0 < p < q and let Z be positive. It follows from the mean value theorem that In(l+i) 1 1 In(l+;)-ln(l+i) - > - : - 1 + (0 1 + (1 -  _ ;!. 1 q 11 q 
2.5. Applications of Derivatives 289 where (0 e (0, i) ,(1 e (i, : ) · Hence ( -  ) In (1 + ) >  (In ( 1 + ) - In ( 1 + ) ) . Consequently, x In ( 1 + x ) >  In ( 1 + x ) p q q p , or in other words, qln (1 + : ) > pin (1 +) . 2.5.12. The inequality e Z  1 + z, z e IR, follows, e.g., from the mean value theorem. Indeed, we have e Z -1 = e.(  1 for :r. > 0) X and e Z -1 = e( < 1 for z < o. x H x = 0 we get the equality. Let In  An = n E ak and G n = n II at k=1 k=l denote the arithmetic and gcometric mean of nonncgative numbers alt a2.-.. tan. H An  0, then e*"-I> 61: > 0 for k = 1 , 2 , ....n. -A n - . Thus E (-1) II n  II " ale Gn 1 = eO = e'.I" = eA;;-1 > - = --!!. - A An' k=1 k=1 n .n.n which gives An > G n . H An. = O. then An = Gn = O. Since in e Z > 1 + x the equality is attained only for x = 0, we see that An = G n if and only if a1 = a2 = . · · = CIn. 
290 Solutions. 2: Differentiation 2.5.13. Hin the inequality e'  1 +t we replace t by X-Z t we obtain xel; S e + e=(z - 1) for x, z E 1ll So the desired result follows if we replace z by In y. 2.5.14. By the mean value theorem, there is a E (-2,0) such that 1f'(a)1 = 1/(0) -/( -2)1  1/(0)/ +2 1 /(-2)1 < 1. Similarly, there is bE (0,2) such that 1/'(b)1  1. Set F(x) = (f(x»2+ (/'(x»2. The function F attains its maximum on [a, b], say, at an zo. Since F(O) = 4, F(a)  2, and F(b)  2, Xo is in the open interval (a, b). Then F'(xo) = 2/'(%0) (f(%o) + f"(xo» = O. Note that /'(xo) #: 0, because /'(xo) = 0 would give F(xo) = l(xo)2  1, a contradiction. Therefore 1(%0) + /"(xo) = O. 2.5.15. (a) The inequality to be proved is e<luivalent to /(x) = (x 2 + 1) arctan x - z > 0, x > o. Since f'(x) = 2xarctanz + 1-1 > 0, we see that I(x) > /(0) = 0 for z > o. (b) By Taylor's formula with the Lagrange form for the remainder, 2 3 ( sing {I 2 sin I ) 4 2 3 (1) 2tanz=2x+ 3 x +2 COS61 +3 COS 3 {1 x >2x+ 3 x and _ x 3 12-e-(:I.. x 3 lei-eT. (2)Slnhx=x+ 6 + 41 2 x<x+ 6 +4i 2 z. We now show that e'lr/2 < 8. To this end note that (see, e.g., I, 2.5.3) In(! +x) > Z;2 Cor x> o. This implies that inS = 3ln2 = 3ln(1 + 1) > 2. Consequently, 8 > e 2 > e7r/2. It then follows that e ; - eT etr/2 2 < "2 < 4, 
2.5 AppUcations of Derivatives 291 which combined with (1) and (2) gives . z3 z4 2 3 sinhZ<.Z+1f+1f<.2:t+ 3 % , because :t + !z3 -lz4 > 0 Cor 0 < z < 2. (c) Set /(z) = In:l: - ; for z > 0, :r i: e. We have /'(z) = e;l!z . Thus f'(x) > 0 if 0 < z < e, and /'(;z;) < 0 if z > e. Consequently, f(x) < fCe) = 0 if x #: e. (d) For z > 1 tbe inequality ro be proved is equivalent to f(z) = 2zhlX-Z 2 + 1 < O. Since /'(z) = 2ln:r + 2 -  and /"(x) =  - 2 < 0, we get /'(z) < /'(1) = 0, and consequently, lex) < 1(1) = O. For 0 < % < 1 the inequality to be proved is equivalent to /(x) = 2z1nz - %2 + 1 > O. Since /"(z) = : - 2 > 0, we get f'(x) < /'(1) = 0, and therefore f(z) > 1(1) = O. 2.5.16. (a) By (c) in the foregoing problem we get In 1r < t which nleaDS that eft > T. (b) By (d) in the foregoing problem we see tbat .J2ln..;2 < i. This gives 2v'i < e. (c) The inequality In 8 > 2 is proved in the solution of (b) in the foregoing problem. 2.5.17. (a) The inequality toO be proved is equivalent to ( b ) In(I+=) .1 1 + - < e-. z Since In(1 + t) < t for t > 0, 
292 Solutions. 2: Differentiation (b) For positive integers m and n, define the function 1 by setting 1(:1:) = (1 +;:;f (1-;t, 1:1:1 < min{m,n}. Then I'(x) < 0 if x > 0 and f'(x) > 0 if z < o. Therefore f(x) < f(O) = 1 for x i: 0, Ixl < rnin{m, n}, which implies the desired result. (c) Set f() = m( v I + Z 2 + 1) -  -lnz, z > O. Then 1'(:1:) = (1- :1:)("'1 +:1: 2 + 1) +z2 . z2( " 1 + z2 + 1 + x 2 ) Clearly, I'(x) > 0 if 0 < x S 1. If x> 1, then (1 - z)( vi I + x 2 + 1) + x 2 > 0 if and only if Z2 > (x - 1)( V I + Z2 + 1). The last inequality is equivalent to :£2 . -1> V l+:£2 z 1 ' which can be proved by squaring both sides. So, /'(x) > 0 for all positive x. Moreover, since lim"In ( 1 + VI +x 2 ) _ 0, 2:-'00 X we see that lim /(z) = o. Consequently, I(z) < 0 for x > o. -+co 2.5.18. (a) Set x I(z) = In(l +) - V ' l+z x> o. Then f'ex) = 2';1 + - 2-  < 0 2(1 +z) v 1 +x ' because v I +z < 1 + ,x > o. Thus I(x) < 1(0) = o. 
2.6. Applications of Derivatives 293 (b) For z > 1 the inequality follows from (a). Indeed, it suffices to replace z by z - 1. If z e (0,1), then we apply the proved inequality to  > 1. 2.5.19. (a) It suffices to apply Taylor's formula to f(z) = (1 + z) In(1 + z). (b) By Taylor's fonnula, z2 sin' Z3 :£2 In(1 +cosz) = In2 - -:4 - (1 +COS{)2 · 3f < 102- 7- 2.&.20. (a) Set fez) = ez - 1 - ze Z . We have f'(x) = -ze Z < O. Hence /(z) < /(0) = o. (b) Setting /(z) = e Z -1- z - z2e z , we get /'(z) = e Z - 1 - 2z - z 2 e z < 1 +ze z -1-2ze z _2ez = -xeZ(l +z) < 0, where the first inequality follows from (a). (c) If /(z) = zel- eZ + 1, then f'(x) = e t (1 +  -e t ) < 0, because ez > 1 + z for z > o. (d) The inequality to be proved is equivalent to the easily verifiable inequality z < (1 +z)In(1 + z). (e) We will prove the equivalent inequality (z + 1)(10(1 + x) -10 2)  z1oz. To this end consider I(z) = (z+1)(1n(1+z)-102)-zloz. ThiS function attains its global maximum at z = 1. Therefore I(z)  1(1) = O. 2.6.21. Taking logarithms of both sides, we rewrite the inequality to be proved in the form (e-z) m(e+z) > (e+z) lo(e-z). Now consider the function / defined by fez) = (e - z) 1o(e + %) - (e + z) 1o(e - z). We have I"(z) > 0 for z e (0, e). Consequently, /'(z) > /'(0) = 0, which implies I(z) > 1(0) = o. 
294 Solutions. 2: Differentiation 2.5.22. Setting f(x) = e z - 1 + In x - 2x + 1, we get f'(x) = e z - I + .!. - 2. x So for x > 1 we have f " ( x ) = e z - J - 1 > 0 because e z - 1 > 1 and  ' -!rr < 1.. 2.5.23. (a) Set I(x) =  tan x + i sin x - x. Then f ' (x) = 2(1- COSX)2 (cosx +!) > 0 ( '11" ) 3 cos 2 x for :c E 0, 2' · This means that f is strictly inct e<)sin g. So I(x) > 1(0) for O<x< : . w (b) We define f(x) = x - 2:8ZZ . Then f'(x) = (cosx - 1)2 > O. (2 + cosx)2 - (c) Putting f(x) = ::z - x for 0 < x < ; , we see that I ' ( ) _ 1 + cos 2 x - 2 Vc osxcosx (1- cosx)2 0 x- > >. 2 vco sx cosx 2 v'c osx cosz 2.5.24. Let I(x) = zQ + (1 - x)Q. Then /' vanishes only at x = . lvloreover, the function attains its global minimum value 20 1 -1 at this point and its global maximum value 1 at the endpoints of [0,1]. 2.5.25. Dividing both sides by :r;l, we see that it suffices to prove (1 + t)Q < 1 + t a for t > O. H I(t) = (1 + t)Q - 1 - ttJ, then J'(t) < o. Thus J(t) < /(0) for t > o. 2.6.26. Col1sideJ.. the function a(a - 1) . l(x)=(I+x)Q-l-ox- 8 x 2 , xe[-l,l]. 
2.5. Applications of Derivatives 295 We have 1(0) = 0, /'(0) = O-and, for x E (-1,1), 0 ( 1 - a ) /"(z) = --0:(1- 0)(1 + X)Q-2 + 4 0 ( 1 - 0 ) < -2 n - 2 a(1 - a) + 4 1 = 4 Q (1 - 0)(1- 2 Q ) <: O. Consequently, f' decreases on tbe interval (-1,1). Thus f'ex) > 0 for x E (-1,0), and f'(x) < 0 Cor x e (0,1). It then follows tbat I attains its maximum at zero. Since /(0) = 0, we see that 1 (x)  0 for z e [-1,1]. 2.5.27 [D.S. Mitrinovic, J.E. Peearic, Rendiconti del Circolo lvlat. di Palermo 42 (1993), 317-337J. Consider the function a(a - 1) I(x) = (1 + x) 01 -I-ox - 2 (1 + By-2x2, x E [-I,B]. We have f'ex) = a(1 + x)Q-l - a - 0(0 - 1)(1 + B)Q-2 X and /"(x) = O(Q - 1) (1 + x)O-2 - (1 + B)a-2) . (a) IT 0 < a < 1 and x E (-I,B), then f"(x) < 0, which means that /' decreases. Thus 0 = /'(0) < /'(x) if z E (-1,0), and 0= /'(0) > /'(x) if x E (0, B). Therefore / attains its maximum at zero. Hence f(x) :S f(O) = 0 for x e [-I,D]. Finally, since (1 + B)tl > 1, we see that Q Q(a - 1) 2 (1 + z) - 1 - oz - 2{I + B)2 :Z: <. J(:e) <: 1(0) = o. (b) As in (a), if 1 < Q < 2 and x E [-1, B], then fez) > f(O) = 0, and consequently, (1 + :c)" - 1 - a:c - (t+ ;;2 :il  lex)  /(0) = o. 
296 Solutions. 2: Differentiation 2.5.28. (a) Define {  if x e ( '0 !: ] I(x) = z. '2 ' 1 if z = o. We will show that I decreases on [0, i). By the mean value the- orem . sin% /'(x) = zcosz - smz = cosz - %"' = x 2 Z cosx - cosO z , where 0 < 0 < z. This implies that I'(x) < 0 on (0, ; ]. Since I ( f) = , the desired inequality follows. (b) The inequality to be proved can be written in the form 3 x 3 sin x > -z -4-. -1[' w3 Set I(x) = sinx - x + 4;;, 1= [0. i] and J = [  . ; ] . We have /(0) = / ( ; ) = 0 nd / () > O. Moreover, /"(0) = 0, I" ( : ) < 0 and 1(4)(x) > 0 for x E I. This implies that I"  0 OD I, which means that I is concave on I. Since /(0) = 0 and / () > 0, we see that I(x) 2: 0 for x e I. Now we show that I' is convex on J. Indeed, since 1(3) ( : ) > 0 and 1(4)(Z) > 0 for x E (i, t) I we see that the third derivative is positive on J. Moreover, we have II ( : ) < 0 and I' (i) = O. It then follows that f'(x) < 0 for z e J, which together with f ( ; ) = 0 gives I  0 aD J. 2.5.29. Assume first that x E (0, j). Then 1f;!8 < 1rX 2 . Hence the inequality sin 1rZ > 1rZ- ";f3 (see, e.g., 2.5.1(b» shows that sin 1rZ > 1I"Z- .;f3 > 1I"x(l-z). To prove the other inequality we consider the function defined by f(x) = 4x - 4x 2 - sin 1I"Z, x E [0, ]4O We have I' (x)  4 - &; - 11' C051rZ and I" (x) = -8 + .«2 sin 1(:£. Thus /"(xo) = 0 if and only if 1 . 8 Zo = -arcsm- 11' 11'2 ' 
2.5. Applications of Derivatives 297 and I" (0) = -8, and f" () = 11'2 - 8 > O. Hence I" (x) < 0 for z e (0. Xn) and j"(x) > 0 for x E (xo. i), or in other words, J' strictly decreases on (O,xo) and strictly increases on (zo, ). Moreover, since 1'(0) = 4-11" > 0 and /'(!) = 0, we see that /'(x).< 0 for x E (XOt ), so also /'(xo) < o. This implies that there is Xl E (O,xo) such that I' (XI) = O. It follows from the monotonicity of f' that 1 increases ou (O,;CI) and decrees on (Xl, l). Since /(0) -- I() -= 0, we get /(x)  0 if z E (0,  ). So we have proved that the inequalities hold for z E (Ot ). It is easy to check that they also hold for X = . Finally, note that the inequalities are not changed if we replace z by 1- x. Therefore they are satisfied for all x E (0,1). n . 2.5.30. Define J(z) = e% - E ir - : (e% - I), z > O. Then 1e=0 n Ie-Ill J'(x) = e Z - L z - !e:r: - _e z +- i=l (k -l)! n n n' and n Ie-i 1 / (f) (x) - e % -  x - = e z - _ e z , - 2 3 . n -  (k _ ,) , ,- t ,..., · Ie=, · n n Moreover, 1(1)(0) = -  < 0, I = 2,3,...,n, and 1'(0) = 0, and /(0) = o. Since j(n)(x) < 0 for z > 0, the derivative /(n-l) strictly decreases, which implies that /(n-l)(x) < /(n-l)(O) < o. This in turn implies the monotonicity of /(n-2) and /(n-2)(x) < 0 for x > O. Ry repeating the reasoning we obtain lex) < /(0) = O,:c > 0 " 2.5.31. Since /'(x) = _e-z, we see that the derivative vanishes only at zero. Moreover, if n is even, then /' (z) < 0 for x  O. So in this caso I does not havo any local oxtrema. On tho other hand, if n is odd, then f'(x) > 0 for z < 0 and I'(x) < 0 for z > O. So in the case where n is odd, 1(0) = 1 is a (global) maximum value of J. 2.5.32. The derivative f'ex) = (m + n)zm-l (1 - %)n-l ( m';.n - z) vanishes only at Xo = 0 (if m > 1), Xl = 1 (if n > 1), and at X2 = m':.n . It is easy to verify that l(z2) = (m':.:;::+11 is the local maximum value of /. Moreover, if m is even, then f(xo) = 0 is the 
298 Solutions. 2: Differentiation local minimum value of 1- On the other hand, if m is odd, any local extremum of / does not occur at zero. Analogous analysis shows that if n is even, then I($J) = 0 is a local minimum value, and if n is odd DO local extremum of J occurs at Xt. 2.5.33. It follows from the result in the foregoing problem that the maximum value (,:::;a:+D of / is attained at x satisfying the equation 5in 2 x = m m+n 2.5.34. For x  0, 1, 1 I ' ( $ ) _ 1 3 - $ - 9. if Z 2 (1 - x) So /' vanishes at x =  . Moreover /'($) > 0 if x E (0, l) and I' (x) < 0 if x E (, 1). Thus I() = -¥ is a local maximum value of I. The function is not diHerentiable at 0 and at 1. Since I(x) > 0 for x E (0,1) and fez) < 0 for x < 0, no local extremum of / occurs at zero. But /(1) = 0 is a local rninim um value of I, because 1(:1:) > 0 = 1(1) for z > 1 and for z e (0,1). 2.5.35. We have I' (z) = arcsin x. Thus zero is the only critical point of /. Since 1(0) = 1, and /(-1) = 1(1) = i,  is the global maximum value and 1 is the global minimum value of / in [-1,1]. 
2.5. Applications of Derivatives 299 2.5.36. For x > 1 the derivative /' is negative, and consequently, f(x} < 1(1) = . For x e (0,1) we have f'() = 0, f'(x) < 0 if x e (0,  ), and f'(x) > 0 if x e (, 1). Thus f() = t is the local minimum value of the function. For x < 0 the derivative /' is positive and therefore  = f(O) > f(x). - So tbe global maximum value of / is /(0) = /(1) = . On the - other hand, since lim f(x) = fun f(x) = 0 and f(x) > 0 for all z-too %-+-00 Z e IR, the greatest lower bound of / (IR) is zero, but the function is not rninimized over III 2.5.37. (a) The global maximum value of the function x  ze-, z 2= 0, is J(I) = . Therefore n 1 L -a, 1 1 1 - ake < -. n. - = -. n 11:=1 - nee (b) As in (a), it is enough to find the global maximum value of rye  ...2 e -:e ,.. > 0 oN r-r ... ,... _ . (c) If one of the numbers ai is zero, the inequality is obvious. So suppose that all ak are positive. Then, taking logarithms of both sides, we obtain the equivalent form of the inequality: fa E{lnalc-) 1n3-1. k=1 Now it suffices to find the global maximum value of x :& .-+ In x - 3 ' :c > o. 2.5.38. We have f'(x) = { :e-tlr «J2+sinHsgnx-cos) Since if Z:F 0 1 if x = o. sin! :I: cos .!. I < V2, z x 
300 Solutions. 2: Differentiation we see that /'(x) > 0 if x > 0, and f'(x)  0 if z < o. Therefore no local extremum of J occurs at Z:F O. Moreover, 0 = /(0) is a global minim um value of f, because f(x) > f(O) = 0 for z #: o. 2.5.39. Note first that f(z) > 1(0) = 0 for x 1: O. Moreover, f'(x) = { % 0 2 (8x + 4xsin  - COS) i£ if  x ;: 0, x=o. Consequently, if n e z \ {O, I}, then 1 , ( 1 ) 1 ( 4 ) - = --1 <0 2n1f' 4n 2 7("2 n7(" 1 and if n E Z \ {-I}, then I' ( 1 ) 1 ( 8 1 ) 0 (2n + 1)# = (2n + 1)211"2 (2n + 1)7(" + >. 2.5.40. Observe tbat sinh x > 0 and tanh z > 0 Cor x > o. Thus the inequality sinh x tanh < X vs inh 2 z + cosh 2 x can be rewritten in the following equivalent form: 1 1 vsinh 2 x + cosh 2 x < cosh x · This inequality is obvious. The other inequalities can be proved by standard arguments. 2.5.41. For 0 < a < b, set x = In.[f. Then b-a b-a fa b-a 1 lr-a 2 < <In -< <-. . 2 { a 2 !b 2 b + a a 2..;ab 2 2ab Dividing by ";a gives the desired inequalities. 
2.&. AppllcatioDS of Derivatives 301 2.&.42. (a) We have 1 ( %P + tiP ) P 1 1 ."t"" .1 lim = lim e. n = e"t Inz, = .;zy, p-tO 2 p-tO because by I'Hospital's rnIet ( .'.J...ttP ) ' .p P In nm!tn % +fI =lim 2 =!In. p-+O P 2 p-+O pi 2 Z1J (b) For p.:p 0, set 1(P) = ( :OPt"" ) 1/.. It suffices to show that F(P) = In /(p) = ! In zP + JJP P 2 is a strictly increasing function. We have F'(p) =  C:. :1/P (ln% + V'1n,) -In :r: P ; "" ). Now let G(p) = :r: P : 1/P (In:r: + V' In fI) -In :r: P ; flP · Then , _ p[ (ln2:r: + flP 1n 2 1/ )<:r: P + flP) - (:/:P In:r: + ""In" Y] GM- + · Our aim is to show that (1n2:r: + V'1n2 fI )( + V') - (In:r: + 1I"lnfl r  O. Applying the Cauchy inequality (%IJJl + %2112)2  (z + xl)(lIf + ,I:> with XI = z', %2 = 11', tli = %llnz, 1/2 = "I InJl, 
302 Solutions. 2: Differentiation we obtain (zi · zi Inz + Vi 'V i Jny r ::; (zP + 11")( zP Jn2 Z + y"ln 2 v). Hence (xPlnz +1flny)2 < (x P + yP)(x P ln 2 z + y P ln 2 y). This means that G'(p)  0 for p > o. Consequently, in this case we get G(p) = y F'(P) > a(O) = o. If p < 0, then G'(p) < 0, and so G(p) = yF'(p) > G(O). S limmin g up, we see that the function p ...... 1(P) is strictly increasing on each of the intervals ( -00,0) and (0,00). It then follows by the definition of 1\;/0 (x, y) (see 2.5.42) that I is strictly increasing on III 2.5.43. For A  1, define zn +yfl +  «x + y)R -:en _ y") I()..) = 2 + (2n - 2) . Using the inequality (x + y)n S 2 n - 1 (x" + V"), one can show that I'() ::5 o. So / is decreasing on [1,00). Since 1(1) = (z + y)n /2", the right inequality follows. To prove the left inequality it is enough to show that lim I()  ( V-xy )n. By the -.oo arithmetic-geometric mean inequality, lim 1(>') = (x + y)n - zn - yR A-foOO 2 n - 2 ()xy"-l + (;)z2 y n-2 + . . . + (nl)xn-ly 2 n -2  2"-2 (:cyn-l )()(x2yn-2)(2) ... (xn-ly)("l) = (JXY)n, where tbe last equality follows from the identity () + 2(;) +... + (n -I)(n  1) = n(2 n - 1 -I), which in turn can be proved using the fact that k(;)=n(;=D, k > l. 
2.5. AppllcatioDS of Derivatives 303 2.&.44. (a) Set f(z) = sin(tan:z:) - % for z e [0, i] . Then 1 /(0) = 0 and /'(%) = cos(tau z) 2 - 1, cos z and therefore, /'(z) 2= 0 if and only if COS(tUIIZ)  cos 2 x. Note now that cos(tan x)  1 - tru;2 Z (Sl-'e t e.g., .2.5.1(a». So it suffices to show that 1 - tA2   c:os 2 :z: Cor % e [0, i] . The last inequality can be rewritten in the form 2005" z - 3005 2 z + 1  0, which is clearly satisfied Cor all % e [0, f] . (b) For:z: e [0, f] , define f(%) = tan(sinz) - z. Then I cosz /(0) = 0 and / (:I:) = cos 2 (si.llz) -1. Consequently, f'(x)  0 if WId only if' > 2 ( . ) 1 + cos(2sinz) cos% _ cas 81D% = 2 · So it suffir.es to prove the lWit inequality. To thlli end note that, by 2.5.1(c), 1 +cos(2sinz)  2 - 2sin 2 z+ sin" z  2OO8z. To see that the last inequalit.y holds for z e [0, f] t standard arguments can be used. 2.&.45. Define fez) = "n z - . Then for :e e (0,1r/2) we bave /'(x) > 0 if and only if 1 cosz -;- > . 3 t % SIn % or in other words, if .and only if sinz -z>o. c osz 
304 Solutions. 2: Differentiation Now, if we put . SlDZ 9( Z ) - -z - -Vc osz ' then g'(z) = (cosz)1 + !(COSZ)-t 8in 2 z - 1 3 and g"(3:) = (cosz)-I sins z. 9 Since 9"(2:) > 0 for z e (0, w/2) , we see that g'(z) > 9'(0) = o. Consequently, g(z) > g(O) = 0 for % e (0, '1'/2). TIiJj in turn implies that I increases on that interval, and so fez) S I (I) = 1- . 2.15.46. It is enough to observe that ( 3Z ) ' ( "' 1 + z 2 - 1) 2 0 arctanz - = > . 1 +2  1 + z2 (1 +Z2) (1 +2  1 +Z 2) 2 2.6.47. If a" = hk for all Ie, then the assertion is clear. So assumE that ale  b. for at least one Ie, and put n I(z) = II (ZOIe + (1 - z)6,,) and g(z) = In I(z) . "=1 Then I  0,,, - hie 9 (z)= L.J za + (l-z)6 1=1 i Ie n ( 6 ) 2 " Ok - Ie and 9 (z)=- E (1- )b · 1=1 zalc + Z t Since g"(z) < 0, the function 9 (so also f) attains its maximum over [0,1] at one of the endpoints if and only if 9'(0) and 9'(1) have the same sign, that is, if g'(O)9'(I)  o. The last inequality means tbat ( t Ot - bt ) ( t Ot-b. ) O. '=1 ak i=1 6i 
2.5. Applications of Derivatives 30S . 2.5.48. By 2.S.1(a) and (c), z2 x 2 x 4 1- - < cosz < 1- - + - z e III 2 - - 2 24' Consequently, to prove our inequality it is enough to show that x 2 x4 y2 y4 x2y2 1-2+24+1-2+ 24 1+1- 2 ' or equivalently, Z4 + y. + 12z2y2 - 12(z2 + y2)  0 for z2 + y2 < 71'. The last inequality can be written in polar coordinates r, 8 as follows: (1) r 2 (2 + 58in 2 29)  24 for r 2  11" and 8 E [0,211"]. Since r 2 (2 + 5 sin 2 28)  711" < 24, we see that (1) is true. 2.5.49. The inequality is obvious if x > 1 or y  1. So assume that x, y e (0, 1) and write y = tx. In view of the symmetry, it is enough to consider the case where 0 < t  1. We have zJJ + yZ = x tz + (tx)Z = (XZ)t + xz. Since the function x  zZ attains its mjnim um value e-l/ = a at : and since F 2= t, we see that x JJ + yZ  at + ta. Moreover, F(t) = at + ta, t e Ilt, has only one local minimum to = 1- e < 0, and F is strictly increasing on (ko, (0), and F(O) = 1. It then follows that x Y + yZ > 1. 2.5.50. For 0 < z < 1, the inequality to be proved can be rewritten in the form I - 2xn + xn+l < (1 - x n ) VI I - xn, or l-x n l-(I-x n ) < . I-x 1- l -zn Since the function t H-  is strictly increasing on (0, 1), it is enough to show that x < \1 1 - zn, or equivalently, 0 < x <  . Finally, note that (1 +  )n > 2 for n  2, which means that  > n:l . 
306 Solutions. 2: Differentiation 2.5.51. For 0 < x < 1, consider the function J (z) Xl z3. 1 g(x) = = 1 - - + - SIn -. x 624z Since o'(x) < 0 for 0 < x < 1, we see that 9 is strictly incr easin g on (0,1). Therefore g(y+z) < g(y) and g(y+z) < g(z), and consequently, yg(y + z) + zg(y + z} < yg(y) + zg(z), which means that fey + z) < fey) + I(z). 2.5.52. We start with the well-known binomial formula (I) (x + y)O = t ( n ) zkyn-k. "=0 k Differentiating (1) with respect to x and multiplying the resulting equality by x, we get (2) nx(x + y)O-1 = tk()xkYo-Ic. 1e=O Now differentiating (1) twice and multiplying the result by x 2 , we get (3) n(n -1)x 2 (x + y)0-2 = tu k(k-I) ()xkyO-Ic. If in (1), (2) and (3) we replace y by 1- x, we obtain 1 = t (:}l'(I- x)o-Ic. k=O nx= tk(:)Xlc(I-X)O-Ic. 1: =0 and n(n - 1)z2 = t k(k - I) ()xlc(1- x) 0-1: . k= O It then follows that t(k - nx)2 ()xk(l- x)o-Ic = nx(l- x)  : . k=O 
2.5. Applications of Derivatives 307 2.5.53. By assumption the equation f(x) = 0 has a unique zero, say , in [a, bJ. a f,r> 0 I'> 0 f" > 0 f'< 0 f" < 0 /'>0 Suppose, for example, that f'(x) > 0 and f"(X) < 0 for x E [a, b]. So we set Xo = a. By Taylor's fonnula with the Lagrange form for the remainder, 1 o = J() = J(xn} + J'(Xn)( - xn) + 2J"(Cn)( - xn)2, where en is an clement of the open interval with the endpoints xn,. By the definition of {x n }, _ f"(c n ) 2  - Xn+l - - 2f'(xn) ( - X n ) > O. So {xn} is bounded above by . Consequently, f(xn} < o. Hence f(xn)  - Xra+l =  - X n + f'(xn) <  - Xn, which means that {Xn} is a strictly increasing sequence. Thus it con- verges, and Jim X n =. The other cases can be proved analogously. noo 
308 Solutions. 2: Differentiation 2.5.54. Clearly, m and M are positive. It follows from the solution mthefuregomgproblth 1 o = f() = f(xn) + f'(xn)( - x n ) + 2 f "(Cn)({ - xn)2, where  is a unique root of the equation f(x) = 0 m [a, b), and en is an elent of the open mterval with the endpoints xn,. Hence I I I f(xn) If" (en) I { ) 2 M ( } 2 Xn+l -  = X n -  - f'(x,,) = 2If'(x n )1 {- X n  2m {-xn · 2.5.55. We will show that sup{2- Z + 2-l : x > O} = 1. Define f(x) = 2-% + 2-, x > O. Clearly, f(l} = 1 and f(x) = J (  ) . Therefore it suffices to show that if x> 1, then f(x) < 1, or in other words, (1) 1 1 - < 1 - - for x > 1. 2 2% By 2.3.7(a) we get ( 1 ) z x 1 - - > 1 - -. 2 z 2 z Now we show that x 1 ( 2 ) 1 - - > - for x > 2. 2%-2 - To this end we write (2) in the form g(x) = 2 z - 1 - x 2= 0 and note that 9 is strictly mcreasmg on [2,00), and g(2) = o. So, inequality (1) is proved for z > 2. Thus f(x) < 1 for z  2. Our task is now to prove that f(x) < 1 for x e (1,2). To this end we define the function h by h(x) = Inf(x) = In (2'" + 2!) - (x +  ) 1n2. Since h'(x) = In2 -2! + 2'" 2% + 2 ' it follows that h'(x} < 0 if and only if (x 2 -1) In 2 < 2x lnx. To prove the last inequality, consider k(x) = (x 2 -1)1n2 - 2xlnx, x E (1,2). 
2.5. Applications of Derivatives 309 Then k'(x) = 2xln2 - 2lnz - 2 and k"(x) = 2(In2 - l/x). Thus k"(x) < 0 if x E (1,1/ln2), and k"(x) > 0 if x E (I/In2,2). Since k'(I) = k'(2) < 0, we get k'(x) < 0 for all x E (1,2). This means that k decreases on this interval; that is, k(z) < k(l) = o. So h'(x) < 0 if z E (1,2), and therefore hex) < he!) = 0, or f(x) < 1 for x e (1,2). Thus we conclude that the inequality (1) holds for all x in (1,00). 2.5.56 [5]. The proof is based on Baile's category theorem. For n E N, define An = {x E [0,1] : /(n)(x) = OJ. By assumption, [0,1] is a union of An. So, by Baire's theorem not every An. is nowhere dense. . Therefore there are a closed interval I and an n such that I C An. Since /(n) is continuous, we have f(n)(x) = 0 for % E I, and consequently, f coincides on I with a polynomial. H I = [0, I], the proof is complete. If not, we can repeat the reasoning on the remaining intervals of [0, 1]. Continuing this procedure, we get a collection of intervals whose union is dense in [0, 1]. Moreover, I coincides on each of these intervals with a polynomial. Our task is now to show that J coincides with the same polynomial in all the intervals. To this end, consider the set B which is left when we remove the interiors of the intervals in the collection. Clearly, B is closed. Moreover, if B is not empty, then each element of B is also a limit point of B. Indeed, if an Xo E B were not a limit point of B, then Xo would be a common endpoint of two intervals, say 11 and 12, such that /(nl)(x) = 0 for x E II and /(n 2 )(x) = 0 for x E 1 2 . Thus I(n)(z) = 0 for x e 11 u 12 and n  max{n.,n2}. Since /(n) is continuous, 1 would coincide with one polynomial on the union of 11 and 12, and consequently, Xo would not belong to B. A contradiction. Since B is closed, if it is not empty we can apply Baire's theorem again. So there is An such that An n B is dense in B n J, where J is an interval. This implies that fen) vanishes on B n J. On the other hand, there is a subinterval K of J which is complementary to B. Therefore there is an integer m such that /(m)(z) = 0 for % e K. H m < n, then I(o)(z) = 0 for x E K. H m > n, then /(n+I)(z) = /(n+2)(x) = ... = j(m)(z) = ... = 0 (or x E BnJ, because each point of B is also its limit point. In particular, /(n+l,(x) = l(n+2)(x) = ... = f(m)(x) = ... = 0 at the endpoints of 
310 Solutions. 2: Differentiation K, say a and b. So, for each $ E K we have o = l j<m)(t)dt = j(m-J) (x) -f(m-l)(a) = j<m-J)(x). Repeating the process, we get f(n)(x) = 0 for x E K also in the case where m > n. Of course, this reasoning applies to every subinterval K of J which is complementary to B. It then follows that I(n) (x) = 0 for x E J, and consequently, there are no points of B in J, a contradiction. Thus B is empty, which means that I = [0, 1] was the only interval to begin with. 2.5.57. Let { 0 if x E [0,1/2], f(x) = (x _ !)2 if x E (1/2,1). Then /'(z) = 0 for x E [0,1/2] and /(3)(X) = 0 for z E (1/2,1]. The following example shows that the conclusion of 2.5.56 is not true if lim fCn){x) = 0 for each z E [0,1]: n-to 00 f(x) = sin i, x E (0,1]. 2.6. Strong Differentiability and Schwarz Differentiability 2.6.1. It suffices to set $2 = a in Definition 1. The converse does not hold (see, e.g., 2.1.13). 2.6.2 [M. Esser, O. Shisha, Amer. Math. Monthly 71 (1964), 904- 906J. Let E > 0 be arbitrarily chosen and let 6 > 0 be such that B = {x: Ix - al < 6} c A and if Xl,X2 E B, Xl :F %2, then f(2:2) - I(XI) _ /-(a> 1 < £. X2 - Xl 
2.6. Strong and Schwarz Differentiability 311 Now if x e A l (that is, if f'(x) exists) and if Ix - al < , then for all X2 such that I X 2 - xl < t, - f(x2) - f(x) _ la(a) 1 < E. X2- X Letting X2  x gives II' (x) - I4I(a)1  E. SO, J f'(x) = f-(a) = zeAl I' (a). Since A. C A l, it then follows that 1im f-Cx) = /-(a) = f'Ca). %-'0 zEA- 2.6.3 [rvl. Esser, O. Shisba, Amer. Math. Monthly 71 (1964), 904- 906}. Since I' is continuous at a, the mean value theorem yields lim f(xl) - f(x2) = lim f'(x) +9(X2 -Xl» = f'(a). (%1.%2)-.(0.0) Xl - X2 (ZI. Z 2)-'(0.0) :z: 17b:2 z 1 Z2 2.6.4 [1\11. Esser, O. Shisha, Amer. Math. lvIonthly 71 (1964), 904- 906]. No. Consider the function I defined on tbe interval (-1,1) by f(x) = f: g(t)dt, where o if t E (-1,0] U (j [ 2k1 '  ) , get) = k=l tift e U [ 2 ' 2k:'1 ) . k=l Then f is continuous on (-1, 1), and lim /(Xl) -/(:£2) = lim 1 1 z1 g(t)dt = o. (%1.%2)-.(0.0) Xl - x2 (%1.%2)-.(0,0) Xl - X2 %2 :r:l%2 ZIZ2 The last equality follows from the fact that 1 %1 x2 x 2 o  g(t)dt < 1; 2 for X2 < Xt. %2 So, f is strongly differentiable at zero. On the other hand, f' does not exist at , n = 3, 4, 5, . .. . 
312 Solutions. 2: Differentiation 2.6.5. The result follows immediately from 2.6.2 and 2.6.3. 2.6.6 [C.L. Belna, M.J. Evans, P.D. Humke, Amer. lvlath. Monthly 86 (1979), 121-123]. Note first that f' is in the first class of Baire, because f ' ( ) - Ii f (z +  ) -lex) z-m 1 . n....oo - n Therefore the set of points of discontinuity of /' is of the first category (see, e.g., 1.7.20). So, the assertion follows from the result in 2.6.3. 2.6.7. Let Q be a real number such that f(a) < 0: < I(b), and denote c = inf{x E (a, b) : f(x) > Q}. Clearly, e :F a and e :F b. By the definition of the greatest lower bound, f(x) < Q for x E [a,e], and there exists a positive sequence {h n } converging to zero such that f(c + hn) > Q. Since' is Schwarz differentiable at e, '8(C) = lim J(e + h n ) -fCe - hn) > o. n....oo 2h n - It is worth noting here that in much the same way one can show that if f(a) > f(b), then there is c E (a, b) such that f.(c) < O. 2.6.8 [C.E. AuIl, Amer. Math. Monthly 74 (1967), 708-711J. If / is identically zero on [a,b], then the statement is obvious. So assume that there is C E (a, b) such that, for example, I(e) > o. Then it follows from the foregoing result that there are XI and X2 such that a < Xl < C < Z2 < b, /.(Xl)  0, and '.(X2)  o. 2.6.9 (C.E. Aull, Amer. lvlath. Monthly 74 (1967), 708-711]. It is easy to see that the auxiliary function F(x) = f(x) - f(a) - f(b) -f(a) (x - a) b-a satisfies the assumptions of the foregoing problem. 2.6.10 [C.E. Aull, Amer. Math. Monthly 74 (1967), 708-711]. Since f. is bounded on (a, b), there is M  0 such that If.(x)1  M for all X E ( a, b). It follows from the result in the foregoing problem that _11.' < f(x) - J(t) < M if ( b) '# J....l . x, tea, ,x t. - x-t - 
2.6. Strong and Schwarz Differentiability 313 Consequently, If(x) - f(t)1  kllx - tl. 2.6.11 [C.E. Aull, Amer. Math. Monthly 74 (1967), 708-711]. By 2.6.9, there are Xl and X2 between X and x+h (x,z+h E (a, b» such that f.{x2) S f{x +  - f(x) S f.{x1). On the other hand, by the continuity of 1 8 there is X3 between x and x + h such that f8(X3) = !(z+hl-/(z) . Upon letting h -+ 0, we get f8(x) = f'ex). 2.6.12. If x, z are in I and x < z, then by 2.6.9 there is X2 E (x, z) such that fez) - f(x)  f8(X2) 2: o. z-z 2.6.13. As above, it suffices to apply the result given in 2.6.9. 2.6.14. No. Consider, for example, the function f given by I(x) = x - 2Jxl, x E (-1,1). It is easy to check that f.(O) = 1 and 1(0) is the maximum value of Ion (-1,1). 2.6.15 [C. Belna, M. Evans, P. Humke, Amer. Math. Monthly 86 (1979), 121-123]. It suffices to show that there is a residual set on which the first equality holds, because the second equality can be obtained by replacing I by -I in the first one. By definition, D.f(x)  D.f(x). Our task is to show that the set A(/) = {x: D./(x) > D.f(x)} 
314 Solutions. 2: Differentiation is of the first category. Observe that A(f) is a countable union of the sets A(f,a) = {x: DlJf(x) > a > D.f(x)}, Q E Q. SO, it is sufficient to show that each of these sets is of the first category. Since A(f, a) = A{g, 0), where g(x) = f(x) -QX, it is enough to show that A{f, 0) is of the first category. To this end, note that 00 A(f, 0) = U Anef,O) n=1 = Q ({ x : f(x - h) < f(x + h) for 0 < h < } n A(f, 0»)- Thus it suffices to show that all the sets An(f,O) are of the first category. Suppose, contrary to our claim, that there is n E N such that An(f, O) is of the second category. Then there is an open interval I such that An(f,O) is also of the second category on every open subinterval of I. Assume additionally that the length of I is less than  and that a, bel with a < b. Let S c 1R be a residual set such that fls is continuous, and choose e E S n ( a, b). Let E > 0 be chosen arbitrarily. Then there is an open subinterval J of the interval (a, b) such that e e J and (1) lex) > fee) -e for x E SnJ. Now let K be an open subinterval of (a, b) such that K = 2K - b = {y : 11 = 2x - b, x E K} c J. Since the set 8n = { x : f(x - h) $ f(x + h) for 0 < h <  } is of the second category on K and S is residual on K, we see that the set (2S n - b) n (S n K) is of the second category, hence non empty. We can pick an xES n K such that zt b E Sn. Consequently, taking h = biz (clearly, ° < h < I/n), we see tbat f(x) S I(b). Moreover, (1) implies that f(e) -g < f(x). In view of the arbitrariness of E > 0, we get fee)  f(b). In an entirely similar mnn er one can show that f(a) < fee). So we have proved that f increases on I. Consequently, D.f(x)  0 for x E I. Thus A(ftO) n 1 = 0, a contradiction. 
2.6. Strong and Schwarz Differentiability 315 2.6.16. The result follows immediately from the foregoing problem. Note that this is a generalization of 2.6.6. 2.6.11 [J. Swetits, Amer. l\1ath. fvlonthly 75 (1968), 1093-1095]. We may assume that I is locally bounded on [Xl, xo) and %0 - XI < 6 < 1. Let X2 be the midpoint of [XI,XO). Then there is M > 0 such that I/(x)1  M for x e [Xl,X2]. Choose h, 0 < h < t, such that I/(X2 + h)1 > 1 + /vI + I/ S (X2)1. Then I{X2 + h) - I(X2 - h) _ J B ( ) 2h X2 > I fe x 2 + h)  fe x 2 - h) _ If'e x 2)1 > 1/(X2 + h)I-I/(X2 - h)1 -1/"(X2)1 2= 1/(X2 + h»)- kJ -1/-(X2)1 > 1. So, I is not uniformly Schwarz differentiable on [a, b]. 2.6.18. One can use the result in 2.6.9 and proceed as in the solution of 2.2.26. 2.6.19. Consider the function defined as follows: ( { 0 if x E Ii \ {O}, f z) = . 1 If x = o. Then f" is identically zero on I:t, so continuous, but J is not uniformly Schwarz differentiable on any interval cont ainin g zero. 2.6.20 [J. Swetits, Amer. Iath. Ionthly 75 (1968), 1093-1095]. As- sume first that / is uniformly Schwarz differentiable on every [a, b] c I. Let Xo e (a, b) and let 6 1 > 0 be such that [xo-61,xo+61] C (a, b). Put 11 = (xo - 6 1 ,xo + 6 1 ). Since f is locally bounded on I, there is M > 0 such that I/(x)1  M for x Ell. Let 6 > 0 be such that I fe x + h); f(x - h) -f'(x) < 1 
316 Solutions. 2: DifFerentiation for Ihl < 6 and z e [a,b]. Then, for z e 1 2 = (%0 - t,Zo +) and Ihll < min{6,6 1 /2}, 1/.(z)1 < 1 + /(:1: + hl)2/(:I: - hi) I < 1 + r Thus ,- is locally bounded on I. We now show that , is continuous on I. Suppose, contrary to our claim, that 1 is discontinuous at an Zo e [a, b) C I. Then there is E > 0 such that for every 6 > 0 there is z' e [a, b) n (zo - 6, Zo + 6) for which I/(z') -/(zo)1 > E. Since I- is local1y bounded, there is Ml > 0 such that 1/-(%)1  M I for:e in the interval with endpoints Z' and xo. Consequently, I f(z') -f(zo) _f. ( z' +%0 ) > e - M 1 z' - xo 2 - Ix' - zol ' which contradicts uniform Schwarz differentiability of 1 on [a, b). S we have shown that f is continuous on I, and, by the result in 2.6.1 80 is f'. This \X>mbined with 2.6.11 shows that f' exists and is contiD DOUB on I. Sufficiency follows imm ediately from the result in 2.6.18. 
Chapter 3 Sequences and Series of Functions 3.1. Sequences of Functions, Uniform Convergence 3.1.1. Suppose first that In  I. Then, given e > 0, there is no such B that Ifn(x) - f(x)1 < E for all n  no and all x e B. Hence, for n > no, dn = sup{l/n(z) -/(x)1 : x e B} ::; e, and consequently, lim dn = O. n...oo Suppose now that lim dn = O. Then n-too I/n(x) -/(x)1 ::; suP{l/n(x) -/(x)1 : x e B} < E for sufficiently large n and for all z E B, which means that {In} is uniformly convergent on B to I. 3.1.2. Given E > 0, we get E E I/n(x) -f(x)1 < 2 and Inn(x) - g(x)1 < 2 - 317 
318 Solutions. 3: Sequences and Series of Functions for n large enough and for all % e A. Thus I/n(%) + On (x) - (/(%) + g(%»1 S l/n(Z) -/(z)1 + 19n(%) - 9(3:)1 < E for n large enough and for all % e A. To see that the analogous assertion does not hold for the product of two uniformly convergent sequences, consider the following func- tions: In{:t) = :t (1 - ;) and 9n(:t) =. :t e (0.1). We have /n =* / and 9n  0, where 1(%) = z and 0(3:) = is. On (0,1) (0,1) the other hand, In(:t)9n(:t) = ; (1-;) . So {/ngn} is pointwise convergent on (0,1) to the function z H> . Since d,. = sup {/In(:t)9n(:t) - ;1 : % e (0. I)} = +00. n e N. the convergence is not uniforn1. 3.1.3. Note first that if {On} converges unifonnly on A to a bounded function 9, then there is C > 0 8uch that for sufficiently large RJ 10n(z)1 S C for an z e A. Given E > 0, by the uniform convergence of {In} and {gn}, we get e £ I/n(z) -/(x)1 < 2C and 19n(z) - g(2:)1 < 2M for sufficiently large n and for all z e A. Hence, for sufficiently large n and for aU % e At I/n(z) · 9n(z) -fez) · g(%)1 S I/n(z) -/(z)1I9n(z)1 + 19n(%) - g(%)I1/(%)1 < £. 
3.1. Sequences of Functions, Uniform Convergence 319 3.1.4. It follows from the Cauchy criterion for convergence of se- quences of real numbers that {In} is pointwise convergent on A, say, to I. Our task is now to show that the convergence is uniform. Let E > 0 be arbitrarily chosen. By assumption, there is no such that if n, m > no, then 1 I/n(x) - Im(x)1 < 2 g for every z E A. By the continuity of the absolute value function we get Jim I/n(x) - Im(x)1 = I/n(z) - f(x)1 < 2 1 E < e m-too for every x e A and for all n > no. 3.1.5. Let {In} be a sequence of bounded functions uniformly con- vergent on A to I. Then, given E > 0, there is no e N such that If(x)1 < Ifno(x) - f(x)1 + Ifno(x)1 < e + If no (x) I for all x E A. Since lno is bounded on A, 50 is f. . . . . . . . . I .....--- . " . '" . '. . " . '" . " . .. . ..., . '.. . '-. n - n 1 The limit function of a pointwise convergent sequence of bounded functions need not be bounded. To see this take, for example, In{x) = win G , n} , x e (0,1), n e N. The sequence {In} converges to the unbounded function x t-+ l/z, 3: E (0,1).. 
. 320 Solutions. 3: Sequences and Series of Functions 3.1.6. For z e Ii, lim In(z) = O. The convergence is not uniform on n-+oo because n = +00. Clearly, the subsequence {/2n-l} is uniformly convergent on Ii. 3.1.7. The proof runs as in 3.1.4. 3.1.8. (a) We have 1 1 + (nx - 1)2 n I(x), where ) { 0 for x E (0, 1], I{x = I for x = o. Since the limit function is not continuous, the convergence is not uniform (see, e.g., 1.2.34). (b) We have z2 -+0 x2 + (nz - 1)2 n-+oo and dn = sup{l/n(x) - 01 : x e [0, I]} = In () = 1. By 3.1.1 the convergence is not uniform. (c) Since x R (I-x) -+ 0 n-+oo and dn = sup{l/n(x) - 01 : z e [0, I]} = In ( nl ) = (n+;n+1 t we see that {In} converges uniformly on [0,1]. (d) The convergence is not uniform because nn+l 1 dn = -» -. (n + l)n+l n-+co e (e) Since dn = I ( n:4 ) -+ 0, the convergence is uniform.. n-+oo (f) The sequence is uniformly convergent because 1 dn = sup{l/n(z) - zl : x e [0, I]} = 1- In(l) = 1 -+ o. n + n-+oo 
3.1. Sequences of Functions, Uniform Convergence 321 (g) The sequence is pointwise convergent to ( { I for z e [0,1), f x) = 1 ? for Z = 1. - So the limit function is not continuous and therefore tbe conver- gence is not uniform (see, e.g., 1.2.34). 3.1.9. (a) One can easily see that In(z) --+ 0 and dn = l. Thus the n..,.oo convergence is not uniform on A. On the other hand, sup{l/n(z)l: z E B} = () n (1- () n). n  2, and therefore the sequence converges uniformly on B. (b) The sequence converges uniformly on JR to the zero function, and so it also converges uniformly on each subset of It 3.1.10. (a) Since dn = arctan '*', {In} converges uniformly on Ii to zero. (b) In(z) -+ z2 t and since In( Jii}-n = n(ln 2-1). the convergence n-.co unn ot be uniform on 1R. (c) We have.ln(z) --+  . The sequence CAI1n ot converge uniformly n-+oo on (0, (0), because In () - n = n(ln 2 -1). (d) In(z) --+ I(z), where n-+oo I (x) = { 1 if Ixl =s 1, Izl if Izi > 1. Set Un = 2v l + z2n. Then, for x > I, u 2n _ z2n 2\1 1 + x 2n - Z = Un - X = n 'un-l + U;n-2z + .. . + z2n-1 1 < 2... n-l + n-2z + . . . + z2n-l - 2n It then follows that d n  sup I/n(z)-j(z)l+ sup Ifn(z)-j(z)1 S 2yt2-1+- 2 1 t Ze(O,I) zE(l,oo) n 
322 Solutions. 3: Sequences and Series of Functions which shows that {In} converges uniformly on 1ll (e) As in (d), one can show that the sequence is uniformly convergent onRto { 2 if Ixl < 2, f(x) = - Ixl if Ixl > 2. (f) We bave d,. = supl vn+ I sinnzcoszl = ( fI\ 1 n   . zeit V n+1 ) n-roo v e Thus the convergence cannot be uniform on B. (g) The sequence is pointwise convergent to In x (see, e.g., I, 2.5.4). By Taylor's formula, d,. = sup In(  -1) -luxl = sup In (elnz -1) -luxl e[l,oJ ze[t.a] = sup I n ( 1 + !.lux - Jn2: e(" -1 ) -lnx < ln 2 a a  , ze(t,o] n 2n 211 because 0 < (n < la . Consequently, Urn d n = 0, which shows n-.oo that In  I, where I(x) = Inx. (l.a) 3.1.11. We have [nl(x») = nl(x) - Pn(x), where 0  Pn{x) < 1. Hence sup I/n(x) -/(x)1 = sup I pn(x) I  !, ze(a.b) zera.b] n n and therefore In =r I. [0.6) 3.1.12. Since sin .j 47r 2 n 2 + :1;2 = sin (27m = sin 2n7r ( x 2 ) 1 + 41l' 2 n 2 + 2n1r - 21nr x 2 ) 1+ 4 " .,-1 1r-n" x 2 . =sm , V4 11. 2 1f2 + x 2 + 2n1l" 
3.1. Sequences of Functions, Uniform Convergence 323 we see that Urn nsin v'4 7r 2 n 2 + % 2 = . Moreover, if ;r. E [0,0], n-too then, using the fact that sinz  % - " we get I n sin ../4'1r2112 + :1: 2 _  <  ( 1- 2 ) +  riG . 471' - 41r _ , ] + n:l + 1 3! 81&3,..3 V 4 Jr 2 n 2 This: establishes the unifonn convergence of tile sequence on [0,0]. For :z: e nt. b). the inl'quality I sin xl  lxi, we obtain I nSin ../471'2112 + :r 2 - =: 1  =: ( 1- 2 ) , 4Jr 471' _ ' I + z2 + 1 V 41r 2 n 2 which shows that tbe CODvergence cannot be waifonn on III 3.1.13. First we show, by induct.ion, that ror any positive integer fi, o < . 'Z - Pn(x) < 2 2V <!, zerO, 1 ] . - Viii - + 1& Z n ( For n = 1 the inequalities are ob\"ious. Now, assunting the inecluaJities hold for 8, we will prove theln for n + 1. It foUows CraIn the induction assumption that o  Vi - Pn(z)  ..fi. Hence, by the de6nition of Pn+l' ,;% - Pn+l(z) = (,;% - Pn(z» (1- (Ji + PR(Z»). Thus Vi - I+J (z)  o. 1oreovert _ 1:= P. ( ) 2JX ( JX ) V% - n+l Z $ 2 + nv'i 1 - T < 2..fi ( 1 Vi ) - 2 + r,.v'Z - 2 + (n + 1) v'Z 2..[i - 2 + (n + l)Ji" Since Ixl = , it follows frOOl the proved inequalities that the sequence or POlYOOOlials {Pn(Z2)} converges wtiformly on [-1,1] to the absolute value function Ixl. 
324 Solutions. 3: Sequences and Series of Functions 3.1.14. By the mean value theorem, f (x + 2 - f{x) - f'{x) = If'({n) - J'(x)l, n where (n e (x, x + ; ). Since the derivative I' is uniformly continuous on R, given E > 0 there is an 71i) such that if n  no, then 1/'«(n) - 1'(x}1 <  for all x E III Thus the uniform convergence on JR is proved. Consider f(x) = z3, x e nt Then do = sup I (x +) - I{x) - J'{x) = sup 3x.!. + ...;. 1 = +00, zER n zeR n n" which shows that the convergence is not uniform. So we see that the assumption of the uniform continuity of I' is essential. 3.1.15. Let e > 0 be arbitrarily chosen. It follows from the uniform convergence of the sequence on JR that there is an no e N such that c I/no(x) - f(x») < 3 for all x E III Now the uniform continuity of !fIf) implies that there exists 6 > 0 such that I/no(x) - lno(z')1 <  whenever Ix - x'i < o. Consequently, I/(x)- l(x')1  1/110 (x)- f(x)I+l/no(x)- lno(x')I+l/no(x')-/(x/)1 < e whenever Ix - x'I < o. 3.1.16. Set gn(x) = Inex) - I(x) for x e K. \Ve will show that {On} converges to zero uniformly on K. Let e > 0 be arbitrarily chosen. Since {gn} is pointwise convergent to zero on K, for x E K there is Dz such that £ o < 9n.(x) < 2 . It follows from the continuity of Dn. and from the monotonicity of the sequence {On} that there is a neighborhood O(x) of z such that (1) 0 < gn(t} < E for .n > n z and t e O(x).. 
3.1. Sequences or Functions, Uniform Convergence 325 Since K is compact, there are finitely mAny points ZI,..., Zn E K such that K C O(ZI) U 0(%2) U · .. U O(Zn). Now if no = max {nZI' n Z2 '... J n Z .} , then (1) holds for aU n > no and all % E K. To see that the compactness of K is essential, consider 1 In(z) = 1 ,z e (0,1), n = 1,2, ... · +nz Then dn = sup I/n(z) -/(z)1 = I, and therefore the convergence ze(o.1 ) is not uniform. The continuity of the Umit function is also essential. Indeed, the sequence In(z) = zn, z E (0,1], n E N, fails to converge uniformly on [0, 1]. The assumption of continuity of / n caJln ot be omitted, 88 the following example shows. The functioDs { 0 if z = 0 or   Z  It In(z) = 1 if 0 < Z < 1. n I 1 ii I are not continuous. They form a monotonic sequence pointwise con- vergent to zero on [0, 1], but the convergence is not uniform. 
326 Solutions. 3: Sequences and Series of Functions Finally, the functions defined by 2-n 2 z In(x) = 11 - 2n 2 (x - 2 ) o for 0 $ x < 2 ' for in < x  , for !<x<l n - n -- . . 1 1 2D D 1 are continuous and form a sequence which is pointwise convergent to the zero function on [0,1]. Note that the sequence {In} is not monotonic and the convergence is 110t uniform. 3.1.17. Let {In} be a sequence of continuous functions uniformly convergent on a cOlnpact set K to the limit function f. Let E > 0 be given. Choose "nO such that (see 3.1.7) g 1/,,(x) -lno(x)1 < 3 for n> 110 and all x E K. Next, since each function In is unifonnly continuous on K, one can choose 6 > 0 such that if x,x' e K and Ix - x'I < 6, then (1) Ilk (x) - Ik(x')1 < i for 1 < k < no. Therefore we get I/n(z) - In(x')1 < Ifn{x) -/no(x)1 + I/..o{x) -/no(x')1 + Ifno(x') -1,a(x')1 < E for Ix - x'i < 6 and n > no. This together with (1) proves the equicontinuity of the sequence {In} on K. 
3.1. Sequences of FunctioDS, Uniform Convergence 327 3.1.18. Let {In.} be a subsequence of {In}, and {xn} a sequence of elements of A converging to x E A. We define the sequence {11m} by setting XI for 1  m  nit X2 for nl < m  "2, Ym = , XI; for nk-l < nJ  nt, . . . . Then the sequence {Ym} converges to x, so lim Im(Ym) = I(x). m"'oo Thus Jim In. (Yn.) = Iim In. (Xk) = l(x). k-+oo k...oo 3.1.19. Note first that if {In} converges continuously on A to /, then {In} converges pointwise to the same limit function. To see this. it is enough to consider constant sequences all of whose terms are equal to an element of A. Let x e A be arbitrarily chosen and let {xn} be a sequence of elements in A converging to ,x. Given E > 0, the pointwise COil vergence of the sequence implies that there is n 1 (which can depend on Xl) such that £ I/nJ (XI) - l(xl)1 < 2- Similarly, there is n2 ('which can depend on X2), n2 > n I, such that E I/n2(x2) - /(x2)1 < 2 - Continuing tbe process, we get the sequence {nk} such that  I/n.,(xk) - l(xk)1 < 2' keN. Moreover 1 by the result in the foregoing problem, E I/n. (XI:) -/(x)1 < 2' k > ko- Consequently, I/(xk)- l(x)1 S I/n.(xk)-/(xj:)I+I/njr(x,;}- l(x)1 <  for k  ko. 
328 Solutions. 3: Sequences and Series of Functions 8.1.20. Let {xn} be a sequence of elements in A converging to % eA. Let £ > 0 be given. It follows from the uniform convergence of {In} that I/(xn) -In(%n)1 S sup I/(u) -In(I/)1 < ! 2 for n  no. ileA. Since / is continuous, E I/(zn) -/(z)1 < 2 for n  "1- Hence if n  max{nChn.}, we have I/n(xn) -/(z)1 S I/n(zn) -/(zn)1 + I/(zn) -/(z)1 < E. The converse of the statement just proved is not true, as the following example shows. Let A = (0, 1) and /n(z) = %1&. It is easy to see that {In} fails to converge uniformly to zero on (0,1). But {In} does converge continuously on (0,1). Indeed, if {zn} is a sequence of points in (0,1) converging to z e (0,1), then there is 0 < a < 1 such that Zn < G. Therefore lim /n(Zn) = o. noo 3.1.21. The implication (i) ==> (ll) has been proved in the foregoing problem. Our task is to prove (ll) ==> (i). We know (see 3.1.19) that the limit function / is continuous on K. Suppose, contrary to our claim, tbat {In} fails to converge uniformly OD K. Then there are Eo > 0, a sequence {ni} of positive integers, and a sequence {XIc} of elements in K 8uch that I/n.(zAJ -/(xi)1 > £0. Since K is compact, we can assume witbout loss of generality that {Zi} converges, say, to z e K. On tbe other hand, by 3.1.18, Eo I/n. (Xi) -/(x)1 < 3 for k > ko. Moreover, the continuity of / implies that £0 I/(z.) -/(z)1 < 3 for Ie> k 1 . Thus, for sufficiently large k, 2 Eo < I/n.. (Xi) -/(Xi)1 S I/n,. (z,) -/(x)1 + 1/(%) -/(x,)1 < 3EO, 
3.1. Sequences of Functions, Uniform Convergence 329 a contradiction. 3.1.22. Assume, for example, that the functions In are increasing on [a, b]. Evidently, 1 is uniformly continuous on [a, b]. Let e > 0 be given. By the uniform continuity of I there is 6 > 0 such that E I/(z) -/(x')1 < 2 whenever Ix - x'i < 6, x, x' E [a, b]. Now choose a = XO < Xl < X2 < ... < XI. = b so that IXi - Xi-II < 6, i = 1,2,... ,k. Since lim In(xi) = I(xi), i = 1,2,. .., k, noo there exists no such that, if n > no, then (1) Ifn(xi) -/(xi)1 <, i = 1,2,..., k. Clearly, for an z E [a,b] there is an i such that Zi-l  X < Xi. Now the mono tonicity of In and (1) imply E E l(x;-I) - 2 < fn(X;-I)  fn(x) < fn(Xi) < f(x;) + 2 for n > 1J{). Since J must be increasing, we have f(Xi-l) =:; I(x)  f(xj), which combined with the uniform continuity of f yields E E -E < /(Xi-l) - f(xd - 2 < In(x) - f(x) < I(zi) - /(Xi-l) + 2 < e. Thus the uniform convergence of {/ n} on [a, b] is proved. 3.1.23. We wiU first show that there is a subsequence {In.} conver- gent on the set of all rationals Q. Since Q is countable, we can write Q= {rl,T2,...}. The sequence {/n(rt)} is bounded, so it contains a convergent subsequence {fn.t(rl)}. Next, since {fn.l(T2)} is bounded, there exists a convergent subsequence {In.2(T2)}. Clearly, {fn.2(Tl)} is also convergent. Repeating the process, we obtain the sequence of sequences {fn.l}, {/n.2},... with the following properties: . {In.k+l} is a subsequence of {fn,k} for k = 1,2,..., . the sequence {/n,k(ri)} is convergent for keN and i = 1,2, . .. J k. 
330 Solutions. 3: Sequences and Series of Functions So the diagonal sequence {/n.n} is convergent on Q. In this way we bave constructed the subsequence {Ira,,} pointwise convergent on Q, say, to f. Clearly, f is increasing on Q. Now we extend J to Ii by setting I(x) = sup{f(r) : r E Q, r < x}. The extended function I is also increasing on nt Now we show that if f is continuous at x, then Iim In,,(z) = j(x). To this end, consider k-+oo two sequences of rationals {Pn} and {qn} converging to x and such that Pn < X < qn. The mODotonicity of In. implies that In" (Pn)  In,,(x)  InlJ(qn). No\v, letting k  00, we get !(Pn) < Jim in! fnla (x)  lim sup In" (x) < f(q,.). k-+(X) k-+oo Next, upon passage to the limit as n  co (see, e. g., 1.1.35), we obtain I(x-)  liminC /n,,{x)  IimsuP/n.(z}  f(:&+). k-+oo k-+oo It then follows that /(z) = lim In. (x) at each point % of conti- "....00 nuity of I. We know that the set D of points of discontinuity of a monotonic function is countable (see, e. g., 1.2.29). Thus we have f(x) = lim fn,,(z) on the set 1R \ D, and since {In,,} is bounded k-..oo on the countable set D, we can use the diagonal method again to choose a subsequence of {In.} pointwise convergent on D. Clearly, this subsequence is convergent on all of 1ll 3.1.24. If K is a compact subset of R, then there is a closed interval [0, b] such tbat K C [0, b]. Clearly, I is uniformly continuous on [a, b]. By the result in 3.1.22 {In.} converges uniformly on [a, b], and so it also converges uniformly on K. The following example shows that {f n.,} may fail to converge uniformly on III Put ( 1 ) '& /n(X) = ; (arctanz + ) , x e R. Each In is strictly increasing on 1R, and 0 < In(x) < 1. The sequence {fn} is pointwise convergent to I(x) = o. However, the convergence is not uniform. 
3.1. Sequences of Functions, Uniform Convergence 331 3.1.25. We first show tbat if {P n} is a sequence of polynomials con- vergent uniformly on Ii, then, beginning with sorne value of tbe index n, all Pn are of the same degree. Indeed, if this were not true, then for every kEN there would exist nk > k such that the degree of PI: would differ from the degree of Pn... Consequently, sup IP nlf (x) - Pk(x)1 = +00, zER contrary to the Cauchy criterion for uniform convergence (see, e.g., 3.1.7). Hence there is no E N such that if n > no, then Pn(X) = an,px P + an.p-lz P - 1 +... + an.IX + an.O. By the Cauchy criterion for uniforrn convergence again, we see that if n  no, then the coefficients an.i, i = 1,2,... ,p, are constant (in- dependent of n), that is, Pn(X) = apx P + ap_lx P - 1 + ... + aJx + an.o. Clearly, such a sequence of pol)"nomials converges uniformly on IR to the polynomial P(x) = apx P + ap_IxP-J + ... + al X + Clo, where ll{) = lim an,O. n -t ex; 3.1.26. Clearl)", (i) => (ii). \Ve now show that (ii) => (ill). Indeed, (1) On.O + an,lC{) +... + an.p = Pn(eo), UntO + Il,..ICI + ... + an.pc,f = Pn(Cl), Qn.O + an.IC p + ... + an.p = Pn(c p ). Since the so-called Vanderrnondc determinant 1 4) t! Co .. . 0 1 I) cf CI Cj... det " . . . . . . . . . . .. . . 1 ? c: c p c:p . .. 
332 Solutions. S: Sequences and Series of Functions is different from zero, the system of linear equations (1) has a unique solution and an.h i = 0,1,2,. . . ,p, can be determined using Cramer's rule. Consequently, (ii) implies the cODvergence of each sequence {anti}' i = 0,1,2,... ,p. The implication (ill) => (i) is easy to prove. 3.1.27. Since {In} is equicontinuous, given E > 0 one can choose 6 > 0 such that for all n E N (1) e I/n(z} -jn{y)1 < 3 whenever Ix - yl < 6, x, y e K. Letting n -? 00, we get (2) e I/(x) -j(y»)  3 . (Note that this shows that I is uniformly continuous on K.) As K is compact, there are finitely many open intervals (Xi - 6, Xi + 6), i = 1,2,..., k, where Xi e K, which cover the set K. By pointwise coDvergence of {In}, there is no such that if n > no, then (3) I/n(x,) -/(xi)1 <, i = 1,2,... ,k. Clearly, for X e K there is an i such that Ix - xii < c. Thus by (l)t (2) and (3), if n > no, then I/n(z) - f(z)1 S I/n(z) -In(x;)1 + I/n(xi) -/(Xi)l+ I/(x,) -/(z)1 < E. 3.1.28. Observe that {In} is equicontinuous on [a,b]. Indeed, by the mean value theorem, I/n(x) -In(y)1 = 1/(')lIx - yl  kllx - yl for all x, y e [a, b) and n E N. Now the desired result follows from the foregoing problem. 3.1.29. (a) Since I/n{x)1  *, tbe sequence is uniformly convergent on III Wehave/(z) = vncosnx. Hence lim I(O) = lirn ,jii = +00. n--J.oo n-4OO Moreover, if x t= 0, then the limit lim f(z) does not exist. n--J.oo Indeed, if lim /(x) = l, then for sufficiently largen we would get n-t-oo 
3.1. Sequences of Functions, Uniform Convergence 333 I cos nxl < i. Thus I cas 2nzi = 1- 2 cos 2 nz > !, a contradiction. So we see that {/} does not converge at any point. (b) Since I/n(x)1  /nt the sequence converges uniformly on [-1,1]. On the other hand, 1 - n 2 x 2 { I for z = 0, Jim 'z-lim - n...oo In( ) - n...oo (1 + n 2 z 2 )2 - 0 for x #: o. The pointwise limit of {/} is discontinuous at zero, and therefore the convergence cannot be uniform. 3.1.30. Assume first that lim I{z) = I. Let E > 0 be given. Then %"'%0 there is 6 > 0 such that if 0 < 1% - %01 < 6, then I/(z) -II < . The uniform convergence of {In} on A implies e I/n(x) - l(x)1 < 2 for n  no, x E A. Hence I/n(x) -II < e whenever 0 < Ix - %01 < 6 and n  no. Since lim In(z) exists, Z"'.%o this implies that lim lim In (X) = I. n-+oo Z"'ZO Assume now that lim Jim In{z) = I. Set lim In(z) = On(XO). n-+oo z-+%O Z"'ZO So we have lim On(XO) = I. Let E > 0 be given. By the uniform n...oo convergence of {In} there is nl such that n > nl implies E (1) I/n(z) - l(z)1 < 3 ' % E A. By the above there is n2 such that if n > n2, then E (2) Ign(xo} -II < 3. Fix 7IG > max{nl,n2}. Since lim Ino(x) = 9no(Zo), we have -+%O E (3) I/no(x) - 9110 (XO) I < 3 if Ix - xol < 6no. By (1), (2) and (3), we see that lim I(x) = I. -+ZO The equality lim lim In(x) = lim 1(%) can be established in n-+oo z...oo %-+00 much the same way. 
334 Solutions. 3: Sequences and Series of Functions 3.1.31. Let E > 0 be given. Choose n such that if n,""  110, then (1) E I/n(xo) - Im(xo)1 < 2 and (2) I/(t) -/:"(t)1 < 2(b  a) ' tela, b). This combined with the nle&1 value theorem applied to the function In - 1m gives Elx - tl E (3) I/n(x) - Im{x) - In(t) + Im(t)1 < 2(6 _ a) < 2 for n,m > no and xtt E [a,bl. Now, by (3) and (1), I/n(z) - /m(x)1 < I/n(z) -/m(x) -/n{xo) + fm(xo)1 + I/n(xo) - I". (xo)1 < E. Thus the Cauchy criterion for unifornl convergence is satisfied (see, e.g.,3.1.7). Let x E [at b) be arbitrarily chosen. Define the functions h and h n by h(t) = let) - f(x) 2 hn(t) = Inet) -In(Z) , t e [a 1 b], t  x. t-;c t-x Then Urn hn{t) = I(x), n = 1,2,... . By (3), t-J.Z E Jhn(t) -ltm(t)1 < 2(b _ a) 1 n,1n 2: "'0, which means that {hn} is uniformly convergent (evidently to h) on [a, b] \ {x}. Applying the result in the foregoing problem to the se- quence {h n } and the set [a, 6) \ {x}, we get lim I(x) = lim h(t) = n-+oo t-+z I'(x). 3.1.32. The equality 1 = (x + (1- X»A = t (  ) Xk(l- x)n-I: k=O gives I(x) = t/(X) (  ) Xk(l- z)n-k. k =O 
3.1. Sequences of Functions, Uniform Convergence 335 Consequently, (1) IB,,(f,x) -f(z)1 <  f ( : ) -f(Z)1 ()Xk(l- z)"-k. By the uniform continuity of I on [0, 1], given e > 0 there is 6 > 0 such that I/(x) - l(x')1 < e whenever Ix - x'i < 6, x, x' e [0, 1]. Clearly, there is Al > 0 such that I/(x)1 < J"l for x E [0,1]. Let x be arbitrarily chosen in [0,1]. Then the set {Oi 1.. 2. . . . . n} can be deconlposed into the two sets A = { k:  - xl < 6 } and B = { k:  - x  6 } · HkeA,then k (;) -f(x) < e, and so (2) E / ( : ) -/(x)1 < E L ()xk(l - x),,-k < €. keA keA HkeB,then .,. (k - nx).. . n 2 6 2  1, and by the inequality given in 2.5.52 we get  j1 () -f(x)1 ()Xk(l- z),,-k 2AJ  2 ( n ) k n-I: kI :S n'26 2 L." (k - nx) k x (1- x)  2n6 2 . keD This combined with (1) and (2) yields Al IBn(/, x) - f(z)1  e + 2n62 ' x E [0,1]. I . 
336 Solutions. 3: Sequences and Series or Functions 3.1.33. If [a, b] = [Ot I), then we take P(z) = Bn(/t z). If (a, b)  [0, I), then we can apply the result in the foregoing problem to the function g(,,) = J(a + lI(b - 0», II e [0, IJ. So, given E > 0, there is a Bernstein's polynomial Bn (g, ,,) such that 19(,1) - Bn(g, ,,)1 < E, II e [0, 1]. Putting z = a + ,,(b - a), we obtain !(Z)-Bn(g, :=: ) <E. 3.2. Series of Functions, Uniform Convergence 3.2.1. (a) If z e (-1, 1], then lim .;z"  O. So the series diverges b: n-+oo the nth term test for divergence. If Izi > 1, then Ixln  2 fa sufficiently large n. Hence 1 < 1 <2- 1 + x" - Ixl" - 1 - Ixl" , and by the comparison test the series converges. (b) Clearly, the series converges if z = o. If z :F 0, then x n 1 - l+z" -1 + -L- z" Therefore by (a) the seriel converges for -1 < x < 1. (e) If z = 0, the series diverges. If x  0, then 2 ft +2:" ()n + t- - 1 + 3nzn - 1 + 1 · 3"2- So the nth term of the series converges to zero if and only if Iii I < 1, that is, if Ixl > . The comparison test shows that the series converges if z e (-00, -2/3) U (2/3, (0). (d) We have Z"-1 1 ( 1 1 ) (1- zn)(I- zn+l) = z(l- x) 1 - ZR - 1- %"+1 · 
3.2. Series of FUnctions, Uniform Convergence 337 Hence N n-l SN(Z) = ]; (1 _ Zn(l - zn+l) 1 ( 1 1 ) = %(1 - z) 1 -:t - 1 - zN+l · Consequently, {  if 1%1 < 1, llm SN(Z) = ,I.!r · N-+oo 1 if 1 % 1 > 1.  So the series converges on . \ {-I, I}. (e) We have %2"-1 1 1 1 - z2*' = 1 - Z2"-1 - 1 - z'J" · Hence { ...L if I x l < 1 llm SN(Z) = l-z t N-too . i!; if Izl > 1. So the series converges on R \ {-I, I}. (f) If z  Ot then the series diverges by the nth term test for diver- gence. For z > 0, by the Cauchy condensation test (see, e.g., It 00 3.2.28) tbe given series converges if and only if I: 2".-1) does. n=2 The root test shows that the latter converges if z > 1 and di- verges if z < 1. If z = 1, then the series diverges. S nmm ing up, we see that the domain of convergence is (1,00). (g) Since z1n n = n 1n %, the series converges if In z < -1 and diverges if In:r  -1. Thus the do mAin of convergence is (0, t). (h) We bave sin 2 ( 271' ''' n2 + z2) = sin 2 2n7l'  ) < 71'2" . Vi +  + 1 - n The comparison test shows that the series converges for all z. 
338 Solutions. 3: Sequences and Series or Functions 3.2.2. (a) Since arctanz + arctan  = i for z > 0, we see that 1r 2 ( 2 1 1 1 2 - arctan(n 1 + x » = arctan "'(I 9) < 4)(1 0»  _ 2 . n- + x- n- + x- n By the lvI-test of \Veierstrass (dominated convergence test) the series is uniformly convergent on R. (b) For x E (2,00), In( 1 + nx) < 1 < 1 nx n - zn-l - 2 n - 1 ' and consequently, the uniform convergence of the series follows from the ?vI-test of \Veierstrass. (c) Since sup{n2:re-n:llzl : x E IR} = n:e'J ' the h-I-tcst of Weierstrass shows that the series converges unifonnly on lIt (d) The series converges pointwise to ( { I if x E [-1, 1] \ {O}, S x) = . o If x = O. Since S fails to be continuous! tbe convergence cannot be Wluorm on [-1, 1]. (e) Note that n 2 I n2 n 2 sup -(x n + x-n) < _(2" + 211) = _2 n + J . 1/2$1%1 < 2 Jni - In! In! 00 Since E  2n+1 converges, for example by the ratio test, the n=1 'I-test of Weierstrass shows that the series converges uniformly on A. (f) The series does not converge uniConnly on A because the Cauchy criterion for uniforn1 convergence fails to hold. Indeed, if 0 < 
3.2. Series of Functions, Uniform Convergence 339 3Z  ; , then ISn+m(x) - Sn(:I:)1 = 2 n +1 sin 3nlx +... + 2 n + m sin 3 n : m x 2 n+l 2 1 2 n + m 2 1 > - +...+ - - 1r 3"+1 Z 1r 3 n + m x > 2n+l 2 . - 1r3 n + 1 x Putting x = in, we obtain ( 1 ) ( 1 ) I 2n+2- 2 3 Sn+m  - 8n  > 31i"  311' . (g) The uniform convergence of the series follows Crom the M-test of Weierstrass. We have ( X 2 ) z2 0'2 In 1+ 2 < 2 < 9 t n In Jl, - 11m .,.. 1 In'" n 00 2 and the Cauchy condensation test shows that E n I n con- n==2 verges. 00 n 3.2.3. Let 5(3:) = E In(x) and 5n(x) = E /t(x). Then n::;l k=l sup{S(x) - Sn(x) : x e [0, I]} = I/(n + 1), which shows that the series converges unifonnly on [0,1]. Since sup{/n(x) : :c e [0, I)} = lln, the M-test of Weierstrass fails. 3.2.4. \Ve have ra S,,(:c) =  «k - 1):1: : l)(kx + 1) n ( 1 1 ) 1 =  (k - l)x + 1 - kx + 1 = 1 - nx + 1 · Hence f{x) = lim Sn(x) = { 0 n-+oo I Clearly, J is not. continuous at. zero. if x = 0, if x > o. 
340 Solutions. S: Sequences and Series of IUnctloDS 3.2.&. (a) The series converges absolutely on R, since f: I ztl sin(nz) $ f: Izl; = e 1zl . n=O n! n=O n. Clearly, the convergence is uniform on each bounded interval. So the continuity of the sum follows from the result in 1.2.34. (b) Since 00 00 L Izl tl ' $ L Izl tl = I! Izl ' n=O n=O the series converges absolutely on (-1,1). Moreover, the convergen( is uniform on each compact subset of (-1,1). Thus tbe sum is COI tinuous on (-It 1). (c) The series converges absolutely for -1/2 < % < 1/2, and, as i (a), one can show that its sum is continuous on (-1/2,1/2). (d) The series converges absolutely for 1/ e - 1 < % < e - I, and Ii sum is continuous on (lIe -I,e -1). 3.2.8. Clearly, the series converges for z = o. Using, for exampl. the resuIt in I, 3.2.16, we see that the series converges if 0 < Ixl < J H 1%1  1, the series diverges. Reasoning similar to that used in tb solution of the preceding problem shows that the sum is continuous on the domain of convergence. 3.2. 'I. Note first that the series  sin(n 2 z)  n 2 n=1 is uniformly coDvergent on R, so its sum 5 is continuous on R. More- o z .i n ,2z ! . .. over, if Sn(Z) = :I , then lim S,,(z) = z5(%). Consequently, n-+oo the sum of the given series is also continuous on R. 
3.2. Series of Functions, Uniform Convergence 341 co 3.2.8. Suppose that E In(x) converges uniformly on A to S. This n=1 means tbat dn = sup 15n(x) - 5(x)1 --+ 0 , zEA n-too n where Sn(x) = E Ik(x}. Since I is bounded, we also have k:::l d'n = sup I/(z)Sn(x) - l(x)5(z)1 -+ o. zEA n-+oo To see that boundedness of f is essential, take A = (0,1], I(z) =  , 00 and In(z) = 2"1 . Then the series E In(z) converges uniformly on n=1 00 AJ but E  I R(Z) fails to converge uniformly on A, because n=1 00 1 2 sup  - II: (x) = sup _ 2 = +00.  z ) z n :tEeO,I) k=n+l zE(O.1 It is easy to see that if } is bounded on At then tbe converse holds. 00 3.2.9. For z e A the series E (_l)R In (x) converges by the Leibniz n=l theorem. roreoverJ by the result in I, 3.4.14, 00 sup Irn(x)1 = sup E (-1)I:+l/k(Z) :S sup In+l(x). zEA zEA k:::n+l :tEA This combined with condition (3) proves the uniform convergence of the given series on A. 3.2.10. The three series (a), (b) and (c) satisfy the assumptions of the assertion in the foregoing problem. n+m sup E cJ:llt(x) zEA k::::n So it suffices to apply the Cauchy criterion for uniform convergence. 3.2.11. By the Cauchy inequality, ( n+m ) 1/2 ( n+m ) 1/2   ci :  /Z(z) · 
342 Solutions. 3: Sequences and Series of Functions 3.2.12. (a) A = [i,) and B = (t, 4).. The series COli verges uniformly on [k, i] , because 00 1 16x - 21"+1 1 sup L -2£:(3x _1)" < sup =. ze[l.!] k=n+l k - ze[l.1] n + 1 n + 1 (b) A = (-co,-i] and B = (-00, -l) . The series converges urn- fonnlyon [-2, -1], because 00 1 ( X + 1 ) " 00 1 sup L k  L k2k " ze(-2.-1J k=n+l Z k=n+l 3.2.13. S umma tion by parts gives n n-l Sn{x) = L Ik (x)gj; (x) = L Gk (x)(IJ: (x) - Ik+l (x» + Gn(x)/n(x). k=1 k=1 This together with assumption (3) implies ISn+m(x) - Sn(x)1 n+m-I L Gk(X)(/k(X) - 11:+1 (x»+Gn+m(x)/n+na(x) - Gn{x)/n(X) k=n S M C  l lMx ) -lk+1(x)1 + I/n+m(x)1 + I/n(x)l) · Now let E > 0 be given. Then it follows from (1) and (2) that, for nl. E N and for sufficiently large n, sup ISn+m(x) - Sn (x) I zEA =s f sup ( n"I: 1 !/k(X) -Ik+l (X) I + I/n+m(x)1 + I/n(X)I ) < €. zEA k=n Thus the Cauchy criterion for unifonn convergence can be applied to 00 L /n(x)gn{x) n=1 
3.2. Series of Functions, Uniform Convergence 343 To prove the Dirichlet test for uniform convergence, Dote that the monotonicityand the uniform convergence to zero of {/n(x)} imply 00 (1) and (2). f\lloreover, since the sequence.of partial sums of E 9n(Z) n=1 is uniformly bounded on At \ve see that condition (3) is also satisfied. 00 Consequently, the series L: /n(X)gn(x) converges uniformly on A. n=l 3.2.14. The Dirichlet test for uniform convergence win be applied. (a) Take 1 In(x) = - and 9n(x) = (_l)R+lxn. n (b) Here we take 1 In(x) = - and Un{x) = sin(nx} n and note that n LSin(kx) < 1 < 1 . k=1 - sin  - sin  (c) Since n 2 Lsin (k 2 x) sin (kx) 1:=1 n - L(cos(k(k -1)x) - cos(k(k + l)z» 1:=1 = 11 - cos (n(n + l)x)J  2 and { nz2 } is decreasing and uniformly convergent to zero, the Dirichlet test shows that the series converges uniformly on III (d) We bave f Sin(nx):ctarJ(nx) n=1 = f ( Sin(nX) (arctan(nx) - ) +  SiD(nX} ) . n=1 n n Since f .; 8i: C nz) converges unifonnly on [6,21r -  (see (b», n=1 . the sequence of its partial sums is uniformly bounded. Moreover, (*) 
344 Solutions. 3: Sequences and Series of Functions the sequence {arctan( nz) - 1T /2} is increasing and satisfies the Cauchy criterion for uniform convergence on [6,211" - 6), because mx arctan«m + n)x) - arctan(nz) = arctan 1 ( ) z2 + m+n n mz < arctan - (m+n)nx 2 1  arctan 116 . So {arctan(nz) -11"/2} converges uniformly to zero. It then fol- lows, by (*), that the given series converges uniformly on A. (e) We have (_l)n+l.!. = (_l)n+l 1 2-. L., n Z L., n%-! nl n= 1 n= 1 00 Since E (-l)R+l;!r converges, the sequence of its partial sums n=1 n is bounded. Moreover, the sequence { n. f } decreases and con- verges unifonnly to zero on [a, 00). (f) Note that for x E [0. co), n "'(_l)i+l = L., ekz k=1 1 - ( J r: eft. < 1. e+l - Moreover, the sequence { "n% } decreases and converges uni- formly to zero on [0,00). 3.2.15. S umm ation by parts yields n n-l Sn(x) = E Ik(x)g" (x) = E Gk(z}(/i(z) - 11:+1 (x» + Gn(z)fn(z), i=l k=l R where Gn(x) = E 9J;(x). Since 11 is bounded on A, condition (2) k=1 implies that there is }II> 0 such that I/n(z)1  M for all x E A and 
3.2. Series of Functions, Uniform Convergence 345 all n E N. Since {G n } converges uniformly on A, say, to G, we obtain Sn+m(x) - Sn{x) n+m-l = L GIc(X)(/k(X) -/k+l(X» + Gn+m(x)/n+m(x) - Gn(x)fn(x) Ie=n n+m-l = L (/Ic(X) - Itc+l(x»(GIc(x) - G(x» Ie=n + (Gn+m(x) - G(x»/n+m(x) - (Gn(z) - G(:c»/n(x). This combined with (2) and the uniform boundedness of {/n(z)} shows that {Sn} satisfies the Cauchy criterion for uniform CODver- gence. To prove the Abel test for uniform convergence, it suffices to note that the monotonicity and the uniform boundedness of {In} imply the pointwise convergence to a bounded function, and so conditions (I) and (2) are satisfied. 3.2.16. (a) The sequence {arctan(nz)} satisfies conditions (1') and (2') in 00 ( 1 ) ,,+1 the Abel test for uniform convergence. Moreover t E :+%2 n- I converges uniformly on IR (see 3.2.10(a». (b) The Abel test for uniform convergence can be applied, because the series 00 (_I)n+l  JR + cou: is uniformly convergent on A (see 3.2.IO(c» and the sequence {CO& } is bounded and monotonic for n > . (c) The series f: (-l)Iv'iiI n n= 1 converges (see, e.g., I, 3.4.8) and the sequence { A% } is mono- tonic and bounded on [0,(0). Thus the Abel test for uniform convergence can be applied. 
346 Solutions. S: Sequences and Series of Functions 3.2.17. The result follows immediately from 3.1.30. 3.2.18. To prove (a) and (b), one can use the results in 3.2.14, 3.2.17, and in I, 3.1.32(a). (c) Since f(zn _ xn+1) = { X for x E [0,1), n=1 0 for z = 1, we get co lim (zn_xn+l)=I. z....l- L..J n=l co (d) Note first that E 2nln_ is uniformly convergent on [0,00), by the n=1 M-test of Weierstrass. Thus, by the foregoing problem, 00 00 lim  1 =.!..=l Z10+  2 R n%  2 n . n=1 n=1 (e) Since x 2 1 sup = - zeR 1 + n 2 z 2 n 2 ' 00 the series E l+Z:l converges uniformly on III Now using the n=1 result in 3.1.30 we obtain . 00 z2 00 1 11'2 zL l+n 2 z 2 = L n 2 = 6'. n=l ,1 00 3.2.19. Observe first that E ClnXR converges uniformly on [0,1]. n=1 This follows immediately from the Abel test for unifonn convergence stated in 3.2.15, with In(x) = x n and 9n(X) = an. Now by 3.2.17 we 00 see that the limit is E an. n=1 3.2.20. Since the In are continuous on [O,IJ, we see that n+m n+m sup L Ik(:C) = sup L Ik(X). ze(O,l) lc=n ze(O,I] k=n 
3.2. Series of Functions, Uniform Convergence 347 00 Thus by the Cauchy criterion, tbe uniform convergence of E In(z) n=1 00 on [0,1) implies tbe uniform convergence of E In(x) on [0,1]. n=1 3.2.21. A = (0,00). The convergence is not uniform. Indeed t if tbe series were uniformly convergent on A, then by the result in the foregoing problem it would converge for z = 0, a contradiction. 3.2.22. Note that 00 rn(:z:) = E I.(z) = I(z) - Sn(z), k=n+l 00 where Sn(Z) denotes tbe nth partial sum of E In(%). By assumption. n=l the sequence {r,,(%)} is monotonic and convergent to zero at each fixed z in [a, b]. Hence the Dini theorem (see, e.g., 3.1.16) implies the uniform convergence of {rn(z)}, and consequently, the uniform convergence of the series on (a, b). 3.2.23. No. Consider 00 L<-l)n(l- z)zn, A = [0,1]. n=0 By the result stated in 3.2.9 thiS series converges uniformly on A. On co the other hand, the sum of the series E (1 - z)%'. is n=O Sex) = { I for z e (0, I), o for z = 1. Since S is Dot continuous, the convergence cannot be uniform. 3.2.24. Since the In are monotonic on (a,b], 00 00 00 Irn(z)1 = E Ik(z) S E 1/,(z)1  E max{I/.(a)I,I/.(b)l}. i=n+l '=n+l '=n+l This shows that if the series converges absolutely at tbe endpoints of the interval [a, b), then it converges absolutely and uniformly on the whole [0, b]. 
348 Solutions. 3: Sequences and Series of Functions 3.2.25. Let A be a bounded set disjoint with the elements of {an}. 00 Since E Jtr converges, we have lim Ian I = +00. Consequently, one n=1 n n"'oo can choose no such that if n  no, then Ix - an 1  1 for z e A. Hence, for sufficiently large nJ 1 =. 1 <. 1 Jz - ani lanl 10:. -11 - lanl 1 - fa!T' 00 where M = sup Ixl. Finally, observe tbat if E y::L, converges, then 10nl zEA 1 E ( lol..r . I_ ) also converges. 1 en 3.2.26. Write 00 co an an 1  n =  ;;0 · 11%-%0 n=1 n=1 and apply the Abel test for uniform convergence (see, e.g., 3.2.15). 3.2.27. It has been shown in the solution of 3.2.1 that the given series converges to a continuous function on fit We now show that the convergence is not uniform on Jlt Observe first that if no is odd, then the sum  sin(n 2 x) L.-, n 2 n=no is different from 7"ero at each Xi = i + 2k1r, kEN. Moreover, (1)  Sin(n2xk) =  sin(n) . L.J n 2 L.-, ( no + 2l ) 2 n- no 1=0 00 '(2) If the series E Z S1n n Z were convergent to I uniformly on Ii, then, n=l given E > 0, there would exist an odd no such that no-I sin(n 2 x) f(x) - L Z n 2 < £ for all % e III n=l 
3.2. Series of Functions, Uniform Convergence 349 In particular, we would get J(Xk) no-l sin( n2 i') :&k - L n 2 n=l and consequently, e <  + 2k1r ' I( ) no-I. ( ") Jr ) Jim Xk = L 8m n- '2 . k-+oo XIc n=1 n 2 On the other hand, by (1), I(x,,) _  sin(n2xk} _ 1 sin(n2i) . ( 2 )  1 - L.J ., - L..., 2 + sm no L..., ( 1) 2 ' Xic n=1 n" n=1 n 2 1=0 no + 2 contrary to 00 sin ( ; )  (no  21)2  o. 3.2.28. The assertion follows immediately from the result in 3.1.31. 00 3.2.29. By the M-test of Weierstrass the series E n2z2 converges n:=1 uniformly on III Moreover, since ( 1 ) ' _ I -2x < 1- n 2 +x2 - (n 2 +X2)2 - n 3t E ( n2z;2 ) ' also converges uniformly on Il Hence by the result in n=1 the foregoing problem I is differentiable on each compact interval and consequently on III 00 3.2.30. Note first that E c:;) converges uniformly on . The n=1 series  ( cos(nx» ) ' =  -nsin(nx) L..., 1 + n 2 L....., 1 + n 2 n=l n=1 converges uniformly on the indicated interval by the Dirichlet test for uniform convergence stated in 3.2.13. Therefore the differentiability of I follows from 3.2.28. 
350 Solutions. 3: Sequences and Series of Functions 00 3.2.31. The series E (_1)n+l1n (1 +  ) converges, e.g., for x = o. n=l The senes f (C-1)n+11n (1 + =))' = fC-1)n+l 1 n=1 n n=l n+x converges uniformly on [0,00) by the result stated in 3.2.9. So the result in 3.2.28 shows that f is differentiable on [0, 00) and 00 00 /'(0) = L(-l)n+l.!. = In2, /'(1) = L(-l}R+l 1 = 1-ln2. n=l n n=1 n + 1 Finally, applying 3.1.30 we find that lim I' (z) = o. zoo 3.2.32. By the Abel test for uniform convergence (see, e.g., 3..2.15)J 00 E (_l)R+l-jn arctan * converges uniformly on III The derived se- n=1 00  ) ,,+1 ries E z':l is also uniformly convergent on IR (see 3.2.10(a». So n=l one can apply 3.2.28. 00 . ( 2) . 3.2.33. Clearly, the series E 81+n converges uniformly on IR. The n=l 00 ':I derived series E 2n ::z ) converges uniformly on each boundE'.d in- n=l terval. Therefore by 3.2.28 I' is continuous on each bounded interval, and thus I' is continuous on Ul 3.2.34. The 1-test of Weierstrass shows tbat the series and the de- rived series 00 1 '" nv'ii(tanx)n-l ? L.J l cos- x n= are uniformly convergent on each compact subinterval of ( -11'/4, 1r /4). Therefore by 3.2.28 I' is continuous on (-1r/4,1t/4). . 3.2.35. The M-test of Weierstrass shows that the given series con- verges uniformly on [0, 00). By the t-test again, we see that the derived series 00 -n% '" -ne L-, 1 + n 2 n=O 
3.2. Series of Functions, Uniform Convergence 351 is uniformly convergent on each interval [a, 00), a > o. Thus / is in C 1 (0, 00). Repeating the above process k times, we conclude that 00 !.=!l . " -n. E - l:n: converges uniformly on each [a, (0), a > o. This shows n =O that f E 0 00 (0,00). If I' (0) existed, then since I(x) - 1(0) 00 e- nz -1 N e- nz -l x =  xU + n 2 ) <  x(1 + n 2 ) for x > 0 and lV > 1, we would get N lim I(x) - 1(0) <" -n . z-+o+ :z; - L.i 1 + n 2 . &= 0 Upon passage to the limit as N -)0 00, we would obtain f'(O)  -00, a contradiction. 3.2.36. Clearly, the series cODverges uniformly on each bounded in- terval. Thus I is continuous on III lvIoreover, for x :F 0,  ( Ixl ) ' =  n 2 sgn(x} - xlxl .  3;2 + n 2 L...J ( x 2 + n 2 ) 2 n=1 n=1 Thus the derived series is uniformly convergent on each bounded in- terval that does not contain zero. Consequently, I' is continuous at each X:F o. Now we show that f'(O) does not exist. Since I(h) - 1(0) = ( Ihl  1 ) h II L..J 11. 2 + n 2 n=1 and (see, e.g., 3.2.17) . 00 1 00 1 11'2 lim ---- h-+OL-, 11. 2 +n2 -  n 2 - 6' n=1 n==l the limit lim f(h)-fCO) does not exist. h-+O h 
352 Solutions. 3: Sequences and Series of Functions 00 3.2.37. Observe first that the series E -b coDverges uniformly on n=1 each interval [zo,oo), Xo > 1 (see, e.g., 3.2.26). Thus the Riemann (-function is continuous on (1,00). For a kEN, the series 00 In #: "(-I)i n LJ n Z n=1 (1) is also uniformly convergent on each [zo, 00), Zo > 1, because ln t n n -° 2 - 1 1 < = n - nZo n -° 2 + 1 for sufficiently large n. Consequently, each kth derivative of the Rie- mann (-function is continuous on (1,00). 3.2.38. By (1) there is an Zo e (0,1] such that f(xo) ¥: O. Now, by (2) and by Taylor's formula with the Lagrange form for the r emain der, we get 1(:£0) = f(n)(n)xg . n. where 9 0 e (0,1). Hence (*) I(n) (8n) = n!f(xo) . zn o Now (3) implies that sup lan/(n) (x) I  O. This means that, given ze[O,i] n-+oo e > 0, there is no such that if n > no, then lanj(n)(8 n )1 < E. It then follows by (.) that n I I I EZo n.an < If(xo)l. 00 3.2.39. Clearly, for Z E Z we have fn(z) = O. So E In(z) = O. Now n=1 let x = i, where r and B and co-prime integers and 8 > 1. If pis a prime number different from 8, then j,,(z)  'is. Indeed, for any aE Z, I !: -  I = Irp - asl > 1:... 8 P 8P -sp 
3.2. Series of Functions, Uniform Convergence 353 Consequently, 00 1 L In(x)  L 8' n=1 pEP P where P denotes the set of all prime numbers different from 8. So 00 (see, e.g., I, 3.2.72) the series E In(z) is divergent for all z E Q\ Z. n=1 For an irrational x, set A={neN: i<nx-1nxJ<H, A(m) = {n E A : n < m}, where dB denotes the number of elements of the set B. It follows from the fact that for an irrational x the numbers nx - [nx] are uni- formly distributed modulo 1 (see, e.g., Theorem 25.1 in P. Billingsley, Probability and Measure, Wiley, New York, 1979, pp. 282-283) that lim A) = . Consequently, E 4 = +00. Note that for n E A, m A [nx] 1 In (x) = x - n > 4;- 00 It then follows that E In (x) diverges for x E R \ Q. n=1 3.2.40. Since 9 is bounded, the series converges uniformly on III to f. Hence I is continuous on III Our task is now to show that f is nowhere differentiable. Let a real number z and a positive integer m be arbitrarily chosen. If there is an integer in (4 m x, 4 m x +  ) , then there is no integer in (4 m x - ,4mx) . So we can always find 6m = :f:4-m 5uch that there is no integer in the open interval with the endpoints 4 m x and 4 m (x + 6m). By the definition of g, I g(4"(X + 6m» - g(4 n x) I = { 0 if n > m, 6m 4" if 0  n S m. Note here that, for a fixed m, 9(4"(x + 6m» - g(4"z) b m 
354 Solutions. 3: Sequences and Senes oC Functions have the same sign for n = 0, 1, . . . ,m. Hence I f(x + 6rn) - f(x) I = f: (  ) n g(4 n (z + 6m» - g(4 n x) 6m n=O 4 6m = f: ( ! ) R g(4 R (x + 6m» - g(4 n x) n=O 4 6m = f (  ) n4n neO 3 m + 1 - 1 2 · Since Urn 6m = 0, it Collows from the above t11at 1im  m-+oo 11-+0 does not exist. This shows that f is nowhere differentiable. ThE graphs of three first partial sums So(z), 51 (x) and 52(X) of the serle!! defining / are sketched below. -I 1 y-8 0 (%) -0.5 0.5 y-8.(.r) I -I -0.5 0.5 I y- S 2(%) 
3.3. Power Series 355 3.3. Power Series 3.3.1. Define R to be tbe supremum of the .set of the r e [0,00) for which {Ianlr"} is a bounded sequence. If R is positive, then for o  p < R there is a positive constant, say C p , such that lanlpn  C p . Hence lim tfiOJ  _ p l. Since the last inequality holds Cor each noo p e (O,R), we get (i) iiiii \II an I  - R 1 · n-+oo Note that inequality (i) holds also for R = O. To show that the in- verse inequality also holds, suppose that R < 00; then for p > R the sequence {Ianlpn} is unbounded. Consequently, it contains a subse- quence such that Inn.lpn.  1. So iiiii Viani  Urn " t/lan.1  !. n-toao Ic-+oo p Since p > R can be arbitrarily chosen, we get (6) iiiii \II an I  - R 1 · n-too Note that (ii) obviously holds for R = 00. Combining (i) and (ii), we see that i = Urn tllanl. Now the root test shows tbat the series n-toea 00 E on(% - %0)" converges absolutely for Ix - %01 < R and diverges n=O ror 1% - :tol > R. 3.3.2. (a) The radius of convergent."e of the series is 1, and it therefore con- verges for Ixl < 1 and diverges ror Ixl > 1. For :E = I, -1 the series diverges. Thus the open interval (-1, 1) is the interval of convergence. (b) The radius of cODvergence is +00, aud therefore the series con- verges for all x e III (c) The domain of convergence is the closed interval (-1/2,1/2]. (d) We have ! = iiiii V'(2 + (_1)")" = 3. R n-+oo 
356 Solutions. 3: Sequences and Series of Functions Thus the series converges on (-1/3, 1/3). Clearly, the series di- verges at the endpoints of the interval of convergence. (e) Since 1.. = liiii 2 + (-1)" =!, R n-+oo 5 + (_I)n+l 4 the series converges on (-4/3,4/3). At the endpoints the series diverges. (r) Since .!. = Um Viani = Iim "= I, R n-+oo n-+oo one can easily find that the interval of convergence is (-1,1). (g) Since ! = liiii Viani = fun '\Y2 n2 = I, R n-+oo n-+oo one can easily find that the interval of convergence is (-1., 1). (h) We have 1 _ ( 1 ) (-I)a" - R = liiii \llanl = lim 1 + - = e. n-+oo n-+oo n Therefore the series converges on (-l/e.,l/e). At the endpoint: the series diverges by tbe nth term test for divergence. Indeed, i z = lie, then ( I + -!- ) 4n 2 Um 02n = lim 2n = e- I / 2 n-+oo n-+oo e 2n and if % = -lIe, then Jim 102nl = e- 1 / 2 . n-+oo 3.3.3. (a) The radius of convergence is ..j2 and the interval of convergence is [1 - ..[2, 1 + ..j2). co (b) The radius of convergence of E n:l "" is 1. Thus tbe series n=1 co E n:'1 ( 2:t )n converges on (-1.,-1/3). Clearly it diverges at n=1 Z = -1 and z = -1/3. 
3.3. Power Series 357 00 n (c) The radius of convergence of E 4,. yn is 3/4. Consequently, the n=l 00 . series E '; zn(l-x)n converges on (-1/2,3/2). One can easily n=1 see that it diverges at the endpoints. (d) Since the radius of convergence is 4, the series converges on ( -3,5). For x = 5 the series diverges because the sequence of its terms { t;) 4n} monotonically increases. For x = -3 we get 00  the series E (_1)n4n, which diverges by the nth term test n=1 for divergence. 00 (e) The radius of convergence of E ..myn is 1. Therefore the series n=1 00 E In(tanz)n converges on the set n=l u (-  +nlf,  +n1T). nez If z = -7 + n1T or z = : + n1l', the series diverges. (f) The domain of convergence is (-00, - tan 1) U (tan 1, co). 3.3.4. (a) Suppose that, for example, RI < R2. Then for Ixl < RI the series 00 E (an + bn)x n converges as the sum of two convergent series. For n=O Rl < Ixl < R2' the series diverges as the sum of a divergent and a convergent series. Thus R = RI = min{Rl,R2}. If RI = R2, then clearly R  Rle To show that the inequality can be strict, set an = -1, b n = 1 for n = 0,1,2,.... Then RI = R2 = 1 and R = 00. (b) Since (see, e.g., I, 2.4.16) ! = lim \llanbnl S IiiD \llanl . lim \llbnl = J:.. · -.!..., R n-too n-too n-too Rt R2 
358 Solutions. 3: Sequences and Series of Functions we obtain H > RIR2. The following example shows that tbe inequality can be strict. Set 62n = 0, a2n+1 = 1, n = 1, b:!n+1 = 0, n = 0, 1,2, . .. . Then R 1 = R2 = 1 and R = 00. 3.3.5. (a) It follows from an an = - . b n b n and from (b) in the foregoing problem that R 1  RR2. To see that the inequality may be strict, consider, for example, the series 00 00 E 6n Xn and E bnx n , where '1=0 n= O { I for even n, an = 21) for odd n and { 2n for even n, b n = 1 for odd n. Then R 1 = R 2 = R = 1/2. (b) It suffices to observe that if Ixl < min {HI, R 2 }, then by the hJlerteDS theorem (see, e.g., I, 3.6.1) the Cauchy product of the 00 00 series E a,.xn and E bnx n converges. The following example n= O n=O shows that the inequality R  min {Rlt R 2 } can be strict. The 00 00 Cauchy product of E anx n and E bnx n , where n= O n=0 ( 3 ) n ( 3 ) n-l ( 1 ) ao = 1, an = - 2 ' ho = 1, b n = 2 2 n + 2n+l 00 is E ()n x n (see, e.g., I, 3.6.11). Here R 1 = 2/3, R 2 = 1/3 and n - O R = 4/3. The next example shows that R can be infinite even while both Rl and R2 are finite. H { 2 for n = 0, an = 2 n for n = 1,2,.. . 
3.3. Power Series 359 and { -I for n = O. b n = 1 for n = 1. 2 p . . , then R 1 = 1/2, R2 = 1 and R = +00. 3.3.6. We will use 3.3.1(2). (a) For 0 < E < L there is no such that if n 2= no. then Ln:e < tllanl < ifLn: e . Hence lim \llanl = 1 and R = 1. n---+oo (b) One can show, as in (a), that R = Q. (c) R = 00. 3.3.7. (a) Since lim \! 12 n a n l = i, the radius of convergence is equal to n-+oo !R. (b) If E > 0 is so small that -h - E > 0, then for infinitely many n, n tll",,1 > n (A - e) . Consequently, Jim n Viani = +00 and R = o. n-too (c) Since lim  = e, we see that the radius of convergence is RI e noo n. (see, e.g., I, 2.4.20). (d) Since there is a sequence of positive integers {nil such that R 1 = lim " V lan.l, k-+oo we conclude that the radius of convergence is R2. 3.3.8. It follows immediately from the result in 3.1.25 that the only such power series are polynomials. 
360 Solutions. S: Sequences and Series of I\mctioDS 3.3.9. The radius of convergence of the series is +00. Termwise dif- ferentiation gives ( ) ' 00 2n+l 00 2n f'(x) =  (:. + 1)11 = 1 +  (2n x _l)1I = 1 +xf(x). 3.3.10. As in the solution of the foregoing problem. for % e R we get OQ n 1"(%) + /'(z) + /(:1:) = L =-- = e Z . n=0 n! 3.3.11. For z e (-1, I), set g(x) = /(zzo) -/(zo) . z-1 Then lim g(z) = zo/'(%o). Moreover (see, e.g., I, 3.6.4), z-+l- 1 1  g(x) = 1 /(zo) - 1 /(zoz) = (/(zo) - Sn(ZO»Z". -z -x L" n=0 So if 0 < x < 1 and m = 0,1,2. . . ., we get 00 g(%) = L(/(ZO) - 8 n (zo»:I:" > (/(zo) - 8m (Zo»zm . n=O Consequently. zo/'(zo) = 1im g(z)  I(zo) - Sm(ZO) > O. ....I- 00 00 00 3.3.12. We first show that E B,ax", E Snz" and E (n + I)T n x" n=O n=O n=O converge for Izi < 1. Since {Tn} is bounded, there is C > 0 such that ITnl  C for aU n. Then, for Izi < I, 00 00 C )n + 1)I T n xn l S L(n + I)Clxl n = (1 -lxl)2 ' n=O n =O 00 Convergence of E Snzn for Izi < 1 fonows from the equality n =O N N LSn Z " = So + L«n + I)T n -nTn_l)Zn. n=O n=1 
3.3. Power Series 361 N N Similarly, since L anx n = Cli) + L (Sn - Sn_l)X n , the convergence n=O n=1 00 00 of E Sn xn implies the convergence of E llnX n for Ixl < 1. n=O n=O The stated equalities follow from the Mertens theorem (see, e.g., I, 3.6.1). 3.3.13. We have 1 Ix 1/,(x)1 $ :f: Ixl" :f: 2kX2. = :f: ( E 2" ) Ixln I I n=O k=O n=1 2.n 00 ( [IOg2n J ) =   2 k Ixl" $ 2  nlxl" = 2 (1 1)2 · Thus the desired inequality is satisfied with M = 2. 00 3.3.14. The uniform convergence of E anx n on [0,1] follows from n=O the Abel test for uniform convergence (see the solution of 3.2.19). To prove (2) it suffices to apply 3.1.30 (see also the solution of 3.2.19). 3.3.15. We first show that (1) lim f(x)  fun Sn. z-t 1- n-+oo We have (see 3.3.12) (2) 00 f(x) = (1 - x) E Sn xR for Ixl < 1. n=O H Urn Sn = +00, then (1) is obvious. If lim Sn = S E Ill, then by n-too n-+oo (2) we get (3) 00 S -f(x) = (1- x) E(S - Sn)x n . n=O 
362 Solutions. 3: Sequences and Series of Functions Let e > ° be given. Then there is no such that Sn < S + E whenever n> no. So, by (3), for x E (0,1), no 00 s- f(x)  (I-x) E(s -Sn)x n -g(I-x) E x n n=O n= no+1 no = (1 - x) E(S - Sn)x n - ex no + 1 n=0 no  (1 - z) E(S - Sn)x n - e. n=0 Consequently, no f{x)  S +E - (1- x) E(S - Sn)x". n=0 Since there is 6 > 0 such that if x E (1 - 6, 1), then "0 (1- x) E(S - Sn)x n < E, n=O we see that f(x) S S + 2e. So (1) is proved in the case of finite lirn Sn- Now if Jim Sn = -00, then clearly, lim Sn = -00. Thus " noo "OO for an M E Ii one can choose nl such that if n > nl, then Sn < M. Consequently, for x E (0, 1) we get nl 00 M -f(x) = (I - x) E(M - Sn)x" + (1 - x) E (kI - Sn)x n n=O n= nl +1 "I  (I- x) E(M - Sn)x n . n= O nl So f(x) < M - (I-x) L (M -Sn)x". Since there is 6 > 0 such tbat n=O if x e (1- 6,1), then, no (1- x) E(M - Sn)x n < e, n=O 
3.3. Power Series 363 we obtain I(z) Al + E. Hence !!!!! I(z) iiiU J(x) AI. %-+J - z-+I- Since M can be arbitrarily chosen, this shows that 1im J(x} = -00. z-tl - This" ends the proof or (1). Tile inequality lim 8n Urn I(x) n-+oo z-+I- can be established analogously. 3.3.16. Set n E klaA: I A _k=O n- · n Then tim An = 0 (see e,g., I, 2.3.2). By assumption, if Zn = 1 - .I n ' n-too then Urn I(zn) = L. Thus, given E > 0, there is no 8uch that if n-too n no, then E E £ I/(zn) - LI < 3' An < 3 and nlanl < 3. n Putting Sn = E at, we get "=0 n 00 Sn - L = /(z) - L + Ea.(l-zk) - E al:z i , Ixl < 1. A:=O n+1 Now note that if:z: e (0,1), then (1- :e') = (l- :r.)(1 + z +... + %'-1) k(l - z). Consequently, n e ISft - LI S I/(x) - LI + (1 - x) E klakl + 3n(1 _ x) · , =0 Finally, taking z = z" giVC8 £ E £ ISn - LI 3 + 3 + 3 = £.
364 Solutions. 3: Sequences and Series of Functions 00 3.3.17. Consider, for example, the series E (-I)"x n . n=O 3.3.18. It follows from the Abel theorem (see 3.3.14) that if the series 00 E an converges, then the limit lim f(x) exists. To show that the n=1 :1:-+1- other implication holds, assume that lim f(x) = 0 e III Then, by %-+1- assumption, for 0 < x < 1 we get Ie E llnx n =:; I(x) =:; g, keN. n=1 Ie Ie Hence E On = Um E an xn < 0, which implies the convergence of n:::::1 z-+I- n _ l 00 E an. n= 1 3.3.19. Define bo = 0, 6n = al + 2C12 + . . . + nan, n e N. Then I( ) E oo 6n - 6n-l n x =00+ x n n=1 00 ( xn xn+l ) =00+ "'6n --  n n+l 00 ( zn _ xn+l xn ) =ao+  b" n+1 + n(n+1) 00 00 6 ( )  6n n  n n = ao + 1- x L...J 1 x + L...J ( 1) x · n=1 n + n=l n n + Since lim 6 + 11 1 = 0, one can show that n-+oo n Iim (1-:1:) f: bft I X" = O. %-+1- I n + n= 
3.3. Power Series 365 Now applying the Tauber theorem, we get 00 b n  n(n + 1) = L - 00. Moreover I N N ) llm b n - lim bn!. _ 1 N-+oo n(n+l) - N-+OO (n n+l = llm ( f. bn - b n - 1 _ bN ) N-+oo n=1 n N + 1 N = Urn Ean. N-too n=1 00 Thus E an = L. n=O 00 3.3.20. It fonows from the convergence of the series E na and the n=1 result in I, 3.5.9(b) that 0 2 + 22 + . . . + n 2 a 2 Urn I --" n = o. ..-+00 n By the Cauchy inequality, ( tka. ) 2 n ( tk2a: ) . k=1 '= 1 Consequently, Urn n-+oo n E kale i=1 n 2 n E k 2 al < Jim i=1 = o. - n-+oo n The desired result follows from the foregoing problem. 
366 Solutions. 3: Sequences and Series of Functions 3.3.21. Let e > 0 be given. By assumption there is no E N such that if n > no, then Ian - Abnl < ebn. Thus, for x E (0,1), 00 I/(x} - Ag(x)1 = L(a n - Abn}x n n=O no < L(a n - Abn}x n + n=0 00 L (an - Abn)x n n=no+l no 00  L Ian - Abnl +E L bnx B n=O n= no+l no < L Ian - Abnl + eg{x). n=O Since lirn g(x) = +00, for x sufficiently close to 1 we get :r-+ 1- no L Ian - Abnl < eg(x). n=O Hence If(x) - Ag(x)1 < 2eo(x) for x sufficiently close to 1. 3.3.22. Note that by the Mertens theorem (see, e.g., I, 3.6.1), 00 00 f(x) = (1 - x) :E Sn xn and O(x) = (1 - x) :E Tn xn n=O O for Ixl < 1. Thus the result stated in the foregoing problem gives lim f(x) = 1im {2 = A. %-+1- g(x) z-H- 9(Z) l-z 3.3.23. Consider 00 f(x) = (1 + X)(1 _ x) = (1- x)  (n + l)x 2n 00 = L(n + 1)(x 2n - z2n+l) n.= 0 
3.3. Power Series 367 and 1 00 y(z) = =  ;J;. I-x L.-, n=O Then lim g /; » ) = f. On the other hand, since S2n+l = Ot S2n = 'n+ 1 z-+l- and Tn = n, the limit lim :iD.. 'l : does not exist. n-too n 3.3.24. For x E (0,1), (1) n f(x) > Lakxk > x n 5 nt k=O because all the coefficients an are nonnegative. Putting x = e--k, we get e- 1 S n < J (e-  ). Thus, by assumption, given £ > 0, there is "0 such that if n :> no, then I A+E e- Sn  I < 2(A + E)n. 1- e-n The last inequality follows from the fact that lim In ( 1-  ) n = n-+oo ... - > -1. So \ve have (2) 5n < A 2 n with some .4 2 > 2(A + c)e. Now by (2) we get 00 f(x) = (1 - x) L Sn xn n=0 n-l 00 < (1- x)Sn L xk + A 2 (1- x) L k:c k k=O k=n A2Xn+l < 58 + A 2 nx n + . I-x H in (1) we put:c = e- a/n , a> 0, as above we obtain ( Q ) A-E n f e- fi > _2. > (A - E)-. l-e n Q 
368 Solutions. 3: Sequences and Series of Functions The last inequality follows from e- f > 1 - . Consequently, (A ) n S A -0 2A2ne- o - £ - < n + 2ne + t Q Q or in other words, A - e - 2A 2 e- o - A 2 oe- 0 8n >n . Q If we take Q sufficiently large, we get 8n > Al n with some positive constant AI. 3.3.25. We start with some considerations which we will need in the proof of the theorem. Assume that tp is continuous on [0, 1) except for one point e e (0,1) at which one-sided limits tp(c+) and tp(c-) exiat and tp(c) = tp(c+) or tp(c) = ep(c-). Our aim is now to show tha given £ > 0, there are polynomials PI and P2 such that i 1 ( (x) - rp(x»d:z: < E and i l (rp(x) - PI (x»d:z: < E. 'To this end suppose, for example, that cp(e-) < cp(c+) and tp(e) : ep(c+). Clearly, one can choose 61 > 0 80 small that tbe inequa1i1 1'P(c - 6 1 ) - cp(z)r < £/4 holds for x e (c - 6 1 , c). Set M = sup{lcp(x) - tp(c)1 : x e (c - 6., c)} and take 6 < min {6 1 ,£/(4M),c,l- c}. Now define ( ) { tp(x) if x e [O,c - cS] U [c,l], g% = max{l(x),tp(x)} if %E(c-6,c), where l(x) is the Unear function such that I( c - 6) = tp( c - 6) and lee) = ep(c). Then 9 is continuous and tp S 9 on [0, 1]. By the approximation theorem of Weierstrass (see, e.g., 3.1.33) there is a polynomial P 2 such that £ 19(z) - P2(%)1 < 2 for x e [0.1]. Likewise, we define hex) = { tp(x) if % e [0, c) U [c + 6,1J, min{l. (2:), cp(x)} if % e [c, c + 6),
3.3. Power Series 369 where II (x) is the linear function such that 11 (c) = \O(c-) and I. (c + 6) = ep(c + 6). Clearly, h is continuous and h < VJ on [0, IJ. By tbe approximation theorem of Weierstrass there is a polynomial PI such that E Ih(x) - PI (x) I < 2 for x E rOt 1]. Moreover, we have 1 1 (g(x) - <p(x»dx = ( (g(%) - <p(:c»clx. o J(c-6,c) H we put A = {x E (c- 6,c) : g(x) = lex)} and B = (c - 0, c) \ A, then we get r (g(x) - cp(x»d.x = ( (g(x) - fP(x»dx J C c-6,c) J A < ( I'(x) - rp(x)ldx J(c-6.c)  1. (II/'(z) - 'P(e - 6)1 + 1'P(e - 6) -I(x)lclz (c-6.c) E 6 E <i+ M 2 < 2 . It follows from the above that f (Pa(z) - tp(z»tb: = l (P2(X) - g(x»dx + l (g(x) - tp(x»dx < E. In an entirely similar manner one can show that l (tp(x) - PI (x»dx < E. We now turn to the proof of the theorem of Hardy and Littlewood. Without loss of generality we can assume that A = 1. We first show that 00 I lim (I-x) L4n Xn p(x n ) = 1 P(t)dt z-+l- n =O 0 
370 Solutions. 3: Sequences and Series of Functions for any polynomial P. Clearly, it is enough to prove the equality for P(x) = xi. We have  1 00 lim (1 - x)  anx n + kn = lim - x (1 - xk+l) " anx(k+l)n %J - L.J zJ- 1 - Zk+l L.-, n=0 n =O = 1 = (1 tltdt. k I 1 10 Now we define <p by { 0 for 0  x < e- I , cp(z) = 1  -I < < 1 z lor e _ x _ · Our task is to show that 00 1 1 (1) lim (1- x) " anxncp(x n ) = f,O(t)dt = 1. zl- L....J o 0 fa= It fol1ows from t.hp. rnnsiclp.rittions prpnt.Prl at. thp hpginning of the solution that, given c > 0, there exist polynomials PI and P2 such that E E PI (x) - 2 < h(x) < cp(x) < 9(x) < P2(X) + 2 and l (P2(Z) - \p(z»dz < e, l (\p(z) - PI (x»dz < e. Sinc an  0, we get 00 00 lin i (1 - x) '" anxn(xn)  1im (1 - x) '" anz n P2(X") +  2 z-+I- L.J zl- L.J n=O n=O . {I £ {I 3E = Jo P2(t)dt + 2 < 10 tp(t)dt + 2 COI1::;quut1y, 00 1 iiiii (1- x) E anxnVJ(xn) < f (t)dt. %1- n=O 10 In much the same way one can show that 00 1 1 lim (1 - z) E anxn(xra)  fP(t)dt. z-+l- n=O 0 
3.3. Power Series 371 So (1) is proved. Therefore 00 N 1 = lim (1_e- 1IN )  One-n/Ntp(e-n/N) = Um (l-e- I / N )  On. N  Noo  n=O n=O Since lirn (1 - e- 1 / N )N = 1, we obtain N-+oo N :E an lim n=O = 1. Noo N 3.3.26. H IRani S C, then Cor x E (0,1), 00 00 1 II"(x)1 ::;  n(n -1)IOnlx n - 2 < C  (n _1)x n - 2 = C (1- :1:)2 " It then foUows by 2.3.23 that 00 lim (1 - x)j'(x) = 0 = lim (1- z) "" t'UlnZn-1. z-+l- .rI- L..J n=1 Now since F(z) =  ( 1- '&an ) xn-l = 1 _ J'(x)  C I-x C' we obtain lim (I - z)F(x) = 1, which combined \\ith the above z-+l- theorem of Hardy and Littlewood yields n :E(I-) lim k=l = 1. n-+oo n Hence n E kak lim =1 = o. n-+oo n To end the proof it suffices to apply the result given in 3.3.19. 
372 Solutions. 3: Sequences and Series of Functions 3.3.27. Suppose, contrary to our claim, that lim an = O. Then, n-+oo given E > O. there is no such that if n > n{). then Ian I < E /2. So. no 1(1- x)f(x)1 < (1- z) L akz k + t k=O which implies lim 1(1 - x)/(z)1 = 0, z-+ 1- contrary to the assumption. 3.4. Taylor Series 3.4.1. Suppose that If(n)(x)1 5 }.If for n E N and x E [a, b]. By Taylor's formula with the Lagrange form for the remainder (see, e.g., 2.3.3 (a» we have n j(k) (xo) k f(x) = L k! (x - xo) + r n (2'), 1:=0 where I ( )1 _ I jCn+l)(xo + 8(x - xo» { _ ) n+l M (b - a)n+l r n x - en + I)! x Xo S en + I)! · Hence lim Tn (x) = O. Consequently, n-+oo n j(k){X) 00 j(k)(xO) f(x) = lim "'" 0 (x - xo)" = " (x - xo)". n-t-oo L..., k!  k! 1:=0 k=O 3.4.2. No, because /(n)(o) = 0 for n = 0,1,2,..., and lex) :F 0 for xO. 00 . 2 3.4.3. By the M-test of Weierstrass the series E Cb5: z) and the n=0 00 2.(2) derived series L -n 8; n:r: converge absolutely and uniformly on n=0 1ll So /' is continuous on nl Repeating the reasoning, we see that / is 
3.4. Taylor Series 373 in CCO( .). Moreover, one can find that /(2i-I){O) = 0 and 1<21&)(0) = 00 .... (-1)' E 7. Thus n=O 1/(D)(O)Iz2i ( n2z ) 2At_n (2k )! > 2k e, z 1: 0, n = 0, 1,2, · .. · If we take n = 21;, we get 1/(2t) (O)Iz21; ( 2kz ) 2i I e I (2k)! > e > 1 for x;' 0 and k> 2; · Therefore the Taylor series of I about zero diverges Cor z  0 and the equality carm ot hold if z ;' o. 3.4.4. Suppose first that % > O. Tbe Lagrange form for the remainder in Thylor's formula of I(z) = (1 + z)Q is ( ) _ o(a -1)... (Q - n) n+l (l + /} ) o-n-l r n % - (n + I)! % u% . For Ixl < 1 we have . ()(o - 1)... (0 - n) n+l_ lim (1 )' Z - o. noo n+. To see this one can apply, for example, I, 2.2.31. Consequently, to prove that lim rn(z) = 0 it suffices to show that {(I + 6z)O-n-l} is noo a bounded sequence. This follows from the follo\\ing obvious inequal- ities: 1 S (1 + 8x)Q S (1 + z)Q   for a  0 and (1+z)Q(1+8%yr1 for 0<0, and (1 + 9Z)-R  1. Thus we have proved the given equality for o < % < 1. Now we turn to the case z < O. The Cauchy form for the remainder in Taylor's formula of I(x) = (1 + z)Q (see, e.g., 2.3.3(b) . 18 r (z) = 0(0 - 1) ... (0 - n) Zn+l (1- 9)n(1 + &)O-ra-l n (n + 1)1 · 
374 Solutions. 3: Sequences and Series of Functions As above, it suffices to show that {(1-lJ)n(1+0x)o-n-l} is a bounded sequence. Since x E (-1,0), we see that (1- 9)" < ( 11; r < 1. Ioreover , 1 < (1 + 6x)O-1 < (1 + X)O-l if Q 1 and (1 + 3:)0-1 < (1 + Ox)n--I < 1 if a > 1. This ends the proof of the equality for x E (-1,0). 3.4.5. Suppose first that :c o. Then the e quality 1%1 = .../ 1- (1- Z2) and Newton's binornial formula with a =: 1/2 (see the foregoing prob lern) give 00 1:r. 1 = 1- 1 (1 _ :c 2 ) _ " 1.3... (2n - 3) (1- x2)n 2 L-i 2 71 n! n=2 = 1 _ 1 (1 _ 2 ) _ (2n - 3)!! (1 _ 2 ) R 2 x (2n)!! :c. 00 l\11oreover, note that the series L C2 n !!! converges because, by the n=2 Wallis formula (see, e.g., 1, 3.8.38), (2n-3)!! 1 . (2n)!! 1m I - Vi " n-+oo 7r (2n-l) vn 1 Therefore the Abel theorem (see, e.g., 3.3.14) shows that the e luality holds also for x = o. 3.4.6. Termwise differentiation shows that f is in Coo ( - R, R). Ailore- over, 00 J(k}(x) = E n(n -l)(n - 2)... (n - k + l)a n x"-k. I:
3.4. Taylor Series 375 Hence 1(1;)(0) = k!ak for k = 0,1,2,... . 3.4.7. Observe that 00 I(x) = L an «X - XO) + xo)n n=O = f: an t () (:c - :cu)k:cg- k n=O k=O =  (  ()anx8-") (x_XO)k, To see that the last equality holds, note that F?' a,,() (x - xo)"x8- k = f lanl(lx - :tol + Ixo!)", Consequently, the double series on the left side of this equality con- verges absolutely for Ix - Xo I + Ixo I < R, and therefore the result in I, 3.7.23 can be applied. No\\. differentiating term by term we get Jlkl(xo) = f: ( )anx-kk! for k = 0,1,2,..., n=k Thus J("xo) = f: ()anx8-k for k = 0,1,2,.... n=k 3.4.8. Set Cia = an - b n and 00 f(x) = E c"xn, x E (-R,R). n=O (1) 'l'hen f(x) = 0 for x e A. Now let B be the set of all limit points of A that are in (-R,R), and put C = (-R,R) \ B. Then.O is open. By assumption B is nonempty. Clearly, (- R R) = B U C. Our task is no\v to prove that B is also open. To this end take Xo E B. By (1) and the result in the foregoing problem, 00 (2) J(x) = :E dn(x - zo)n, Ix - xol < R -Ixol. n=O 
376 Solutions. 3: Sequences and Series of Functions Now we show that d n = 0 for n = 0, 1,2, . . . . If this were not the case, then we would find the least nonnegative integer k for which dk :/: 0 and we would get f(x) = (x - xo)kg(x), where 00 g(x) = L dk+n(X - xo)n, Ix - xol < R -ixol. n=O Since 9 is continuous at Xo and g(xo) = dk # 0, there would exist 6 > .0 Sllc.h that O(x)  0 for Ix - zol < 6, contrary to. the fact that Zo is in B. Thus dn = 0 for n = 0,1,2,... , and consequently, f(x) = 0 for Ix - xol < R -Ixol. So we bave proved that B is open. Since ( - R, R) is a connected set, we see that C = 0 and B = (- R, R). 3.4.0. We will apply the result stated in 3.4.6. (a) Since 00 :c 2n + 1 sin x = L(-I)R (2n + I)! ' x e JR, n=O we get 00 x 6n + 3 sinx 3 =  (_I)R (2n + I)! ' x e III (b) In view of the identity 5iIi 3 x =  sin x - l sin 3x, z E Ii, we get 3 00 x2n1 sm 3 X = - "(_1)n1(32n -1) x E 1ll 4  (2n+ I)!' (c) We have sinxcos3z = l(sin4z - sin2x), z E nl Thus 1 00 x2n+l sinxcos3x = - L(-1)n(4 2n + 1 - 22n+l) Z e IR. 2 n=O (2n + I)!' (d) We have sin 6 x + cos 6 x = i + i cos4x, x E Ii, and 00 ., n Z" COSX = L(-I)R (2n)! ' x e III n=0 
3.4. Taylor Series 377 Consequently,  I,) 00 . 2n . 6 e i).:J  ( ) n 4 2n :t: 1m SID :r + cos :r = 8 + 8 t:o -1 (2n)!,:r En. (e) Since 00 n In(1 + z) - E<-1)'I+l=-, z E (-It 1), n n=1 we get 1 1 + x 1 00 ::c 2n + 1 2 10 I-x = 2(10(1+x)-ln(1-z» = E 2n+ l ' x E (-1,1). n-O (f) Clearly, m{! + x + x 2 ) = In 11:s , x E (-1t 1). Therefore, as in (e) we have 00 In(1 + % + x 2 ) = E Qllx n ., :c E (-1.,1), n=1 where { -  for n = 3k, k = 1, 2, 3, .. . , an=  for n =F 3k, k = 1,2, 3 p .. . 1_5z1+6Z2 = 1!3% - 1':2% ' we obtain (g) Since 00 1 =  (3"+1 _ 2"+1 )x n z e (-1/3, 1/3). 1 - 5x + 6x 2 ,L" , n=O (h) We know that OOzn e'Z = E., x E IR, O n. n= and 1 co l_x =EX'\ xe(-l,l). n=O By the Mertens theorem (see, e.g., I, 3.6.1) the Cauchy product of these two series converges for Ixl < 1, and ez =  ( 1 + .!. + .!. + .. . + .!.. ) xn. I-x  l! 2! n! 
378 Solutions. 3: Sequences and Series of Functions 3.4.10. (a) \Ve have /(%+1)= (%+2)+1 = f: e(n-:-2) z'" zE.. n=O n. Hence fez) = f: e(n -:- 2) (z -I)", z E R. R=O n. (b) As in S.4.9(b), one can show that /(z + 1) = e t (-I)"  (_)i ) z", z E (-1,1,). Thus J(%) =e  (-I)" (_)i ) (z -I)", z e (0,2). (c) Apply the identity 005% _ oos 1 005(% -1) - sin 1 sin(z - 1) z 1 + (z - 1) . (d) Reas oning similar to that presented in the solution of 3.4.9(h) yields ¥ = f: « _1)"+1 t  ) (x _l)ft, z e (0,2). n=1 J:; I 3.4.11. (a) By 3.4.4, 1  (-1)tI(2n -I)!! = 1 + L...i x n vI +% n=1 (2n)!! for 1:z:1 < 1. Hence 1 = 1 f: (2n-l)!! rn 11' 1 - z2 + 0- 1 (2n)!! · 
3.4. Taylor Series 379 Set 5 ) - (2n -1)!! 2nt-1 (x - x + L-, (2' )"(2 1) x J n... n+ n= and note that (arcsin:c)' = Vl Z2 = 5'(x). So arcsinx = 5(x) + C. lvloreovcr,sinceS(O) = 0 = arcsin 0, weobtainS(x) = arcsinx. (b) Set x. S(:c)=L:(-l)n 1 :t2n+l. n::O 2n + 1 In view of the well known identity 1 oc 1 + 2 = L<-1)n x 2n, Ixl < 1, o x 0 n= we get (arctarax)' = 11.:r 2 = S'{x). Thus Sex) = arctan x + C. Since arctan 0 = 5(0) = 0, we see that C = O. To obtain the first equality it is enough to put z = in (a). co To obtain the second one, observe tbat L (-1)" 2ra J converges and n=O apply the Abel theorem (see, c.g, 3.3.14) to the power series (b). 3.4.12. (a) Applying the Taylor series expansion for arctanx (given in the foregoing problem) and for 111(1 + x2.), we obtain 1 2 00 (-I)n- 1 x2 n xarctan:c - 2 ln (1 + x ) = 2n(2n -1) ' x E (-1.1). (b) Applying the Taylor serle:) expansion for arcsin a: (gi veu in the foregoing problem) and Newton's binomial formula (see 3.4.4), we get 2 oc ( ?o _ 3) 11 . V I 2 x '"" _n .. 2n ( ) xarcsrnx+ - -x =1+2+ (2n)!!(2n-l) X ,xE -1,1. '1--
380 Solutions. 8: Sequences and Series of FUnctions 8.4.18. (a) Let 00 1 f(x) = 2: xn+1, Ixl < 1. '11,=1 n(n + 1) Then 00 1 f'(x) = 2: -x n = -In(l- x), Ixl < 1. n '11,=1 So f (x) = (1 - x) In(1 - x) + x for Ixl < 1. Now the Abel theorem gives 00 (_1)'11,+1 2: ( 1) = 2In2-l. '11,=1 n n+ (b) For x E IR, we have 2: 00 (-I)nn 2'11,+1 _ 1 2: 00 (-1)'11, 2'11, 1 2: 00 (-1)'11, 2'11,+1 x - -x x - - X '11,=0 (2n + I)! 2 '11,=0 (2n)! 2 '11,=0 (2n + I)! = (xcosx - sin x). Putting x = 1, we get (-I)nn 1 . (2n + I)! = 2 (cOS 1- sml). (c) It follows from the equality 1 1 ( 1 1 ) n 2 + n - 2 = 3 n - 1 - n + 2 that if 0 < Ix I < 1, then f: (-I)n xn-l = 1 f: (-I)n xn-l _ 1 f: (_I)n xn-l '11,=2 n 2 + n - 2 3 '11,=2 n - 1 3 '11,=2 n + 2 1 00 '11, 1 00 '11, = - 2:(_l)n-l + - 2:(_I)n-l 3 '11,=1 n 3x 3 '11,=4 n = In (1 + x) + -.2.... ( In (1 + x) _ x + x 2 _ X 3 ) 3 3x 3 2 3 .
3.4. Taylor Series 381 This combined with the Abel theorem gives 00 (-l)n :2 5 --ln2-- L-, n 2 + n - 2 - 3 18. n=2 (d) The sum is 11'/2 -ln2. To see this, apply 3.4.12(a) and the Abel theorem. (e) By Newton's binomial formula (see 3.4.4), 1 L oo (-1)n(2n -I)!! 2 = 1 + x n for Ixl < 1, v I + x 2 n=1 (2n)!! and hence, by the .L\bel theorem, f (-1)R(2n -I)!! - ...!... n=l (2n)!! - V2,. (£) Clearly, 00 (3x ) n+l L =3xe 3z , zenL O n! n= So 3 f: (3x)n( + 1) = (3a;<;:3%)' = e3 (3 + 93:). O n. n= Putting x = 1 gives f 3 n (n + 1) = 4e 3 . n=o n! 3.4.14. The interval of convergence of the series is (-1.,1). Let S(z) denote its sum in that interval. Then 5'(:z=) = 2 an -1)!)2 (2x)2r.-l L.J (2n -1)! n=1 and S"(z) = 4 «n -1)!)2 (2x)2n-2. L-, ( 2n - 2 ) 1 n=1 It then follows that (1 - x 2 )S"(x) - xS'(x) = 4, Ixl < 1.
382 Solutions. 3: Sequences and Series of Functions Multiplying both sides of this equality by (1 - X2)- produces ( V 1 - x2S '(x»)' = V1 X2 ' Consequently, S ' ( ) 4 . C x = V arcsin x + V 2 ' 1-x 2 I-x and therefore Sex) = 2(arcsinx)2 + C arcsin x + D. Since S'(O) = S(O) = 0, we obtain Sex) = 2(arcsinx)2. If x =:1:1 we get the series «n - I)!)2 4 n Li ( 2n ) ! ' n=l which converges by the Gauss criterion (see, e.g., I, 3.2.25). Indeed, we have a n +l = 1 - + 0 ( ) . an 4n n 2 So the Abel theorem gives f «n _1)!)2 4 n = 11"2 . n=l (2n)! 2 3.4.15. For a E I, f(x) = f(a) + f'i ) (x - a) + · . . + f(: a) (x - a)n + 14. (x), where I I X Rn(x) = .. f(n+l) (s)(x - s)nds. n. a Applying the change of variable formula twice, we obtain 1 (x-a Rn(x) = n! 10 f(n+l)(u -+ a) (x - u - a)ndu (x a)n+l 1 1 = - f(n+l) «x - a)t + a)(1- t)ndt. n! 0
3.4. Taylor Series 383 The monotonicity of f(n+1) implies that if a < x < b, bEl, then, (x a)n+1 1 1 o < Rn(x) < - , f(n+l)«b - a)t + a) (1- t)ndt n. 0 = G=: f+1 (b). Clearly, Rn(b) < f(b). Thus O < (x) < G=: f f(b) for a<x<b, a,bEI, and therefore, lim Rn(x) = O. This shows that Taylor's series con- n-+oo verges to f uniformly on each compact subinterval of I. Since a < b can be arbitrarily chosen in I, the analyticity of f follows from 3.4.7. 3.4.16. The proof is similar to that of 3.4.1. 3.4.17 [18]. Let Xo be arbitrarily chosen in I. By assumption, there is an r > 0 such that 00 fen) ( ) f(x) = L ,x o (x - xo)n for Ix - xol < r. n. n=O Differentiating m times yields ( ) L oo f(n) (xo) n-m f m (x) = n(n - 1) ... (n - m + l)(x - xo) . n! n=m Hence If(m) (x) 1 < f: If(n xo)l n(n -1)... (n - m + l)lx - xol n - m . n=m It follows from the definition of the radius of convergence of the power series (see, e.g., 3.3.1) that for 0 < p < r there is a positive C such that If(n) (xo)1 < !2. , - P n n. Consequently, If(m)(x)1 < f: n(n -1)... (n - m + l)lx - xoln-m. n=mP
384 Solutions. 3: Sequences and Series of Functions Therefore, in view of the identity  I '" n(n -1)... (n - m + l ) xn-m = '1"'. I xl < 1, L-i ( 1 - X ) m+l ' n=m we arrive at IJ(m)(z)1 S; p-m ! p"m n(n -1) · .. (n - m + 1) Ix - xoln-m _ Cm! < Cpm! pm (1- J7GL )m+l - (p-pl)m for I - ol < PI < p. So we can take J = (zu - pIJZO + Pl.), A = Cp and B = p - Pl. 3.4.18 [18]. Set 1 1 lex) = 1- A(x -1) for Ix -11 < A and 1 get) = 1 _ t for It I < 1. 1-t h(t) = (1 0 g)(t) = 1 _ (A + l)t . Then Clearly, co co f(x) = LAR(x -l)n, g(t) = L tn. n-O "-0 Moreover, 1 h(t) = 1- (A + l)t t }.- (A + l)t 00 co - 2: (1 + A)nt n - 2: (1 + A)Rt n . 1 n=O n=O 00 = 1 + 2: A(1 + A)n-1t n . n=1 Since g(n)(o) = n!, 1(1)(g(O» = 1(1)(1) = n!AR and h(n)(o) = n!A(l + A)n-l, application of the Faa di Bruno formula gives the desired equality. 
3.4. Taylor Series 385 3.4.19 [18]. Let Xo be arbitrarily chosen in I and let Yo = /(xo). It follows from 3.4.17 that there are intervals 11 C I and J 1 C J (cont ainin g Xo and 1)0, respective)-) and poitive constants A, B, C and D such that . If(n)(x)1  A ; for x E 11 and I Ig(n)(y)1 < C  for y e J 1 . By the formula of Faa di Bruno, In) _ E n! (Ie) ( /(1)(%» ) lcl ( /(2)(z» ) 2 ( /cn)(:z;» ) Ie.. h (;,;) - 9 (J{%» ---... . . .. . kl !k2! . . . k n ! I! 2! n! where k = k 1 + k 2 +. . . + k n and the sum is taken over all k.,  I . . . , k n such that k l + 2k 2 + . · · + nk n = n. This combined with the result in the preceding problem gives n n! C k! ( A ) lei ( A ) 1e2 ( A ) i" I/t( >(x)1  E k 1 !k2! ... k n ! Dk 8 1 lfi . . . Bn n! Ck! Ak n!C k! Ai = E kl!!'. . k n ! Dk Bn = Bn L k l !k2! .. · k n ! Dk _ n!C A ( A ) n-l - Bn D 1 + D . It now follows from tbe result in 3.4.16 that h is real analytic on I. 3-4..20. It foUows from 3.4.15 tbat g(x) = f( -x) is real analyt.ic on the interval -I = {x : -x e I}. Since x  -x is real analytic, the result follows from the foregoing problem. 3.4.21 [18]. Consider g(t) = 1 - V l- 2t, It I < 1/2, and f(x) = 11 z' 13:1 < 1. Tben 1 h(t) = j(g(t» = = g'(t). V I - 2t So 9(n+I){t) = h(n)(t). Moreover, by the Newton binomial formula ( 3.4.4), get} = - f: (  ) (-2t)n. n=1 
386 Solutions. .3: Sequences and Series of Functions 00 Clearly, f(x) = E x n _ Consequently, g(n)(o) = -f1!(!)( _2)" and ,.=0 J(nJ(g(O» = n!. Finally, by the formula of Fail di Bnlno, -<n+l)! (  ) <_2)IJ+I=g(n+1 J (O)=h(n)(o) n+l = n! k! ( _ (  ) (_2» ) lcl ... ( _ ( k ) (-2)ra ) k ll L.J k 1 !k 2 ! .. . k n ! 1 n {-l)kk' ( l ) kl ( ! ) kn = (-2)"n'  · 2 . ... 2 . L., kl!k2!... k n ! 1 11' where k = kl +k2+.. .+k n and the sum is taken over all kl,k2,.. .,k n such that kl + 2k:2 + . . . + nk n = n. 3.4.22 [18]. Observe first that if J satisfies the assumptions stated in the problem, then its inverse 9 exists in an open interval cont inin g J(xo). l\iIoreover, 1 g'(y} = h(g(y», where h(x) = f'(x) . It is nlso clcar that since / is in Coo, so i g. Our task is now to prove that 9 satisfies the assumptions of 3.4.16. 'Ve know, by 3.4.19, that h is analytic in SOnle open interval containing Xo (as a CODlposition of two analytic functions). Therefore, by 3.4.17, there are positive constants .4. and B such that Ihln)(x)1 < A* (1) in some open interval 10 C I cont ainin g Xo. No",. induction will be used to sbow that there is an open interval K containin f(xo) such that (2) ( 1 ) (2A}n I g(n) ( y) 1 < n!(-l ) n-I 2 for y E K. - n 8 n - 1 We choose K so that g(K) is contained In 10. Then, by (1), we have Ig'(y)1 = Ih(g(y»)1 < At which proves (2) for n = 1. Assuming (2) to hold for k = 1,2,..., fi, we will proy- it for n + 1. By the foregoing 
3.4. Taylor Series 387 problem we get Ig(r1+I)(y)1 = I(h 09)(n)(Y)1 n! k! (( ) (2A) ) k 1 ... ( ( _l)n-l ( i ) (2A n ) kts k 1 !k2! . . . k n ! BI: 1 n Bn I = (_1)nn! (2A)R A'" (-l)kk! ( ! ) " k. ... ( l ) 1-" Bn L., k 1 !k 2 !... kr&! 1 n (2A)r& ( 1 ) = (-I)nn! A2(n+l) 2 B'I n + 1 = (-l)U(n + 1)! (2A)JI+l ( ) . Bn n + 1 This completes the proof of (2). Thus the analyticity of 9 on K follows from 3..:1.16. 3.4.23. It follows from /-1 (x) = f'(x) that I maps the interval (0,00) onto itself and tbat f is ill Coo on that interval. Hence f'(x) > 0, and J is strictly increasing 011 (0,00). Differentiating the equality f(/'(x» = x, .we see that I"(x} > 0 for z E (0,00). We claim that (-l)"f(n){x) > 0 for x E (0,00) and n > 2. We will prove this by induction, using the forrnula of Faa di BnJno (see 2.1.38). Suppose that (-I),n j(nt) (x) > 0 for m = 2,3,..., n. Then 0='" n! f(k)(f'(x» ( fll X» ) kl ( J(3)(X» ) k2 L-, k 1 !k:!! . . . k n - 1 ! I! 2! ... ( f(n) (x) ) k.._. + f'(f'(x»f 1 n+1J(x), (n - 1)! where k = k 1 + . . · + k n - 1 and the sum is taken over nll k J , . . . , k n - 1 such that k 1 + 2k 2 " + . . . + (n - 1 )kn-t = n. Since the sign of each term under E is sgn ( -l)k( _1)2k 1 (_1)3k 2 ... (_l)nku_l) = (-1)", we get. . sgn (/'(/'(x»)/(8+1) (x) ) = sgnft n + 1 )(x) = _(_l)n.
388 Solutions. 3: Sequences and Series of Functions Now the result in 3.4.20 shows that / is analytic on (0,00). 3.4.24. We know 1 by the foregoing problem, that each I satisfying the assumption is analytic on CO, 00). We first show that there is exactly one number a > 0 such that f(x) < x if x e (0, a), and f(x) > z if x > a. To this end, observe that by the monotonicity of / we have lim f(x) = 0, which to ether with the equality J'(J{x»f'{x) = z-+o+ xf'(x) gives (1) j(f(z» = foz tf(t)dt. Now if I(x) were greater than z for 0 <:c < Ii then (1) would imply foz j'(t)(t -1) > 0, contrary to the fact that f'ex) > 0 for :r > O. On the other hand, if /(:#;) <: z tor all z e (0,00), then (1) would imply j(z) > j(f(z» = foZ t!,(t)dt > foZ f(t)f'(t)dt = (f(Z»2 , which in turn would give f{x) < 2" for x> 0, contrary to the fact that I{(O, 00» = (0,00). CU1l5equel1tly, by t,h iJlt rl1J diaLe value properLy there is a fixed point a of /. Since fez) < x for x E (0, a), we see that I'(Y) = J-I(y) > y for y E (O,a). Likewise, J/(y) < y for y > a. Now we turn to the proof of the uniqueness. Suppose, contrary to our claim, that there are two such functions /1 and 12- Let al and Cl2- be the fixed points of fl and 12, respectively. Clearly, we can assume that al > Cl2. Set 9 = 11 - 12. H al = B2 = a, then g{a) = 0 and /-1 = /' implies that g(n)(a) = 0 for n E N. So since 9 is analytic, 9 is a constant function (equal to zero) on (0, (0). If a1 > tl2, then /l(X) < z < 12(:C) and f;(x) :;> x > /2(x) for [a2,ul). Th r rur g(x) < 0 and g'(x) > 0 for x E [a2,al). Since lirn g(x) = 0, there z-+o+ is b E (O, 2) such that g'(b) = 0 and g'(x) > 0 for x e (b,a}), and g(x) < 0 for x E [b,al). Set f (b) = / (b) = 61. Then b ' e (b' )J because b <: J (b) = b' < 14(0.3) = Ila- Henc-.e g(h') < o. On the other hand, /1(11) = Il(Ji(b» = b and J2(b') = J2(f (b» = b, a contradiction.
3.4. Taylor Series 389 3.4.25. If f(x) = axc, then f'ex) = acx c - l and f-l(x) = a-x. This gives c = l+2v'5 and a = cl-c. 3.4.26. By Taylor's formula proved in 2.3.10, N 1 ( ) 2n+l In(1+X)=2 2n+1 2:X + RN (x) , where . 2 ( X ) 2N+3 RN(X) = (2N + 1)(1 + 8X)2N+3 2 . Clearly, lim RN(X) = 0 for x E (0,2). Consequently, Noo 00 1 ( x ) 2n+l In(1+X)=2 2n+1 2+x · 3.4.27 ['lUng-Po Lin, Amer. Math. Monthly 81 (1974), 879-883]. By definition, z- y 1/ L(x, y) Inz-lny 2 P(x - y) Mp(x,y) - CIPIYP )l/P - (xp +yP)l/Pln : for distinct positive x and y and for p ¥- O. Dividing the numerator and denominator by y and putting z = ( : ) P , we obtain L(x,y) _ 2 1 / P (Zl/P -1) Mp(x,y) - (z + 1)lf p lnz 1 / p . Now writing l+w z= 1-'w ( z-l ) w = z + l ' 0 < Iwl < 1 
390 Solutions. 3: Sequences and Series of Functions .and multiplying the numerator and denorninator by (1-2w I/P , we ar- rive. at L(x,y) _ P21/P(( r/P -1) Alp (x, y) - ( !:I:!!! + 1 ) 1/ P In .!.:f:.!!!. J-rv J-w pC (1+w}llP _(1-W)I /J1 ) 2w _ f(w,p) - In(1+w)-ln(l-w) - g(w ) · 2w Clearly, 00 1 g(w) = "'" w 2n t ;0 2n + 1 and by 3.4.4, f(w, p) = 1 + 1 ( ( 1 - 1 ) ( 1. - 2 ) . . . ( ! - 2-n ) . ] ) w 2n . 211 + 1 p p p (2n)! n= 1 Consequently, to prove that f(w,p) < g(w) it suffices to show tbat for any positive integer n, ( ! - 1 ) ( ! - 2 ) . . . ( .!. - 2n ) 1 :S 1 p p p (2n)! and tbe strict inequality holds for at lest one n. For n = 1, we have ( -1)( -2). .< 1 for p > , because 1 3 1 ( 1 ) - - - + 1 = 1 - - 3 - - < 1, 2p2 2p 2p p if 0 < ! < 3 p Hence ( - 1) ( - 2) ( - 3 ) · .. ( - 211) Qn = (2n)! = (1 - ;) (1 - 2 ) (1 - ) .. · (1 - ) .... ...'-.. ".. Q, I for p 2: !. SO Q1 < 1 for p > i, and the last formula shows that Q n < 1 for 11 = 2, 3, . .. .
3.4. Taylor Series 391 3.4.28 ['lUng-Po Lin, Amer. Math. Monthly 81 (1974),879-883]. We adopt notation from the solution of the foregoing problem. We have Ql > 1 for p < i. Thus there is 0 < h < 1 such that if 0 < w < h, then f(w,p) > yew). Now observe that the inequality 0 < w < h can be rewritten in the form l<z<r1', where r= G r/p and z=( : r. This means that there is an r > 1 such that L(x,y) > Mp(x,y) if 1 < ; < T. 3.4.29 ['lUng-Po Lin, Amer. Math. Monthly 81 (1974),879-883]. Putting (l+W : I I = (l-w ' 0 < w < 1, we get (1+w)2 1 x- y i"- 1 (1-w)2 L(x, y) Inx-lny In =- 4(w+ 1:. w 3 + 1:. w 5 +...) _ _ 11 _ 3 5 Mo(x,y) - (Xy)1/2 - ( t2 - + : 1 1 - .11 1 - w 2 1 + -w 2 + -w 4 + . . . 3 5 1 + w 2 + w 4 + w 6 + . .. = 1 1 1 > 1, 1 + '3W2 + sw 4 + 7 W6 + . . . which combined with 2.5.42 and 2.5.43 implies the desired result. 3.4.30 ['lUng-Po Lin, Amer. Math. Monthly 81 (1974),879-883]. Under the notation introduced in the solution of 3.4.27, we have L(x,y) p ((1 + W)l/P - (1- W)l/P) = !:I:!£ ----t O. M ( X y) In w w-tl p , l-w Since z = ( r = +:: , we get L(x,y) < Mp(x,y) for sufficiently large z.
. ebliogra by - Books References (I] J. Band, S. W Zlridr " . onGlizy mQ mtJtycznej., \V". dawDictwa Naukowo-TechniczDe, \ V a, 1994. [2) VeL Bernik, O. V. fe1nikov t L K. Zuk,. Sbomik olimpiodnyda zad po m , Narodnaja Minsk, 1980. (3) P. BUer, A. \Vitkowski. Pro&kms in A athematicol Anal. $", 11 c 1 Dekker, In New York and Ba:;el, 1990. [4J T. J. Brom, "ic:h, An Introduction to the 77&eorr oJ Infinite Smu, Macmillan and Co.. Limited, London. 1949. (5) R. P. Boas. A Prime1" oj Real Analytic Funcliou, Birkhiuser V rl Basel Boston Bern 1992. (6) L. Carleson. T. \V. Gamelin, Comp," DynomiCl, Springer- Ver1a& New York Berlin Heidelberg, 1993. (7) B. P. Demidovi Sbomik zedot i 'Uprt.tZnmij po matem "lukomu Gnd. i.ru, Naub, Moskva, 1969. (8) J. DieudoDD6, Foundafionl of Afothm Anol Academic Press, New York San Francisco London J 1969. [91 A. J. Dorogovcev, Mate matiWkij onalU. SpnIt1Olnoe po&Obe, VyR.aja aJroJa, Kiev, 1985. (10) A. J. Dorogovcey. Motem4tiwiQ ancli& SbomiI: :Qda VyRaj amI Kiev, 1987. (11) G. "I. Fichtenholz. Di/fumtial-urul.lnkgrohchmmg, 1, D, lI1, V.B.B. Deutscher Verlag Wiss., Berlin, 1966-1968. 393
394 BibUography - Books (12) B. R. Ge1baumt J. M. B. Olmst,ed, Theomn.t 4fld Coun in MtJt1aem,o,tiu, Springer-Verlag, New York Berlin Heidelberg, 1990. [13) E. Hille, AaGlpU Vol. Blaisdell Pub rr.-;hing CompaD1, New York Toronto London, 1964. [14) W. J. Kac:'lor , "I. T. Nowak, Problems in Itfothemolicol Anal 1. Reel Numbers, S tmd Seriu. American Mathematical Society, Pro\idenc Rl, 2000. (15) G.lOambauer, 1\fathem4tia11 Analys "farccl Dekker, IDe., Nmv York, 1975. (16) G. Fbmb auer J Problmu aM in AnaltIsis. Marcel DH1Mr . Inc., New York and Ba.. , 1979. (17] K. Knopp, Theorie und Anwendung tier Unmdliden Springer- Verlag, Berlin aDd Heidelberg, 1947. (18) S. G. Kran1z, B. Parks, A Primer 01 Real AIUIlytic Punclions. Birkhau&et V lag, 1992. (19) L. D. Kudriavtsev t A. D. Kut , v. L Ch , M. I. Shabnnin, Problenuu ds An4li.ri.t M0Um4tU». Llmiu, Conlita uidGd, Dftrivolnli- dad (Spanish), Mir, losl"'Va, 1989. {2O) K. Kuratowski, Introdtcelion to Colculu&, Pergamon Press, Oxford- Edmb urgh-New York; Polish Scientific Publishers. \Varsaw. 1969. [21J S. Lojasiewica. Aa Introduditm to the Theoru oj Real Fundimu. A \\'lIey-lDterscience PublicatioD, Job '\\1i1ey & SoDS, Ltd.. Chichester. 1988. [22] D. S. Mitrinovit, 1 P. Noordhoff Ltd., Gron.iD- gen, 1964- (23) G. P61ya, G. Szego, Probkml and theomn8 ita 4n41ym I, Spriger- Verlag, Berlin Heidelberg New York, 1978. [24] R. Remmert, Tbeorr of Comples Springer-Verlag, New York Berlin Heidelberg, 1991. (25) JJ. Rivkind t Zod46 po matem4tit'momu onc&lUu, V jSaja SkoJ8t Afuisk, 1973. (28) W. L Bo.zboy, G. D. KlIldevanidze, N. G. Panfilov, Sbomii Z444t mf1tem Gti ft1VA olimpi44, Izdat. Univ. Drozhby Narodav, Moskva. 1987. [27] W. Rudin, D/ Matlamu1liaJl AnGlgria, lcGraw..Hill Book Company, New York, 1964. (28) W. Dzymowski. Coma prcprint.
Biblio grap hy - Books 395 [29J W. A. SadorniCij, A. S. Podkolzin, Zad4a muknluiit;h olimpiod po N lJoskva, 1978. [30) R. SikorsJd, Ftm1cje P\VN, \ V& .a., 1958- [31) H. SiJvennao, Complez ooriah1e3, HoughtoD MifBin Company, Bostons 1976. [32) E.C. 'l'itdmuush, The Theof1l DI Amc Oxford UDiverr-ity p London, 1944. (33) G. A. Tonojan t W. N. Scrgoov, StudenWkije m4ttmotitul;ije olimpio- tly, IzdateIstwo Erevanskogo Universitcta, Erewn, 1985.
Index Cantor Nt. 148 Caudl)' criterion for uniform conve... lence,82 Cauchy funetional equation. 27 Cauchy theorem. 8 c:ontinuoUi conversence. 84 conversence In the Cftlr o "11M, 30 Increaalnl. 3 midpolnt-eoDvex, 88 monotone. 3 piecewise Itrietly monotone, 17 real analytic. 105 Rmicontlnuoua. 18 atrictly convex. 81 strictly decreutnl. 3 Itrictly Inereains, 3 Itrlctl)' monotone. 3 aubadditive. 81 uniformly continuoul. 24 uniformly differentiable. 48 fundamental period, 13 Abel teat for uniform conWI'pDce, 82 Abel theorem, " approximation theorem of Welent...... 87 Balre property, 78 Balre &laeorem. 23 Bemateln polynomial. 87 Bernstein theorem, 10& Hamel bull. 187 Hardy and Littlewood theorem. 101 Holder Inequality. 85 deleted nelshborhood. 3 Dlnl derivdiw, 281 Dinl theorem. 84 Dirichlet Rries. N DIrichlet test for unlfonn co nverpn ce, 81 Incommensurate. 13 Intermediate value ProPenyl 14 Iterate. D equlcontlnult)'. 84 extended real Dumber lyatem. 18 Jenaen equation. 28 JeDHD inequality. 82 loprithmle mean, 74 Pal di Bruno fonnula, 44 fixed point. 1& function concave,el continuOUl In &he CeUro MnM. 30 convex, 14.81 decreaalnl. 3 81"1t Balre e..... 34 Minkowakl Inequality. 85 pnerallud. 66 modulUl of continult)', 27 Newton binomial formula. 102 Newton method, 78 397 
398 Index - .. .. . - - oscillation. 23, 83 power mean. 74 radius or onverpnce, 87 remainder tenn of Ta,ylor _rin Cauchy form, 53 Integral fonn, &3 Lqranae rorm, 63 Peano fonn, 62 SchIBmilch-Roche torm, 62 residual let, 78 Riemann zeta-runctlon, 88 , Schwan deliYatlve, 77 lower. 77 uniform, 79 upper, 77 strons derivative, 77 lower, 77 upper, 77 .ymmetrlc: derifttlve, 77 Tauber theorem, 100 uniform converaence. 81 Weienrtraulunctlon, H 
ProblelDs in MatheDlatical Analysis II Continuity and Differentiation W. J. Kac or and M. T. No'."ak 'We learn by doing. We Icc.l111 nlathcnlatic by doing prob- leln. Anu we learn Inore rnathenl.ltjc by doing mure problcnls. Thi i the equcl to P,.(}bll111.\ ill J1111/U 11I1Il;ClIl llZllysis J (Volume 4 in the Student Mathclnatical Library eries). If you w'ull to hone your understanding of contin- 110U and differentiable function thi book cOl1t'lin hundrl-d of problcnl to help you do o. The elnphasis here is on real functions of a inglc variable. The book i Inainly geared to\\'drd students studying the basic principles of analysis. fowever, given it clection of problelll. organi/ation and level it would be an idea] choice for tutand} or problcnl-sol\ ing sClninar particularly those oeared tov:"lfd the Putnan1 exam. It is alo uitablc for sclf-stud). The presentation of the Inaterial i, designed to help student conlpl chcnsion. to encourage theln to ask their o\\'n question. and to tart recdrch. The collection of prob- lelns \\'ill llso help tcachers who wish to incorporate problcln intn their lecture. The prohlenls are grouped into ection dccording to the nlcthod of solution. Solution for the problenl are provided. ISBN 0-8218-2051-6 11111111111 """ 11111111111